Download as pdf or txt
Download as pdf or txt
You are on page 1of 213

TRIGONOMETRY

02 MARCH 2024
REVISION: 4313

AZIZ MANVA
AZIZMANVA@GMAIL.COM

ALL RIGHTS RESERVED


Get all the files at: https://bit.ly/azizhandouts
Aziz Manva (azizmanva@gmail.com)

TABLE OF CONTENTS
3.4 Equations: Trig Identities 101
TABLE OF CONTENTS ................................. 2 3.5 Inequalities 111
1. RIGHT TRIANGLE TRIGONOMETRY .... 3 3.6 Graphs: Modelling 114
3.7 Graphs: Transformations 129
1.1 Trigonometric Ratios 3 3.8 Angle Equations 131
1.2 Trigonometric Ratios-II 10 3.9 Composite Trigonometric Functions 135
1.3 Triangles 13
1.4 Polygons 17 4. IDENTITIES AND MORE ................... 138
1.5 Heights and Distances: Basics 25
4.1 The Pythagorean Identity 138
1.6 Heights and Distances: II 35
4.2 Double Angle and Half Angle Identities 148
1.7 3D Problem Solving 44
4.3 Sum and Difference Identities 157
1.8 Bearings and Coordinate Geometry 53
4.4 More Identities 164
2. UNIT CIRCLE TRIGONOMETRY.......... 55
5. INVERSE TRIG FUNCTIONS ............. 170
2.1 Radians and the Unit Circle 55
5.1 Inverse Trigonometric Functions (ITF) 170
2.2 Unit Circle Identities 63
5.2 Equations 179
2.3 Law of Sines 69
5.3 Domain and Range 180
2.4 Law of Cosines 73
2.5 Geometry 80 6. FURTHER TOPICS .............................. 190
3. GRAPHS AND EQUATIONS .................. 82 6.1 Parametric Equations 190
3.1 Basic Graphs 82 6.2 Polar Coordinates; Circles 198
6.3 Graphing in Polar Coordinates 208
3.2 Equations: Basics 86
3.3 Equations: Change of Domain/Variable 98 6.4 Trigonometric Series 212
6.5 Further Topics 213

P a g e 2 | 213
Get all the files at: https://bit.ly/azizhandouts
Aziz Manva (azizmanva@gmail.com)

1. RIGHT TRIANGLE TRIGONOMETRY


Example 1.1
sin2 𝑥 − cos 2 𝑥 tan 𝑥 − 1
=
1 + 2 sin 𝑥 cos 𝑥 tan 𝑥 + 1

tan 𝑥 − 1
𝑅𝐻𝑆 =
tan 𝑥 + 1
sin 𝑥
Substitute tan 𝑥 = cos 𝑥
and simplify:
sin 𝑥 sin 𝑥 − cos 𝑥
cos 𝑥 −1 cos 𝑥 sin 𝑥 − cos 𝑥
= = =
sin 𝑥 sin 𝑥 + cos 𝑥 sin 𝑥 + cos 𝑥
+1
cos 𝑥 cos 𝑥
sin 𝑥+cos 𝑥
Note that on the LHS we have sin2 𝑥 − cos2 𝑥. Hence, multiply by sin 𝑥+cos 𝑥:
sin 𝑥 − cos 𝑥 sin 𝑥 + cos 𝑥 sin2 𝑥 − cos2 𝑥
= ∙ =
sin 𝑥 + cos 𝑥 sin 𝑥 + cos 𝑥 (sin 𝑥 + cos 𝑥)2
Expand:
sin2 𝑥 − cos2 𝑥
=
sin2 𝑥 + 2 sin 𝑥 cos 𝑥 + cos 2 𝑥
Substitute sin2 𝑥 + cos 2 𝑥 = 1:
sin2 𝑥 − cos 2 𝑥
= = 𝐿𝐻𝑆
1 + 2 sin 𝑥 cos 𝑥

1.1 Trigonometric Ratios


A. The Greek Letters
You might have already see the Greek Letter 𝜋 used in the formulas related to circles:
𝐴𝑟𝑒𝑎 = 𝐴 = 𝜋𝑟 2
𝐶 = 𝐶𝑖𝑟𝑐𝑢𝑚𝑓𝑒𝑟𝑒𝑛𝑐𝑒 = 2𝜋𝑟

1.2: Greek Letters


In trigonometry, we often use Greek Letters for measures of angles. Some of the common ones are mentioned
below:
𝛼 = 𝑎𝑙𝑝ℎ𝑎
𝛽 = 𝑏𝑒𝑡𝑎
𝛾 = 𝑔𝑎𝑚𝑚𝑎
𝜙 = 𝑝ℎ𝑖
𝜎 = 𝑠𝑖𝑔𝑚𝑎
𝜃 = 𝑡ℎ𝑒𝑡𝑎

➢ 𝛼, 𝛽, 𝛾 are the first three letters of the Greek alphabet.

B. The Trigonometric Functions


Trigonometry was first used by the Egyptians. They lived next to the river Nile, and every year when the Nile
flooded their agricultural marking were lost, which distinguished one field from another. To recover the
marking, they developed a sophisticated system of geometry including trigonometry.
Trigonometry is based on the study of triangles. Initially, right-angled triangles, but then building up and

P a g e 3 | 213
Get all the files at: https://bit.ly/azizhandouts
Aziz Manva (azizmanva@gmail.com)

generalising. Apart from geometry, trigonometry has applications in many fields.

1.3: Defining the Trigonometric Functions


If we have a right-angled triangle with one angle with a measure of 𝜃
(see diagram), then we can define certain ratios that are very useful.

opp adj sin 𝜃 opp


sin 𝜃 = , cos 𝜃 = , tan 𝜃 = =
hyp hyp cos 𝜃 adj

We can also define the reciprocals of these trigonometric functions:


1 1 1
csc 𝜃 = , sec 𝜃 = , cot 𝜃 =
sin 𝜃 cos 𝜃 tan 𝜃

1.4: The Six Trigonometric Functions


The six trigonometric functions are:
sin 𝜃 , cos 𝜃 , tan 𝜃
csc 𝜃 , sec 𝜃 , cot 𝜃

1.5: Mnemonic: Reciprocal Functions


Obtaining the reciprocal of sin 𝜃 and cos 𝜃 changes the initial letter.

1 hyp
𝐜sc 𝜃 = =
𝐬in 𝜃 opp
1 hyp
𝐬ec 𝜃 = =
𝐜os 𝜃 adj

Example 1.6
Find the six trigonometric ratios of 𝜃 for the diagram shown alongside.

Part A
In Δ𝐴𝐵𝐶, by the Pythagorean Theorem:
𝐴𝐵 = 8, 𝐵𝐶 = 15 ⇒ 𝐴𝐶 = 17 (𝑃𝑦𝑡ℎ𝑎𝑔𝑜𝑟𝑒𝑎𝑛 𝑇𝑟𝑖𝑝𝑙𝑒𝑡)

Now, we can use the definitions of the trigonometric functions to calculate their values:
𝑜𝑝𝑝 8 1 17
sin 𝜃 = = ⇒ csc 𝜃 = =
ℎ𝑦𝑝 17 sin 𝜃 8
𝑎𝑑𝑗 15 1 15
cos 𝜃 = = ⇒ sec 𝜃 = =
ℎ𝑦𝑝 17 cos 𝜃 17
𝑜𝑝𝑝 8 1 12
tan 𝜃 = = ⇒ cot 𝜃 = =
𝑎𝑑𝑗 15 tan 𝜃 5

Example 1.7
Find the value(s) of the six trigonometric functions for the smaller angle of a right-angled triangle with two
sides 5 and 12.

P a g e 4 | 213
Get all the files at: https://bit.ly/azizhandouts
Aziz Manva (azizmanva@gmail.com)

Case I
Suppose the sides which are 5 and 12 are the legs of the right-angled triangle. A
Then, by Pythagoras Theorem: 13
𝐻𝑦𝑝 = √52 + 122 = √25 + 144 = √169 = 13 5

Let ∠𝐴𝐶𝐵 = 𝜃: B C
𝑜𝑝𝑝 5 12
sin 𝜃 = =
ℎ𝑦𝑝 13
𝑎𝑑𝑗 12
cos 𝜃 = =
ℎ𝑦𝑝 13
𝑜𝑝𝑝 5
tan 𝜃 = =
𝑎𝑑𝑗 12
hyp 13
csc 𝜃 = =
opp 5

ℎ𝑦𝑝 13
sec 𝜃 = =
𝑎𝑑𝑗 12
adj 12
cot 𝜃 = =
opp 5

Case II
12 is the hypotenuse ⇒ 𝐿𝑒𝑔𝑠 𝑎𝑟𝑒 5 𝑎𝑛𝑑 √119

1.8: Primitive Pythagorean Triplets Primitive Pythagorean Triplets


➢ Three integers which can be the sides of a right 3, 4, 5 20, 21, 29 11, 60, 61 36, 77, 85
triangle form a Pythagorean Triplet. 5, 12, 13 12, 35, 37 33, 56, 65 13, 84, 85
➢ A Pythagorean Triplet (𝑎, 𝑏, 𝑐) where 8, 15, 17 9, 40, 41 16, 63, 65 39, 80, 89
𝐻𝐶𝐹(𝑎, 𝑏, 𝑐) = 1 is called a primitive 7, 24, 25 28, 45, 53 48, 55, 73 65, 72, 97
Pythagorean Triplet.

2(3,4,5) = (6,8,10)

1.9: Reverse Calculations


Given the value of 𝜃, we can use the trigonometric functions to find the length of side of a triangle.

Example 1.10
If the angle shown in the diagram has a measure of 𝜃°, then find, in terms of 𝜃, the
length of
A. Side 𝐴𝐵
B. Side 𝐵𝐶

𝑜𝑝𝑝 𝐴𝐵
sin 𝜃 = = ⇒ 𝐴𝐵 = 3 sin 𝜃
ℎ𝑦𝑝 3
𝐵𝐶
cos 𝜃 = ⇒ 𝐵𝐶 = 3 cos 𝜃
3

P a g e 5 | 213
Get all the files at: https://bit.ly/azizhandouts
Aziz Manva (azizmanva@gmail.com)

1.11: Reference Triangle


If we are given one of the trigonometric ratios, we can draw a triangle with two sides that meet these
conditions, use the Pythagorean theorem to find the third side, and then find the remaining trigonometric
ratios.

➢ A reference triangle is a triangle that meets the conditions for the information that you have.

Example 1.12
In each case, find the other five trigonometric ratios.
3
A. sin 𝜃 = 5
24
B. sin 𝜃 = 25

Part A
3 𝑜𝑝𝑝
sin 𝜃 =
=
5 ℎ𝑦𝑝
Draw a 𝑟𝑒𝑓𝑒𝑟𝑒𝑛𝑐𝑒 𝑡𝑟𝑖𝑎𝑛𝑔𝑙𝑒 (see diagram) with
𝑜𝑝𝑝 = 3, ℎ𝑦𝑝 = 5

Now, we have a Pythagorean Triplet:


(3,4,5)
Now that we have all three sides, the six trigonometric ratios are:
4 3 1 5 1 5 4
cos 𝜃 = , tan 𝜃 = , csc 𝜃 = = , sec 𝜃 = = , cot 𝜃 =
5 4 sin 𝜃 3 cos 𝜃 4 3

Part C
Draw a right triangle, and then 𝑜𝑝𝑝 = 24, ℎ𝑦𝑝 = 25 ⇒ 𝑎𝑑𝑗 = 7
7 24 1 25 1 25 7
cos 𝜃 = , tan 𝜃 = , sec 𝜃 = = , csc 𝜃 = = , cot 𝜃 =
25 7 sin 𝜃 24 cos 𝜃 7 24

Example 1.13
In each case, find the other five trigonometric ratios.
𝑝
sin 𝜃 =
𝑞

1.14: Trig Functions for 𝟒𝟓°


1 √2
sin 45° = = ≈ 0.70711
√2 2
1
cos 45° =
√2
tan 45° = 1

P a g e 6 | 213
Get all the files at: https://bit.ly/azizhandouts
Aziz Manva (azizmanva@gmail.com)

Draw an isosceles right triangle with angles 45°, 45°,and 90°. This kind of triangle is
called:
45 − 45 − 90 𝑇𝑟𝑖𝑎𝑛𝑔𝑙𝑒

Suppose:
𝐴𝐵 = 1 ⇒ 𝐵𝐶 = 1 ⇒ 𝐴𝐶 = √12 + 12 = √2
Now, the values can be obtained from the triangle by using the definitions.

1.15: Trig Functions for 𝟑𝟎° and 𝟔𝟎°


√3 1
sin 60° = , cos 60° = , tan 60° = √3
2 2
1 √3 1
sin 30° = , cos 30° = , tan 30° =
2 2 √3

Draw equilateral Δ𝐴𝐶𝐵 with side 1.


Drop a perpendicular from vertex A, meeting the opposite side at
point X. Then:
𝛥𝐴𝐶𝐷 𝑖𝑠 𝑎 30 − 60 − 90 𝑡𝑟𝑖𝑎𝑛𝑔𝑙𝑒.
In an isosceles triangle, the altitude is also the median:
1
𝐴𝐷 = 𝐷𝐵 =
2
In right Δ𝐴𝐵𝑋, by the Pythagorean Theorem:
1 2 3 √3
𝐴𝑋 = ℎ = √𝐴𝐵2 − 𝐵𝑋 2 = √12 − ( ) = √ =
2 4 2
Now, the values can be obtained from the triangle by using the definitions.

Example 1.16
Find the exact value:
A. cos 30°
B. cot 30°
C. csc 60°
D. cos 60°
E. tan 45°
F. sec 60°
G. cos 45°

√3 1
cos 30° = cos 60° =
2 2
1 1 tan 45° = 1
cot 30° = = = √3 1 1
tan 30° 1 sec 60° = = =2
√3 cos 60° 1
1 1 2 2
csc 60° = = = 1
sin 60° √3 √3 cos 45° =
2 √2

P a g e 7 | 213
C. Inverse Trigonometry Functions

Example 1.17
√3
Find the value of 𝜃 if cos 𝜃 = 2
.

√3
cos 𝜃 = ⇒ 𝜃 = 30°
2

1.18: Inverse Trigonometry Functions


The trigonometric functions associate a value with an angle. The inverse trigonometric functions do the reverse
process.

Example 1.19
√3
Find the value of 𝜃 if cos 𝜃 = 2
.

√3
cos 𝜃 =
2
Apply the cos−1 function on both sides:
√3
𝜃 = cos −1 ( ) = 30°
2

Example 1.20
Find 𝜃 in each case below:
A. cot 𝜃 = √3
2
B. csc 𝜃 =
√3
1
C. cos 𝜃 = 2
D. tan 𝜃 = 1

cot 𝜃 = √3 ⇒ 𝜃 = cot −1 √3 = 30°


2 √3 √3
csc 𝜃 = ⇒ sin 𝜃 = ⇒ 𝜃 = sin−1 ( ) = 60°
√3 2 2
1 1
cos 𝜃 = ⇒ 𝜃 = cos−1 ( ) = 60°
2 2
tan 𝜃 = 1 ⇒ 𝜃 = tan−1 1 = 45°

Example 1.21
Solve for 𝜃:
A. sin 𝜃 = 𝑥
B. cos 𝜃 = 𝑦
C. tan 𝜃 = 𝑧

𝜃 = sin−1 𝑥
𝜃 = cos −1 𝑦
Get all the files at: https://bit.ly/azizhandouts
Aziz Manva (azizmanva@gmail.com)

𝜃 = tan−1 𝑧
D. Co-Function Identities

1.22: Co-function Identities


sin(90 − 𝜃) = cos 𝜃
cos(90 − 𝜃) = sin 𝜃

In 𝑅𝑖𝑔ℎ𝑡 Δ𝐴𝐵𝐶, right − angled at B


𝐿𝑒𝑡 ∠𝐶 = 𝜃 ⇒ ∠𝐴 = (90 − 𝜃)

𝑜𝑝𝑝 𝐵𝐶
sin ∠𝐴 = sin ∠(90 − 𝜃) = = = cos 𝜃
ℎ𝑦𝑝 𝐴𝐶
𝑎𝑑𝑗 𝐴𝐵
cos ∠𝐴 = cos ∠(90 − 𝜃) = = = sin 𝜃
ℎ𝑦𝑝 𝐴𝐶

Example 1.23
A. cos 43° − sin 47°

cos 43° − sin 47°


Rewrite 43 = 90 − 47:
= cos(90° − 47°) − sin 47°
Use cos(90 − 𝜃) = sin 𝜃:
sin 47° − sin 47° = 0

1.24: 𝐜𝐨𝐬(𝟗𝟎 − 𝜽) = 𝐬𝐢𝐧 𝜽

Right Triangle Trigonometry Proof


𝐴𝐵
sin 𝜃 = cos(90 − 𝜃) =
𝐴𝐶
𝐵𝐶
cos 𝜃 = sin(90 − 𝜃) =
𝐴𝐶

Second Quadrant Proof


Coordinates of A
= 𝐴(𝑥, 𝑦) = 𝐴(cos 𝜃 , sin 𝜃)
∠𝐴𝑂𝑃 = 𝜃
∠𝐴𝑂𝐵 = 180 − 𝜃
∠𝐵𝐴𝑂 = 90 − (180 − 𝜃) = 𝜃 − 90

Using cos 𝜃 = cos(−𝜃):


cos(90 − 𝜃) = cos(𝜃 − 90)

𝑬𝒒𝒖𝒂𝒕𝒊𝒐𝒏 𝑰
From Δ𝐴𝑂𝐵:
𝑎𝑑𝑗 𝐴𝐵
cos(𝜃 − 90) = = = 𝐴𝐵 = sin 𝜃
⏟ ℎ𝑦𝑝 1
𝑬𝒒𝒖𝒂𝒕𝒊𝒐𝒏 𝑰𝑰

P a g e 9 | 213
Get all the files at: https://bit.ly/azizhandouts
Aziz Manva (azizmanva@gmail.com)

cos(90 − 𝜃) = sin 𝜃

1.25: 𝐭𝐚𝐧(𝟗𝟎 − 𝜽) = 𝒄𝒐𝒕 𝜽

sin(90 − 𝜃) cos 𝜃
𝐭𝐚𝐧(90 − 𝜃) = = = cot 𝜃
cos(90 − 𝜃) sin 𝜃

1.26: Translation
sin(90 + 𝜃) = cos 𝜃

The graph of 𝑦 = sin 𝑥 when translated to the left 90°gives


𝑦 = cos 𝑥

E. Degree-Minute-Second Notation

Example 1.27: Degree-Minute-Second Notation


The degree-minute second notation is a notation for precise communication of values of angles. It is not so
common now.

⏟ = 60 ⏟′ = 3600
⏟ ′′
𝑫𝒆𝒈𝒓𝒆𝒆𝒔 𝑴𝒊𝒏𝒖𝒕𝒆𝒔 𝑺𝒆𝒄𝒐𝒏𝒅𝒔
1 Degree = 60 Minutes
1 Minute = 60 Seconds

Convert from Degree-Minute-Second notation to Decimal Notation


A. 6°15′ 30′′
Convert from Decimal Notation to Degree-Minute-Second notation
B.

30′ 15.5′ 15 ° 0.5 ° 1 °


6°15′ 30′′ = 6°15′ + = 6°15.5′ = 6° + = 6° + ( ) + ( ) = 6° + 0.25° + ( )
60 60 60 60 120

1.2 Trigonometric Ratios-II


A. Some Further Special Triangles
In this section, we cover trigonometric ratios which are used less frequently.
Sometimes these ratios are required to solve a problem. Sometimes, they are not required, but make the
solution shorter if you use them.

1.28: Trig Functions of 𝟐𝟐. 𝟓 and 𝟔𝟕. 𝟓


√2 − 1
sin 22.5° =
√4 − 2√2
1
cos 22.5° =
√4 − 2√2
tan 22.5° = √2 − 1

P a g e 10 | 213
Get all the files at: https://bit.ly/azizhandouts
Aziz Manva (azizmanva@gmail.com)

Draw Rectangle 𝐴𝐵𝐶𝐷 with length √2 and width 1.


On side BC, pick point P such that 𝐵𝑃 = 1.

In isosceles Δ𝐴𝐵𝑃:
𝐴𝑃 = √2(𝐴𝐵) = √2

In isosceles Δ𝐷𝐴𝑃
180 − 45 135
∠𝐴𝐷𝑃 = = = 67.5°
2 2

In Δ𝑃𝐷𝐶
∠𝑃𝐷𝐶 = 90 − 67.5 = 22.5°
∠𝐷𝑃𝐶 = 180 − 90 − 22.5 = 67.5°
𝐷𝐶 = 𝐴𝐵 = 1
𝑃𝐶 = 𝐵𝐶 − 𝐵𝑃 = √2 − 1

By Pythagoras Theorem, in Δ𝑃𝐷𝐶


2
𝐷𝑃 = √12 + (√2 − 1) = √1 + 2 − 2√2 + 1 = √4 − 2√2

1.29: Trig Functions of 𝟏𝟓° and 𝟕𝟓


√3 − 1
sin 15° = cos 75° =
2√2
√3 + 1
cos 15° = sin 75° =
2√2
tan 15° = cot 75° = 2 − √3
tan 75° = cot 15° = 2 + √3

Draw
➢ ∠𝐴𝐵𝐶 𝑤𝑖𝑡ℎ 90°
➢ ∠𝐴𝐵𝐷 𝑤𝑖𝑡ℎ 45°
➢ ∠𝐷𝐵𝐸 𝑤𝑖𝑡ℎ 30°
➢ ∠𝐵𝐷𝐹 with 90° intersecting BE at F.
➢ Rectangle 𝑋𝑌𝑍𝐵 such that the left lies on the y-axis, the
bottom lies on the x-axis, D lies on the top, and F lies on the
right side.

∠𝑋𝐷𝐵 = 180 − 90 − 45 = 45°


In 45 − 45 − 90 Δ𝑋𝐷𝐵:
√3
𝐵𝑋 = 𝑋𝐷 =
√2

P a g e 11 | 213
Get all the files at: https://bit.ly/azizhandouts
Aziz Manva (azizmanva@gmail.com)

∠𝐹𝐷𝑌 = 180 − 45 − 90 = 45°


∠𝑌𝐹𝐷 = 180 − 45 − 90 = 45°
In 45 − 45 − 90 Δ𝐷𝑌𝐹:
1
𝐷𝑌 = 𝑌𝐹 =
√2
1
√3 √3 + 1
𝐵𝑍 = 𝑋𝐷 + 𝐷𝑌 = + =
√2 √2 √2
√3 1 √3 − 1
𝐹𝑍 = 𝑋𝐵 − 𝑌𝐹 = − =
√2 √2 √2
And note that
∠𝐹𝐵𝑍 = 90 − 45 − 30 = 15°
∠𝐵𝐹𝑍 = 180 − 90 − 15 = 75°
Hence, Δ𝐵𝐹𝑍 is 15° − 75° − 90°

√3 − 1
sin 15° = cos 75° =
2√2
√3 + 1
cos 15° = sin 15° =
2√2
√3 − 1 √3 − 1 √3 − 1 3 − 2√3 + 1 4 − 2√3
tan 15° = = ∙ = = = 2 − √3
√3 + 1 √3 + 1 √3 − 1 2 2
1 2 + √3
cot 15° = = = 2 + √3
2 − √3 1
1 1
tan 75° = = = 2 + √3
tan 15° 2 − √3
P a g e 12 | 213
Get all the files at: https://bit.ly/azizhandouts
Aziz Manva (azizmanva@gmail.com)

1.3 Triangles
A. Basics
Given some side lengths and angles related to a right triangle, the process
of finding the remaining values is called “solving” the right triangle. This
can be accomplished using the trigonometrical ratios.

1.30: Triangle Nomenclature


It is 𝑡𝑟𝑎𝑑𝑖𝑡𝑖𝑜𝑛𝑎𝑙 that for a general triangle diagram:
➢ We use capital letters for the vertices 𝐴, 𝐵, 𝐶
➢ We use small letters for the lengths of the sides opposite the
vertices 𝑎, 𝑏, 𝑐
➢ We use Greek letters for the angles 𝛼, 𝛽, 𝛾

Example 1.31
A right-angled triangle has an angle of 𝜃, 𝜃 ≠ 90°. Find the lengths of the other two sides in terms of its
hypotenuse ℎ.

𝐴𝐵
sin 𝜃 = ⇒ 𝐴𝐵 = ℎ sin 𝜃

𝐵𝐶
cos 𝜃 = ⇒ 𝐵𝐶 = ℎ cos 𝜃

1.32: Solving a Triangle


Finding the values of the angles and the sides of a triangle is called “solving” the triangle.

Example 1.33
A right-angled triangle 𝐴𝐵𝐶, right-angled at 𝐴, has ∠𝐵𝐶𝐴 = 28°. If 𝐵𝐶 = 10 𝑚𝑒𝑡𝑒𝑟𝑠, find the perimeter of the
triangle in terms of ∠𝐵𝐶𝐴.

𝐴𝐵
sin 28° = ⇒ 𝐴𝐵 = 10 sin 28°
10
𝐵𝐶
cos 28° = ⇒ 𝐵𝐶 = 10 cos 28°
10

The perimeter is
10 + 10 sin 28° + 10 cos 28° = 10(1 + sin 28° + cos 28°)

P a g e 13 | 213
Get all the files at: https://bit.ly/azizhandouts
Aziz Manva (azizmanva@gmail.com)

B. Area of a Triangle

1.34: Area of a Triangle


Area of a triangle with sides 𝑎, 𝑏 and included angle 𝜃 is given by:
1
𝐴 = 𝑎𝑏 sin 𝐶
2
Where
𝑎, 𝑏 𝑎𝑟𝑒 𝑎𝑑𝑗𝑎𝑐𝑒𝑛𝑡 𝑠𝑖𝑑𝑒𝑠 𝑜𝑓 𝑎 𝑡𝑟𝑖𝑎𝑛𝑔𝑙𝑒
𝐶 𝑖𝑠 𝑡ℎ𝑒 𝑎𝑛𝑔𝑙𝑒 𝑏𝑒𝑡𝑤𝑒𝑒𝑛 𝑎 𝑎𝑛𝑑 𝑏

Case I: Right Angled Triangle


Before we proceed note that, if:
𝐶 = 90° ⇒ sin 𝐶 = sin 90° = 1
And the formula becomes:
1 1 1
𝐴 = 𝑎𝑏 sin 𝐶 = 𝑎𝑏 ∙ 1 = 𝑎𝑏
2 2 2
Which is the formula for a right-angled triangle that we already know from
geometry.

Case II: Three further cases


A. One of the angles which is not 𝜃 is a right angle.
B. Both of the angles which are not 𝜃 are acute.
C. One of the angles which is not 𝜃 is obtuse.

For all three cases, we have:


𝑜𝑝𝑝 ℎ
sin 𝜃 = = ⇒ ℎ = 𝑏 sin 𝜃
ℎ𝑦𝑝 𝑏

1
Substitute 𝐻𝑒𝑖𝑔ℎ𝑡 = ℎ = 𝑏 sin 𝜃 , 𝐵𝑎𝑠𝑒 = 𝑎 in 𝐴 = 2 ℎ𝑏:
1 1
= ⏟ (𝑏 sin 𝜃) (𝑎)
⏟ = 𝑎𝑏 sin 𝜃
2 𝑯𝒆𝒊𝒈𝒉𝒕 𝑩𝒂𝒔𝒆 2

Example 1.35: Triangles


A. The adjacent sides of a triangle are 3 𝑓𝑒𝑒𝑡 and 4 𝑓𝑒𝑒𝑡 long. The angle between the two sides is 45°. Find
the area of the triangle.
B. Two sides of a triangular field are 5 𝑚 and 7𝑚 long, and the angle between them is 30°. Find the area of
the field.
C. Anshuman is playing in the garden. He runs 7 feet north, makes a sharp turn at a 60° angle from north,
and runs approximately northeast another 10 feet. If he now runs back to his starting position, what is
the area in feet of the enclosed figure that he has run around.
D. A triangle has sides of length 2 𝑐𝑚 and 3 𝑐𝑚. The angles of the triangles are in the ratio 3: 2: 1. What is

P a g e 14 | 213
Get all the files at: https://bit.ly/azizhandouts
Aziz Manva (azizmanva@gmail.com)

the sum of the possible values of the area of the triangle.

Part A
1 1 √2
𝐴 = 𝑎𝑏 sin 45° = ∙ 3 ∙ 4 ∙ = 3√2
2 2 2
Part B
1 1 1 35 3
𝐴 = 𝑎𝑏 sin 30° = ∙ 5 ∙ 7 ∙ = =8
2 2 2 4 4
Part C
1 1 √3 35√3
𝐴 = 𝑎𝑏 sin 120° = ∙ 7 ∙ 10 ∙ =
2 2 2 2
Part D
Let the angles be:
3: 2: 1 = 3𝑥: 2𝑥: 𝑥
3𝑥 + 2𝑥 + 𝑥 = 180 ⇒ 6𝑥 = 180 ⇒ 𝑥 = 30
The angles of the triangle are
{30,60,90}

The three possibilities are:


1 1 1
𝑎𝑏 sin 30° + 𝑎𝑏 sin 60° + 𝑎𝑏 sin 90°
2 2 2
1
= 𝑎𝑏(sin 30° + sin 60° + sin 90°)
2
1 1 √3
= ∙ 2 ∙ 3( + + 1)
2 2 2
√3 + 3
= 3( )
2
3√3 + 9
=
2

Example 1.36: Law of Sines


We can use the formula for the area of a triangle to prove the law of sines. By considering the area of a triangle
in three different ways, show that:
𝑎 𝑏 𝑐
= =
𝑆𝑖𝑛 𝐴 𝑆𝑖𝑛 𝐵 𝑆𝑖𝑛 𝐶

We calculate the area of the triangle in three different ways:


1 1 1
𝑎𝑏 sin 𝐶 = 𝑎𝑐 sin 𝐵 = 𝑏𝑐 sin 𝐴
2 2 2

Equate the second and the third part:


𝑎 𝑏
𝑎𝑐 sin 𝐵 = 𝑏𝑐 sin 𝐴 ⇒ =

sin 𝐴 sin 𝐵
𝑬𝒒𝒖𝒂𝒕𝒊𝒐𝒏 𝑰
Equate the first and the second part:
𝑏 𝑐
𝑎𝑏 sin 𝐶 = 𝑎𝑐 sin 𝐵 ⇒ =

sin 𝐵 sin 𝐶
𝑬𝒒𝒖𝒂𝒕𝒊𝒐𝒏 𝑰𝑰
Combine Equation I and II into a tripartite equality:

P a g e 15 | 213
Get all the files at: https://bit.ly/azizhandouts
Aziz Manva (azizmanva@gmail.com)

𝑎 𝑏 𝑐
= =
sin 𝐴 sin 𝐵 sin 𝐶

Example 1.37
Triangle ABC is a right angle, right angled at 𝐵. Side 𝐴𝐵 has length 1. 𝐶𝐷 is
a diameter of the circle, and 𝐵 is the center of the circle. ∠𝐵𝐴𝐶 is 𝛼. Find
the diameter of the circle in terms of 𝛼.

𝐵𝐶 𝐵𝐶
tan 𝛼 = =
𝐴𝐵 1
𝑟 = 𝐵𝐶 = tan 𝛼
𝐷𝑖𝑎𝑚𝑒𝑡𝑒𝑟 = 2𝑟 = 2 tan 𝛼

Example 1.38
The diagram alongside has three triangles such that ∠ 𝐴𝐵𝐶 = ∠𝐵𝐷𝐶 = ∠𝐷𝐸𝐶 =
90°. Find an expression for 𝐷𝐸 in terms of 𝐴𝐵 and 𝜃.

𝐷𝐸
sin 𝜃 = ⇒ 𝐷𝐸 = 𝐷𝐶 sin 𝜃
𝐷𝐶
𝐷𝐶
Substitute cos 𝜃 = 𝐵𝐶 ⇒ 𝐷𝐶 = 𝐵𝐶 cos 𝜃:
= (𝐵𝐶 cos 𝜃) sin 𝜃

𝐴𝐵 𝐴𝐵
Substitute tan 𝜃 = 𝐵𝐶 ⇒ 𝐵𝐶 = tan 𝜃:
𝐴𝐵
= cos 𝜃 sin 𝜃 = 𝐴𝐵 cot 𝜃 cos 𝜃 sin 𝜃 = 𝐴𝐵 cos2 𝜃
tan 𝜃

Example 1.39
𝑀𝑎𝑟𝑘 𝑎𝑙𝑙 𝑐𝑜𝑟𝑟𝑒𝑐𝑡 𝑜𝑝𝑡𝑖𝑜𝑛𝑠
𝐷𝐸
Can 𝐴𝐵 be a:
A. Positive Integer
B. Negative Integer
C. Rational Number
D. Irrational Number
1+√5
E. Integer multiple of the Golden ratio (𝑤ℎ𝑒𝑟𝑒 𝑔𝑜𝑙𝑑𝑒𝑛 𝑟𝑎𝑡𝑖𝑜 = )
2

𝐷𝐸 𝐴𝐵 cos 2 𝜃
= = cos2 𝜃
𝐴𝐵 𝐴𝐵

cos 2 𝜃 = 1 ⇒ 𝜃 = 0 ⇒ 𝑁𝑜𝑡 𝑉𝑎𝑙𝑖𝑑 ⇒ 𝑂𝑝𝑡𝑖𝑜𝑛 𝐴 𝑖𝑠 𝑖𝑛𝑐𝑜𝑟𝑟𝑒𝑐𝑡


cos2 𝜃 < 0 ⇒ 𝑁𝑜𝑡 𝑉𝑎𝑙𝑖𝑑 ⇒ 𝑂𝑝𝑡𝑖𝑜𝑛 𝐵 𝑖𝑠 𝑖𝑛𝑐𝑜𝑟𝑟𝑒𝑐𝑡

If
1 1
𝜃 = 60° ⇒ cos 𝜃 = ⇒ cos2 𝜃 = ⇒ 𝑅𝑎𝑡𝑖𝑜𝑛𝑎𝑙 𝑁𝑢𝑚𝑏𝑒𝑟 ⇒ 𝑂𝑝𝑡𝑖𝑜𝑛 𝐶 𝑖𝑠 𝑐𝑜𝑟𝑟𝑒𝑐𝑡
2 4

P a g e 16 | 213
Get all the files at: https://bit.ly/azizhandouts
Aziz Manva (azizmanva@gmail.com)

0 < cos 2 𝜃 < 1 ⇒ cos2 𝜃 𝑐𝑎𝑛 𝑏𝑒 𝑖𝑟𝑟𝑎𝑡𝑖𝑜𝑛𝑎𝑙 ⇒ 𝑂𝑝𝑡𝑖𝑜𝑛 𝐷 𝑖𝑠 𝑐𝑜𝑟𝑟𝑒𝑐𝑡

1 + √5
> 1 > cos2 𝜃 ⇒ 𝑂𝑝𝑡𝑖𝑜𝑛 𝐸 𝑖𝑠 𝑖𝑛𝑐𝑜𝑟𝑟𝑒𝑐𝑡
2

𝑂𝑝𝑡𝑖𝑜𝑛 𝐶, 𝐷

1.4 Polygons
A. Quadrilaterals

1.40: Area of a Parallelogram


The area of a parallelogram is the product of two adjacent sides and the sine of the included angle.
𝑠1 𝑠2 sin ∠𝐷
Where
𝑠1 𝑎𝑛𝑑 𝑠2 𝑎𝑟𝑒 𝑎𝑑𝑗𝑎𝑐𝑒𝑛𝑡 𝑠𝑖𝑑𝑒𝑠
𝐷 𝑖𝑠 𝑡ℎ𝑒 𝑖𝑛𝑐𝑙𝑢𝑑𝑒𝑑 𝑎𝑛𝑔𝑙𝑒

Draw parallelogram 𝐴𝐵𝐶𝐷 with sides 𝑠1 and 𝑠2 . Draw the


height from vertex 𝐴 ⊥ 𝐷𝐶.

In Right Triangle Δ𝐴𝐷𝑃:


𝑜𝑝𝑝 ℎ
sin ∠𝐷 = = ⇒ ℎ = sin ∠𝐷 × 𝑠2
ℎ𝑦𝑝 𝑠2

Area of Parallelogram
(sin ∠𝐷 × 𝑠2 ) × 𝑠⏟1 = 𝑠1 𝑠2 sin ∠𝐷
= ℎ𝑏 = ⏟
𝑯𝒆𝒊𝒈𝒉𝒕 𝑩𝒂𝒔𝒆

Example 1.41: Finding area


A. Find the area of a parallelogram with sides of 4𝑚 and 6𝑚 and an angle between the two sides of 60°.
B. A parallelogram has adjacent sides of length 3 𝑐𝑚 and 5 𝑐𝑚, and angle between the two sides as 30°.
Find the area of the parallelogram.
C. A parallelogram has adjacent sides of length 5 𝑐𝑚 and 7 𝑐𝑚, and angle between the two sides as 27°.
Find a trigonometric expression for the area of the parallelogram.

Part A
√3
𝐴 = 𝑠1 𝑠2 sin ∠𝐷 = 4 × 6 sin 60° = 4 × 6 × = 12√3
2

Part B
1
𝐴 = 𝑠1 𝑠2 sin ∠𝐷 = 3 × 5 × = 7.5
2
Part C
𝐴 = 𝑠1 𝑠2 sin ∠𝐷 = 5 × 7 × sin 27° = 35 sin 27°

Example 1.42
A. The area of a parallelogram is 1 square meter. The base of the parallelogram is 5 meters. The angle
between the base and the adjacent side is 30°. Find the length of the adjacent side.

P a g e 17 | 213
Get all the files at: https://bit.ly/azizhandouts
Aziz Manva (azizmanva@gmail.com)

𝐴𝑟𝑒𝑎 = 𝑎𝑏 sin 𝐶
1 2
1=𝑎×5× ⇒𝑎 = 𝑚
2 5

1.43: Rhombus
➢ A rhombus is a special case of a parallelogram.
➢ All sides of a rhombus are equal.

Example 1.44: Rhombus


A. A rhombus has a side length of 12𝑚. An interior angle of the rhombus is 30°. Find the area of the
rhombus.
B. Find the side length of a rhombus with an area of 𝐴 and an internal angle of 𝛼°.

Part A
1
𝐴 = 𝑠1 𝑠2 sin ∠𝐷 = 12 ∙ 12 ∙ = 72 𝑚2
2
Part B
𝐴
𝐴 = 𝑠 2 × sin 𝛼 ⇒ 𝑠 = √ 𝑢𝑛𝑖𝑡𝑠
sin 𝛼

1.45: Kites

Example 1.46: Kite

1.47: Area of a General Quadrilateral


Show that the area of a quadrilateral is one half the product of the length of its diagonals and the sine of the
angle between the diagonals.
1
𝐴𝑟𝑒𝑎 = sin 𝜃 [(𝑎 + 𝑏)(𝑐 + 𝑑)]
2
1
Apply the formula for area of a triangle 𝐴 = 2 𝑎𝑏 sin 𝜃 to the four
triangles that make up the quadrilateral:
1 1 1 1
= 𝑎𝑐 sin 𝜃 + 𝑎𝑑 sin(180 − 𝜃) + 𝑐𝑏 sin(180 − 𝜃) + 𝑑𝑏 sin 𝜃
2 2 2 2
1
Substitute sin(180 − 𝜃) = sin 𝜃. This gives a 2 sin 𝜃 in each term,
which we factor out:
1 1
= sin 𝜃 [𝑎𝑐 + 𝑎𝑑 + 𝑐𝑏 + 𝑑𝑏] = sin 𝜃 [(𝑎 + 𝑏)(𝑐 + 𝑑)]
2 2

Example 1.48: Area of General Quadrilateral

B. Trigonometry with Rectangles

P a g e 18 | 213
Get all the files at: https://bit.ly/azizhandouts
Aziz Manva (azizmanva@gmail.com)

Example 1.49
A rectangular gate of width 𝑤 and height ℎ has two cross braces going from the top left to bottom right, and top
right to bottom left, respectively. Find the angles between the braces in terms of ℎ and 𝑤 given that 𝜃 is the
angle made by the braces with the base.

Since Δ𝐴𝐷𝐶 ≅ Δ𝐵𝐷𝐶,


∠𝐵𝐷𝐶 = ∠𝐴𝐶𝐷 = 𝜃
In Δ𝐵𝐷𝐶:
𝑜𝑝𝑝 ℎ ℎ
tan 𝜃 = = ⇒ 𝜃 = tan−1
𝑎𝑑𝑗 𝑤 𝑤

The angle between the diagonals is



= 𝛼 = 180 − 2𝜃 = 180 − 2 (tan−1 )
𝑤

𝛽 = 180 − 𝛼 = 2𝜃 = 2 (tan−1 )
𝑤

Example 1.50
Find the perimeter of a rectangle with diagonal 𝑑, and an angle 𝜃 between diagonal and side.

Draw a diagram of a rectangle with width 𝑤 and length 𝑙. Then:


𝑤
sin 𝜃 = ⇒ 𝑤 = 𝑑 sin 𝜃
𝑑
𝑙
cos 𝜃 = ⇒ 𝑙 = 𝑑 cos 𝜃
𝑑
Then the perimeter is:
𝑃 = 2(𝑙 + 𝑤) = 2(𝑑 sin 𝜃 + 𝑑 cos 𝜃) = 2𝑑(sin 𝜃 + cos 𝜃)

Example 1.51
Sandy wants to draw a 2 cm margin for 30 cm long art paper. He doesn’t have a ruler, so while he starts 2 cm
from the edge of the page, he ends 3 cm away from the edge of the page. Find the angle that the line made by
Sandy makes with the parallel to the edge of the paper.

Draw a diagram. We want to find the angle 𝜃.


Method I
1
tan 𝜃 =
30
Take the tan inverse of both sides:
1
𝜃 = tan−1 ( )
30
Method II
𝐻𝑦𝑝 = √901
1
∙ 30 ∙ √901 sin 𝜃 = 15
2
1
sin 𝜃 =
√901
−1
1
𝜃 = sin ( )
√901

Example 1.52
P a g e 19 | 213
Get all the files at: https://bit.ly/azizhandouts
Aziz Manva (azizmanva@gmail.com)

A rectangular piece of paper is divided into a square and a rectangle. The rectangle is folded along the diagonal
to create a triangle. Show that the ratio of the perimeter of the triangle to the perimeter of the square is
independent of the length of the diagonal.

The ratio is:


𝑃(𝑇𝑟𝑖𝑎𝑛𝑔𝑙𝑒) 𝑑 + 𝑠 + 𝑙
=
𝑃(𝑆𝑞𝑢𝑎𝑟𝑒) 4𝑠
This ratio has 𝑑 in it, which we would like to eliminate.

𝑠 𝑙
Substitute cos 𝜃 = 𝑑 ⇒ 𝑠 = 𝑑 cos 𝜃 and sin 𝜃 = 𝑑 ⇒ 𝑙 = 𝑑 sin 𝜃:
𝑑 + 𝑑 cos 𝜃 + 𝑑 sin 𝜃 1 + sin 𝜃 + cos 𝜃
=
4𝑑 cos 𝜃 4 cos 𝜃

The last expression does not contain 𝑑. Hence, it is independent of 𝑑.

1.53: Double Angle Identity


sin 2𝑥 = 2 sin 𝑥 cos 𝑥

Challenge 1.54
In a rectangle 𝐴𝐵𝐶𝐷, 𝐸 is the midpoint of 𝐴𝐵. 𝐹 is a point in 𝐴𝐶 such that 𝐵𝐹 is perpendicular to 𝐴𝐶, and 𝐹𝐸
perpendicular to 𝐵𝐷. Suppose 𝐵𝐶 = 8√3. Find 𝐴𝐵. (IOQM 2017/13)

Angle Chasing
Let ∠𝐷𝐵𝐴 = ∠𝐶𝐴𝐵 = 𝛼.
In Δ𝐸𝑁𝐵
∠𝑁𝐸𝐵 = 90 − 𝛼
∠𝐹𝐸𝐴 = 180 − (90 − 𝛼) = 90 + 𝛼

In Δ𝐴𝐹𝐸:
∠𝐴𝐹𝐸 = 180 − 𝛼 − (90 + 𝛼) = 90 − 2𝛼

In Δ𝐴𝐹𝐵
∠𝐴𝐹𝐵 = 90° (𝐵𝐹 ⊥ 𝐴𝐶)
∠𝐸𝐹𝐵 = 90 − (90 − 2𝛼) = 2𝛼

Right Triangle Trigonometry


Let 𝐴𝐸 = 𝐸𝐵 = 𝑥.
𝐵𝑁
𝐼𝑛 𝑟𝑖𝑔ℎ𝑡 𝛥𝐸𝑁𝐵: cos 𝛼 = ⇒⏟𝐵𝑁 = 𝑥 cos 𝛼
𝑥 𝑬𝒒𝒖𝒂𝒕𝒊𝒐𝒏 𝑰
𝐵𝐹
𝐼𝑛 𝑟𝑖𝑔ℎ𝑡Δ𝐴𝐹𝐵: sin 𝛼 = ⇒ 𝐵𝐹 = 2𝑥 sin 𝛼
2𝑥
𝐵𝑁
𝐼𝑛 𝑟𝑖𝑔ℎ𝑡 𝛥𝐵𝐹𝑁: sin 2𝛼 = ⇒ 𝐵𝑁 = 𝐵𝐹 sin 2𝛼 ⇒ ⏟ 𝐵𝑁 = (2𝑥 sin 𝛼)(sin 2𝛼)
𝐵𝐹 𝐸𝑞𝑢𝑎𝑡𝑖𝑜𝑛 𝐼𝐼

𝑥 cos 𝛼 = (2𝑥 sin 𝛼)(sin 2𝛼)


cos 𝛼 = (2 sin 𝛼)(2 sin 𝛼 cos 𝛼)
1 1
= sin2 𝛼 ⇒ sin 𝛼 = ⇒ 𝛼 = 30°
4 2

P a g e 20 | 213
Get all the files at: https://bit.ly/azizhandouts
Aziz Manva (azizmanva@gmail.com)

Special Right Triangles


𝐵𝐶 𝐵𝐶 8√ 3
tan 30° = ⇒ 𝐴𝐵 = = = (8√3)√3 = 24
𝐴𝐵 tan 30° 1
√3
C. Regular Polygons

Example 1.55
In regular hexagon 𝐴𝐵𝐶𝐷𝐸𝐹, what is ∠𝐴𝐷𝐸?

All regular hexagons are similar. Without loss of generality, consider a regular
hexagon with side length 1.

Each angle of a regular hexagon is:


(𝑛 − 2)(180) (6 − 2)(180)
= = = 120
𝑛 6

Draw 𝐴𝐸 ⊥ 𝐸𝐷.
∠𝐴𝐸𝐹 = 120
⏟ − 90 = 30°
∠𝐹𝐸𝐷
Draw 𝐹𝑀 ⊥ 𝐴𝐸
∠𝑀𝐹𝐸 = 180 − 90 − 30 = 60°

𝑀𝐸 √3 √3
sin ∠𝑀𝐹𝐸 = ⇒ 𝑀𝐸 = 𝐹𝐸 ∙ sin 60° = 1 ∙ =
𝐹𝐸 2 2
√3
𝐴𝐸 = 2 ∙ 𝑀𝐸 = 2 × = √3
2
𝐴𝐸
tan ∠𝐴𝐷𝐸 = = √3 ⇒ ∠𝐴𝐷𝐸 = 60°
𝐸𝐷
Method II
Recognize that
𝐴𝐷 = 2
1
cos ∠𝐴𝐷𝐸 = ⇒ ∠𝐴𝐷𝐸 = 60°
2

1.56: Apothem
The apothem of a regular polygon is the line segment from the center to the midpoint of one of its side.

Example 1.57
Area of a regular hexagon

1.58: Some Trig Values


tan 36° = √5 − 2√5
tan 22.5° = √2 − 1
tan 15° = 2 − √3

Example 1.59

P a g e 21 | 213
Get all the files at: https://bit.ly/azizhandouts
Aziz Manva (azizmanva@gmail.com)

Find the area of a regular octagon with side 𝑠.

By connecting the vertices, we get 8 congruent triangles. The angle


1
subtended by each triangle is of a circle.
8
360
∠𝐴𝑂𝐵 = = 45
8
Draw 𝑂𝑃 ⊥ 𝐴𝐵
𝐴𝑂𝐵 45
∠𝑃𝑂𝐵 = 𝜃 = ∠ = = 22.5°
2 2
Also, note that:
𝑃𝐵 𝑃𝐵 𝑃𝐵 𝑃𝐵
tan 𝜃 = ⇒ 𝑂𝑃 = = =
𝑂𝑃 tan 𝜃 tan 22.5° √2 − 1

𝑠
Substitute 𝐴𝐵 = 𝑠 ⇒ 𝑃𝐵 = 2:
𝑠
2 𝑠 √2 + 1 𝑠(√2 + 1) 𝑠(√2 + 1)
= ∙ = =
√2 − 1 2(√2 − 1) √2 + 1 2(2 − 1) 2

The area of the octagon is eight times the area of area of Δ𝐴𝑃𝐵:
1 𝑠(√2 + 1)
= ×𝑠× × 8 = 2𝑠 2 (√2 + 1)

2 2
𝑨𝒓𝒆𝒂 𝒐𝒇 𝚫𝑨𝑷𝑩

Example 1.60
Find the area of a regular dodecagon with side 𝑠.

1
By connecting the vertices, get 12 congruent triangles. The angle subtended by each triangle is 12 of a circle.
360
∠𝐴𝑂𝐵 = = 30
12
Draw 𝑂𝑃 ⊥ 𝐴𝐵
𝐴𝑂𝐵 30
∠𝑃𝑂𝐵 = 𝜃 = ∠ = = 15°
2 2
Note that:
𝑃𝐵 𝑃𝐵 𝑃𝐵 𝑃𝐵
tan 𝜃 = ⇒ 𝑂𝑃 = = =
𝑂𝑃 tan 𝜃 tan 15° 2 − √3
𝑠
Substitute 𝐴𝐵 = 𝑠 ⇒ 𝑃𝐵 = :
2
𝑠
2 𝑠 2 + √3 𝑠(2 + √3) 𝑠(2 + √3)
= ∙ = =
2 − √3 2(2 − √3) 2 + √3 2(4 − 3) 2
The area of the dodecagon is twelve times the area of Δ𝐴𝑃𝐵:
1 𝑠(2 + √3)
= ×𝑠× × 12 = 3𝑠 2 (2 + √3)
2 2

Example 1.61
Find the area of a regular pentagon with side 𝑠.

P a g e 22 | 213
Get all the files at: https://bit.ly/azizhandouts
Aziz Manva (azizmanva@gmail.com)

By connecting the vertices, get 5 congruent triangles. The angle


1
subtended by each triangle is 5 of a circle.
Draw 𝑂𝑃 ⊥ 𝐴𝐵.
360
∠𝑃𝑂𝐵 = 𝜃 = = 36°
10
Also, note that:
𝑃𝐵 𝑃𝐵 𝑃𝐵 𝑃𝐵
tan 𝜃 = ⇒ 𝑂𝑃 = = =
𝑂𝑃 tan 𝜃 tan 36° √5 − 2√5

𝑠
Substitute 𝐴𝐵 = 𝑠 ⇒ 𝑃𝐵 = 2, and rationalize:
𝑠
2 𝑠 √5 + 2√5 𝑠 (√5 + 2√5)
= ∙ =
√5 − 2√5 2√5 − 2√5 √5 + 2√5 2√5

Rationalize one more time:



𝑠 (√5 + 2√5) √5 𝑠 ( 5(5 + 2√5))
= ∙ =
2√5 √5 10

The area of the pentagon is five times the area of Δ𝐴𝑃𝐵:

𝑠 (√5(5 + 2√5))
1 1
= ×𝑠× × 5 = 𝑠 2 (√5(5 + 2√5))
2 10 4

1.62: Regular Polygons: General Formula


1
× 𝑃𝑒𝑟𝑖𝑚𝑒𝑡𝑒𝑟 × 𝐴𝑝𝑜𝑡ℎ𝑒𝑚
2

D. Regular Polygons and the Circle


Example 1.63

Divide the dodecagon into 12 congruent triangles, each of which subtends an angle at the center of:
360°
= 30°
12

Each of the triangles has area:

P a g e 23 | 213
Get all the files at: https://bit.ly/azizhandouts
Aziz Manva (azizmanva@gmail.com)

1 1 1 1 𝑟2
𝑎𝑏 sin 𝜃 = 𝑟 2 sin 30° = 𝑟 2 ( ) =
2 2 2 2 4

And the area of the entire polygon will be:


𝑟2
12 ∙ = 3𝑟 2
4

Example 1.64
A circle with radius 𝑟 has a regular polygon inscribed in it. Show that the area of the polygon is:
1 2 2𝜋
𝑟 𝑛 sin ( )
2 𝑛

Consider a regular polygon inscribed in a circle. For example, the diagram has an
octagon inscribed in a circle. Draw the radii from the center of the circle to the
vertices of the polygon.

The radii are equal, and the angle subtended by each triangle is equal, since it is a
regular polygon. Hence, by 𝑆𝐴𝑆, the triangles are all congruent.

The area of a single triangle


1 1 2𝜋
= 𝑎𝑏 sin 𝜃 = 𝑟 2 sin ( )
2 2 𝑛
The area of the entire polygon
1 2𝜋 1 2𝜋
= ⏟
𝑛 × 𝑟 2 sin ( ) = 𝑟 2 𝑛 sin ( )
𝑁𝑜. 𝑜𝑓

2 𝑛 2 𝑛
𝑇𝑟𝑖𝑎𝑛𝑔𝑙𝑒𝑠 𝐴𝑟𝑒𝑎 𝑜𝑓
𝑎 𝑇𝑟𝑖𝑎𝑛𝑔𝑙𝑒

Example 1.65
A circle with radius 𝑟 has a regular polygon circumscribing it. Show that the area of
the polygon is:
𝜋
𝑟 2 𝑛 tan
𝑛

Consider a regular polygon circumscribed around a circle. For example, the diagram has
a hexagon circumscribed around a circle. Draw the radii from the center of the circle to
the vertices of the polygon.
Note the radii are all equal, and the angle subtended by each triangle is equal, since it is
a regular polygon. Hence, by 𝑆𝐴𝑆 Theorem, the triangles are all congruent.
1
𝜋 2 𝐵𝑎𝑠𝑒 𝜋
tan = ⇒ 𝐵𝑎𝑠𝑒 = 2𝑟 tan
𝑛 𝑟 𝑛
The area of a single triangle
1 𝜋 𝜋
= × ⏟ 𝑟 × 2𝑟 tan = 𝑟 2 tan
2 𝐻𝑒𝑖𝑔ℎ𝑡 ⏟ 𝑛 𝑛
𝐵𝑎𝑠𝑒
The area of the polygon
𝜋 𝜋
= ⏟
𝑛 × 𝑟 2 tan = 𝑟 2 𝑛 tan
𝑁𝑜. 𝑜𝑓
⏟ 𝑛 𝑛
𝑇𝑟𝑖𝑎𝑛𝑔𝑙𝑒𝑠 𝐴𝑟𝑒𝑎 𝑜𝑓
𝑎 𝑇𝑟𝑖𝑎𝑛𝑔𝑙𝑒

P a g e 24 | 213
Get all the files at: https://bit.ly/azizhandouts
Aziz Manva (azizmanva@gmail.com)

1.5 Heights and Distances: Basics


A. Angle of Elevation and Depression

1.66: Angle of Elevation and Depression


➢ Angle of Elevation: Angle between the horizontal and a line of sight above the horizontal
➢ Angle of Depression: Angle between the horizontal and a line of sight below the horizontal

The straight line from Rapunzel to the Hero is a transversal cutting the
two parallel horizontal lines. Hence:
𝑑 𝑎𝑛𝑑 𝑒 𝑎𝑟𝑒 𝑎𝑙𝑡𝑒𝑟𝑛𝑎𝑡𝑒 𝑖𝑛𝑡𝑒𝑟𝑖𝑜𝑟 𝑎𝑛𝑔𝑙𝑒𝑠
𝑇ℎ𝑒𝑦 𝑎𝑟𝑒 ≅

𝑑 = ⏟
𝑒
𝑨𝒏𝒈𝒍𝒆 𝒐𝒇 𝑨𝒏𝒈𝒍𝒆 𝒐𝒇
𝑫𝒆𝒑𝒓𝒆𝒔𝒔𝒊𝒐𝒏 𝑬𝒍𝒆𝒗𝒂𝒕𝒊𝒐𝒏
We are interested in using triangles to solve real life problems. The
definitions above are used in the applications below.

(Calc) Example 1.67


A bird sitting in its nest on a tree observes a worm at an angle of depression of 32°. The bird flies a distance of
23 meters to the worm and back to its nest. Find the:
A. height of the nest from the ground.
B. ground distance between the worm and the tree

𝐴𝑛𝑔𝑙𝑒 𝑜𝑓 𝐷𝑒𝑝𝑟𝑒𝑠𝑠𝑖𝑜𝑛 = ∠𝐴𝐶𝐵 = 32°


𝐴𝐵 23
sin 32° = ⇒ 𝐴𝐵 = sin 32° = 6.09 𝑚
23 2
2

𝐵𝐶 23
cos 32° = ⇒ 𝐵𝐶 = cos 32° = 9.75 𝑚
23 2
2

Example 1.68
An observation station observes an angle of elevation of 𝛽 to a balloon that was released from 𝑥 miles away.
Assuming the balloon rose in a straight line, determine the angle of depression from the balloon to the
observation station. Then, find the height ℎ reached by the balloon, and the distance 𝑑 from the observation
station to the balloon.

Draw a line on the ground, and another parallel to it at the


height of the balloon. By alternate interior angles:
𝐴𝑛𝑔𝑙𝑒 𝑜𝑓 𝑑𝑒𝑝𝑟𝑒𝑠𝑠𝑖𝑜𝑛 = 𝐴𝑛𝑔𝑙𝑒 𝑜𝑓 𝐸𝑙𝑒𝑣𝑎𝑡𝑖𝑜𝑛 = 𝛽

Find the height and the distance:



tan 𝛽 = ⇒ ℎ = 𝑥 tan 𝛽
𝑥

P a g e 25 | 213
Get all the files at: https://bit.ly/azizhandouts
Aziz Manva (azizmanva@gmail.com)

𝑥 𝑥
cos 𝛽 = ⇒𝑑=
𝑑 cos 𝛽

Example 1.69
A boat floating on the water observes the light at the top of the lighthouse of height ℎ at an angle of elevation of
α. The horizontal distance of the boat from the cliff that has the lighthouse at its edge is 𝑑. Find the height of the
cliff.

Let the height of the cliff be 𝑐.


ℎ+𝑐
tan 𝛼 =
𝑑
Clearing fractions:
ℎ + 𝑐 = 𝑑 tan 𝛼
Solving for 𝑐:
𝑐 = 𝑑 tan 𝛼 − ℎ

Example 1.70
A ladder 𝑙 units long makes an angle 𝛾 with the ground when resting against the window of a wall at a
√(𝑙+𝑑)(𝑙−𝑑)
horizontal distance 𝑑 units away. Show that 𝛾 = tan−1 ( 𝑑
).

By the Pythagorean Theorem,


ℎ𝑒𝑖𝑔ℎ𝑡 𝑜𝑓 𝑤𝑎𝑙𝑙 = √𝑙 2 − 𝑑 2

√𝑙 2 − 𝑑 2
tan 𝛾 =
𝑑
Solving for 𝛾:
√𝑙2 − 𝑑 2 √(𝑙 + 𝑑)(𝑙 − 𝑑)
𝛾 = tan−1 ( ) = tan−1 ( )
𝑑 𝑑

Example 1.71
A bird finds a shiny toy at point X, and flies back to its nest on a tree, flying at an angle of 𝛾 to the ground and
travelling a distance of 𝑑𝑓 . A boy at point X runs 𝑑 units fter the bird, and then climbs 𝑑𝑐 units to get his toy
𝑑(1−sin 𝛾)
back. Show that 𝑑𝑓𝑙𝑦 − 𝑑𝑐 = cos 𝛾
.

𝑑𝑐
tan 𝛾 = ⇒ 𝑑𝑐 = 𝑑 tan 𝛾
𝑑
𝑑 𝑑
cos 𝛾 = ⇒ 𝑑𝑓 =
𝑑𝑓 cos 𝛾
Substitute the above two into 𝑑𝑓 − 𝑑𝑐 :
𝑑 𝑑(1 − sin 𝛾)
− 𝑑 tan 𝛾 =
cos 𝛾 cos 𝛾

Example 1.72
A flag post breaks at a point and touches the ground 𝑑 units away from its base. If the angle made by the broken
part of the flag post with the ground is 𝜃, then find the length of the flag post before it was broken.

P a g e 26 | 213
Get all the files at: https://bit.ly/azizhandouts
Aziz Manva (azizmanva@gmail.com)

𝑢
tan 𝜃 = ⇒ 𝑢 = 𝑑 tan 𝜃
𝑑
𝑑 𝑑
cos 𝜃 = ⇒ 𝑏 =
𝑏 cos 𝜃
𝑑 1 + sin 𝛼
𝑢 + 𝑏 = 𝑑 tan 𝜃 + = 𝑑( )
cos 𝜃 cos 𝛼

B. 𝒕𝒂𝒏 𝜽 as Slope

1.73: 𝒕𝒂𝒏 𝜽 as Slope


𝑆𝑙𝑜𝑝𝑒 = tan 𝜃

𝑦2 − 𝑦1 𝑅𝑖𝑠𝑒
𝑆𝑙𝑜𝑝𝑒 = =
𝑥2 − 𝑥1 𝑅𝑢𝑛
𝑅𝑖𝑠𝑒
tan 𝜃 =
𝑅𝑢𝑛

Example 1.74: Railway Engineering


The Manitou and Pike's Peak Railway goes from its base station in Colorado (elevation 𝑐 feet) to Pikes Peak
(elevation 𝑝 feet) of the Rocky Mountains. The horizontal distance between the base station and Pike's peak is 𝑑
feet. Find the angle of elevation from the base station to Pikes Peak.

𝑝−𝑐
tan 𝜃 =
𝑑
−1
𝑝−𝑐
𝜃 = tan ( )
𝑑

Example 1.75: Railway Engineering


From coordinate geometry, recall that tan 𝜃 represents the slope of a line when the triangle is set up
appropriately. In railway engineering, the 𝑔𝑟𝑎𝑑𝑒 𝑜𝑓 𝑎 𝑡𝑟𝑎𝑐𝑘 is the slope of a railroad track. It can be expressed
in various ways, including as a percentage of its rise for the length of its run. Find the angle of elevation for a
train going uphill at a grade of 3%.

Let the train start at point A, and go till point B. We can take any length that
we want for the horizontal distance travelled. Let the distance be 1.
Then:
tan 𝜃 = 0.03 ⇒ 𝜃 = tan−1 0.03

Example 1.76
A rise of 600 feet is required to get a railroad line over a mountain. The grade can be kept down by lengthening
the track and curving it around the mountain peak. The additional length of track required to reduce the grade
from 3% to 2% is approximately (answer to one significant digit): (AHSME 1951/38)

P a g e 27 | 213
Get all the files at: https://bit.ly/azizhandouts
Aziz Manva (azizmanva@gmail.com)

Calculate horizontal distance required


For a 2% grade
600 100
600 = 2% 𝑜𝑓 𝐴𝐵 ⇒ 𝐴𝐵 = = 600 × = 30,000
2% 2

For a 3% grade
600 100
600 = 3% 𝑜𝑓 𝐴𝐵 ⇒ 𝐴𝐵 = = 600 × = 20,000
3% 3

Difference
= 30,000 − 20,000 = 10000

Approximate to get actual track length


The actual track length
= 𝐴𝐶
In this context:
𝐴𝐶 ≈ 10,000

C. Two Triangles: Horizontal Movement


Another way to increase the complexity of a situation is through having more than one triangle where the
trigonometric ratios need to be manipulated. The standard question of this type often requires calculating a
quantity in two different ways, and then equating them. (This is equivalent to solving a system of equations).

Example 1.77
A person standing on the bank of a river observes that the angle of elevation of the top of a tree on the opposite
bank of the river is 60°, and when he retires 40 meters away from the tree the angle of elevation becomes 30°.
The breadth of the river is: (JEE Main 2004)

Draw a diagram. Let


𝐻𝑒𝑖𝑔ℎ𝑡 𝑜𝑓 𝑡𝑟𝑒𝑒 = ℎ
𝐵𝑟𝑒𝑎𝑑𝑡ℎ 𝑜𝑓 𝑟𝑖𝑣𝑒𝑟 = 𝑏

Find the height of the tree using the triangle with the greater angle of elevation:


tan 60° = ⇒⏟
ℎ = 𝑏 tan 60° = √3𝑏
𝑏 𝑬𝒒𝒖𝒂𝒕𝒊𝒐𝒏 𝑰

Find the height of the tree using the triangle with the smaller angle of
elevation:
ℎ 𝑏 + 40
tan 30° = ⇒ℎ=
𝑏 + 40 ⏟ √3
𝑬𝒒𝒖𝒂𝒕𝒊𝒐𝒏 𝑰𝑰

But the height in both the cases must be same.


Compare Equations I and II and set the RHS equal:
𝑏 + 40
√3𝑏 =
√3
Cross multiply and solve the resulting linear equation:
3𝑏 = 𝑏 + 40 ⇒ 𝑏 = 20

P a g e 28 | 213
Get all the files at: https://bit.ly/azizhandouts
Aziz Manva (azizmanva@gmail.com)

Example 1.78: General Case


The angle of inclination to a mountain of height ℎ is 𝜃1 . After moving a distance 𝑑 closer to the mountain, the
angle increases to 𝜃2 , with the point on the ground below the mountain peak 𝑥 units away. Show that
tan 𝜃2 tan 𝜃1
ℎ=𝑑
tan 𝜃2 − tan 𝜃1

In the right triangle with angle 𝜃2:


ℎ ℎ
tan 𝜃2 = ⇒ 𝑥 =
𝑥 ⏟ tan 𝜃2
𝑬𝒒𝒖𝒂𝒕𝒊𝒐𝒏 𝑰
In the right triangle with angle 𝜃1:
ℎ ℎ − tan 𝜃1 𝑑
tan 𝜃1 = ⇒ tan 𝜃1 𝑑 + tan 𝜃1 𝑥 = ℎ ⇒ 𝑥 =
𝑑+𝑥 ⏟ tan 𝜃1
𝑬𝒒𝒖𝒂𝒕𝒊𝒐𝒏 𝑰𝑰

Equate the RHS of Equations I and II:


ℎ ℎ − tan 𝜃1 𝑑
=
tan 𝜃2 tan 𝜃1

Cross-multiply:
ℎ tan 𝜃1 = ℎ tan 𝜃2 − tan 𝜃1 tan 𝜃2 𝑑

Collate all ℎ terms on the 𝑅𝐻𝑆, factor out ℎ:


tan 𝜃2 tan 𝜃1 𝑑 = ℎ(tan 𝜃2 − tan 𝜃1 )

Solve for ℎ:
tan 𝜃2 tan 𝜃1
ℎ=𝑑∙
tan 𝜃2 − tan 𝜃1

Example 1.79
𝐴𝐵 is a vertical pole with 𝐵 at the ground level and 𝐴 at the top. A man finds that the angle of elevation from a
certain point 𝐶 on the ground is 60°. He moves away from the pole along the line 𝐵𝐶 to a point 𝐷 such that 𝐶𝐷 =
7𝑚. From 𝐷, the angle of elevation of the point 𝐴 is 45°. Then the height of the pole is: (JEE Main 2008)

Note: Rationalize the denominator of the answer

tan 𝜃1 tan 𝜃2
ℎ=𝑑
tan 𝜃2 − tan 𝜃1

Substitute 𝑑 = 7, tan 𝜃1 = tan 45° = 1, tan 𝜃2 = tan 60° = √3:


(1)(√3)
=7
√3 − 1

Rationalize the denominator:


7√3(√3 + 1) 21 + 7√3
= =
2 2

1.80: Double Angle Identity for 𝒕𝒂𝒏

P a g e 29 | 213
Get all the files at: https://bit.ly/azizhandouts
Aziz Manva (azizmanva@gmail.com)

2 tan 𝜃
tan 2𝜃 =
1 − tan2 𝜃

Example 1.81
3
The central pole of a conical tent is 2 meter high. The pole is supported by ropes tied to its top and nails on the
ground. If, on the ground from the foot of the pole, the distances of the surface of the tent and the nail(s) are in
the ratio of 1:3, and if the angles of depression form the top of the pole to the nails, and the surface of the tent
are in the ratio of 1:2, then the length of one such rope is: (JMET 2011/78)

From the rope and the pole:


3
1
tan 𝜃 = 2 ⇒ 𝑥 =
3𝑥 2 tan 𝜃

From the tent and the pole:


3
3
tan 2𝜃 = 2 ⇒ 𝑥 =
𝑥 2 tan 2𝜃

But both values of 𝑥 must be equal. Hence:


1 3
= ⇒ tan 2𝜃 = 3 tan 𝜃
2 tan 𝜃 2 tan 2𝜃

Use the double angle identity for tan 𝜃, and then solve for 𝜃:
2 tan 𝜃
= 3 tan 𝜃
1 − tan2 𝜃
2 = 3 − 3 tan2 𝜃
1
tan2 𝜃 =
3
1
tan 𝜃 =
√3
𝜃 = 30°
1
sin 𝜃 =
2

Use the formula for sin 𝜃 in the triangle with the rope:
3
𝑜𝑝𝑝 𝑙(𝑝𝑜𝑙𝑒) 2 1
sin 𝜃 = = = = ⇒ 𝑙(𝑟𝑜𝑝𝑒) = 3
ℎ𝑦𝑝 𝑙(𝑟𝑜𝑝𝑒) 𝑙(𝑟𝑜𝑝𝑒) 2

1.82: Sum and Difference Identities


sin(𝛼 ± 𝛽) = sin 𝛼 cos 𝛽 ± cos 𝛼 sin 𝛽
cos(𝛼 ± 𝛽) = cos 𝛼 cos 𝛽 ∓ sin 𝛼 sin 𝛽

Note that
➢ Sine of a sum, RHS has a plus sign.
➢ Sine of a difference, RHS has a negative sign
➢ Cosine of a sum, RHS has a plus sign.
➢ Cosine of a difference, RHS has a negative sign

Example 1.83

P a g e 30 | 213
Get all the files at: https://bit.ly/azizhandouts
Aziz Manva (azizmanva@gmail.com)

𝑀𝑎𝑟𝑘 𝑡ℎ𝑒 𝑐𝑜𝑟𝑟𝑒𝑐𝑡 𝑜𝑝𝑡𝑖𝑜𝑛


The angle of elevation of the top of a vertical tower of a point 𝑃 on the horizontal ground was observed to be 𝛼.
After moving a distance 2 meters from 𝑃 towards the foot of the tower, the angle of elevation changes to 𝛽. Then
the height (in meters) of the tower, is:
2 sin 𝛼 sin 𝛽
A. sin(𝛽−𝛼)
2 sin 𝛽
B. cos (𝛽−𝛼)
2 sin(𝛽−𝛼)
C. sin 𝛼 sin 𝛽
cos (𝛽−𝛼)
D. sin 𝛼 sin 𝛽
(JEE Main, April 11, 2014)

tan 𝜃2 tan 𝜃1
ℎ=𝑑 , 𝜃2 > 𝜃1
tan 𝜃2 − tan 𝜃1

sin 𝜃
Substitute 𝜃2 = 𝛽, 𝜃1 = 𝛼, 𝑑 = 2 in the above. Expand using tan 𝜃 = cos 𝜃:
sin 𝛽 sin 𝛼

cos 𝛽 cos 𝛼
=2
sin 𝛽 sin 𝛼

cos 𝛽 cos 𝛼

Add the fractions and cancel the denominator:


sin 𝛽 sin 𝛼
cos 𝛽 cos 𝛼 sin 𝛽 sin 𝛼
=2 =2
sin 𝛽 cos 𝛼 − sin 𝛼 cos 𝛽 sin 𝛽 cos 𝛼 − sin 𝛼 cos 𝛽
cos 𝛽 cos 𝛼

Use the property sin(𝛼 − 𝛽) = sin 𝛼 cos 𝛽 − cos 𝛼 sin 𝛽. The positions of 𝛼 and 𝛽 are interchanged. Hence:
sin 𝛽 sin 𝛼
=2 ⇒ 𝑂𝑝𝑡𝑖𝑜𝑛 𝐴
sin(𝛽 − 𝛼)
D. Time, Speed and Distance

Example 1.84
A man is walking towards a vertical pillar in a straight path, at a uniform speed. At a certain point 𝐴 on the path,
he observes that the angle of elevation of the top of the pillar is 30°. After walking for 10 minutes from 𝐴 in the
same direction, at a point 𝐵, he observes that the angle of elevation of the top of the pillar is 60°. Then the time
taken (in minutes) by him, from 𝐵 to reach the pillar is: (JEE Main 2016)

Distance is proportional to time, so let


𝐵 = 10 𝑚𝑒𝑡𝑒𝑟𝑠, 𝐵𝐶 = 𝑡 𝑚𝑒𝑡𝑒𝑟𝑠 ⇒ 𝑊𝑖𝑙𝑙 𝑡𝑎𝑘𝑒 𝑡 𝑚𝑖𝑛𝑢𝑡𝑒𝑠

1ℎ 10 + 𝑡
𝐼𝑛 Δ𝐷𝐴𝐶: tan 30° = =⇒ℎ=
√3 10 + 𝑡 √3

𝐼𝑛 Δ𝐷𝐵𝐶: tan 60° = √3 = ⇒ ℎ = √3𝑡
𝑡

10 + 𝑡
= √3𝑡 ⇒ 𝑡 = 5
√3

Example 1.85
A bird is sitting on top of a vertical pole 20 m which makes an angle of elevation 45° from a point 𝑂 on the

P a g e 31 | 213
Get all the files at: https://bit.ly/azizhandouts
Aziz Manva (azizmanva@gmail.com)

ground. It flies off horizontally straight away from the point 𝑂. After one second, the elevation of the bird from 𝑂
𝑚
is reduced to 30°. Then the speed (𝑖𝑛 𝑠 ) of the bird is: (JEE Main 2014)

Since we have a 45 − 45 − 90 triangle


𝐷𝑖𝑠𝑡𝑎𝑛𝑐𝑒 𝑓𝑟𝑜𝑚 𝑂 𝑡𝑜 𝑃𝑜𝑙𝑒 = 20 𝑚
In Δ𝐴𝐷𝐹
1 20
tan 30° = =
√3 20 + 𝑑
20 + 𝑑 = 20√3
𝑑 = 20(√3 − 1)
𝐷𝑖𝑠𝑡𝑎𝑛𝑐𝑒 20(√3 − 1) 𝑚
𝑆𝑝𝑒𝑒𝑑 = = = 20(√3 − 1)
𝑇𝑖𝑚𝑒 1 𝑠

Example 1.86
An aeroplane flying at a constant speed, parallel to the horizontal ground, √3 𝑘𝑚 above it is observed at an
elevation of 60° from a point on the ground. If, after five seconds, its elevation at the same point, is 30°, then the
𝑘𝑚
speed (in ) of the aeroplane is: (JEE Main April 15, 2018)
ℎ𝑟

𝐵𝐶 𝐵𝐶 √3
Δ𝐴𝐵𝐶: tan 60° = ⇒ 𝐴𝐶 = = =1
𝐴𝐶 tan 60° √3

√3 1 √3
Δ𝐴𝐷𝐸: tan 30° = ⇒ = =1+𝑑 =3⇒𝑑 =2
1 + 𝑑 √3 1 + 𝑑

𝑑 2 𝑘𝑚 2 𝑘𝑚 𝑘𝑚
𝑆= = = × 3600 = 1440
𝑡 5 𝑠 5 ℎ𝑟 ℎ𝑟

Example 1.87
𝑚
A bomber flying ℎ feet high has an angle of depression of 𝜃 to its target, and releases a missile flying at 𝑠1 𝑠 . The
plane is exactly above a military station, from which troops leave to reach the target at the same time as the
𝑚
missile is released. The troops have a speed of 𝑠2 . The missile reaches the target first. The troops will reach
𝑠
𝑓 (𝜃) 𝑓 (𝜃)
𝑡= ℎ ( 1𝑠 − 2𝑠 ) seconds later. Find the value of 𝑡.
2 1

𝐴𝑛𝑔𝑙𝑒 𝑜𝑓 𝐸𝑙𝑒𝑣𝑎𝑡𝑖𝑜𝑛 = 𝐴𝑛𝑔𝑙𝑒 𝑜𝑓 𝐷𝑒𝑝𝑟𝑒𝑠𝑠𝑖𝑜𝑛 = 𝜃

We use the trig ratios to find the values that we want1


𝑑ℎ𝑜𝑟𝑖𝑧𝑜𝑛𝑡𝑎𝑙
cot 𝜃 = ⇒ 𝑑ℎ𝑜𝑟𝑖𝑧𝑜𝑛𝑡𝑎𝑙 = ℎ cot 𝜃


sin 𝜃 = ⇒ 𝑑𝑆𝐿 = ℎ csc 𝜃
𝑑𝑆𝐿

𝐷
Using 𝑇 = 𝑆 , the time difference

𝑑ℎ𝑜𝑟𝑖𝑧𝑜𝑛𝑡𝑎𝑙
1 If you use tan 𝜃 =

, then your trigonometric function will be in the denominator, and you will have to
manipulate to move it to the numerator.

P a g e 32 | 213
Get all the files at: https://bit.ly/azizhandouts
Aziz Manva (azizmanva@gmail.com)

𝑑ℎ𝑜𝑟𝑖𝑧𝑜𝑛𝑡𝑎𝑙 𝑑𝑆𝐿 ℎ cot 𝜃 ℎ csc 𝜃 cot 𝜃 csc 𝜃


= − = − = ℎ( − )
⏟ 𝑠2 𝑠1
⏟ 𝑠2 𝑠1 𝑠2 𝑠1
𝑻𝒓𝒐𝒐𝒑 𝑻𝒊𝒎𝒆 𝑴𝒊𝒔𝒔𝒊𝒍𝒆 𝑻𝒊𝒎𝒆

E. Two Triangles: Vertical Movement

Pending

Example 1.88
43: From the base of a pole of height 20 meter, the angle of elevation of the top of a tower is 60°. The pole
subtends an angle of 30° at the top of the tower. Then the height of the tower is: (JEE Main, June 29, 2022, Shift-
II)

1.89: Trig Functions of 𝟏𝟓° and 𝟕𝟓°


√3 − 1
sin 15° = cos 75° =
2√2
√3 + 1
cos 15° = sin 75° =
2√2
tan 15° = 2 − √3
tan 75° = 2 + √3

Example 1.90
A tower PQ stands on a horizontal ground with base Q on the ground. The point R divides the tower into parts
such that 𝑄𝑅 = 15𝑚. If from a point A on the ground the angle of elevation of R is 60°, and the part PR of the
tower subtends an angle of 15° at A, then the height of the tower is: (JEE Main July 25, 2022, Shift-I)

15 15 15 15√3
tan 60° = ⇒ 𝐴𝑄 = = = = 5√3
𝐴𝑄 tan 60° √3 3

𝑃𝑄
tan 75° =
𝐴𝑄
𝑃𝑄
= 2 + √3
5√3
𝑃𝑄 = 10√3 + 15

1.91: Double Angle Identity for 𝒕𝒂𝒏


2 tan 𝜃
tan 2𝜃 =
1 − tan2 𝜃

Example 1.92
A vertical pole fixed to the horizontal ground is divided in the ratio 3: 7 by a mark on it with lower part shorter
than the upper part. If the two parts subtend equal angles at a point on the ground 18𝑚 away from the base of
the pole, then the height of the pole is (as a number): (JEE Main 2017; JEE Main Aug 21, 2021, Shift-I)

P a g e 33 | 213
Get all the files at: https://bit.ly/azizhandouts
Aziz Manva (azizmanva@gmail.com)

In Δ𝐴𝐵𝐶
2 tan 𝜃 10𝑥
tan 2𝜃 = =
1 − tan2 𝜃 18
3𝑥 𝑥
Substitute tan 𝜃 = 18 = 6 from Δ𝐷𝐵𝐶:
𝑥
2 (6) 10𝑥
2 =
𝑥 18
1 − (6)
10𝑥 3
6𝑥 = 10𝑥 −
36
10𝑥 3
= 4𝑥 ⇒ 10𝑥 2 = 144 ⇒ √10𝑥 = 12 ⇒ 𝑥 = 12√10
36

1.93: Sum and Difference Identities for 𝒕𝒂𝒏


tan 𝛼 + tan 𝛽
tan(𝛼 + 𝛽) =
1 − tan 𝛼 tan 𝛽
tan 𝛼 − tan 𝛽
tan(𝛼 − 𝛽) =
1 + tan 𝛼 tan 𝛽

Example 1.94
3𝑡ℎ 3
The upper 4 portion of a vertical pole subtends an angle tan−1 5 at a point in the horizontal plane through its
foot and at a distance of 40𝑚 from the foot. The possible height(s) of the vertical pole are (as a number): (JEE
2003, Adapted)

𝛼 + 𝛽 = 𝜃 ⇒ 𝛽 = 𝜃 − 𝛼 ⇒ tan 𝛽 = tan(𝜃 − 𝛼)

RHS
3
Substitute 𝛽 = tan−1 5
3 3
tan 𝛽 = tan (tan−1 ) =
5 5
RHS
Use the difference of angles formula:
tan 𝜃 − tan 𝛼
tan(𝜃 − 𝛼) =
1 + tan 𝜃 tan 𝛼
1
ℎ ℎ ℎ
Substitute tan 𝜃 = 40
= 40 = 160:
, tan 𝛼 4

ℎ ℎ 3ℎ
− 3ℎ 6400 3(40ℎ)
= 40 160 = 160 = ∙ =
ℎ ℎ ℎ2 160 6400 + ℎ 2 6400 + ℎ2
1 + (40) (160) 1 +
6400

Combine LHS and RHS, cross-multiply and solve the resulting quadratic:
3 3(40ℎ)
= ⇒ ℎ2 − 200ℎ + 6400 = 0 ⇒ ℎ ∈ {40,160}
5 6400 + ℎ2

Example 1.95
Let a vertical tower 𝐴𝐵 of height 2ℎ stands on a horizontal ground. Let from a point 𝑃 on the ground a man can
see upto height ℎ of the tower with an angle of elevation 2𝛼. When from 𝑃, he moves a distance 𝑑 in the

P a g e 34 | 213
Get all the files at: https://bit.ly/azizhandouts
Aziz Manva (azizmanva@gmail.com)

direction of 𝐴𝑃 he can see the top 𝐵 of the tower with an angle of elevation 𝛼. If 𝑑 = √7ℎ, then the possible
values of tan 𝛼 are: (answer as a number): (JEE Main, July 27,2022, Shift-I, Adapted)

In Δ𝐶𝑃𝐴
ℎ ℎ ℎ ℎ(1 − tan2 𝛼)
tan 2𝛼 = ⇒ 𝑃𝐴 = = =
𝑃𝐴 tan 2𝛼 2 tan 𝛼 2 tan 𝛼
1 − tan2 𝛼

In Δ𝐵𝐻𝐴:
2ℎ 2ℎ ℎ (2 − √7(tan 𝛼))
tan 𝛼 = ⇒ 𝑃𝐴 = − √7ℎ =
𝑃𝐴 + √7ℎ tan 𝛼 tan 𝛼

Equate the RHS of Equations I and II and substitute 𝑥 = tan 𝛼:


ℎ(1 − 𝑥 2 ) 2ℎ − √7ℎ𝑥
=
2𝑥 𝑥
2
1 − 𝑥 = 4 − 2√7𝑥
𝑥 2 − 2√7𝑥 + 3 = 0
2√7 ± √28 − (4)(1)(3) 2√7 ± 4
tan 𝛼 = 𝑥 = = = √7 ± 2
2 2

Example 1.96
From the top 𝐴 of a vertical wall 𝐴𝐵 of height 30𝑚, the angles of depression of the top 𝑃 and bottom 𝑄 of a
vertical tower 𝑃𝑄 are 15° and 60° respectively. 𝐵 and 𝑄 are on the same horizontal level. If 𝐶 is a point on 𝐴𝐵
such that 𝐶𝐵 = 𝑃𝑄, then the area (𝑖𝑛 𝑚2 ) of the quadrilateral 𝐵𝐶𝑃𝑄 is (answer as a number): (JEE Main, April
6, 2023, Shift-I)

In Δ𝐴𝐵𝑄
30
tan 60° = √3 = ⇒ 𝐵𝑄 = 10√3
𝐵𝑄

In Δ𝐴𝐶𝑃
𝐴𝐶
tan 15° = 2 − √3 =
𝐶𝑃
𝐴𝐶 = (2 − √3)𝐶𝑃 = (2 − √3)(10√3) = 20√3 − 30
𝐶𝐵 = 30 − 𝐴𝐶 = 30 − (20√3 − 30) = 60 − 20√3

(𝐵𝑄)(𝐶𝐵) = 10√3(60 − 20√3) = 600√3 − 600

1.6 Heights and Distances: II


A. Optics

Example 1.97
50: A ray of light coming from the point (2,2√3) is incident at an angle 30° on the line 𝑥 = 1 at the point A. The
ray gets reflected on the line 𝑥 = 1 and meet the 𝑥-axis at the point 𝐵. Then, the line 𝐴𝐵 passes through the

P a g e 35 | 213
Get all the files at: https://bit.ly/azizhandouts
Aziz Manva (azizmanva@gmail.com)

point:
1
A. (3, − )
√3
√3
B. (4, − )
2
C. (3, −√3)
D. (4, −√3) (JEE Main Sep 6, 2022, Shift-I)

𝑦2 − 𝑦1
𝑆𝑙𝑜𝑝𝑒 𝑜𝑓 𝐴𝐵 = = −√3
𝑥2 − 𝑥1

𝑦 − 𝑦1 = 𝑚(𝑥 − 𝑥1 )
𝑦 − 0 = −√3(𝑥 − 2)
𝑦 = −√3𝑥 + 2√3

Check the options and substitute


𝑂𝑝𝑡𝑖𝑜𝑛 𝐶

B. Looking Up and Down

Example 1.98
A house of height 100 𝑚 subtends a right angle at the window of an opposite house. If the height of the window
be 64 m, then the distance between the two houses is: (NTA Abhyas, JEE/Math/Heights and Distances)

Let 𝑑 be the distance between the two houses.


64
In right Δ𝐷𝐴𝐵: tan 𝜃 =
𝑑
𝑑
In right Δ𝐶𝐷𝐸 tan 𝜃 =
36
Equate the two values:
64 𝑑
=
𝑑 36
𝑑 2 = 64 × 36
𝑑 = 8 × 6 = 48

Example 1.99
The angle of elevation of a cloud 𝐶 from a point 𝑃, 200 m above a still lake is 30°. If the angle of depression of
the image of 𝐶 in the lake from the point 𝑃 is 60°, then 𝑃𝐶 (in meters) is equal to: (JEE Main, Sep 4, 2020, Shift-
II)

P a g e 36 | 213
Get all the files at: https://bit.ly/azizhandouts
Aziz Manva (azizmanva@gmail.com)

In Δ𝑃𝐷𝐶′:
ℎ + 400
tan 60° = √3 = ⇒ 3ℎ = ℎ + 400 ⇒ 2ℎ = 400
√3ℎ

𝑃𝐶 = 400

Example 1.100
𝜋
In the figure, 𝜃1 + 𝜃2 = 2 and √3(𝐵𝐸) = 4(𝐴𝐵). If the area of Δ𝐶𝐴𝐵 is 2√3 −
𝜃
3 𝑢𝑛𝑖𝑡 2 , when 𝜃2 is the largest, then the perimeter (in units) of Δ𝐶𝐸𝐷 is equal to:
1
(JEE Main, April 10, 2023, Shift-II, Integer Response)

The given condition has a relation between 𝐵𝐸 and 𝐴𝐵. Hence, write the parts of 𝐵𝐸 in terms of 𝐴𝐵:
𝐴𝐶
tan 𝜃1 = ⇒⏟𝐵𝐷 = 𝐴𝐶 = 𝐴𝐵 tan 𝜃1
𝐴𝐵
𝑬𝒒𝒖𝒂𝒕𝒊𝒐𝒏 𝑰
𝐶𝐷 𝐶𝐷 𝐴𝐵
∠𝐶𝐸𝐷 = 𝜃1 ⇒ tan 𝜃1 = ⇒ 𝐷𝐸 = =
𝐷𝐸 ⏟ tan 𝜃1 tan 𝜃1
𝑬𝒒𝒖𝒂𝒕𝒊𝒐𝒏 𝑰𝑰

𝐴𝐵 1
𝐵𝐸 = 𝐵𝐷 + 𝐷𝐸 = 𝐴𝐵 tan 𝜃1 + = 𝐴𝐵 (tan 𝜃1 + )
tan 𝜃1 tan 𝜃1

4
Substitute √3(𝐵𝐸) = 4(𝐴𝐵) ⇒ 𝐵𝐸 = 𝐴𝐵:
√3
4 1
𝐴𝐵 = 𝐴𝐵 (tan 𝜃1 + )
√3 tan 𝜃1
4 3+1 1 1
= = √3 + = tan 𝜃1 +
√3 √3 √3 tan 𝜃1

Hence, the solutions are2:


1
tan 𝜃1 = 𝑥 ∈ {√3, }
√3

𝜃2 1
To make
𝜃1
largest, we make tan 𝜃1 smallest, giving tan 𝜃1 = ⇒ 𝜃1 = 30° ⇒ 𝜃2 = 60°.
√3
Use the area of the triangle condition:

4 1
2 We can also do this by solving the equation. Substitute tan 𝜃1 = 𝑥 to get = 𝑥 + ⇒ √3𝑥 2 − 4𝑥 + √3 = 0
√3 𝑥
Factor using 𝑃𝑟𝑜𝑑𝑢𝑐𝑡 = √3√3 = 3, 𝑆𝑢𝑚 = −4 = −3 − 1:
√3𝑥 2 − 3𝑥 − 𝑥 + √3 = 0
√3𝑥(𝑥 − √3) − 1(𝑥 − √3) = 0
(√3𝑥 − 1)(𝑥 − √3) = 0

P a g e 37 | 213
Get all the files at: https://bit.ly/azizhandouts
Aziz Manva (azizmanva@gmail.com)

1 1 1 1
2√3 − 3 = [𝐶𝐴𝐵] = (𝐴𝐵)(𝐴𝐶) = (𝐴𝐵)(𝐴𝐵 tan 𝜃1 ) = 𝐴𝐵2 ( )
2 2 2 √3
2
𝐴𝐵 = 2 3(2 3 − 3) = 12 − 6 3
√ √ √

Use (𝑎 − 𝑏)2 = 𝑎2 − 2𝑎𝑏 + 𝑏2


2𝑎𝑏 = 6√3 ⇒ 𝑎𝑏 = 3√3 ⇒ 𝐶𝐷 = 𝐴𝐵 = 3 − √3

Using the properties of a 30 − 60 − 90 triangle:


𝑃𝑒𝑟𝑖𝑚𝑒𝑡𝑒𝑟 = 𝐶𝐷 + 𝐷𝐸 + 𝐶𝐸 = 𝐶𝐷 + √3𝐶𝐷 + 2𝐶𝐷 = 𝐶𝐷(1 + √3 + 2) = 𝐶𝐷(3 + √3)
= (3 − √3)(3 + √3) = 9 − 3 = 6

C. Two Poles

1.101: Trig Functions of 𝟐𝟐. 𝟓 and 𝟔𝟕. 𝟓


√2 − 1
sin 22.5° = cos 67.5° =
√4 − 2√2
1
cos 22.5° = sin 67.5° =
√4 − 2√2
tan 22.5° = √2 − 1

Example 1.102
44: Let 𝐴𝐵 and 𝑃𝑄 be two vertical poles, 160𝑚 apart from each other. Let 𝐶 be the middle point of 𝐵 and 𝑄,
𝜋
which are feet of the two poles. Let 8 and 𝜃 be the angles of elevation from 𝐶 to 𝑃 and 𝐴, respectively. If the
height of Pole 𝑃𝑄 is twice the height of pole 𝐴𝐵, then tan2 𝜃 is equal to: (JEE Main, June 28, 2022, Shift-I)

In Δ𝐴𝐵𝐶

tan 𝜃 = ⇒ ℎ = 80 tan 𝜃
80
In Δ𝑃𝑄𝐶
𝜋 2ℎ 𝜋
tan = ⇒ ℎ = 40 tan
8 80 8
𝜋
80 tan 𝜃 = 40 tan
8
1 𝜋 1
tan 𝜃 = tan = (√2 − 1)
2 8 2
1 1
tan2 𝜃 = (2 − 2√2 + 1) = (3 − 2√2)
4 4

Pending

Example 1.103
48: Two vertical poles are 150m apart and the height of one is three times that of the other. If from the middle
point of the line joining their feet, an observer finds the angles of elevation of their tops to be complementary,
then the height of the shorter pole (in meters) is: (JEE Main, Feb 24, 2021, Shift-I)

P a g e 38 | 213
Get all the files at: https://bit.ly/azizhandouts
Aziz Manva (azizmanva@gmail.com)

1.104: Similarity Property

In Δ𝐷𝐴𝐶 and Δ𝑃𝐴𝐹


∠𝐷𝐶𝐴 = ∠𝑃𝐹𝐴 = 90°
∠𝐷𝐴𝐶 = ∠𝑃𝐴𝐹 (𝑆𝑎𝑚𝑒 𝐴𝑛𝑔𝑙𝑒)
Δ𝐷𝐴𝐶~Δ𝑃𝐴𝐹

𝑚 𝑃𝐹 𝑚
= ⇒ 𝑃𝐹 = 𝐶𝐷 ( )
𝑚 + 𝑛 𝐶𝐷 ⏟ 𝑚+𝑛
𝑬𝒒𝒖𝒂𝒕𝒊𝒐𝒏 𝑰

In Δ𝐵𝐴𝐶 and Δ𝑃𝐹𝐶


∠𝐵𝐴𝐶 = ∠𝑃𝐹𝐶 = 90°
∠𝐵𝐶𝐴 = ∠𝑃𝐶𝐹 (𝑆𝑎𝑚𝑒 𝐴𝑛𝑔𝑙𝑒)
Δ𝐵𝐴𝐶~Δ𝑃𝐶𝐹

𝑛 𝑃𝐹 𝑛
= ⇒ 𝑃𝐹 = 𝐴𝐵 ( )
𝑚 + 𝑛 𝐴𝐵 ⏟ 𝑚+𝑛
𝑬𝒒𝒖𝒂𝒕𝒊𝒐𝒏 𝑰𝑰
From Equation I and II:
𝑚 𝑛
𝐶𝐷 ( ) = 𝐴𝐵 ( )
𝑚+𝑛 𝑚+𝑛
𝑚 𝐴𝐵
=
𝑛 𝐶𝐷

Example 1.105
52: Two vertical poles 𝐴𝐵 = 15𝑚 and 𝐶𝐷 = 10𝑚 are standing apart on a horizontal ground with points 𝐴 and 𝐶
on the ground. If 𝑃 is the point of intersection of 𝐵𝐶 and 𝐴𝐷, then the height of 𝑃 (in meters) above the line 𝐴𝐶
is: (JEE Main, Sep 4, 2020, Shift-I)

𝑚 𝐴𝐵 15 3 3
= = = ⇒𝑚= 𝑛
𝑛 𝐶𝐷 10 2 2

3 3
𝑚 2 𝑛 𝑛 3
𝑃𝐹 = 𝐶𝐷 ( ) = 10 ( ) = 10 (2 ) = 10 ( ) = 6
𝑚+𝑛 3 5 5
2𝑛 + 𝑛 2
𝑛

D. Three Triangles: Multiple Angles of Elevation


Two triangles increase the complexity of a question. Three triangles
increase the complexity of a question still further.

Example 1.106
If the angles of elevation of the top of a tower from three collinear points 𝐴, 𝐵 and 𝐶, on a line leading to the foot
of the tower are 30°, 45° and 60°, respectively, then the ratio 𝐴𝐵: 𝐵𝐶 is: (JEE Main 2015)

Method I
Without loss of generality, let the height of the tower be 1. Then

P a g e 39 | 213
Get all the files at: https://bit.ly/azizhandouts
Aziz Manva (azizmanva@gmail.com)

1 1 1 1
𝑄𝐴 = = √3, 𝑄𝐵 = = 1, 𝑄𝐶 = =
tan 30° tan 45° tan 60° √3

𝐴𝐵 𝑄𝐴 − 𝑄𝐵 √3 − 1 √3 − 1
= = = = √3
𝐵𝐶 𝑄𝐵 − 𝑄𝐶 1 − 1 √3 − 1
√3 √3

Method II: Longer


𝐴𝐵 𝑐
Let 𝑄𝐶 = 𝑎, 𝐶𝐵 = 𝑏, 𝐴𝐵 = 𝑐 ⇒ 𝐵𝐶 = 𝑏


tan 60° = ⇒⏟
ℎ = 𝑎 tan 60° = √3𝑎
𝑎 𝐸𝑞𝑢𝑎𝑡𝑖𝑜𝑛 𝐼

tan 45° = ℎ = (𝑎 + 𝑏) tan 45° = 𝑎 + 𝑏
⇒⏟
𝑎+𝑏 𝐸𝑞𝑢𝑎𝑡𝑖𝑜𝑛 𝐼𝐼
ℎ 𝑎+𝑏+𝑐
tan 30° = ⇒ ℎ = (𝑎 + 𝑏 + 𝑐) tan 30° =
𝑎+𝑏+𝑐 ⏟ √3
𝐸𝑞𝑢𝑎𝑡𝑖𝑜𝑛 𝐼𝐼𝐼

From Equations I and II:

𝑏 𝑏
√3𝑎 = 𝑎 + 𝑏 ⇒ (√3 − 1)𝑎 = 𝑏 ⇒ 𝑎 = = (√3 + 1)
⏟ √3 − 1 2
𝑬𝒒𝒖𝒂𝒕𝒊𝒐𝒏 𝑰𝑽
From Equations I and III:

𝑎+𝑏+𝑐 𝑏+𝑐
√3𝑎 = ⇒ 3𝑎 = 𝑎 + 𝑏 + 𝑐 ⇒ 𝑎 =
√3 ⏟ 2
𝑬𝒒𝒖𝒂𝒕𝒊𝒐𝒏 𝑽

From Equations IV and V


𝑏 𝑏+𝑐 𝑐
(√3 + 1) = ⇒ 𝑏(√3 + 1) = 𝑏 + 𝑐 ⇒ √3𝑏 = 𝑐 ⇒ = √3
2 2 𝑏

1.107: Angle Bisector Theorem


An angle bisector of a triangle divides the side opposite it in the ratio of the other
two sides of the triangle.

In the diagram, if 𝐴𝐷 is a bisector of ∠𝐵𝐴𝐶:


𝐵𝐷 𝐷𝐶 𝐵𝐷 𝑐
= ⇔ = ⇔ 𝐵𝐷: 𝐷𝐶 = 𝑐: 𝑏
𝑐 𝑏 𝐷𝐶 𝑏

Example 1.108
If the angles of elevation of the top of a tower from three collinear points 𝐴, 𝐵 and 𝐶, on a line leading to the foot
of the tower are 30°, 45° and 60°, respectively, then the ratio 𝐴𝐵: 𝐵𝐶 is: (JEE Main 2015)

P a g e 40 | 213
Get all the files at: https://bit.ly/azizhandouts
Aziz Manva (azizmanva@gmail.com)

∠𝑃𝐶𝐵 = 120°
∠𝐶𝑃𝐵 = 180 − 120 − 45 = 15°
∠𝐵𝑃𝐴 = 180 − 135 − 30 = 15°
∠𝐶𝑃𝐵 = ∠𝐵𝑃𝐴 = 15° ⇒ 𝑃𝐵 𝑏𝑖𝑠𝑒𝑐𝑡𝑠 ∠𝐶𝑃𝐴

By the angle bisector theorem:


𝐴𝐵: 𝐵𝐶 = 𝑃𝐴: 𝑃𝐶
ℎ 𝑃𝐴
sin 30° = ⇒ℎ=
𝑃𝐴 2
ℎ √3
sin 60° = ⇒ℎ= 𝑃𝐶
𝑃𝐶 2
𝑃𝐴 √3 𝑃𝐴
= 𝑃𝐶 ⇒ = √3 ⇒ 𝑃𝐴: 𝑃𝐶 = √3: 1
2 2 𝑃𝐶
E. Three Triangles: Incline

Example 1.109
The angle of elevation of the top of hill from a point on the horizontal plane passing through the foot of the hill
is found to be 45°. After walking a distance of 80 meters towards the slope, up a slope inclined at an angle of 30°
to the horizontal plane, the angle of elevation of the top of the hill becomes 75°. Then the height of the hill (in
meters) is (JEE Main, Sep 6, 2020, Shift-I)

ℎ − 40
tan 75° = 2 + √3 =
ℎ − 40√3
2ℎ − 80√3 + √3ℎ − 120 = ℎ − 40
2ℎ − 80√3 + √3ℎ − 120 = ℎ − 40
ℎ + √3ℎ = 80 + 80√3
ℎ(1 + √3) = 80(1 + √3)
ℎ = 80

Example 1.110
The angle of elevation of the top of 𝑃 of a vertical tower 𝑃𝑄 of height 10 from a point 𝐴 on the horizontal ground
is 45°. Let 𝑅 be a point on 𝐴𝑄 and from a point 𝐵, vertically above 𝑅, the angle of elevation is 60°. If ∠𝐵𝐴𝑄 =
30°, 𝐴𝐵 = 𝑑 and the area of the trapezium 𝑃𝑄𝑅𝐵 is 𝛼, then the ordered pair (𝑑, 𝛼) is (Answer as a numerical
expression) (JEE Main, July 27, 2022, Shift-II)

√3 𝑑
Since Δ𝐴𝐵𝑅 is a 30 − 60 − 90 triangle, substitute 𝐴𝑅 = = 𝑋𝑄 = 2:
2
𝑑, 𝐵𝑅
𝑑 20 − 𝑑
𝑃𝑋 𝑃𝑄 − 𝑋𝑄 10 − 20 − 𝑑
tan 60° = = = 2 = 2 =
𝐵𝑋 𝑄𝐴 − 𝑅𝑄 √3 20 − √3𝑑 20 − √3𝑑
10 − 2 𝑑 2

P a g e 41 | 213
Get all the files at: https://bit.ly/azizhandouts
Aziz Manva (azizmanva@gmail.com)

Substitute tan 60° = √3:


20 − 𝑑
√3 = ⇒ 20√3 − 3𝑑 = 20 − 𝑑 ⇒ 𝑑 = 10√3 − 10
20 − √3𝑑

The area of the trapezium 𝑃𝑄𝑅𝐵


𝐵𝑅 + 𝑄𝑃 20 − √3𝑑 𝑑 + 20
= (𝑅𝑄) ( )=( )( )
2 2 4

20 − √3(10√3 − 10) (10√3 − 10) + 20


=( )( )
2 4
1 1 200
= (10√3 − 10)(10√3 + 10) = (300 − 100) = = 25
8 8 8
F. Tangents to a Circle

1.111: Tangents to a Circle


➢ The tangents to a circle drawn from a point outside the circle are congruent.
➢ Tangent Perpendicularity Theorem: The tangent to a circle is perpendicular to the radius at the point of
tangency.

Example 1.112
A spherical balloon of radius 𝑟 subtends an angle 𝛼 at the eye of an observer. If the angle of elevation of the
center of the balloon be 𝛽, then the height of the center of the balloon in terms of 𝑟, 𝛼, 𝛽 is: (NTA Abhyas,
JEE/Math/Heights and Distances, Adapted; JEE Main July 25, 2021, Shift-I, Adapted)

Let
𝑂 = 𝑐𝑒𝑛𝑡𝑒𝑟 𝑜𝑓 𝑏𝑎𝑙𝑙𝑜𝑜𝑛
𝑙 = 𝐷𝑖𝑠𝑡𝑎𝑛𝑐𝑒 𝑓𝑟𝑜𝑚 𝑂𝑏𝑠𝑒𝑟𝑣𝑒𝑟 𝑡𝑜 𝐵𝑎𝑙𝑙𝑜𝑜𝑛
ℎ = ℎ𝑒𝑖𝑔ℎ𝑡 𝑜𝑓 𝑏𝑎𝑙𝑙𝑜𝑜𝑛
𝛼 = ∠𝐴𝐵𝐶
The lines drawn from point B to the sphere are tangent to the sphere.
The radii at the point of contact are perpendicular to the tangents (by
the tangent perpendicularity theorem).

𝐴𝐵𝐶 𝛼
Δ𝐵𝐴𝑂 ≅ Δ𝐵𝐶𝑂 ⇒ ∠𝐴𝐵𝑂 = ∠ =
2 2
𝛼 𝑟 𝑟
In right Δ𝐴𝐵𝑂: sin ( ) = ⇒ 𝑙 = 𝛼
2 𝑙 sin ( 2 )
ℎ 𝑟 sin 𝛽
In right Δ𝑂𝐵𝐷: sin 𝛽 = ⇒ ℎ = 𝑙 sin 𝛽 = 𝛼
𝑙 sin ( 2 )

1.113: Absolute and Percentage Error


Absolute error is the difference between an actual value and an approximated value:
𝐴𝑏𝑠𝑜𝑙𝑢𝑡𝑒 𝐸𝑟𝑟𝑜𝑟 = |𝑟 − 𝑟𝐴𝑝𝑝𝑟𝑜𝑥 |

Percentage is absolute error written as a percentage


𝑟 − 𝑟𝐴𝑝𝑝𝑟𝑜𝑥
𝑃𝑒𝑟𝑐𝑒𝑛𝑡𝑎𝑔𝑒 𝐸𝑟𝑟𝑜𝑟 = | | × 100
𝑟
P a g e 42 | 213
Example 1.114: Radius of the Earth
A. Show that if the angle of depression from a mountain with height ℎ meters to the ground is 𝛼°, then the
ℎ cos 𝛼
radius of earth is 𝑟 = 1−cos 𝛼.
B. Estimate the radius of the earth in km if ℎ = 965 and 𝛼 = 89°.
C. Use the actual radius of the earth (6371 𝑘𝑚) to find the absolute error and the percentage error in the
estimation in Part B.

Draw the Earth, with a mountain on top. Let:


𝑂 = 𝐶𝑒𝑛𝑡𝑒𝑟 𝑜𝑓 𝐸𝑎𝑟𝑡ℎ
𝑀 = 𝑇𝑖𝑝 𝑜𝑓 𝑀𝑜𝑢𝑛𝑡𝑎𝑖𝑛
𝑟 = 𝑅𝑎𝑑𝑖𝑢𝑠 𝑜𝑓 𝐸𝑎𝑟𝑡ℎ
ℎ = ℎ𝑒𝑖𝑔ℎ𝑡 𝑜𝑓 𝑡ℎ𝑒 𝑚𝑜𝑢𝑛𝑡𝑎𝑖𝑛
𝛼 = 𝑎𝑛𝑔𝑙𝑒 𝑜𝑓 𝑑𝑒𝑝𝑟𝑒𝑠𝑠𝑖𝑜𝑛

Note that from the mountain you can see uptil T, and hence:
𝑀𝑇 𝑖𝑠 𝑡𝑎𝑛𝑔𝑒𝑛𝑡 𝑡𝑜 𝑡ℎ𝑒 𝐸𝑎𝑟𝑡ℎ

By the tangent perpendicularity theorem:


∠𝑀𝑇𝑂 = 90°
∠𝑇𝑀𝑂 = 90 − 𝛼
∠𝑇𝑂𝑀 = 180 − 90 − (90 − 𝛼) = 𝛼

Part A
In right Δ𝑀𝑇𝑂:
𝑟
cos 𝛼 =
𝑟+ℎ
Clear fractions:
𝑟 cos 𝛼 + ℎ cos 𝛼 = 𝑟
Collate all 𝑟 terms on the RHS, and factor out 𝑟
ℎ cos 𝛼 = 𝑟(1 − cos 𝛼)
Solve for 𝑟:
ℎ cos 𝛼
𝑟=
1 − cos 𝛼
Part B
ℎ cos 𝛼 965 cos 1°
𝑟= = = 6335 𝑘𝑚
1 − cos 𝛼 1 − cos 1°
Part C
𝐴𝑏𝑠𝑜𝑙𝑢𝑡𝑒 𝐸𝑟𝑟𝑜𝑟 = |6371 − 6335| = |36| = 36
𝑟 − 𝑟𝐴𝑝𝑝𝑟𝑜𝑥 36
𝑃𝑒𝑟𝑐𝑒𝑛𝑡𝑎𝑔𝑒 𝐸𝑟𝑟𝑜𝑟 = | | × 100 = × 100 = 0.56%
𝑟 6371

Example 1.115
Measuring Diameter of a Pipe

G. Multiple Triangles

Example 1.116
Get all the files at: https://bit.ly/azizhandouts
Aziz Manva (azizmanva@gmail.com)

1.7 3D Problem Solving


A. Cross Sections
A standard approach to 3D problems is take one or more suitable 2Dcross sections, work with them, and arrive
at the required answer. This technique is powerful because it reduces a difficult problem into a known problem.

1.117: Cross Section


A cross section is a 2D “slice” of a 3D object

When working with three dimensional objects, drawing is a challenge. One way to simplify is draw a 2D cross
section of a 3D object.

Example 1.118
Draw a cross section of a sphere.

The cross section of a sphere is always a circle.

Example 1.119
The conics (parabola, hyperbola, circle and ellipse) are cross section of two right circular cones placed with tips
together.

P a g e 44 | 213
Get all the files at: https://bit.ly/azizhandouts
Aziz Manva (azizmanva@gmail.com)

Example 1.120
What is the cross section of a right square pyramid:
A. Parallel to the Base
B. Perpendicular to the base

Part A
𝑆𝑞𝑢𝑎𝑟𝑒
Part B
𝑇𝑟𝑖𝑎𝑛𝑔𝑙𝑒
B. Pyramids

1.121: Right Pyramid


The apex of a right pyramid is exactly above the center of the base.

➢ Height is length of the line segment drawn the apex to the


base, perpendicular to the base.
➢ Slant height is the distance on a lateral face from the
center of the face to the apex.

Example 1.122
A right square pyramid has side length of base 𝑠 and height 𝑠.
Find:
A. the slant height
B. the angle made by a face with the base
C. the angle made an edge length (not on the base) of the
face with the base

P a g e 45 | 213
Get all the files at: https://bit.ly/azizhandouts
Aziz Manva (azizmanva@gmail.com)

Part A
Take a cross section perpendicular to the base and
parallel to a side, passing through the center of the
square base: (left diagram).
𝑠 2 5𝑠 2 𝑠√5
𝑆𝑙𝑎𝑛𝑡 𝐻𝑒𝑖𝑔ℎ𝑡 = √𝑠 2 + ( ) = √ =
2 4 2

Part B
Use the same cross section as in part A.
𝑠
tan 𝜃 = 𝑠 = 2 ⇒ 𝜃 = tan−1 2
2
Part C
Take a cross section perpendicular to the square base, and parallel to the diagonal of the base, passing through
the center of the square base.
𝑠
tan 𝜃 = 𝑠 = √2 ⇒ 𝜃 = tan−1 √2
√2

Example 1.123: Pyramid


A square pyramid has all sides length 𝑠. What is the angle formed by the slant edge of the pyramid with a base
diagonal?

Drop a perpendicular from the vertex of the pyramid to the base. It will touch the base at the center of the
square, which is also the point where the diagonals bisect each other.
𝑙(𝐷𝑖𝑎𝑔𝑜𝑛𝑎𝑙 ) 𝑠√2
𝑙(𝐷𝑖𝑎𝑔𝑜𝑛𝑎𝑙 ) = 𝑠√2 ⇒ =
2 2
𝑠√2
𝑎𝑑𝑗 𝑠√2 1 √2 1 𝜋
cos 𝜃 = = 2 = × = = ⇒ cos −1 𝜃 = 45° =
𝑜𝑝𝑝 𝑠 2 𝑠 2 √2 4
C. Cubes

1.124: Parallel Lines, Intersecting lines and skew lines


(𝟑𝑫 𝑮𝒆𝒐𝒎𝒆𝒕𝒓𝒚)
Two lines which go in the same direction are parallel lines.
Two lines which have exactly one point in common are intersecting
lines.
Two lines which are neither parallel nor intersecting are skew lines.

For example, in the diagram of a cube:


➢ AE and HG are skew lines
➢ AD and DC are intersecting line
➢ AD and BC are parallel lines

1.125: Intersecting Lines


Two intersecting lines lie in a single plane and the angle between the two lines can be determined by using 2D
geometry with respect to that plane.

A plane is determined by a line and a point not on it.

P a g e 46 | 213
Get all the files at: https://bit.ly/azizhandouts
Aziz Manva (azizmanva@gmail.com)

So, two points decide a line, and the direction of the line. A second line which intersects the first has one point
in common with the first line.
A point on the second line which is not on the first line determines a plane (in which both the lines lie).

1.126: Types of Diagonals


In a three-dimensional object, a diagonal which
➢ is contained entirely in a face is a face diagonal.
➢ Travels through space outside of a face is a space diagonal

Example 1.127
Find the angles between the space diagonals of a cube.

Draw a cube. Without loss of generality, let the side length of the cube be
1

Consider the plane formed by points


𝐴, 𝐸, 𝐺
And note that C also lies in the same plane.

Note that 𝐴𝐸𝐺𝐶 forms a rectangle.


𝐴𝐸 = 1, 𝐴𝐶 = √2, 𝐴𝐺 = √3

𝑎𝑐 √2
𝑋𝑂 = =
2 2
𝑒𝑎 1
𝑒𝑋 = =
2 2
1
1
tan ∠𝑒𝑂𝑋 = tan 𝜃 = 2 =
√2 √2
2
1
𝜃 = 2 ∙ tan−1
√2
1
∠𝑒𝑂𝑎 = 2𝜃 = 2 ∙ tan−1
√2

Example 1.128
A cuboid has length 𝑙, width 𝑤, and height ℎ. Find the angles between its space diagonals.

P a g e 47 | 213
Get all the files at: https://bit.ly/azizhandouts
Aziz Manva (azizmanva@gmail.com)

There are two angles that we need to find. By similarity of triangles:


∠𝐸1 𝑂𝑋 = ∠𝐸1 𝐶1 𝐴1

𝑏
tan 𝜃 =
√𝑙 2 + ℎ2
𝑏
2𝜃 = 2 ∙ tan−1 ( )
√𝑙 2 + ℎ2

If 2𝜃 is the smaller angle then, the correct answer will be:


√𝑙 2 + ℎ2
2𝜃 = 2 ∙ tan−1 ( )
𝑏

The second angle (without diagram):


Consider rectangle 𝐴𝐵𝐺𝐻
𝐴𝐵 = 𝑙, 𝐴𝐻 = √ℎ2 + 𝑏 2
𝑙
2𝜃 = 2 ∙ tan−1 ( )
√ℎ2 + 𝑏 2

Example 1.129: Cuboid


The longest diagonal, the base diagonal and an edge are three sides of a triangle in a cuboid with length 𝑙, width
𝑤, and height ℎ. Find the angles of that triangle.

𝐶𝑢𝑏𝑜𝑖𝑑: 𝐵𝑎𝑠𝑒
⏟ 𝐷𝑖𝑎𝑔𝑜𝑛𝑎𝑙 = √𝑙 2 + 𝑤 2 ⇒ 𝐿𝑜𝑛𝑔𝑒𝑠𝑡
⏟ 𝐷𝑖𝑎𝑔𝑜𝑛𝑎𝑙 = √𝑙 2 + 𝑤 2 + ℎ2
𝑃𝑦𝑡ℎ𝑎𝑔𝑜𝑟𝑎𝑠 𝑃𝑦𝑡ℎ𝑎𝑔𝑜𝑟𝑎𝑠

Example 1.130: Cube


Find the angles of the triangle formed by the longest diagonal, the base diagonal and an edge of a cube.

𝐶𝑢𝑏𝑒: 𝐵𝑎𝑠𝑒
⏟ 𝐷𝑖𝑎𝑔𝑜𝑛𝑎𝑙 = √2𝑎2 = 𝑎√2 ⇒ 𝐿𝑜𝑛𝑔𝑒𝑠𝑡
⏟ 𝐷𝑖𝑎𝑔𝑜𝑛𝑎𝑙 = √3𝑎2 = 𝑎√3
𝑃𝑦𝑡ℎ𝑎𝑔𝑜𝑟𝑎𝑠 𝑃𝑦𝑡ℎ𝑎𝑔𝑜𝑟𝑎𝑠

D. Tower at center of Circle


This topic is the beginning of questions that require 3D analysis. Till now we were working using one
dimension on the ground, and one dimension as height. This is equivalent to taking a cross section.
Questions in 3D require multiple cross sections.

Example 1.131: Points on a circle


𝑟 cables (where 𝑟 represents the number of cables) keep the central pole of a tent in position. The ground
distance between the anchor points and the foot of the pole is 𝑑 meters. The angle of elevation from an anchor
point to the pole is α. If cable costs $𝐷 per meter, find the cost of the cable in terms of 𝑟, 𝑑, 𝛼 and 𝐷.

P a g e 48 | 213
Get all the files at: https://bit.ly/azizhandouts
Aziz Manva (azizmanva@gmail.com)

Let
𝑂𝑃 𝑏𝑒 𝑡ℎ𝑒 𝑝𝑜𝑙𝑒, 𝑤𝑖𝑡ℎ 𝑂 𝑡ℎ𝑒 𝑏𝑎𝑠𝑒 𝑎𝑛𝑑 𝑃 𝑡ℎ𝑒 𝑡𝑜𝑝

Let 𝑙 be the length of a single cable. Then in Δ𝑃𝑂𝐶:


𝑑 𝑑
cos 𝛼 = ⇒ 𝑙 = 𝑚
𝑙 cos 𝛼

Since each of the cables has the same length, the length of 𝑟 cables is:
𝑟𝑑
𝑟𝑙 = 𝑚
cos 𝛼

Cost of the cables


𝑟𝑑 𝐷 𝑟𝑑𝐷
= 𝑚× $= = 𝑟𝑑𝐷 sec 𝛼 $
cos 𝛼 𝑚 cos 𝛼

Example 1.132: Angle subtended by chord


A tower stands at the center of a circular park. 𝐴 and 𝐵 are two points on the boundary of the park such that
𝐴𝐵(= 𝑎) subtends an angle of 60° at the foot of the tower, and the angle of elevation of the top of the tower from
𝐴 or 𝐵 is 30°. The height of the tower (in terms of 𝑎) is: (JEE Main 2007)

𝑅𝑎𝑑𝑖𝑢𝑠 = 𝑂𝐴 = 𝑂𝐵
Hence Δ𝐴𝑂𝐵 is isosceles.
∠𝐴𝑂𝐵 is 60°, Δ𝐴𝑂𝐵 is equilateral.

In Δ𝐴𝑂𝑃:

tan 30° =
𝑎
𝑎
ℎ = 𝑎 tan 30° =
√3

E. Single Tower (Different angles of elevation)

Example 1.133
Frodo is staring due North at Mt. Doom, which has height ℎ. Sam is staring due West at Mt. Doom. Frodo and
Sam are 𝑑 units apart, both have the same height above sea level and their angles of elevation to Mt. Doom are 𝛼
𝑑
and 𝛽 respectively. Show that the height of Mount Doom is .
√cot 𝛼 2 +cot 𝛽 2

P a g e 49 | 213
Get all the files at: https://bit.ly/azizhandouts
Aziz Manva (azizmanva@gmail.com)

Mt. Doom is due North of Frodo and due West


of Sam: we get a right triangle
(𝑙𝑒𝑓𝑡 𝑑𝑖𝑎𝑔𝑟𝑎𝑚 − 2𝐷).
Draw a 3D diagram (𝑟𝑖𝑔ℎ𝑡 𝑑𝑖𝑎𝑔𝑟𝑎𝑚)

𝐵𝐹
𝐼𝑛 Δ𝐷𝐵𝐹: cot 𝛼 = ⇒ 𝐵𝐹 = ℎ cot 𝛼

𝐵𝑆
𝐼𝑛 Δ𝐷𝐵𝑆: cot 𝛽 = ⇒ 𝐵𝑆 = ℎ cot 𝛽

By the Pythagorean Theorem in Δ𝐹𝐵𝑆


⏟2 cot 2 𝛼 + ⏟
ℎ ℎ2 cot2 𝛽 = 𝑑 2
𝑩𝑭𝟐 𝑩𝑺𝟐
𝑑
ℎ=
√cot2 𝛼 + cot 2 𝛽

Example 1.134
The angle of elevation of the top 𝑃 of a tower from the feet of one person standing due south of the tower is 45°
and from the feet of another person standing due west of the tower is 30°. If the height of the tower is 5𝑚, then
the distance in meters between the two persons is equal to: (JEE Main, April 11, 2023, Shift-II)

Draw a diagram. Let:


𝐹𝑜𝑜𝑡 𝑜𝑓 𝑡ℎ𝑒 𝑡𝑜𝑤𝑒𝑟 = 𝑂
𝑇𝑜𝑝 𝑜𝑓 𝑡ℎ𝑒 𝑡𝑜𝑤𝑒𝑟 = 𝑃
𝑃𝑒𝑟𝑠𝑜𝑛 𝑠𝑡𝑎𝑛𝑑𝑖𝑛𝑔 𝑑𝑢𝑒 𝑠𝑜𝑢𝑡ℎ 𝑏𝑒 𝑎𝑡 𝑝𝑜𝑖𝑛𝑡 𝐵
𝑃𝑒𝑟𝑠𝑜𝑛 𝑠𝑡𝑎𝑛𝑑𝑖𝑛𝑔 𝑑𝑢𝑒 𝑤𝑒𝑠𝑡 𝑏𝑒 𝑎𝑡 𝑝𝑜𝑖𝑛𝑡 𝐴
In Δ𝑃𝐵𝑂:
5 5 5
tan ∠𝑃𝐵𝑂 = ⇒ 𝑂𝐵 = = =5
𝑂𝐵 tan 45° 1
In Δ𝑃𝐴𝑂:
5 5 5
tan ∠𝑃𝐴𝑂 = ⇒ 𝑂𝐴 = = = 5√3
𝑂𝐴 tan 30° 1
√3
In Δ𝐴𝑂𝐵, by Pythagoras Theorem:
2
𝐴𝐵 = √𝑂𝐴2 + 𝑂𝐵2 = √52 + (5√3) = √25 + 75 = √100 = 10

Example 1.135
38: The angle of elevation of the top of a tower from a point 𝐴 due north of it is 𝛼 and from a point 𝐵 at a
3
distance of 9 units due west of A is cos−1 ( ). If the distance of the point B from the tower is 15 units, then
√13
cot 𝛼 is equal to: (JEE Main, July 29, 2022, Shift-I)

In Δ𝐵𝐴𝑂, by Pythagorean Triplet:


3(3,4,5) = (9,12,15) ⇒ 𝑂𝐴 = 12

P a g e 50 | 213
Get all the files at: https://bit.ly/azizhandouts
Aziz Manva (azizmanva@gmail.com)

3
Take the cos of both sides of 𝛽 = cos −1 ( ) and find tan 𝛽 using a reference triangle:
√13
3 2
cos 𝛽 = ⇒ tan 𝛽 =
√13 3

2 ℎ
𝐼𝑛 𝛥𝑃𝐵𝑂: tan 𝛽 = = ⇒ ℎ = 10
3 15
12 6
𝐼𝑛 𝛥𝑃𝑂𝐴: cot ∠𝑃𝐴𝑂 = cot 𝛼 = =
10 5

Challenge 1.136
A horizontal park is in the shape of a triangle 𝑂𝐴𝐵 with 𝐴𝐵 = 16. A vertical lamp post 𝑂𝑃 is erected at the point
O such that ∠𝑃𝐴𝑂 = ∠𝑃𝐵𝑂 = 15°, and ∠𝑃𝐶𝑂 = 45°, where 𝐶 is the midpoint of 𝐴𝐵. Then 𝑂𝑃2 is equal to: (JEE
Main, July 28, 2022, Shift-II)

In Δ𝑃𝐴𝑂 and Δ𝑃𝐵𝑂:


∠𝑃𝑂𝐴 = ∠𝑃𝑂𝐵 = 90°
∠𝑃𝐴𝑂 = ∠𝑃𝐵𝑂 = 15°
𝑃𝑂 = 𝑃𝑂
Δ𝑃𝐴𝑂 ≅ Δ𝑃𝐵𝑂 (𝑆𝐴𝐴)
𝑂𝐴 = 𝑂𝐵 (𝐶𝑃𝐶𝑇)
Hence:
Δ𝑂𝐴𝐵 𝑖𝑠 𝑖𝑠𝑜𝑐𝑒𝑙𝑒𝑠 ⇒ ∠𝑂𝐶𝐴 = 90°

In right Δ𝐴𝑂𝐶, by the Pythagorean Theorem:


𝑂𝐴2 = 𝑂𝐶 2 + 𝐴𝐶 2

𝑬𝒒𝒖𝒂𝒕𝒊𝒐𝒏 𝑰
Note that:
𝐼𝑛 Δ𝑃𝑂𝐶: ∠𝑃𝐶𝑂 = ∠𝑂𝑃𝐶 = 45° ⇒ ⏟
𝑂𝑃 = 𝑂𝐶
𝑬𝒒𝒖𝒂𝒕𝒊𝒐𝒏 𝑰𝑰
𝑂𝑃 𝑂𝑃
In Δ𝑃𝐴𝑂: tan 15° = ⇒ 𝑂𝐴 = = 𝑂𝑃 cot 15°
𝑂𝐴 ⏟ tan 15°
𝑬𝒒𝒖𝒂𝒕𝒊𝒐𝒏 𝑰𝑰𝑰
𝐴𝐶 2 = 64
𝐴𝐶 = 8 ⇒ ⏟
𝑬𝒒𝒖𝒂𝒕𝒊𝒐𝒏 𝑰𝑽
Substitute Equations II, III and IV in Equation I:
64
(𝑂𝑃 cot 15°)2 = 𝑂𝑃2 + 64 ⇒ 𝑂𝑃2 =
cot 2 15° − 1
2
Substitute cot2 15° − 1 = (2 + √3) − 1 = 4 + 4√3 + 3 − 1 = 6 + 4√3:
64 64 32 32(3 − 2√3) 32(2√3 − 3)
𝑂𝑃2 = = = = =
cot2 15° − 1 6 + 4√3 3 + 2√3 9 − 12 3

P a g e 51 | 213
Get all the files at: https://bit.ly/azizhandouts
Aziz Manva (azizmanva@gmail.com)

Pending

Example 1.137
54: ABC is a triangular park with 𝐴𝐵 = 𝐴𝐶 = 100 𝑚𝑒𝑡𝑒𝑟𝑠. A vertical tower is situated at the midpoint of BC. If
the angles of elevation of the top of the tower at A and B are cot −1 3√2 and csc −1 2√2 respectively, then the
height of the tower (in meters) is: (JEE Main 2018; Jan 10 2019, April 10, 2019)

F. Multiple Towers

Example 1.138
A surveyor wanting to find the width of a river plants two poles (of equal height) 𝑑 meters apart on one bank of
the river. The surveyor’s assistant standing directly opposite one of the poles, measures the angles of elevation
to the poles to be 𝛼° and 𝛽°, with 𝛼 > 𝛽. Determine the width 𝑤 of the river.

Combined Diagram
Let the poles be:
𝐴𝐴′ : 𝑊𝑖𝑡ℎ 𝐴 𝑡ℎ𝑒 𝑏𝑎𝑠𝑒 𝑎𝑛𝑑 𝐴′ 𝑡ℎ𝑒 𝑡𝑜𝑝
𝐵𝐵′ : 𝑊𝑖𝑡ℎ 𝐵 𝑡ℎ𝑒 𝑏𝑎𝑠𝑒 𝑎𝑛𝑑 𝐵′ 𝑡ℎ𝑒 𝑡𝑜𝑝

Draw a 3D diagram. Let


the height of each pole be ℎ.
the assistant be standing at point 𝑀.

In Δ𝑀𝐴𝐵, by the Pythagorean Theorem:


𝑀𝐵 = √𝑑2 + 𝑤 2

Split the Triangles



𝐹𝑟𝑜𝑚 Δ𝑀𝐴𝐴′ : tan 𝛼 = 𝑤 ⇒ ⏟
ℎ = 𝑤 tan 𝛼
𝑬𝒒𝒖𝒂𝒕𝒊𝒐𝒏 𝑰

𝐹𝑟𝑜𝑚 Δ𝑀𝐵𝐵′ : tan 𝛽 = ℎ = √𝑑2 + 𝑤 2 tan 𝛽
⇒⏟
√𝑑2 + 𝑤 2 𝑬𝒒𝒖𝒂𝒕𝒊𝒐𝒏 𝑰𝑰

From Equation I and II:


𝑤 tan 𝛼 = √𝑑 2 + 𝑤 2 tan 𝛽
Square both sides:
𝑤 2 tan2 𝛼 = (𝑑 2 + 𝑤 2 ) tan2 𝛽
𝑤 2 (tan2 𝛼 − tan2 𝛽) = 𝑑2 tan2 𝛽
tan2 𝛽 𝑑2 𝑑 tan 𝛽
𝑤=√ 2 2
=
tan 𝛼 − tan 𝛽 √tan2 𝛼 − tan2 𝛽

G. Cones

1.139: Making a Cone from a Sector

Example 1.140
A three-quarter sector of a circle of radius 4 inches together with its interior can be rolled up to form the lateral

P a g e 52 | 213
Get all the files at: https://bit.ly/azizhandouts
Aziz Manva (azizmanva@gmail.com)

surface area of a right circular cone by taping together along the two radii shown. What is the volume of the
cone in cubic inches? (AMC 10B 2020/10, AMC 12B 2020/9)

Example 1.141
Surface Area of a Cone

1.8 Bearings and Coordinate Geometry


A. Bearings

1.142: True Bearings


➢ True bearings give the direction from north.
➢ True bearings are always written using three digits.

1.143: Reverse Bearings


If the bearing from A to B is 𝛼, then the bearing from B to A is:
➢ 𝛼 + 180 if 0° ≤ 𝛼 < 180°
➢ 𝛼 − 180 if 180° ≤ 𝛼 < 360°

➢ Getting a reverse bearing means going the opposite direction.


➢ Hence, it is equivalent to a 180° turn.

Example 1.144
In each part below, draw a sketch, and find the reverse bearing. If more than one bearing is given, find the
reverse bearing for each.
A. The bearing from A to B is 042°. Sketch a diar
B. The bearing from A to B is 123°. The bearing from B to C is 22°. The bearing from C to A is 260°.

Example 1.145
A. A whaler leaves port 𝑃 at a bearing of 108°, and travels for 50 miles to point
𝑊, where it harpoons a whale. It then changes direction, and travels 120
miles at a bearing of 198° to reach 𝐴. Find the bearing to home port for the
whaler.
B. (𝐵𝑒𝑎𝑟𝑖𝑛𝑔𝑠 𝑇𝑦𝑝𝑒 𝐼)
C. (𝐵𝑒𝑎𝑟𝑖𝑛𝑔𝑠 𝑇𝑦𝑝𝑒 𝐼)

Draw a diagram.
By co-interior angles, the angle at point 𝐴 and the angle at point 𝑃 are supplementary:
𝑇ℎ𝑒 𝑎𝑛𝑔𝑙𝑒 𝑎𝑡 𝐴 = 180 − 108 = 72°
By angles around a point:
∠𝑃𝑊𝐴 = 360 − 72 − 198 = 90°

P a g e 53 | 213
Get all the files at: https://bit.ly/azizhandouts
Aziz Manva (azizmanva@gmail.com)

(5,12,13) is a Pythagorean triplet:


10(5,12,13) = (50,120,130)

50 5 5
sin 𝜃 = = ⇒ 𝜃 = sin−1 ≈ 22.62°
130 13 13
22.62 − 18 = 4.62
And finally, the bearing from 𝐴 to 𝑃 is:
360 − 4.62 = 355.38°

B. Coordinate Geometry

Example 1.146
The adjoining diagram shows a circle on the coordinate plane with its
center three units to the right and five units above the origin. F
Find the coordinates of points T, Q and P.

The coordinates of the center of the circle are (3,5)


𝑌
sin 𝜙 = ⇒ 𝑌 = 5 sin 𝜙 ⇒ 𝑃𝑦 = 5 sin 𝜙 + 5
5
𝑋
cos 𝜙 = ⇒ 𝑋 = 5 cos 𝜙 ⇒ 𝑃𝑥 = 5 cos 𝜙 + 3
5

𝑃(𝑥, 𝑦) = (5 cos 𝜙 + 3,5 sin 𝜙 + 5)


𝑄(𝑥, 𝑦) = (3 − 5 cos 𝜙 , 5 sin 𝜙 + 5)

C. Applications

Example 1.147: Applications


A. (𝐶ℎ𝑎𝑙𝑙𝑒𝑛𝑔𝑒) Distance between Parallel Lines

P a g e 54 | 213
Get all the files at: https://bit.ly/azizhandouts
Aziz Manva (azizmanva@gmail.com)

2. UNIT CIRCLE TRIGONOMETRY


2.1 Radians and the Unit Circle
A. Angles

2.1: Angles in Standard Position


An angle is in standard position when
➢ The 𝑣𝑒𝑟𝑡𝑒𝑥 is at the origin
➢ The initial rays forming the angle has its endpoint on the
𝑥-axis, and extends along the 𝑥-axis in the positive
direction

Example 2.2
Are the angles below in standard position. If not, explain why not?

Not in standard position because


A. none of the rays extends in the positive direction of the 𝑥 −axis.
B. the vertex is not at the origin.
C. The ray extends on 𝑥 − 𝑎𝑥𝑖𝑠, but not in the positive direction.

2.3: Rotation of Second Ray


For an angle in standard position, the terminal ray can be in any direction. However, the angle is measured using
a specific convention:
➢ Counter-Clockwise Rotation = Positive
➢ Clockwise Rotation = Negative

Example 2.4
In the adjoining diagram, the angle is shown to be 120° in the positive
direction. Calculate the value of the angle in the negative direction.

Going 120° in the positive direction is equivalent to going


360 − 120 = 240° 𝑖𝑛 𝑡ℎ𝑒 𝑛𝑒𝑔𝑎𝑡𝑖𝑣𝑒 𝑑𝑖𝑟𝑒𝑐𝑡𝑖𝑜𝑛

That is, the value of the angle is:


−240°

2.5: Positive and Negative Values


Any angle can be represented using both positive and negative values.

P a g e 55 | 213
Get all the files at: https://bit.ly/azizhandouts
Aziz Manva (azizmanva@gmail.com)

Example 2.6
Some angle values are given below. Convert them to positive if they are negative, and negative if they are
positive.
A. 30°
B. 150°
C. −50°
D. −110°

−(360 − 30) = −330


−(360 − 150) = −210
360 − 50 = 310
360 − 110 = 250

2.7: Co-terminal Angles


If two or more angles have the same terminal side, they are said to be co-terminal.
Co-terminal angles are obtained by adding or subtracting multiples of 360.

Example 2.8
Give a positive angle between 0 and 360 for:
A. −90°
B. −270°
C. −1000°
D. 1000°

−90° + 360 = 270°


−270 + 360 = 90°
−1000 + 360(3) = 80°
1000 − 360(2) = 280°

Example 2.9
Give a negative angle between 0 and −360 for:
A. 80°
B. 170°
C. 4000°

80 − 360 = −280°
170 − 360 = −190°
4000 − 360(12) = −320°

2.10: Quadrants and Angle Classification


The coordinate plane is divided into four quadrants (shown alongside).
An angle can be classified on the basis of the quadrant that its terminal ray lies in.
➢ 0 < 𝜃 < 90°: Quadrant I
➢ 90 < 𝜃 < 180°: Quadrant II
➢ 180 < 𝜃 < 270°: Quadrant III
➢ 270 < 𝜃 < 360°: Quadrant IV

P a g e 56 | 213
Get all the files at: https://bit.ly/azizhandouts
Aziz Manva (azizmanva@gmail.com)

Example 2.11
Classify the following diagrams on the basis of the quadrants that the angles lie in.

Example 2.12
Classify the following angles on the basis of the quadrants that the angles lie in.

2.13: Quadrants and Angle Classification


An angle can be classified on the basis of the quadrant that its terminal ray lies in using negative angles as well
➢ −360 < 𝜃 < −270°: Quadrant I
➢ −270 < 𝜃 < −180°: Quadrant II
➢ −180 < 𝜃 < −90°: Quadrant III
➢ −90 < 𝜃 < 0°: Quadrant IV

Example 2.14
Classify the following angles on the basis of the quadrants that the
angles lie in.

B. Unit Circle Trigonometry

2.15: Unit Circle


➢ A circle with radius 1 is called a unit circle.
➢ The circle is often drawn with its center at the origin.

2.16: Right Triangle Trigonometry


Right trigonometry defines the trigonometric ratios using a right triangle for 𝜃 between 0 and 90°. This is
correct, but we will now generalize.

2.17: Trigonometric Ratios in the Unit Circle


For any point on the Unit Circle, determine the trigonometric functions by
drawing a reference triangle.

➢ The reference triangle is always right-angled.


➢ The reference triangle goes in the 𝑥 direction, same as the 𝑥
coordinate of the point on the unit circle.
➢ The reference triangle goes in the 𝑦 direction, to the same extent as
the y coordinate of the point on the unit circle.
➢ The hypotenuse of the reference triangle is always drawn from the
origin to the unit circle. Hence, it always has length 1.

2.18: Coordinates of a Point


The coordinates of a point on the unit circle
= (𝑥, 𝑦) = (cos 𝜃 , sin 𝜃)
Where
𝜃 𝑖𝑠 𝑡ℎ𝑒 𝑎𝑛𝑔𝑙𝑒 𝑚𝑎𝑑𝑒 𝑏𝑦 𝑡ℎ𝑒 𝑟𝑎𝑑𝑖𝑢𝑠 𝑤𝑖𝑡ℎ 𝑡ℎ𝑒 𝑝𝑜𝑠𝑖𝑡𝑖𝑣𝑒 𝑥 − 𝑎𝑥𝑖𝑠

P a g e 57 | 213
Get all the files at: https://bit.ly/azizhandouts
Aziz Manva (azizmanva@gmail.com)

𝑇ℎ𝑒 𝑎𝑛𝑔𝑙𝑒 𝑖𝑠 𝑝𝑜𝑠𝑖𝑡𝑖𝑣𝑒 𝑖𝑛 𝑡ℎ𝑒 𝑐𝑜𝑢𝑛𝑡𝑒𝑟 − 𝑐𝑙𝑜𝑐𝑘𝑤𝑖𝑠𝑒 𝑑𝑖𝑟𝑒𝑐𝑡𝑖𝑜𝑛

opp 𝑦
sin 𝜃 = = =𝑦
hyp 1
adj 𝑥
cos 𝜃 = = =𝑥
hyp 1

Example 2.19
4 3
A point has coordinates ( , ) on the unit circle.
5 5
A. Draw the unit circle and the point on it.
B. Find the values of the six trigonometric functions

3
sin 𝜃 = 𝑦 =
5
4
cos 𝜃 = 𝑥 =
5
𝑦 0.6 6 3
tan 𝜃 = = = =
𝑥 0.8 8 4

2.20: Trigonometric Functions in any Quadrant


➢ To get the value of a trigonometric function in the first quadrant, you draw a reference triangle.
➢ You can draw a similar reference triangle for any quadrant that a point is in.

2.21: All Silver Tea Cups (Mnemonic) Silver All


In the first quadrant, ALL functions are positive. QII QI
In the second quadrant, only (SILVER) (sin 𝜃) is positive. Tea Cups
In the third quadrant, only (TEA) tan 𝜃 is positive. QIII QIV
In the fourth quadrant, only (CUPS) cos 𝜃 is positive.

(𝑥, 𝑦) = (cos 𝜃 , sin 𝜃)


➢ This mnemonic is useful for remembering the signs of the trigonometric functions based on the
quadrant they are in.

P a g e 58 | 213
Get all the files at: https://bit.ly/azizhandouts
Aziz Manva (azizmanva@gmail.com)

Example 2.22
A. In the second quadrant, which functions are positive?
B. cos 𝜃 is positive in which quadrants?

sin 𝜃
𝐼 𝑠𝑡 𝑎𝑛𝑑 𝐼𝑉 𝑡ℎ

2.23: Angles on the axes


cos 0 = 1, sin 0 = 0
cos 90° = 0, sin 90° = 1
cos 180° = −1, sin 180° = 0
cos 270° = 0, sin 270° = −1

➢ All Silver Tea Cups will not give you values for angles whose
terminal points lie on the axes.
➢ They have to picked up using the unit circle diagram alongside.

∠𝐴𝑂𝐴 = 𝜃1 = 0°
(𝑥, 𝑦) = (cos 𝜃1 , sin 𝜃1 ) = (cos 0° , sin 0°) = (1,0)
cos 0° = 1, sin 0° = 0

∠𝐵𝑂𝐴 = 𝜃2 = 90°
𝐵(𝑥,𝑦) = (cos 𝜃2 , sin 𝜃2 ) = (cos 90° , sin 90°) = (0,1)

∠𝐶𝑂𝐴 = 𝜃3 = 180°
𝐶(𝑥,𝑦) = (cos 𝜃3 , sin 𝜃3 ) = (cos 180° , sin 180°) = (−1,0)

∠𝐴𝑂𝐷 = 𝜃4 = 270°
𝐷(𝑥,𝑦) = (cos 𝜃4 , sin 𝜃4 ) = (cos 270° , sin 270°) = (0, −1)

C. Radians
Radians are a way of measuring angles. They are a dimensionless measure.

2.24: Radian Measure


Radians measure the number of times a radius will go around the arc of the degree
measure of the angle in a circle.

𝐶 = 2𝜋𝑟

Measure of Degrees Radians Measure of Angle Degrees Radians


Angle
Full Circle 360 2𝜋 One-Sixth of a Circle 60 𝜋
3
Half Circle 180 𝜋 One-Eighth of a Circle 45 𝜋
4
Quarter 90 𝜋 One-Twelfth of a 30 𝜋
Circle 2 Circle 6

P a g e 59 | 213
Get all the files at: https://bit.ly/azizhandouts
Aziz Manva (azizmanva@gmail.com)

Example 2.25
A. What unit are radians written in?
B. Is it necessary that radians be in terms of 𝜋.

Part A
Radians are a dimensionless measure. Hence, they do not have any units, and the word radian is usually not
written.
Unlike degrees, which are written using the ° symbol.

Part B
No.
180
1 𝑅𝑎𝑑𝑖𝑎𝑛 = ≈ 57.32 °
𝜋

2.26: Converting from Radians to Degrees


180 ° 180
𝑥 𝑅𝑎𝑑𝑖𝑎𝑛𝑠 = (𝑥 × ) ⇒ 𝐶𝑜𝑛𝑣𝑒𝑟𝑠𝑖𝑜𝑛 𝐹𝑎𝑐𝑡𝑜𝑟 =
𝜋 𝜋

Deriving the Conversion Factor


360 180
𝑂𝑛𝑒 𝐶𝑖𝑟𝑐𝑙𝑒 = 2𝜋 𝑅𝑎𝑑𝑖𝑎𝑛𝑠 ⇒ 360° = 2𝜋 𝑅𝑎𝑑𝑖𝑎𝑛𝑠 ⇒ 1 𝑅𝑎𝑑𝑖𝑎𝑛 = =
2𝜋 𝜋

Formula
180 ° 𝜋
𝑥 𝑅𝑎𝑑𝑖𝑎𝑛𝑠 = (𝑥 × ) ⇒ 𝑦° = 𝑦 × 𝑅𝑎𝑑𝑖𝑎𝑛𝑠
⏟ 𝜋 ⏟ 180
𝟏𝟖𝟎 𝝅
𝑪𝒐𝒏𝒗𝒆𝒓𝒔𝒊𝒐𝒏 𝑭𝒂𝒄𝒕𝒐𝒓= 𝑪𝒐𝒏𝒗𝒆𝒓𝒔𝒊𝒐𝒏 𝑭𝒂𝒄𝒕𝒐𝒓=
𝝅 𝟏𝟖𝟎
The conversion factor for converting from degrees to radians is simply the reciprocal of the conversion factor
from radians to degrees.

Example 2.27
Convert as directed:

𝜋
From Radians to E. From Degrees to O. 45°
4
Degrees 𝜋 Radians P. 120°
F.
A. 𝜋 6
2𝜋
J. 180° Q. 150°
B. 2𝜋 G. 3 K. 360° R. 140°
𝜋 5𝜋
C. 2 H. L. 30° S. 170°
6
𝜋 3𝜋 M. 90°
D. I.
3 4 N. 60°

𝜋 180
× = 60°
From Radians to Degrees 3 𝜋
180 𝜋 180
𝜋× = 180° × = 45°
𝜋 4 𝜋
180 𝜋 180
2𝜋 × = 360° × = 30°
𝜋 6 𝜋
𝜋 180 2𝜋 180
× = 90° × = 120°
2 𝜋 3 𝜋

P a g e 60 | 213
Get all the files at: https://bit.ly/azizhandouts
Aziz Manva (azizmanva@gmail.com)

5𝜋 180 𝜋 𝜋
× = 150° 60° × = 𝑅𝑎𝑑𝑖𝑎𝑛𝑠
6 𝜋 180 3
3𝜋 180 𝜋
× = 135° 45° =
4 𝜋 4
2𝜋
From Degrees to Radians 120° =
180° = 𝜋 3
5𝜋
360° = 2𝜋 150° =
𝜋 6
30° = 𝜋 7
6 140° × = 𝜋
𝜋 180 9
90° = 𝜋 17
2 170° × = 𝜋
180 18

2.28: Adding and Subtracting 𝟐𝝅


You can add or subtract a full circle which is 2𝜋 any number of times. This can be used to change the value of an
angle into an equivalent angle.

➢ This is mentioned before

Example 2.29
Convert to an angle in the range [0,2𝜋).
A. 3𝜋
𝜋
B. − 4
7𝜋
C. 2
5𝜋
D. − 3
13𝜋
E. 2

Part A Part B
We can convert to degrees, find the angle we are 𝜋 𝜋 8𝜋 7𝜋
− + 2𝜋 = − + =
looking for, and convert back to radians: 4 4 4 4
180 7𝜋 7𝜋 4𝜋 3𝜋
3𝜋 = 3𝜋 × = 540° = 180° = 𝜋 − 2𝜋 = − =
𝜋 2 2 2 2
Or we can directly work in radians: 5𝜋 5𝜋 6𝜋 𝜋
− + 2𝜋 = − + =
3𝜋 − 2𝜋 = 𝜋 3 3 3 3
The second method is so much shorter! Try to work 13𝜋 𝜋
= 6.5𝜋 = 6.5𝜋 − 6𝜋 = 0.5𝜋 =
directly in radians as much as possible. 2 2

Example 2.30
Supplementary Angles
Find the supplementary angles for:
𝜋
A. 3
𝜋
B. 2
𝜋
C.
4
𝜋
D. 5
Complementary Angles
Find the complementary angles for:

P a g e 61 | 213
Get all the files at: https://bit.ly/azizhandouts
Aziz Manva (azizmanva@gmail.com)
𝜋
E. 6
3𝜋
F. 8

Supplementary
𝜋 2𝜋
𝜋− =
3 3
𝜋 𝜋
𝜋− =
2 2
𝜋 3𝜋
𝜋− =
4 4
𝜋 4𝜋
𝜋− =
5 5
Complementary Angles
𝜋 𝜋 3𝜋 𝜋 2𝜋 𝜋
− = − = =
2 6 6 6 6 3
𝜋 3𝜋 4𝜋 3𝜋 𝜋
− = − =
2 8 8 8 8
D. Geometry in Radians

2.31: Area of a Sector


The area of a sector with radius 𝑟 and angle 𝜃
1
= 𝑟2𝜃
2

𝜃 𝜃 1
𝜋𝑟 2 ∙ = 𝜋𝑟 2 ∙ = 𝑟2𝜃
360 2𝜋 2

Example 2.32

2.33: Area of a Segment


The area of a segment of a circle with radius 𝑟 and angle 𝜃
1
= 𝑟 2 (𝜃 − sin 𝜃)
2

1 2 1
= 𝑟 𝜃 − 𝑟 2 sin 𝜃

2 ⏟
2
𝑆𝑒𝑐𝑡𝑜𝑟 𝑇𝑟𝑖𝑎𝑛𝑔𝑙𝑒
1
= 𝑟 2 (𝜃 − sin 𝜃)
2

Example 2.34
𝜋
Triangle 𝑂𝐵𝐶 has hypotenuse ℎ and ∠𝑂𝐵𝐶 = 2 . ∠𝑂𝐶𝐵 is 𝛼. 𝑂 is the
center of the given circle. 𝛽 is the angle subtended by arc 𝐵𝐷𝐴. Find the
area of minor segment 𝐵𝐷𝐴 in terms of the given variables.

𝑂𝐵
Substitute sin 𝛼 = ⇒ 𝑟 = 𝑂𝐵 = ℎ sin 𝛼 in the formula for the area of a sector

1 ℎ2 sin2 𝛼
= (ℎ sin 𝛼)2 (𝛽 − sin 𝛽) = (𝛽 − sin 𝛽)
2 2

P a g e 62 | 213
Get all the files at: https://bit.ly/azizhandouts
Aziz Manva (azizmanva@gmail.com)

2.35: Arc Length


The arc length of a arc of a circle sector with radius 𝑟 and angle 𝜃
𝑟𝜃

𝜃 𝜃
2𝜋𝑟 ∙ = 2𝜋𝑟 ∙ = 𝑟𝜃
360 2𝜋

2.2 Unit Circle Identities


A. Unit Circle Identities

2.36: Adding/Subtracting 𝟑𝟔𝟎°


sin(360°𝑛 + 𝜃) = sin 𝜃 , 𝑛 ∈ ℤ
cos(360°𝑛 + 𝜃) = cos 𝜃 , 𝑛 ∈ ℤ

Consider an angle 𝑥°. The angle does not change if you:


➢ Add a multiple of 360°
➢ Subtract a multiple of 360°

Example 2.37
Degrees
Find a comparable angle that measures between 0° and 360°, but represents the same angle.
A. −90°
B. 540°
C. 405°
D. 510°
E. −50°
F.

Degrees

Example 2.38
Radians
Find a comparable angle that measures between 0 and 2𝜋, but represents the same angle.
A. 3𝜋
B. 5𝜋
7𝜋
C. 2
𝜋
D. − 2
23𝜋
E. 6

Radians
7𝜋 3𝜋
− 2𝜋 =
2 2
𝜋 3𝜋
2𝜋 − =
2 2
23𝜋
6

P a g e 63 | 213
Get all the files at: https://bit.ly/azizhandouts
Aziz Manva (azizmanva@gmail.com)

Example 2.39
Given the measure of an angle, find three positive angles, and three negative angles that represent the same
angle.
A. 230°

230° + 360°
230° + 720°
230° + 1080°
230° − 360°
230° − 720°
230° − 1080°

2.40: Reflecting across the 𝒙 −axis


sin(−𝜃) = − sin 𝜃
cos(−𝜃) = cos 𝜃
tan(−𝜃) = − tan 𝜃

This property applies to angles in any quadrant.


But it is directly useful for converting angles in Quadrant IV
into angles in Quadrant I.

sin(−𝜃) − sin 𝜃
tan(−𝜃) = = = − tan 𝜃
cos(−𝜃) cos 𝜃

Example 2.41
Simplify:

Degrees E. csc(−45°)
A. sin(−30°) F. tan(−30°)
B. cos (−30°) G. sin 330°
C. csc(−60°)
D. sec(−45°)

Degrees
1 1 1 1
sin −30° = − sin 30° = − csc −45° = = = = −√2
2 sin −45 − sin 45 − 1
√3 √2
cos −30° = cos 30° = 1
2 tan −30 = − tan 30 = −
1 1 1 2 √30
csc −60° = = = =−
sin −60 −sin 60 √3 √3 sin 330° = sin −30°
− 2
1 1 1
sec −45° = = = = √2
cos −45 cos 45 1
√2

Example 2.42
Simplify:

P a g e 64 | 213
Get all the files at: https://bit.ly/azizhandouts
Aziz Manva (azizmanva@gmail.com)

Radians
𝜋
A. sin (− )
6
𝜋
B. cos (− 4 )
𝜋
C. tan (− 3 )
5𝜋
D. sin ( )
3
15𝜋
E. tan 4

Radians
𝜋 𝜋
sin (− ) = − sin
6 6
𝜋 1
cos (− ) = − cos 45° = −
4 √2
𝜋
tan (− ) = − tan(60°) = −√3
3
5𝜋 𝜋 √3
sin = sin (− ) = − sin(60°) = −
3 3 2
15𝜋 𝜋
tan = tan (− ) = − tan 45° = −1
4 4

2.43: Reflecting across the 𝒚 −axis


sin(180° − 𝜃) = sin 𝜃
cos(180° − 𝜃) = − cos 𝜃
tan(180 − 𝜃) = − tan 𝜃
This property is directly useful for converting angles in Quadrant II into angles in Quadrant I.

Let the point A have coordinates


(𝑥, 𝑦)
Let
∠𝐶𝑂𝐵 = 𝜃 ⇒ sin 𝜃 = 𝑦, cos 𝜃 = 𝑥
∠𝐵𝑂𝐷 = 180° − 𝜃 ⇒ sin(180 − 𝜃) = 𝑦
Hence,
sin 𝜃 = sin(180 − 𝜃)
cos(180° − 𝜃) = − cos 𝜃
sin(180 − 𝜃) sin 𝜃
tan(180 − 𝜃) = = = − tan 𝜃
cos(180 − 𝜃) − cos 𝜃

Example 2.44
Simplify:

Degrees F. sec 120°


A. sin 150° G. csc 150°
B. cos 120° Radians
C. sin 135° 2𝜋
H. sin 3
D. sin 120 5𝜋
E. tan 150° I. tan
6

P a g e 65 | 213
Get all the files at: https://bit.ly/azizhandouts
Aziz Manva (azizmanva@gmail.com)
3𝜋 17𝜋
J. cos 4
K. cos 6

Degrees 1 1 1
cot 120 = = =−
1 tan 120 − tan 60 √3
sin 150° = sin(180 − 30) = sin 30° =
2 Radians
1 2𝜋 2𝜋 𝜋
cos 120° = cos(180 − 60) = − cos 60° = − √3
2 sin = sin (𝜋 − ) = sin ( ) = sin 60° =
1 3 3 3 2
sin 135° = sin(180 − 45) = sin 45 = 5𝜋 5𝜋 𝜋 1
√2 tan = tan (𝜋 − ) = tan = tan 30° =
6 6 6 √3
√3 3𝜋 3𝜋 1
sin 120° = sin(180 − 60) = sin 60 = cos = − cos (𝜋 − ) = − cos(45°) =
2 4 4 √2
1
tan 150° = − tan 30° = − 17𝜋 5𝜋 5𝜋
√3 cos = cos = − cos (𝜋 − )
6 6 6
1 1 1
sec 120° = = = = −2 𝜋 √3
cos 120° − cos 60° − 1 = − cos = − cos 30° = −
2 6 2
1 1 1
csc 150° = = = =2
sin 150 sin 30 1
2

Example 2.45
Let 𝛼 and 𝛽 be supplementary angles. Determine and prove the relationship between 𝐴 and 𝐵 if:
𝐴 = sin 𝛼 cos 𝛽 , 𝐵 = cos 𝛼 sin 𝛽

Since the angles are supplementary:


𝛼+𝛽 =𝜋 ⇒𝛼 =𝜋−𝛽

𝐴 = sin(𝜋 − 𝛽) cos 𝛽 = sin 𝛽 cos 𝛽


𝐵 = cos(𝜋 − 𝛽) sin 𝛽 = − cos 𝛽 sin 𝛽 = −𝐴

2.46: Reflecting across the origin


sin(180° + 𝜃) = − sin 𝜃
cos(180° + 𝜃) = − cos 𝜃
tan(180° + 𝜃) = tan 𝜃

This property is directly for converting angles in Quadrant


III into angles in Quadrant I.

sin(180° + 𝜃) − sin 𝜃 sin 𝜃


tan(180° + 𝜃) = = = = tan 𝜃
cos(180° + 𝜃) − cos 𝜃 cos 𝜃

Example 2.47
Simplify:

Degrees Radians
A. sin 225° 5𝜋
D. sin 4
B. cos 240°
C. cos 210°
P a g e 66 | 213
Degrees Radians
1 5𝜋 𝜋 𝜋 √2
sin 225° = sin(180 + 45) = − sin 45° = − sin = sin (𝜋 + ) = − sin ( ) = −
√2 4 4 4 2
cos(240) = cos(180 + 60) = − cos 60° =

2.48: Cofunction Identities


𝜋
sin ( − 𝜃) = cos 𝜃
2
𝜋
cos ( − 𝜃) = sin 𝜃
2

B. Using the Identities

Example 2.49
1 1 1
tan 15° + + + tan 195° = 2𝑎, then (𝑎 + ) is: (𝐉𝐄𝐄 𝐌𝐚𝐢𝐧, 𝐉𝐚𝐧 𝟑𝟎, 𝟐𝟎𝟐𝟑 − 𝐈)
tan 75° tan 105° 𝑎

Write your answer as a single real number.

1 1 1
= =−
tan 105° − tan(180° − 105°) tan 75°
tan 195° = tan(180 + 15) = tan 15°

1 1
tan 15° + − + tan 15° = 2𝑎
tan 75° tan 75°
2 tan 15° = 2𝑎
𝑎 = tan 15° = 2 − √3
1 1 2 + √3 2 + √3
= × =
𝑎 2 − √3 2 + √3 1

1
𝑎+ = 2 − √3 + 2 + √3 = 4
𝑎
Get all the files at: https://bit.ly/azizhandouts
Aziz Manva (azizmanva@gmail.com)

C. More Identities

2.50: Adding multiples of 𝝅


When adding integers multiples of 𝜋 to an angle, the sine and cosine are either the original angle or the negative
of the original angle:

For 𝑛 ∈ ℤ:
sin(𝑛𝜋 + 𝜃) = (−1)𝑛 sin 𝜃
cos(𝑛𝜋 + 𝜃) = (−1)𝑛 cos 𝜃

We consider cases. 𝑛 can be even or odd.

Case I: 𝒏 is even
𝑛 is of the form 2𝑘, 𝑘 ∈ ℤ
𝐿𝐻𝑆 = sin(2𝑘𝜋 + 𝜃) = sin 𝜃 = (−1)2𝑘 sin 𝜃 = (−1)𝑛 sin 𝜃 = 𝑅𝐻𝑆

Case II: 𝒏 is odd


𝑛 is of the form 2𝑘 + 1, 𝑘 ∈ ℤ
𝐿𝐻𝑆 = sin((2𝑘 + 1)𝜋 + 𝜃) = sin(2𝑘𝜋 + 𝜋 + 𝜃) = sin(𝜋 + 𝜃) = − sin 𝜃 = (−1)2𝑘+1 sin 𝜃 = (−1)𝑛 sin 𝜃 = 𝑅𝐻𝑆

D. Applications

Example 2.51
The valve cap on a bicycle tire is 𝑎 units away from the center of the wheel. The wheel has an outer radius of 𝑏
units, with 𝑎 < 𝑏. The line drawn from the center of the wheel to the valve cap exactly points in the positive 𝑥
direction. The bicycle is pedaled so that wheel turns through an angle 𝜃.
A. Show that the height of the valve cap from the ground is 𝑏 + 𝑎 sin 𝜃.
B.

The left blue segment is simply:


(𝑏 − 𝑎) + 𝑎 = 𝑏
The right blue segment is:
𝑦
sin 𝜃 = ⇒ 𝑦 = 𝑎 sin 𝜃
𝑎
And hence, the height of the valve cap from the ground is:
𝑏 + 𝑎 sin 𝜃

2.52: Length of Arc

𝐶𝑖𝑟𝑐𝑢𝑚𝑓𝑒𝑟𝑒𝑛𝑐𝑒 = 2𝜋𝑟
𝜃 𝜃 𝜃
𝐿𝑒𝑛𝑔𝑡ℎ 𝑜𝑓 𝐴𝑟𝑐 = 𝐶 × = 2𝜋𝑟 × = 𝜋𝑟 ×
360 360 180

𝜃 𝜃
𝐿𝑒𝑛𝑔𝑡ℎ 𝑜𝑓 𝐴𝑟𝑐 = 𝐶 × = 2𝜋𝑟 × = 𝑟𝜃
2𝜋 2𝜋

2.53: Area of a Sector

P a g e 68 | 213
Get all the files at: https://bit.ly/azizhandouts
Aziz Manva (azizmanva@gmail.com)

𝜃
⏟2 ×
𝐴𝑟𝑒𝑎 𝑜𝑓 𝑎 𝑆𝑒𝑐𝑡𝑜𝑟 = 𝜋𝑟
𝑨𝒓𝒆𝒂 𝒐𝒇

360
𝒂 𝑪𝒊𝒓𝒄𝒍𝒆 𝑷𝒓𝒐𝒑𝒐𝒓𝒕𝒊𝒐𝒏𝒂𝒍𝒊𝒕𝒚
𝑪𝒐𝒏𝒔𝒕𝒂𝒏𝒕
2
𝑟 𝜃
𝐴𝑟𝑒𝑎 𝑜𝑓 𝑎 𝑆𝑒𝑐𝑡𝑜𝑟 =
2

Example 2.54
Find the length of arc, and the area of a sector of a circle with radius 10, and central angle 18°.

Length of Arc
𝜃 18°
𝐿𝑒𝑛𝑔𝑡ℎ 𝑜𝑓 𝑎𝑟𝑐 = 𝜋𝑟 × = 𝜋(10) × =𝜋
180 180
𝜋 𝜋
18° = ⇒ 𝐿𝑒𝑛𝑔𝑡ℎ 𝑜𝑓 𝑎𝑟𝑐 = 𝑟𝜃 = 10 × =𝜋
10 10
Area of Sector
𝜃 18°
𝐴𝑟𝑒𝑎 𝑜𝑓 𝑎 𝑆𝑒𝑐𝑡𝑜𝑟 = 𝜋𝑟 2 × = 𝜋(10)2 × = 5𝜋
360 360
2 𝜋
𝑟 2 𝜃 (10) (10)
𝐴𝑟𝑒𝑎 𝑜𝑓 𝑎 𝑆𝑒𝑐𝑡𝑜𝑟 = = = 5𝜋
2 2

2.55: Angular Speed


𝜃
𝐴𝑛𝑔𝑢𝑙𝑎𝑟 𝑠𝑝𝑒𝑒𝑑 = 𝜔 =
𝑡

Example 2.56
A. Ferris Wheel
B. Carousel
C. Spinning Top
D. Rotation of the Earth

2.3 Law of Sines


A. Law of Sines
The law of sines relates the sine of three angles of a triangle to their side lengths. The longest side is opposite
the largest angle, and the shortest side is opposite the shortest angle.

2.57: Solution of Triangles


Solution of triangles refers to determining the lengths of the sides of a triangle, and the measures of the angles
of the triangle.

A triangle can be solved using:


➢ Law of Sines
➢ Law of Cosines

P a g e 69 | 213
Get all the files at: https://bit.ly/azizhandouts
Aziz Manva (azizmanva@gmail.com)

2.58: Law of Sines


𝑎 𝑏 𝑐
= =
sin 𝐴 sin 𝐵 𝑆𝑖𝑛 𝐶

Calculate the area of the triangle in three different ways:


1 1 1
𝑎𝑏 sin 𝐶 = 𝑎𝑐 sin 𝐵 = 𝑏𝑐 sin 𝐴
2 2 2

Equate the second and the third part:


𝑎 𝑏
𝑎𝑐 sin 𝐵 = 𝑏𝑐 sin 𝐴 ⇒ =

sin 𝐴 sin 𝐵
𝑬𝒒𝒖𝒂𝒕𝒊𝒐𝒏 𝑰
Equate the first and the second part:
𝑏 𝑐
𝑎𝑏 sin 𝐶 = 𝑎𝑐 sin 𝐵 ⇒ =

sin 𝐵 sin 𝐶
𝑬𝒒𝒖𝒂𝒕𝒊𝒐𝒏 𝑰𝑰
Combine Equation I and II into a tripartite equality:
𝑎 𝑏 𝑐
= =
sin 𝐴 sin 𝐵 sin 𝐶

Example 2.59
The angles of a triangle are 50°, 60° and 70°. The longest side of the triangle has length 3 meters. Find a
trigonometric expression for the length of the other two sides.

𝑎 𝑏 3
= =
sin 50° √3 𝑆𝑖𝑛 70°
2

𝑏 3 3√3
= ⇒𝑏=
√3 𝑆𝑖𝑛 70° 2 ∙ 𝑆𝑖𝑛 70°
2
3 sin 50°
𝑎=
𝑆𝑖𝑛 70°

(Calc) Example 2.60


The angles of a triangle measure 104, 51 and 25 degrees. The perimeter of the triangle is 10m. Find, rounded to
two decimal places, the length of each side of the triangle.

Let the sides of the triangle have lengths 𝑎, 𝑏, 𝑐. Introduce a constant 𝑘 such that:
sin 104 sin 51 sin 25
= = =𝑘
𝑎 𝑏 𝑐

Solve for 𝑎, 𝑏, 𝑐:
sin 104 sin 51 sin 25
𝑎= , 𝑏= , 𝑐=
𝑘 𝑘 𝑘

Since the perimeter of the triangle is 10, we must have:


𝑎 + 𝑏 + 𝑐 = 10

Substitute the values of 𝑎, 𝑏 and 𝑐 from above:

P a g e 70 | 213
Get all the files at: https://bit.ly/azizhandouts
Aziz Manva (azizmanva@gmail.com)

sin 104 sin 51 sin 25


+ + = 10
𝑘 𝑘 𝑘

Using a calculator, solve for 𝑘 to get:


𝑘 = 0.217

Calculate the side lengths of the triangle:


sin 104 sin 104
𝑎= = = 4.47
𝑘 0.217

Example 2.61: Trigonometric Equations


Triangle 𝐴𝐵𝐶 has 𝐴𝐵 = 2 ⋅ 𝐴𝐶. Let 𝐷 and 𝐸 be on 𝐴𝐵 and 𝐵𝐶, respectively, such that ∠𝐵𝐴𝐸 = ∠𝐴𝐶𝐷. Let 𝐹 be
the intersection of segments 𝐴𝐸 and 𝐶𝐷, and suppose that △ 𝐶𝐹𝐸 is equilateral. What is ∠𝐴𝐶𝐵? (AMC 10A
2010/14)

Angle Chasing
Let
∠𝐴𝐶𝐷 = ∠𝐵𝐴𝐸 = 𝑥
By Angles in a Linear Pair:
∠𝐴𝐹𝐶 = 180 − 60 = 120
By sum of angles in a triangle:
∠𝐶𝐴𝐹 = 180 − 120 − 𝑥 = 60 − 𝑥
By Adjacent Angles:
∠𝐶𝐴𝐵 = ∠𝐶𝐴𝐸 + ∠𝐸𝐴𝐵 = 60 − 𝑥 + 𝑥 = 60
By sum of angles in a triangle:
∠𝐴𝐵𝐶 = 180 − (60 + 𝑥) − 60 = 60 − 𝑥

Law of Sines
Trigonometric Equations

Example 2.62: True Bearings


A ship leaves port and travels on a bearing of 034° for 100 km. It then changes course, and sails for 150 km at a
bearing of 015°. Find the distance and the bearing of the port from the ship.

Example 2.63: True Bearings


A hunter finds deer tracks outside his camp at a bearing of 240° 𝑇. He follows the track for two hours before
spotting boar tracks at a bearing of 140° 𝑇 from his then position. He switches over to tracking the boar, which
he does for three hours. When he finally catches up with the boar, he notes that his camp is 2 km away at a
bearing of 0° 𝑇. Find the difference in the distance between the hunter's return path from the boar kill, and the
path to the boar kill. Also, find the ratio of speeds when tracking boar and deer respectively.

Let A be the starting point. Then B is the point where the hunter switches from tracking deer to tracking boar,
and C is the point of the boar kill.

∠𝐵𝐴𝐶 = 60°

P a g e 71 | 213
Get all the files at: https://bit.ly/azizhandouts
Aziz Manva (azizmanva@gmail.com)

Draw 𝐷𝐵 ∥ 𝐴𝐶. By alternate interior angles:


∠𝐷𝐵𝐴 = ∠𝐵𝐴𝐶 = 60°
∠𝐴𝐵𝐶 = 140° − 60° = 80°
∠𝐵𝐶𝐴 = 180 − 80 − 60 = 40°
By the Law of Sines, in Δ𝐴𝐵𝐶:
2 𝑎 𝑐
= =
sin 80 sin 60 sin 40
2
𝐷𝑖𝑠𝑡𝑎𝑛𝑐𝑒 𝑇𝑟𝑎𝑐𝑘𝑖𝑛𝑔 𝐵𝑜𝑎𝑟 = 𝑎 = (sin 60) ( ) = 1.75
sin 80
2
𝐷𝑖𝑠𝑡𝑎𝑛𝑐𝑒 𝑇𝑟𝑎𝑐𝑘𝑖𝑛𝑔 𝐷𝑒𝑒𝑟 = 𝑐 = (sin 40) ( ) = 1.31
sin 80
2
𝑎 + 𝑐 = (sin 60 + sin 40) ( ) = 3.06
sin 80

Difference
= 3.06 − 2 = 1.06

We can find the ratio as:


𝑎 𝑏 1.75 1.31
𝑇𝑟𝑎𝑐𝑘𝑖𝑛𝑔 𝐵𝑜𝑎𝑟𝑑: 𝑇𝑟𝑎𝑐𝑘𝑖𝑛𝑔 𝐷𝑒𝑒𝑟 = : = : = 10.5: 7.86 = 1050: 786 = 525: 393
3 2 3 2

Example 2.64: True Bearings


Long John’s Silver’s treasure map shows a treasure chest 1200 meters inland at a bearing of 132° 𝑇 from a point
on an inaccessible cliff. A beach at a bearing of 63° 𝑇 from the cliff is 1700 meters from the treasure chest. Find
the direction in which he must go after reaching the beach.

Draw a diagram with


𝐶 = 𝐶𝑙𝑖𝑓𝑓, 𝑇 = 𝑇𝑟𝑒𝑎𝑠𝑢𝑟𝑒, 𝐵 = 𝐵𝑒𝑎𝑐ℎ
∠𝐵𝐶𝑇 = 27 + 42 = 69°

sin 69 sin 𝐵 12 sin 69


= ⇒ 𝐵 = sin−1 ( ) = 41.22°
1700 1200 17

By angles around point 𝐵:


∠𝐹𝐵𝑇 = 360 − 90 − 27 − 41.22 = 201.78°

Example 2.65: Compass Bearings

2.66: Extended Law of Sines


𝒂 𝒏 𝒄
= = = 𝟐𝑹
𝐬𝐢𝐧 𝑨 𝐬𝐢𝐧 𝑩 𝑺𝒊𝒏 𝑪

2.67: Length of Circumradius: Trigonometric Proof


𝒂
= 𝟐𝑹
𝐬𝐢𝐧 𝑨

P a g e 72 | 213
Get all the files at: https://bit.ly/azizhandouts
Aziz Manva (azizmanva@gmail.com)

𝑎 𝑎𝑏𝑐 𝑎𝑏𝑐 𝑎𝑏𝑐


𝑅= = = =
2 sin 𝐴 2𝑏𝑐 sin 𝐴 2 × 2 (1 𝑏𝑐 sin 𝐴) 4Δ
2

Challenge 2.68
Olympiad Problem from Putnam.
Requires some Calculus at the very end

2.4 Law of Cosines


A. Basics
The law of cosines is a generalization of the Pythagoras Theorem.

2.69: Law of Cosines


The law of cosines lets us find the length of the third side of a triangle if we know two sides and the included
angle.
𝑎2 = 𝑏 2 + 𝑐 2 − 2𝑏𝑐 cos 𝐴

We consider two cases:


Case I: The altitude of the triangle is inside the base
In right Δ𝐵𝐷𝐶:
𝑎2 = ℎ2 + (𝑏 − 𝑟)2 = ℎ2 + 𝑏 2 − 2𝑏𝑟 + 𝑟 2

Substitute

sin 𝐴 = ⇒ ℎ = 𝑐 sin 𝐴 ⇒ ℎ2 = 𝑐 2 sin2 𝐴
𝑐
𝑟
cos 𝐴 = ⇒ 𝑟 = 𝑐 cos 𝐴 ⇒ 𝑟 2 = 𝑐 2 cos2 𝐴
𝑐

𝑎2 = 𝑐 2 sin2 𝐴 + 𝑏 2 − 2𝑏𝑐 cos 𝐴 + 𝑐 2 cos2 𝐴

Factor 𝑐 2 from the first and last term in the RHS:


𝑎2 = 𝑐 2 (sin2 𝐴 + cos 2 𝐴) + 𝑏 2 − 2𝑏𝑐 cos 𝐴

Substitute sin2 𝐴 + cos2 𝐴 = 1:


𝑎2 = 𝑐 2 + 𝑏 2 − 2𝑏𝑐 cos 𝐴

Case II: The altitude of the triangle is outside the base


By Pythagoras Theorem in right Δ𝐵𝐶𝐷:
𝑎2 = 𝑟 2 + ℎ2

𝑬𝒒𝒖𝒂𝒕𝒊𝒐𝒏 𝑰

We want to eliminate 𝑟 and ℎ, and introduce 𝑏 and 𝑐.


By Pythagoras Theorem in right Δ𝐵𝐷𝐴:
(𝑏 + 𝑟)2 + ℎ2 = 𝑐 2
ℎ2 = 𝑐 2 − (𝑏 + 𝑟)2
ℎ2 = 𝑐 2 − 𝑏 2 − 2𝑏𝑟 − 𝑟 2
ℎ2 + 𝑟 2 = 𝑐 2 − 𝑏 2 − 2𝑏𝑟

𝑬𝒒𝒖𝒂𝒕𝒊𝒐𝒏 𝑰𝑰

P a g e 73 | 213
Get all the files at: https://bit.ly/azizhandouts
Aziz Manva (azizmanva@gmail.com)

From Equations I and II:


𝑎2 = 𝑐 2 − 𝑏 2 − 2𝑏𝑟

𝑏+𝑟
Substitute cos 𝐴 = ⇒ 𝑏 + 𝑟 = 𝑐 cos 𝐴 ⇒ 𝑟 = 𝑐 cos 𝐴 − 𝑏
𝑐
𝑎2 = 𝑐 2 − 𝑏 2 − 2𝑏(𝑐 cos 𝐴 − 𝑏)
𝑎2 = 𝑐 2 − 𝑏 2 − 2𝑏𝑐 cos 𝐴 + 2𝑏 2
𝑎2 = 𝑐 2 + 𝑏 2 − 2𝑏𝑐 cos 𝐴

2.70: Law of Cosines: Alternate Version


Equivalently, if we know the three sides of a triangle, the law of cosines lets us find the cosine of any angle of the
triangle.
𝑏 2 + 𝑐 2 − 𝑎2
cos 𝐴 =
2𝑏𝑐

𝑎2 = 𝑏 2 + 𝑐 2 − 2𝑏𝑐 cos 𝐴

Take cos 𝐴 to the LHS, and the 𝑎2 term to the RHS:


2𝑏𝑐 cos 𝐴 = 𝑏 2 + 𝑐 2 − 𝑎2

Divide both sides by 2𝑏𝑐:


𝑏 2 + 𝑐 2 − 𝑎2
cos 𝐴 =
2𝑏𝑐

Example 2.71
Show that the Pythagorean Theorem is a special case of the law of cosines.

𝑎2 = 𝑏 2 + 𝑐 2 − 2𝑏𝑐 cos 𝐴

The Pythagorean Theorem is applicable to right triangles, which are triangles with a right angle.
Let ∠𝐴 = 90° and substitute in the law of cosines

𝑎2 = 𝑏 2 + 𝑐 2 − 2𝑏𝑐 cos 90°

Substitute cos 90° = 0


𝑎2 = 𝑏 2 + 𝑐 2 − 2𝑏𝑐 ∙ 0
𝑎2 = 𝑏 2 + 𝑐 2

Example 2.72
The base of a triangle is 80, and one of the base angles is 60°. The sum of the lengths of the other two sides is
90. The shortest side is: (AHSME 1959/36)

P a g e 74 | 213
Get all the files at: https://bit.ly/azizhandouts
Aziz Manva (azizmanva@gmail.com)

Let the sides of the triangle be:


80, 𝑥, 90 − 𝑥

By the Law of Cosines:


(90 − 𝑥)2 = 𝑥 2 + 802 − 2(𝑥)(80) cos 60°

Expand
1
8100 − 180𝑥 + 𝑥 2 = 𝑥 2 + 6400 − 160𝑥 ( )
2
1700 = 100𝑥
𝑥 = 17
90 − 𝑥 = 73

2.73: Classifying triangles using Law of Cosines

Example 2.74
AHSME

B. Working with Triangles

Example 2.75: Finding Sides


A. Find the third side of a triangle with sides 7 and 15 and included angle 60 degrees.
B. One side of a triangular garden is 10 meters long. The side adjacent to it is 5 meters long. The angle
between the two sides is 45°. Find the third side of the garden.

Part A
𝑎2 = 𝑏 2 + 𝑐 2 − 2𝑏𝑐 cos 𝐴

Substitute 𝑏 = 7, 𝑐 = 15, 𝐴 = 60°:


1
𝑎2 = 72 + 152 − (2)(7)(15)(cos 60) = 49 + 225 − 2 ∙ 7 ∙ 15 ∙ = 169
2
𝑎 = 13

P a g e 75 | 213
Get all the files at: https://bit.ly/azizhandouts
Aziz Manva (azizmanva@gmail.com)

Part B
𝑎2 = 𝑏 2 + 𝑐 2 − 2𝑏𝑐 cos 𝐴

Substitute 𝑏 = 5, 𝑐 = 10, 𝐴 = 45°:


√2
𝑎2 = 52 + 102 − (2)(5)(10)(cos 45°) = 125 − 100 = 125 − 50√2
2

Take the square root both sides:

𝑎 = √125 − 50√2 = √25(5 − 2√2) = 5√5 − 2√2

Example 2.76: Finding Sides


Arun is at the beach. He throws his frisbee and it lands 7 meters away. Arun’s dog is chasing a wave at the edge
of the ocean, twelve meters aways. If the angle between his dog and his frisbee is 30° for Arun, then how much
does his dog have to run to get the frisbee back to Arun.

𝑎2 = 𝑏 2 + 𝑐 2 − 2𝑏𝑐 cos 𝐴

Substitute 𝑏 = 7, 𝑐 = 12, 𝐴 = 30°:


𝑎2 = 72 + 122 − (2)(7)(12)(cos 30)
√3
= 49 + 144 − 2 ∙ 7 ∙ 12 ∙
2
= 193 − 84√3

𝑎 = √193 − 84√3

The distance to be run by the dog

= 7 + 𝑎 = 7 + √193 − 84√3

Example 2.77: Finding Angles


A. A right triangle has hypotenuse 17, and one side 15. Find the ratio of the other two angles in the
triangle.

Since it is a Pythagorean Triplet, the third side of the triangle must be 8.


𝑎2 = 𝑏 2 + 𝑐 2 − 2𝑏𝑐 cos 𝐴
15 = 82 + 172 − 2(8)(17)(cos 𝐴)
2

272(cos 𝐴) = 128
128
𝐴 = cos−1 ( )
272

(Calc) Example 2.78: Multiple Triangles


Looking down from a 62-meter-high tower, you can see point 𝐴 with an angle of depression of 24.3°. By
swinging the telescope 76.51° in a horizontal arc, you can see point 𝐵 with an angle of depression of 30°.
Calculate the distance 𝐴𝐵 if both points 𝐴 and 𝐵 have the same height above mean sea level.

P a g e 76 | 213
Get all the files at: https://bit.ly/azizhandouts
Aziz Manva (azizmanva@gmail.com)

Let
𝐵𝑎𝑠𝑒 𝑜𝑓 𝑇𝑜𝑤𝑒𝑟 = 𝑄, 𝑇𝑜𝑝 𝑜𝑓 𝑇𝑜𝑤𝑒𝑟 = 𝑃
Find the angles we need:
∠𝐴𝑃𝑄 = 90 − 24.3 = 65.7°
∠𝐵𝑃𝑄 = 90 − 30 = 60°
In Δ𝐴𝑃𝑄:
𝐴𝑄
tan 65.7° = ⇒ 𝐴𝑄 = 62 tan 65.7°
62
In Δ𝐵𝑃𝑄:
𝐵𝑄
tan 60° = ⇒ 𝐵𝑄 = 62 tan 60° = 62√3
62
By the Law of Cosines in Δ𝐴𝑄𝐵:
𝐴𝐵 = √𝐴𝑄 2 + 𝐵𝑄 2 + 2(𝐴𝑄)(𝐵𝑄)(cos 76.51°) ≈ 153.322 ≈ 153

C. Quadrilaterals

Example 2.79
Find 𝑃𝑄 given that 𝐴𝐵 = 245 𝑚𝑒𝑡𝑒𝑟, ∠𝑃𝐴𝐵 = 114°, ∠𝑄𝐴𝐵 = 32.5°, ∠𝐴𝐵𝑄 = 107°, ∠𝐴𝐵𝑃 = 37°.

First, find the angles we need:


∠𝑃𝑄𝐵 = 107⏟ − 37
⏟ = 70
∠𝐴𝐵𝑄 ∠𝐴𝐵𝑃
∠𝐵𝑄𝐴 = 180 − 32.5
⏟ − 107
⏟ = 40.5
∠𝑄𝐴𝐵 ∠𝐴𝐵𝑄

Apply the Law of Sines twice:


245 𝐵𝑄 245 × sin 32.5°
Δ𝐴𝐵𝑄: = ⇒ 𝐵𝑄 =
sin 40.5° sin 32.5° sin 40.5°

245 𝐵𝑃 245 × sin 114°


Δ𝐴𝐵𝑃: = ⇒ 𝐵𝑃 =
sin 29° sin 114° sin 29°
By the Law of Cosines in Δ𝑃𝐵𝑄:
𝑃𝑄 = √𝐵𝑃2 + 𝐵𝑄 2 + 2(𝐵𝑃)(𝐵𝑃)(cos 70°) ≈ 436.126 ≈ 436

D. Polygons

Example 2.80: Hexagons


Equiangular hexagon 𝐴𝐵𝐶𝐷𝐸𝐹 has side lengths 𝐴𝐵 = 𝐶𝐷 = 𝐸𝐹 = 1 and 𝐵𝐶 = 𝐷𝐸 = 𝐹𝐴 = 𝑟. The area of Δ𝐴𝐶𝐸
is 70% of the area of the hexagon. What is the sum of all possible values of 𝑟? (AMC 10A 2010/19)3

3 For a geometric solution, refer the Note on Polygons in Geometry.

P a g e 77 | 213
Get all the files at: https://bit.ly/azizhandouts
Aziz Manva (azizmanva@gmail.com)

𝐴(Δ𝐶𝐸𝐷) + 𝐴(Δ𝐴𝐹𝐸) + 𝐴(Δ𝐴𝐵𝐶)


= 100%
⏟ − 70%
⏟ = 30% 𝑜𝑓 𝐴(𝐴𝐵𝐶𝐷𝐸𝐹)
𝐴(𝐴𝐵𝐶𝐷𝐸𝐹) 𝐴(Δ𝐴𝐶𝐸)

Each exterior angle of 𝐴𝐵𝐶𝐷𝐸𝐹


360
= = 60
6

Hence, Δ𝐶𝐸𝐷 ≅ Δ𝐴𝐹𝐸 ≅ Δ𝐴𝐵𝐶 by SAS Theorem since each triangle has
𝑠1 = 1, 𝑠2 = 𝑟, 𝐼𝑛𝑐𝑙𝑢𝑑𝑒𝑑 𝑎𝑛𝑔𝑙𝑒 = 180 − 60° = 120°

Therefore,
3𝐴(Δ𝐶𝐸𝐷) = 30% ⇒ 𝐴(𝐶𝐸𝐷) = 10% 𝐴(𝐴𝐵𝐶𝐷𝐸𝐹)

By CPCT:
𝐴𝐶 ≅ 𝐶𝐸 ≅ 𝐴𝐸 ⇒ Δ𝐴𝐶𝐸 𝑖𝑠 𝑒𝑞𝑢𝑖𝑙𝑎𝑡𝑒𝑟𝑎𝑙
By the trigonometric formula for area of a triangle:
1 √3
𝐴(𝐶𝐸𝐷) = (1)(𝑟)(sin 120°) = 𝑟
2 4
By the law of cosines in Δ𝐶𝐸𝐷:
𝐶𝐸 2 = 12 + 𝑟 2 − 2(1)(𝑟)(cos 120°) = 𝑟 2 + 𝑟 + 1
By the formula for area of an equilateral triangle:
√3 2 √3 2
𝐴(𝐴𝐶𝐸) = 𝑠 = (𝑟 + 𝑟 + 1)
4 4

But, 𝐴(𝐴𝐶𝐸)
⏟ =7× 𝐴(𝐶𝐸𝐷)
⏟ . Substituting:
70% 𝑜𝑓 𝐴(𝐴𝐵𝐶𝐷𝐸𝐹) 10% 𝑜𝑓 𝐴(𝐴𝐵𝐶𝐷𝐸𝐹)
√3 2 √3
(𝑟 + 𝑟 + 1) = 7 𝑟
4 4
𝑟 2 + 𝑟 + 1 = 7𝑟
𝑟 2 − 6𝑟 + 1 = 0
By Vieta’s Formulas, the sum of the roots of the above quadratic is:
𝑏 −6
− =− =6
𝑎 1

E. Circles

Example 2.81: Circles


A round table has radius 4. Six rectangular place mats are placed on the table. Each
place mat has width 1 and length x as shown. They are positioned so that each mat has
two corners on the edge of the table, these two corners being end points of the same
side of length 𝑥. Further, the mats are positioned so that the inner corners each touch
an inner corner of an adjacent mat. What is 𝑥? (AMC 10A 2008/25)4

4 For a geometric solution, refer the Note on Polygons in Geometry.

P a g e 78 | 213
Get all the files at: https://bit.ly/azizhandouts
Aziz Manva (azizmanva@gmail.com)

Let the circle have center 𝑂.


𝑂𝐷 = 𝑅𝑎𝑑𝑖𝑢𝑠 = 4

BC forms one side of a regular hexagon.


Δ𝐵𝐶𝑂 𝑖𝑠 𝑒𝑞𝑢𝑖𝑙𝑎𝑡𝑒𝑟𝑎𝑙 ⇒ 𝐵𝐶 = 𝑂𝐶 = 𝑥

In Δ𝐷𝐶𝑂:
∠𝑂𝐶𝐷 = 60°
⏟ + 90°
⏟ = 150°
∠𝑂𝐶𝐵 ∠𝐵𝐶𝐷
𝐿𝑎𝑤 𝑜𝑓 𝐶𝑜𝑠𝑖𝑛𝑒𝑠 ⇒ 42 = 12 + 𝑥 2 − 2(1)(𝑥)(cos 150°)
√3
16 = 1 + 𝑥 2 − 2(𝑥) (− )
2
𝑥 2 + √3𝑥 − 15 = 0

Apply the quadratic formula with 𝑎 = 1, 𝑏 = √3, 𝑐 = −15


−√3 ± √3 − (4)(1)(−15) −√3 ± √63 −√3 ± 3√7
𝑥= = =
2 2 2
3√7 − √3

⏟ 2
𝑃𝑜𝑠𝑖𝑡𝑖𝑣𝑒 𝑉𝑎𝑙𝑢𝑒

Example 2.82
Circle 𝐶 with radius 2 has diameter 𝐴𝐵. Circle 𝐷 is internally tangent to circle 𝐶 at 𝐴. Circle 𝐸 is internally
tangent to circle 𝐶, externally tangent to circle 𝐷, and tangent to 𝐴𝐵. The radius of circle 𝐷 is three times the
radius of circle 𝐸, and can be written in the form √𝑚 − 𝑛, where 𝑚 and 𝑛 are positive integers. Find 𝑚 + 𝑛.
(AIME II 2014/8)5

Example 2.83: TSD


Two skaters, Allie and Billie, are at points A and B, respectively, on a flat, frozen lake. The distance between A
and B is 100 meters. Allie leaves A and skates at a speed of 8 meters per second on a straight line that makes a
60-degree angle with AB. At the same time Allie leaves A, Billie leaves B at a speed of 7 meters per second and
follows the straight path that produces the earliest possible meeting of the two skaters, given their speeds. How
many meters does Allie skate before meeting Billie? (AIME 1989/6)

2.84: Brahmagupta’s Formula: Area of a Cyclic Quadrilateral


If a cyclic quadrilateral has sides 𝑎, 𝑏, 𝑐 and 𝑑, then the area of the quadrilateral is given by
𝑎+𝑏+𝑐+𝑑
√(𝑠 − 𝑎)(𝑠 − 𝑏)(𝑠 − 𝑐)(𝑠 − 𝑑), 𝑠=
2

5 Also solved using Descartes’ Theorem in that section. And solved with the Pythagorean Theorem in the Note on Triangles.

P a g e 79 | 213
Get all the files at: https://bit.ly/azizhandouts
Aziz Manva (azizmanva@gmail.com)

1 1 1 1
𝑑𝑏 sin 𝜃 + 𝑎𝑐 sin 𝜃 + 𝑎𝑑 sin(180 − 𝜃) + 𝑐𝑏 sin(180 − 𝜃)
2 2 2 2

Substitute sin(180 − 𝜃) = sin 𝜃:


1 1 1 1
𝑑𝑏 sin 𝜃 + 𝑎𝑐 sin 𝜃 + 𝑎𝑑 sin(180 − 𝜃) + 𝑐𝑏 sin(180 − 𝜃)
2 2 2 2

2.5 Geometry
A. Altitude

2.85: Altitude Basics


➢ The line segment drawn from the vertex of a triangle perpendicular to the opposite side is an altitude of
the triangle.
➢ Every triangle has three altitudes

2.86: Altitude Concurrency


The three altitudes of a triangle are concurrent at the orthocenter of
the triangle.

In the diagram alongside


𝐴𝑙𝑡𝑖𝑡𝑢𝑑𝑒𝑠 𝑎𝑟𝑒 𝐴𝐷, 𝐵𝐸, 𝐶𝐹

The point of concurrency of the three altitudes


= 𝑂𝑟𝑡ℎ𝑜𝑐𝑒𝑛𝑡𝑒𝑟 = 𝐻

2.87: Orthocenter dividing altitude


In Δ𝐴𝐵𝐶 with orthocenter 𝐻, the ratio in which the orthocenter divides the altitude is:
𝐴𝐻 cos 𝛼
=
𝐻𝐷 cos 𝛽 cos 𝛾

Step I: Angle Chasing with Geometry


In right Δ𝐴𝐻𝐸:
∠𝐴𝐻𝐸 = 90 − ∠𝐷𝐴𝐶 = 90 − (90 − 𝛾) = 𝛾
By vertically opposite angles:
∠𝐵𝐻𝐷 = ∠𝐴𝐻𝐸 = 𝛾
Step II: Trigonometry
𝐴𝐸
Substitute Δ𝐴𝐵𝐸: cos 𝛼 = 𝐴𝐵 ⇒ 𝐴𝐸 = 𝐴𝐵 cos 𝛼 in:
𝐴𝐸 𝐴𝐸 𝐴𝐵 cos 𝛼
Δ𝐴𝐻𝐸: sin 𝛾 = ⇒ 𝐴𝐻 = =
𝐴𝐻 ⏟ sin 𝛾 sin 𝛾
𝑬𝒒𝒖𝒂𝒕𝒊𝒐𝒏 𝑰
𝐵𝐷
Substitute Δ𝐴𝐵𝐷: cos 𝛽 = ⇒ 𝐵𝐷 = 𝐴𝐵 cos 𝛽 in:
𝐴𝐵
𝐵𝐷 𝐵𝐷
Δ𝐵𝐻𝐷 tan 𝛾 = ⇒ 𝐻𝐷 = = 𝐴𝐵 cos 𝛽 cot 𝛾
𝐻𝐷 ⏟ tan 𝛾
𝑬𝒒𝒖𝒂𝒕𝒊𝒐𝒏 𝑰𝑰

From Equations I and II:

P a g e 80 | 213
Get all the files at: https://bit.ly/azizhandouts
Aziz Manva (azizmanva@gmail.com)

𝐴𝐵 cos 𝛼
𝐴𝐻 sin 𝛾 𝐴𝐵 cos 𝛼 1 cos 𝛼
= = × =
𝐻𝐷 𝐴𝐵 cos 𝛽 cot 𝛾 sin 𝛾 𝐴𝐵 cos 𝛽 cot 𝛾 cos 𝛽 ∙ cos 𝛾

Example 2.88
The sides of a triangle are of lengths 13,14 and 15. The altitudes of the triangle meet at point 𝐻. If 𝐴𝐷 is the
altitude to the side of length 14, the ratio 𝐻𝐷: 𝐻𝐴 is (AHSME 1964/35)

𝑏 2 + 𝑐 2 − 𝑎2
𝐴𝐻 cos 𝛼 2𝑏𝑐
= =
𝐻𝐷 cos 𝛽 cos 𝛾 𝑎2 + 𝑐 2 − 𝑏 2 𝑎2 + 𝑏 2 − 𝑐 2
×
2𝑎𝑐 2𝑎𝑏

Convert division into multiplication:


𝑏 2 + 𝑐 2 − 𝑎2 2𝑎𝑐 2𝑎𝑏
= × 2 ×
2𝑏𝑐 𝑎 + 𝑐 2 − 𝑏 2 𝑎2 + 𝑏 2 − 𝑐 2

Simplify and rearrange:


2𝑎2 (𝑏 2 + 𝑐 2 − 𝑎2 )
= 2
(𝑎 + 𝑐 2 − 𝑏 2 )(𝑎2 + 𝑏 2 − 𝑐 2 )

Substitute 𝑎 = 14, 𝑏 = 15, 𝑐 = 13:


2(142 )(225 + 169 − 196) 2(142 )(198) 2(14)(198) (198) 11
= = = = =
(196 + 169 − 225)(196 + 225 − 169) (140)(252) (10)(252) (5)(18) 5

We need:
𝐻𝐷 𝐴𝐻 5
= 𝑅𝑒𝑐𝑖𝑝𝑟𝑜𝑐𝑎𝑙 𝑜𝑓 =
𝐴𝐷 𝐻𝐷 11

P a g e 81 | 213
Get all the files at: https://bit.ly/azizhandouts
Aziz Manva (azizmanva@gmail.com)

3. GRAPHS AND EQUATIONS


3.1 Basic Graphs
A. 𝒔𝒊𝒏 𝒙

Example 3.1
From the unit circle, write:
A. sin 0
𝜋
B. sin 2
C. sin 𝜋
3𝜋
D. sin 2

sin 0 =0
𝜋
sin = 1
2
sin 𝜋 = 0
3𝜋
sin = −1
2

Example 3.2
Graph sin 𝑥.

Example 3.3
Using the graph of sin 𝑥 identify its:
A. Domain
B. Range
C. Period
D. Zeroes
E. Maxima
F. Minima

P a g e 82 | 213
Get all the files at: https://bit.ly/azizhandouts
Aziz Manva (azizmanva@gmail.com)

Part A
Domain is the set of all valid inputs for the function. All real numbers are valid inputs for sin 𝑥. Hence, the
domain is
𝑥∈ℝ

Part B
Range is the set of valid outputs for the function. The range of sin 𝑥 is:
[−1,1]

Part C
Period is where the function repeats itself. The function has a period
2𝜋

Part D
Zeroes of the function are the places where the function intersects the 𝑥 axis.
{… , −2𝜋, −𝜋, 0, 𝜋, 2𝜋, … } ⇒ 𝑛𝜋, 𝑛 ∈ ℤ

Any integer multiple of 𝜋 is a zero.

Part E
The maxima are the values where sin 𝑥 takes its maximum value. The maximum value is
𝑦=1
Which is obtained when:
𝜋
𝑥 ∈ { + 2𝑛𝜋, 𝑛 ∈ ℤ}
2
Part E
The minima are the values where sin 𝑥 takes its minimum value. The minimum value is
𝑦 = −1
Which is obtained when:
3𝜋
𝑥 ∈ { + 2𝑛𝜋, 𝑛 ∈ ℤ}
2

3.4: Summary: 𝐬𝐢𝐧 𝒙


𝐷𝑜𝑚𝑎𝑖𝑛: ℝ
𝑅𝑎𝑛𝑔𝑒: [−1,1]
𝑃𝑒𝑟𝑖𝑜𝑑: 2𝜋
𝑛𝜋, 𝑛 ∈ ℤ
𝐴𝑚𝑝𝑙𝑖𝑡𝑢𝑑𝑒: 1
𝜋
𝑀𝑎𝑥𝑖𝑚𝑎: 𝑦 = 1 ⇔ 𝑥 ∈ { + 2𝑛𝜋, 𝑛 ∈ ℤ}
2
3𝜋
𝑀𝑖𝑛𝑖𝑚𝑎: 𝑦 = −1 ⇔ 𝑥 ∈ { + 2𝑛𝜋, 𝑛 ∈ ℤ}
2

B. 𝒄𝒐𝒔 𝒙

Example 3.5
From the unit circle, write:

P a g e 83 | 213
Get all the files at: https://bit.ly/azizhandouts
Aziz Manva (azizmanva@gmail.com)

A. cos 0
𝜋
B. cos 2
C. cos 𝜋
3𝜋
D. cos 2

cos 0 = 1
𝜋
cos = 0
2
cos 𝜋 = −1
3𝜋
cos =0
2

Example 3.6
Graph cos 𝑥.

3.7: 𝐜𝐨𝐬 𝒙 as a transformation of 𝒔𝒊𝒏 𝒙


The graph of cos 𝑥 is obtained by shifting the graph of sin 𝑥 to the left. Hence
𝜋
cos 𝑥 = sin (𝑥 + )
2
𝜋
sin 𝑥 = cos (𝑥 − )
2

Example 3.8
Using the graph of cos 𝑥 identify its:
A. Domain
B. Range
C. Period
D. Zeroes

P a g e 84 | 213
Get all the files at: https://bit.ly/azizhandouts
Aziz Manva (azizmanva@gmail.com)

E. Maxima
F. Minima

Part A
Domain is the set of all valid inputs for the function. All real numbers are valid inputs for sin 𝑥. Hence, the
domain is
𝑥∈ℝ

Part B
Range is the set of valid outputs for the function. The range of sin 𝑥 is:
[−1,1]

Part C
Period is where the function repeats itself. The function has a period
2𝜋

Part D
Zeroes of the function are the places where the function intersects the 𝑥 axis.
𝜋
{ + 𝑛𝜋, 𝑛 ∈ ℤ}
2

Any integer multiple of 𝜋 is a zero.

Part E
The maxima are the values where sin 𝑥 takes its maximum value. The maximum value is
𝑦=1
Which is obtained when:
𝑥 ∈ 2𝑛𝜋, 𝑛 ∈ ℤ
Part E
The minima are the values where sin 𝑥 takes its minimum value. The minimum value is
𝑦 = −1
Which is obtained when:
𝑥 ∈ {𝜋 + 2𝑛𝜋, 𝑛 ∈ ℤ}

3.9: Summary of 𝐜𝐨𝐬 𝒙


𝐷𝑜𝑚𝑎𝑖𝑛: ℝ
𝑅𝑎𝑛𝑔𝑒: [−1,1]
𝑃𝑒𝑟𝑖𝑜𝑑: 2𝜋
𝜋
𝑍𝑒𝑟𝑜𝑒𝑠: { + 𝑛𝜋, 𝑛 ∈ ℤ}
2
𝐴𝑚𝑝𝑙𝑖𝑡𝑢𝑑𝑒: 1
𝑀𝑎𝑥𝑖𝑚𝑎: 𝑦 = 1 ⇔ 𝑥 ∈ 2𝑛𝜋, 𝑛 ∈ ℤ
𝑀𝑖𝑛𝑖𝑚𝑎: 𝑦 = −1 ⇔ 𝑥 ∈ 𝜋 + 2𝑛𝜋, 𝑛 ∈ ℤ

P a g e 85 | 213
Get all the files at: https://bit.ly/azizhandouts
Aziz Manva (azizmanva@gmail.com)

C. 𝒕𝒂𝒏 𝒙

3.10: 𝐭𝐚𝐧 𝒙
𝜋
𝐷𝑜𝑚𝑎𝑖𝑛: ℝ − {𝑛𝜋 + , 𝑛 ∈ ℤ}
2
𝑅𝑎𝑛𝑔𝑒: ℝ
𝑃𝑒𝑟𝑖𝑜𝑑: 𝜋
𝑍𝑒𝑟𝑜𝑒𝑠: 𝑛𝜋, 𝑛 ∈ ℤ
𝑀𝑎𝑥𝑖𝑚𝑢𝑚: 𝑁𝑜 𝑚𝑎𝑥𝑖𝑚𝑢𝑚
𝑀𝑖𝑛𝑖𝑚𝑢𝑚: 𝑁𝑜 𝑚𝑖𝑛𝑖𝑚𝑢𝑚

D. 𝒔𝒆𝒄 𝒙

𝟏
3.11: 𝐬𝐞𝐜 𝒙 =
𝐜𝐨𝐬 𝒙
𝜋
𝐷𝑜𝑚𝑎𝑖𝑛: ℝ − {𝑛𝜋 + , 𝑛 ∈ ℤ}
2
𝑅𝑎𝑛𝑔𝑒: (−∞, −1] ∪ [1, ∞)

3.2 Equations: Basics


A. Useful Properties

P a g e 86 | 213
Get all the files at: https://bit.ly/azizhandouts
Aziz Manva (azizmanva@gmail.com)

3.12: Adding/Subtracting 𝟑𝟔𝟎°


sin(360°𝑛 + 𝜃) = sin 𝜃 , 𝑛 ∈ ℤ
cos(360°𝑛 + 𝜃) = cos 𝜃 , 𝑛 ∈ ℤ

Consider an angle 𝑥°. The angle does not change if you:


➢ Add a multiple of 360°
➢ Subtract a multiple of 360°

3.13: Reflecting across the 𝒙 −axis


sin(−𝜃) = − sin 𝜃
cos(−𝜃) = cos 𝜃
tan(−𝜃) = − tan 𝜃

This property applies to angles in any quadrant.


But it is directly useful for converting angles in Quadrant IV into
angles in Quadrant I.

sin(−𝜃) − sin 𝜃
tan(−𝜃) = = = − tan 𝜃
cos(−𝜃) cos 𝜃

3.14: Reflecting across the 𝒚 −axis


sin(180° − 𝜃) = sin 𝜃
cos(180° − 𝜃) = − cos 𝜃
tan(180 − 𝜃) = − tan 𝜃
This property is directly useful for converting angles in Quadrant II into angles in Quadrant I.

Let the point A have coordinates


(𝑥, 𝑦)
Let
∠𝐶𝑂𝐵 = 𝜃 ⇒ sin 𝜃 = 𝑦, cos 𝜃 = 𝑥
∠𝐵𝑂𝐷 = 180° − 𝜃 ⇒ sin(180 − 𝜃) = 𝑦
Hence,
sin 𝜃 = sin(180 − 𝜃)
cos(180° − 𝜃) = − cos 𝜃
sin(180 − 𝜃) sin 𝜃
tan(180 − 𝜃) = = = − tan 𝜃
cos(180 − 𝜃) − cos 𝜃

3.15: Reflecting across the origin


sin(180° + 𝜃) = − sin 𝜃
cos(180° + 𝜃) = − cos 𝜃
tan(180° + 𝜃) = tan 𝜃

This property is directly for converting angles in Quadrant III


into angles in Quadrant I.

sin(180° + 𝜃) − sin 𝜃 sin 𝜃


tan(180° + 𝜃) = = = = tan 𝜃
cos(180° + 𝜃) − cos 𝜃 cos 𝜃

P a g e 87 | 213
Get all the files at: https://bit.ly/azizhandouts
Aziz Manva (azizmanva@gmail.com)

B. Number of Solutions with 𝒔𝒊𝒏 𝒙


Basic equations focus more on expressions involving sin 𝑥 and cos 𝑥. the other four trigonometric functions are
also important, but occur with less frequency.

3.16: Number of Solutions


Consider the solutions of the equation below in the domain
𝜃 ∈ [0,2𝜋), 𝜃 ∈ ℝ:
sin 𝜃 = 𝑐, cos 𝜃 = 𝑐
We consider three distinct cases:
➢ The above equation has two solutions in for −1 <
𝑐<1
➢ It has exactly one solution for 𝑐 = ±1
➢ If 𝑐 > 1 or 𝑐 < −1, then there are no solutions6

Example 3.17
Consider the solutions of the equation below in the domain 𝜃 ∈ [0,2𝜋), 𝜃 ∈ ℝ:
sin 𝜃 = 𝑐, cos 𝜃 = 𝑐
Explain why if 𝑐 > 1 or 𝑐 < −1, then there are no real solutions
The range of 𝑦 = sin 𝑥 is
[−1,1]

If 𝑐 > 1, the blue line 𝑦 = 𝑐 is always above the graph of


𝑦 = sin 𝑥
This means that it never intersects the graph.
Which means there are no solutions.

If 𝑐 < −1, the purple line 𝑦 = 𝑐 is always below the graph of


𝑦 = sin 𝑥
This means that it never intersects the graph.
Which means there are no solutions.

Hence, if 𝑐 > 1 or 𝑐 < −1, there are no real solutions.

Example 3.18
Consider the solutions of the equation below in the domain 𝜃 ∈ [0,2𝜋), 𝜃 ∈ ℝ:
sin 𝜃 = 𝑐
Explain why the above equation has exactly one solution for 𝑐 = ±1.

If sin 𝑥 = 1, the line 𝑦 = 1 intersects the graph of 𝑦 = sin 𝑥 at


𝜋
𝑥=
2
If sin 𝑥 = −1, the line 𝑦 = −1 intersects the graph of 𝑦 = sin 𝑥 at
3𝜋
𝑥=
2
From the graph we can see that these are the only possible solutions.

6 There can be complex solutions.

P a g e 88 | 213
Get all the files at: https://bit.ly/azizhandouts
Aziz Manva (azizmanva@gmail.com)

Example 3.19
Consider the solutions of the equation below in the domain 𝜃 ∈ [0,2𝜋), 𝜃 ∈ ℝ:
sin 𝜃 = 𝑐
The above equation has two solutions for −1 < 𝑐 < 0. Which quadrants do these lie in?

When −1 < 𝑐 < 0, there are two solutions, shown by the places
where the line intersects the graph.
One solution lies in the third quadrant, and one solution lies in
the fourth quadrant.

Example 3.20
Consider the solutions of the equation below in the domain 𝜃 ∈ [0,2𝜋), 𝜃 ∈ ℝ:
sin 𝜃 = 𝑐1
The above equation has two solutions for 0 < 𝑐 < 1. Which quadrants do these lie in?

When 0 < 𝑐 < 1 there are two solutions, shown by the


places where the line intersects the graph.
One solution lies in the first quadrant, and one solution
lies in the second quadrant.

Example 3.21
Consider the following “solution” below to an equation. Without using a calculator, explain the step where the
mistake is and why.
𝑆𝑡𝑒𝑝 𝐼: sin 𝑥 = 0.78
5𝜋 6𝜋
𝑆𝑡𝑒𝑝 𝐼𝐼: 𝑥 ∈ { , }
4 4

0 < 0.78 < 1, and hence the solutions to it must lie in the first and the second quadrant.
However:
5𝜋 3𝜋
𝜋< < ⇒ 3𝑟𝑑 𝑄𝑢𝑎𝑑𝑟𝑎𝑛𝑡 ⇒ 𝐼𝑛𝑣𝑎𝑙𝑖𝑑 𝑆𝑜𝑙𝑢𝑡𝑖𝑜𝑛
4 2

6𝜋 3𝜋
= ⇒ 𝑦 − 𝑎𝑥𝑖𝑠 𝑏𝑜𝑡𝑡𝑜𝑚 𝑣𝑎𝑙𝑢𝑒 ⇒ 𝐼𝑛𝑣𝑎𝑙𝑖𝑑 𝑆𝑜𝑙𝑢𝑡𝑖𝑜𝑛
4 2

Example 3.22
Without solving the equations below, identify the number of solutions to the equation, and the quadrants those
solutions will lie in. If the solution does not lie in a quadrant, mention the value of 𝑥 in [0,2𝜋].
1
A. sin 𝑥 = 3
2
B. sin 𝑥 = − 3
C. sin 𝑥 = √1
D. sin 𝑥 = √8
E. sin 𝑥 = −1

P a g e 89 | 213
Get all the files at: https://bit.ly/azizhandouts
Aziz Manva (azizmanva@gmail.com)

F. sin 𝑥 = 0

Part A
1
0< < 1 ⇒ 𝑄𝐼, 𝑄𝐼𝐼
3
Part B
2
−1 < − < 0 ⇒ 𝑄𝐼𝐼𝐼, 𝑄𝐼𝑉
3
Part C
𝜋
𝑥=
2
Part D
𝑁𝑜 𝑆𝑜𝑙𝑢𝑡𝑖𝑜𝑛𝑠
Part E
3𝜋
𝑥=
2
Part F
3 𝑆𝑜𝑙𝑢𝑡𝑖𝑜𝑛𝑠 ⇒ 𝑥 ∈ {0, 𝜋, 2𝜋}
C. Equations with 𝒔𝒊𝒏 𝒙

3.23: All Silver Tea Cups (Mnemonic)


In the first quadrant, ALL functions are positive. Silver All
In the second quadrant, only (SILVER) (sin 𝜃) is positive. QII QI
In the third quadrant, only (TEA) tan 𝜃 is positive. Tea Cups
In the fourth quadrant, only (CUPS) cos 𝜃 is positive.
QIII QIV

3.24: Reflecting across the 𝒚 −axis


sin(180° − 𝜃) = sin 𝜃
This property is directly useful for converting angles in Quadrant II into angles in Quadrant I and vice versa

Let the point A have coordinates


(𝑥, 𝑦)
Let
∠𝐶𝑂𝐵 = 𝜃 ⇒ sin 𝜃 = 𝑦, cos 𝜃 = 𝑥
∠𝐵𝑂𝐷 = 180° − 𝜃 ⇒ sin(180 − 𝜃) = 𝑦
Hence,
sin 𝜃 = sin(180 − 𝜃)

Example 3.25
Solve for the variable. In each case, the domain of interest is [0,2𝜋).
1
sin 𝜃 =
2

P a g e 90 | 213
Get all the files at: https://bit.ly/azizhandouts
Aziz Manva (azizmanva@gmail.com)

First check where the solutions are:


1
sin 𝜃 = > 0
2
sin 𝜃 > 0 𝑖𝑛 𝑄𝐼 𝑎𝑛𝑑 𝑄𝐼𝐼
𝑆𝑜𝑙𝑢𝑡𝑖𝑜𝑛𝑠 𝑎𝑟𝑒 𝑖𝑛 𝑄𝐼 𝑎𝑛𝑑 𝑄𝐼𝐼.
By looking at the equation, we know that a first quadrant
solution is:
𝜋
𝜃1 =
6
Find the second quadrant solution
Note that sin 𝜃 remains unchanged after the reflection across the 𝑦 axis (sin(𝜋 − 𝜃) = sin 𝜃):
𝜋 5𝜋 𝜋 5𝜋
𝜃2 = 𝜋 − = ⇒𝜃∈{ , }
6 6 6 6

Example 3.26
Solve for the variable. In each case, the domain of interest is [0,2𝜋).
√3
sin 𝛾 =
2

Since sin 𝛾 is positive, we must have solutions in


𝑄𝐼 𝑎𝑛𝑑 𝑄𝐼𝐼
First Quadrant Solution
Take the sin inverse of both sides to get the first quadrant
solution:
√3 𝜋
𝛾1 = sin−1 =
2 3
Find the second quadrant solution
Note that sin 𝜃 remains unchanged after the reflection
across the 𝑦 axis (sin(𝜋 − 𝜃) = sin 𝜃):
𝜋 2𝜋 𝜋 2𝜋
𝛾2 = 𝜋 − = ⇒𝛾∈{ , }
3 3 3 3

Example 3.27
Solve for the variable. In each case, the domain of interest is [0,2𝜋).
sin 𝜙 = 𝑐, 0 < 𝑐 < 1

Take the sin inverse of both sides to get the first quadrant solution:
𝜙1 = sin−1 𝑐
Find the second quadrant solution
Note that sin 𝜃 remains unchanged after the reflection across the 𝑦 axis (sin(𝜋 − 𝜃) = sin 𝜃):
𝜙2 = 𝜋 − sin−1 𝑐 ⇒ 𝜙 ∈ {sin−1 𝑐 , 𝜋 − sin−1 𝑐}

3.28: Negative Values


If you have a negative value of sin 𝑥, the solutions will be in third and fourth quadrants. However, begin by
finding a first quadrant solution for the equivalent positive value, and then convert it to third and fourth
quadrant solutions.

P a g e 91 | 213
Get all the files at: https://bit.ly/azizhandouts
Aziz Manva (azizmanva@gmail.com)

3.29: Reflecting across the 𝒙 −axis


sin(−𝜃) = − sin 𝜃

This property applies to angles in any quadrant.


But it is directly useful for converting angles in Quadrant IV into
angles in Quadrant I.

3.30: Reflecting across the origin


sin(180° + 𝜃) = − sin 𝜃

Example 3.31
Solve for the variable. In each case, the domain of interest is [0,2𝜋).
√3
sin 𝛽 = −
2

sin is negative in the third and fourth quadrants. But, start by


finding a first quadrant solution.
√3 𝜋
sin 𝛽 = ⇒𝛽=
2 3

Reflect it across the fourth quadrant by reflecting the first


quadrant solution across the 𝑥 axis (sin(−𝜃) = − sin 𝜃):
𝜋 √3 𝜋 √3
sin = ⇒ sin (− ) = −
3 2 3 2

Convert the above solution to the domain of interest by using the identity sin(2𝜋 + 𝜃) = sin 𝜃:
𝜋 5𝜋
2𝜋 − =
3 3

Use the identity sin(𝜋 + 𝜃) = − sin 𝜃 to find the solution in


the third quadrant:
𝜋 4𝜋 √3
sin (𝜋 + ) = sin =−
3 3 2

4𝜋 5𝜋
𝛽∈{ , }
3 3

Example 3.32
Solve for the variable. In each case, the domain of interest is [0,2𝜋).
sin 𝜃 = √2

sin 𝜃 = √2 ≈ 1.41
But

P a g e 92 | 213
Get all the files at: https://bit.ly/azizhandouts
Aziz Manva (azizmanva@gmail.com)

−1 ≤ sin 𝜃 ≤ 1
Hence, the equation has no solutions.

D. 𝒄𝒐𝒔 𝒙

3.33: Number of Solutions


Consider the solutions of the equation below in
the domain 𝜃 ∈ [0,2𝜋), 𝜃 ∈ ℝ:
cos 𝜃 = 𝑐

We consider four distinct cases:


➢ (𝐵𝑙𝑢𝑒 𝐿𝑖𝑛𝑒) 0 < 𝑐 < 1 has two solutions
(1st and 4th quadrant)
➢ (𝑉𝑖𝑜𝑙𝑒𝑡 𝐿𝑖𝑛𝑒 ) −1 < 𝑐 < 0 has two solutions (2nd and 3rd quadrant)
➢ 𝑐 = ±1 has exactly one solution
➢ 𝑐 > 1 𝑂𝑅 𝑐 < −1 has no solutions7

Example 3.34
Solve over the domain [0,2𝜋):
1
cos 𝛼 =
√2

cos 𝛼 > 0 ⇒ 𝑆𝑜𝑙𝑢𝑡𝑖𝑜𝑛𝑠 𝑖𝑛 𝑄𝐼 𝑎𝑛𝑑 𝐼𝑉


𝜋
𝑄𝐼 𝑆𝑜𝑙𝑢𝑡𝑖𝑜𝑛: 𝛼1 =
4
To get the QIV solution, reflect the QI solution across the 𝑥
axis [cos(−𝜃) = cos 𝜃]:
𝜋 𝜋 7𝜋
𝛼2 = − = 2𝜋 − =
4 ⏟ 4 4
𝑨𝒅𝒅 𝟐𝝅
𝜋 7𝜋
𝛼∈{ , }
4 4

Example 3.35
Solve over the domain [0,2𝜋):
1
cos 𝛼 =
2

Solutions will be first and fourth quadrant.


𝑄𝐼: 60°
𝑄𝐼𝑉: −60° = −60 + 360 = 300

𝜋
𝑄𝐼:
3
𝜋 𝜋 5𝜋
𝑄𝐼𝑉: = − = − + 2𝜋 =
3 3 3

7 There can be complex solutions.

P a g e 93 | 213
Get all the files at: https://bit.ly/azizhandouts
Aziz Manva (azizmanva@gmail.com)

Example 3.36
Solve over the domain [0,2𝜋):
√3
cos 𝛼 =
2

Solutions will be first and fourth quadrant.


𝑄𝐼: 30°
𝑄𝐼𝑉: −30° = −30 + 360 = 330°

Example 3.37
Solve over the domain [0,2𝜋):
1
A. cos 𝛼 = − 2
1
B. cos 𝛽 = −
√2
√3
C. cos 𝛾 = −2

Solutions will be second and third fourth quadrant.

Part A
We first find a first quadrant solution:
1
𝑄𝐼: cos 𝛼 = − ⇒ 𝛼 = 60°
2

Convert the first quadrant solution to corresponding second and third quadrant solutions
Use cos(180 − 𝜃) = − cos 𝜃 to get the second quadrant solution:
𝑄𝐼𝐼: 180 − 60 = 120°

Use cos(180 + 𝜃) = − cos 𝜃 to get the third quadrant solution:


𝑄𝐼𝐼: 180 + 60 = 240°

Combine the solutions to get:


𝛼 ∈ {120°, 240°}
Part B
𝑄𝐼: 45°
𝑄𝐼𝐼: 180 − 45 = 135°
𝑄𝐼𝐼: 180 + 45 = 225°
𝛼 ∈ {135°, 225°}
Part C
𝑄𝐼: 30°
𝑄𝐼𝐼: 180 − 30 = 150°
𝑄𝐼𝐼: 180 + 30 = 210°
𝛼 ∈ {150°, 210°}

Example 3.38: Zeroes of the Trig Functions


A. Determine the zeroes of the six trigonometric functions.
B. Explain why some of the functions do not have zeroes.

𝒔𝒊𝒏 𝒙

P a g e 94 | 213
Get all the files at: https://bit.ly/azizhandouts
Aziz Manva (azizmanva@gmail.com)

From the graph of 𝑦 = sin 𝑥, the roots are


precisely the integer multiples of 𝜋. We
can write this as:
sin 𝜃 = 0 ⇒ 𝜃 = 𝑛𝜋, 𝑛 ∈ ℤ

𝒄𝒐𝒔 𝒙
Again, use the graph. The roots of 𝑦 = cos 𝑥 are:
𝑛𝜋
cos 𝜃 = 0 ⇒ 𝜃 = , 𝑛 𝑖𝑠 𝑜𝑑𝑑 𝑖𝑛𝑡𝑒𝑔𝑒𝑟
2

𝒕𝒂𝒏 𝒙
sin 𝜃
tan 𝜃 = 0 ⇒ = 0 ⇒ sin 𝜃 = 0 ⇒ 𝜃 = 𝑛𝜋, 𝑛 ∈ ℤ
cos 𝜃
𝑛𝜋
cot 𝜃 = 0 ⇒ 𝜃 = , 𝑛 𝑖𝑠 𝑜𝑑𝑑 𝑖𝑛𝑡𝑒𝑔𝑒𝑟
2
For sin 𝜃 ≠ 0:
1
csc 𝜃 = 0 ⇒ = 0 ⇒ 1 = 0 ⇒ 𝑵𝒐 𝑺𝒐𝒍𝒖𝒕𝒊𝒐𝒏𝒔
sin 𝜃
For cos 𝜃 ≠ 0:
1
sec 𝜃 = 0 ⇒ = 0 ⇒ 1 = 0 ⇒ 𝑵𝒐 𝑺𝒐𝒍𝒖𝒕𝒊𝒐𝒏𝒔
cos 𝜃
Part B
The functions that do not have zeroes are reciprocal functions of the form:
1
𝑓(𝑥)
1
These do not have a zero since 𝑓(𝑥) is never zero.

E. Principal, General and Particular Solutions

3.39: Principal and General Solutions


➢ A solution over the domain [0,2𝜋) is called a principal solution.
➢ A solution over the domain (−∞, ∞) is called the general solution.

Example 3.40: Principal and General Solutions


Find principal and general solutions for the following equations.
1
A. sin 𝜃 = 2
√3
B. sin 𝜙 = −
2
2√3+2
C. 2 cos 𝑥 + 1 = 2
+ cos 𝑥

Part A 𝜋 5𝜋
𝜃=𝜋− =
Principal Solutions 6 6
1 𝜋 Hence, the principal solutions are:
sin 𝜃 = ⇒𝜃= 𝜋 5𝜋
2 6 𝜃={ , }
The above value of 𝜃 is in Quadrant I. We know that 6 6
sin 𝜃 is positive in Quadrants I and II. General Solutions
Use sin 𝜃 = sin(180 − 𝜃) to the find the quadrant II Because the sin function is periodic with period 2𝜋,
solution: adding 2𝜋 to any solution gives us another solution.
We can generalize this as:

P a g e 95 | 213
Get all the files at: https://bit.ly/azizhandouts
Aziz Manva (azizmanva@gmail.com)

𝜋 5𝜋 4𝜋 5𝜋
𝜃 = { + 2𝑛𝜋, + 2𝑛𝜋} , 𝑛 ∈ ℤ 𝜃={ , }
6 6 3 3
Part B General Solutions
Principal Solutions Because the sin function is periodic with period 2𝜋,
𝜋 4𝜋 adding 2𝜋 to any solution gives us another solution.
𝜙=𝜋+ =
3 3 We can generalize this as:
The above value of 𝜃 is in Quadrant III. sin 𝜃 is 4𝜋 5𝜋
negative in Quadrants III and IV. 𝜃 = { + 2𝑛𝜋, + 2𝑛𝜋} , 𝑛 ∈ ℤ
3 3
Use sin 𝜃 = sin(180 − 𝜃) to the find the quadrant IV Part C
solution: Treat cos 𝑥 like a variable, and solve for it:
4𝜋 𝜋 2√3 + 2
𝜙=𝜋− =− 2 cos 𝑥 + 1 = + cos 𝑥
3 3 2
This is in Quadrant IV. But it does not lie in the
2 cos 𝑥 + 1 = √3 + 1 + cos 𝑥
domain.
cos 𝑥 = √3
Hence, add 2𝜋 to bring it in the required domain:
𝜋 5𝜋 Since √3 > 1, and
𝜙 = 2𝜋 − = −1 ≤ cos 𝑥 ≤ 1
3 3
Hence, the principal solutions are: the above equation has no real solutions.

Example 3.41: Trivial Quadratics and Absolute Value Equations


Find the principal, and the general solution for the following equations:
1
A. cos2 𝑥 = 4
1
B. sin2 𝑥 = 4
C. |tan 𝑥| = 1

Part A
1 1
cos 𝑥 = ±√ = ±
4 2
Principal Solution:
𝜋 2𝜋 4𝜋 5𝜋
cos 𝑥 = { , , , }
3 3 3 3
General Solution:
𝜋 2𝜋
cos 𝑥 = { + 𝑛𝜋 , + 𝑛𝜋, 𝑛 ∈ ℤ}
3 3
Part B
1 1
sin 𝑥 = ±√ = ±
4 2
Principal Solution:
𝜋 5𝜋 7𝜋 11𝜋
sin 𝑥 = { , , , }
6 6 6 6
General Solution:
𝜋 5𝜋
cos 𝑥 = { + 𝑛𝜋 , + 𝑛𝜋}
6 6
Part C
tan 𝑥 = ±1
Case I: tan 𝑥 = 1 ⇒ sin 𝑥 = cos 𝑥

P a g e 96 | 213
Get all the files at: https://bit.ly/azizhandouts
Aziz Manva (azizmanva@gmail.com)

𝜋 5𝜋
𝑥∈{ , }
4 4
Case II: tan 𝑥 = −1 ⇒ sin 𝑥 = − cos 𝑥
3𝜋 7𝜋
𝑥∈{ , }
4 4
Combine to write the principal solutions
𝜋 3𝜋 5𝜋 7𝜋
𝑥∈{ , , , }
4 4 4 4
General Solution:
𝜋 3𝜋
𝑥 ∈ { + 𝑛𝜋, + 𝑛𝜋} , 𝑛 ∈ ℤ
4 4

3.42: Particular Solutions


A solution of a trigonometric equation over a given domain is called a particular solution.

Example 3.43

Particular Solutions Trig Functions of Expressions


Solve the equations below over the given domains. Find the general solution to the equations below:
√3 1
A. sin 𝜃 = , −2𝜋 ≤ 𝜃 < 𝜋 C. cos(2𝑥) = 2
2
1 𝑥 √3
B. tan 𝜃 = , −𝜋 ≤ 𝜃 < 3𝜋 D. cos ( ) = −
√3 3 2

Part A 𝜋 13𝜋
2𝜋 + =
First quadrant solution is: 6 6
𝜋 7𝜋 19𝜋 1
2𝜋 + = = (3 + ) 𝜋 > 3𝜋
3 6 6 6
Second quadrant solution is: Subtract 2𝜋:
𝜋 2𝜋 𝜋 11𝜋
𝜋− = − 2𝜋 = − < −𝜋
3 3 6 6
Subtract 2𝜋 to find the negative angles: 7𝜋 5𝜋
𝜋 5𝜋 − 2𝜋 = −
6 6
− 2𝜋 = − 5𝜋 𝜋 7𝜋 13𝜋
3 3 𝜃 = {− , , , }
2𝜋 4𝜋 6 6 6 6
− 2𝜋 = −
3 3 Part C
Combine the above solutions to get the final Find the principal solutions first:
answer: 1 𝜋 5𝜋 𝜋 5𝜋
5𝜋 4𝜋 𝜋 2𝜋 cos 2𝑥 = ⇒ 2𝑥 ∈ { , } ⇒ 𝑥 ∈ { , }
𝜃 = {− , − , , } 2 3 3 6 6
3 3 3 3 Convert the principal solutions to general solutions:
Part B 𝜋 5𝜋
First quadrant solution is: 𝑥 ∈ { + 2𝑛𝜋, + 2𝑛𝜋}
6 6
𝜋 Part D
6 𝑥 √3
Other solution is the third quadrant: cos ( ) = −
𝜋 7𝜋 3 2
𝜋+ = 𝑐𝑜𝑠 is negative in the second and the third
6 6 quadrants.
Add 2𝜋:

P a g e 97 | 213
Get all the files at: https://bit.ly/azizhandouts
Aziz Manva (azizmanva@gmail.com)

𝑥 5𝜋 7𝜋 5𝜋 7𝜋
∈ { + 2𝑛𝜋, + 2𝑛𝜋} 𝑥∈{ + 6𝑛𝜋, + 6𝑛𝜋}
3 6 6 2 2

3.3 Equations: Change of Domain/Variable


A. Change of Domain

Example 3.44
sin 𝑥 = −0.5, −𝜋 ≤ 𝑥 ≤ 0

In the range 0 < 𝑥 < 2𝜋, the solutions are:


𝑥 ∈ {210°, 330°}
Subtract 360°
5𝜋 𝜋
𝑥 ∈ {−150°, −30°} = {− ,− }
6 6

Example 3.45
1
Solve sin(2𝑥) = − 2 , −𝜋 < 𝑥 < 2𝜋

The expression inside the sin function is in terms of 2𝑥, while the domain is in terms of 𝑥. We need to get the
domain in terms of 2𝑥:
1
sin(2𝑥) = − , −2𝜋 < 2𝑥 < 4𝜋
2
Find the principal solutions first:
7𝜋 11𝜋
2𝑥 ∈ { , } , 0 ≤ 2𝑥 ≤ 2𝜋
12 12
Add 2𝜋 to find the solutions in the domain 2𝜋 ≤ 2𝑥 ≤ 4𝜋.
Subtract 2𝜋 to find the solutions in the domain −2𝜋 ≤ 2𝑥 ≤ 0.
Hence, the final solution set in terms of 𝑦:
−17𝜋 −13𝜋 7𝜋 11𝜋 31𝜋 35𝜋
2𝑥 = { , , , , , }
12 12 12 12 12 12
−17𝜋 −13𝜋 7𝜋 11𝜋 31𝜋 35𝜋
𝑥∈{ , , , , , }
24 24 24 24 24 24

Example 3.46
Find the sum of the four smallest values in 𝑆 given that 𝑥 is in degrees and:
𝑆 ∈ {𝑥: − 4 = 8(sin 3(𝑥 + 20)) − 8, 𝑥 ∈ ℝ+ }

Determine the domain of 3(𝑥 + 20):


𝑥 > 0 ⇒ 𝑥 + 20 > 20 ⇒ 3(𝑥 + 20) > 60°

1
4 = 8(sin 3(𝑥 + 20)) ⇒ = sin 3(𝑥 + 20)
2
Principal Solutions
3(𝑥 + 20) ∈ {30,150}

Determine the solutions in the domain of interest by adding 360 to the principal solutions:
3(𝑥 + 20) ∈ {150, 390,510, 750}
(𝑥 + 20) ∈ {50, 130,170, 250}
𝑥 ∈ {30, 110,150, 230}

P a g e 98 | 213
Get all the files at: https://bit.ly/azizhandouts
Aziz Manva (azizmanva@gmail.com)

Example 3.47

B. Change of Variable

Example 3.48
𝑥 𝜋
Solve 2 cos (2 + 2 ) − √3 = 0 , −𝜋 < 𝑥 < 4𝜋

Change of Variable
Let the expression inside the cosine function be equal to 𝑦:
𝑥 𝜋
+ = 𝑦 ⇒ 𝑥 + 𝜋 = 2𝑦 ⇒ 𝑥 = 2𝑦 − 𝜋
2 2
Finding the Domain
To find the domain in terms of 𝑦, substitute the above value of 𝑦 in the expression for the domain in terms of 𝑥:
−𝜋 < 2𝑦 − 𝜋 < 4𝜋
Add 𝜋 throughout:
0 < 2𝑦 < 5𝜋
Divide by 2:
5
0<𝑦< 𝜋
2
Solve the changed equation over the changed domain
√3 𝜋 11𝜋 13𝜋
2 cos 𝑦 − √3 = 0 ⇒ cos 𝑦 = ⇒𝑦={ , , }
2 6 6 6
Substitute each of the above solutions in 𝑥 = 2𝑦 − 𝜋:
𝜋 11𝜋 13𝜋 2𝜋 8𝜋 10𝜋
2{ , , } − 𝜋 = {− , , }
6 6 6 3 3 3

Example 3.49: Using the original domain


𝜋 3
sin2 (2𝑥 − ) = , −𝜋 ≤ 𝑥 ≤ 𝜋
3 4

Take the square root both sides:


𝜋 √3
sin (2𝑥 − ) = ±
3 2
Take the sin inverse of both sides, and note that we get a solution in each quadrant:
𝜋 𝜋 2𝜋 4𝜋 5𝜋
2𝑥 − ∈ { , , , } + 2𝜋𝑛, 𝑛 ∈ ℕ
3 3 3 3 3
𝜋 4𝜋 2𝜋 5𝜋
Since + 𝜋 = , + 𝜋 = , we can write this as:
3 3 3 3
𝜋 𝜋 2𝜋
2𝑥 − ∈ { , } + 𝜋𝑛, 𝑛 ∈ ℕ
3 3 3
𝜋
Add 3 :
2𝜋
2𝑥 ∈ { , 𝜋} + 𝜋𝑛, 𝑛 ∈ ℕ
3
Since the second solution requires multiples of 𝜋, substitute 0 instead of 𝜋:
2𝜋
2𝑥 ∈ { , 0} + 𝜋𝑛, 𝑛 ∈ ℕ
3
Divide by 2:

P a g e 99 | 213
Get all the files at: https://bit.ly/azizhandouts
Aziz Manva (azizmanva@gmail.com)

𝜋 𝜋𝑛
𝑥 ∈ {0, } + ,𝑛 ∈ ℕ
3 2
The solutions in the domain −𝜋 ≤ 𝑥 ≤ 𝜋 are:
𝜋 𝜋
𝐹𝑟𝑜𝑚 𝑥 = 0: {−𝜋, − , 0, , 𝜋}
2 2
𝜋 2𝜋 𝜋 𝜋 5𝜋
𝐹𝑟𝑜𝑚 𝑥 = : {− , − , , }
3 3 6 3 6
C. Reducible to Quadratic

(Calculator) Example 3.50


tan2 𝑥 + 4 = 2 sec 2 𝑥 + tan 𝑥 , 𝑥 ∈ [0,2𝜋]

Substitute sec 2 𝑥 = tan2 𝑥 + 1:


tan2 𝑥 + 4 = 2 tan2 𝑥 + 2 + tan 𝑥
0 = tan2 𝑥 + tan 𝑥 − 2
Substitute 𝑦 = tan 𝑥:
𝑦2 + 𝑦 − 2 = 0
(𝑦 + 2)(𝑦 − 1) = 0
𝑦 ∈ {−2,1}

𝑦 = tan 𝑥 = −2 ⇒ 𝑥 = tan−1 (−2)


Add 𝜋 ≈ 3.14 and 2𝜋 ≈ 6.28 to the above:
tan−1 (−2) + 𝜋 = 2.03
tan−1 (−2) + 2𝜋 = 5.18

𝜋 5𝜋
𝑦 = tan 𝑥 = 1 ⇒ 𝑥 = ,
4 4

Example 3.51
Solve:
sin 𝑥 cos 𝑥 3 sin3 𝑥 3 sin 𝑥
cos 𝑥 sin3 𝑥 − − + =0
2 4 cos 𝑥 8 cos 𝑥

Note that cos 𝑥 ≠ 0. Multiply both sides by cos 𝑥:


sin 𝑥 cos2 𝑥 3 3 3
cos 2 𝑥 sin3 𝑥 − − sin 𝑥 + sin 𝑥 = 0
2 4 8
Factor out sin 𝑥:
cos 2 𝑥 3 2 3
(sin 𝑥) (cos 2 𝑥 sin2 𝑥 − − sin 𝑥 + ) = 0
2 4 8
3
Factor out cos2 𝑥 from the first two terms, and − from the last two terms:
4
1 3 1
(sin 𝑥) [(cos 𝑥) (sin2 𝑥 − ) − (sin2 𝑥 − )] = 0
2
2 4 2
2 1
Factor out sin 𝑥 − 2:
1 3
(sin 𝑥) (sin2 𝑥 − ) (cos 2 𝑥 − ) = 0
2 4

P a g e 100 | 213
Get all the files at: https://bit.ly/azizhandouts
Aziz Manva (azizmanva@gmail.com)

Apply the zero-product property:


sin 𝑥 = 0 ⇒ 𝑥 = 𝑛𝜋, 𝑛∈ℤ

1 1 𝜋 𝑛𝜋
sin2 𝑥 − = 0 ⇒ sin 𝑥 = ± ⇒𝑥= + , 𝑛∈ℤ
2 √2 4 2

3 √3
cos2 𝑥 −= 0 ⇒ cos 𝑥 = ± ⇒
4 2
𝜋 7𝜋
𝑥 = + 𝑛𝜋, 𝑥 = + 𝑛𝜋, 𝑛∈ℤ
6 6

3.4 Equations: Trig Identities


A. Trig Identities

3.52: Pythagorean Identity


sin2 𝑥 + cos 2 𝑥 = 1
cos 2 𝑥 = 1 − sin2 𝑥
sin2 𝑥 = 1 − cos2 𝑥

Example 3.53
The number of solutions of the equation sin 𝑥 = cos2 𝑥 in the interval (0,10) is: (JEE Main, June 29, 2022-II)

Substitute cos2 𝑥 = 1 − sin2 𝑥:


sin 𝑥 = 1 − sin2 𝑥

Collate all terms on the LHS to get a quadratic in sin 𝑥:


sin2 𝑥 + sin 𝑥 − 1 = 0

Use the quadratic formula with 𝑎 = 1, 𝑏 = 1, 𝑐 = 1, and substitute √5 ≈ 2.23:


−1 ± √1 − (4)(1)(−1) −1 ± √5 −1 ± 2.23
sin 𝑥 = = ≈ ∈ {0.61, −1.61}
2 2 2

Reject −1.61 since

−1 ≤ sin 𝑥 ≤ 1

sin 𝑥 = 0.61 ⇒ 𝑉𝑎𝑙𝑖𝑑

In the range:
[0,2𝜋) ⇒ 𝑄𝐼, 𝑄𝐼𝐼 ⇒ 2 𝑆𝑜𝑙𝑢𝑡𝑖𝑜𝑛𝑠
[2𝜋, 3𝜋) ≈ [6.28,9.42) ⇒ 𝑄𝐼, 𝑄𝐼𝐼 ⇒ 2 𝑆𝑜𝑙𝑢𝑡𝑖𝑜𝑛𝑠

P a g e 101 | 213
Get all the files at: https://bit.ly/azizhandouts
Aziz Manva (azizmanva@gmail.com)

Total number of solutions


=2+2=4

Example 3.54
𝜋 7𝜋
The number of values of 𝑥 in the interval ( 4 , 4
) for which 14 csc 2 𝑥 − 2 sin2 𝑥 = 21 − 4 cos 2 𝑥, is: (JEE Main,
June 25, 2022-I)

14
− 2 sin2 𝑥 − 4 sin2 𝑥 = 21 − 4 cos2 𝑥 − 4 sin2 𝑥
sin2 𝑥
14
− 6 sin2 𝑥 = 17
sin2 𝑥

Let 𝑡 = sin2 𝑥:
14
− 6𝑡 = 17
𝑡
14 − 6𝑡 2 = 17𝑡
6𝑡 2 + 17𝑡 − 14 = 0
𝑃 = 6 × 14 = 84 = (21)(−4)

6𝑡 2 + 21𝑡 − 4𝑡 − 14 = 0
3𝑡(2𝑡 + 7) − 2(2𝑡 + 7) = 0
(3𝑡 − 2)(2𝑡 + 7) = 0
7 2
sin2 𝑥 = 𝑡 ∈ {− , }
2 3
7
Reject sin2 𝑥 = − :
2
2
sin2 𝑥 =
3

𝜋 1 𝜋 1 2
sin = ⇒ sin2 = <
4 √2 4 2 3
𝜋 𝜋 2
sin = 1 ⇒ sin2 = 1 >
2 2 3

Hence, there is one solution in the interval:


𝜋 𝜋
<𝑥<
4 2

Using the similar logic, there are 3 other solutions.


𝑇𝑜𝑡𝑎𝑙: 4 𝑆𝑜𝑙𝑢𝑡𝑖𝑜𝑛𝑠

3.55: Double Angle Identity for 𝒔𝒊𝒏


sin 2𝑥 = 2 sin 𝑥 cos 𝑥

Example 3.56
Find the principal solutions to
sin 2𝑥 + sin 𝑥 + 2 cos 𝑥 + 1 = 0

P a g e 102 | 213
Get all the files at: https://bit.ly/azizhandouts
Aziz Manva (azizmanva@gmail.com)

Substitute sin 2𝑥 = 2 sin 𝑥 cos 𝑥:


2 sin 𝑥 cos 𝑥 + sin 𝑥 + 2 cos 𝑥 + 1 = 0

Factor:
sin 𝑥 (2 cos 𝑥 + 1) + 1(2 cos 𝑥 + 1) = 0
(sin 𝑥 + 1)(2 cos 𝑥 + 1) = 0

Use the zero-product property:


3𝜋
sin 𝑥 + 1 = 0 ⇒ sin 𝑥 = −1 ⇒ 𝑥 =
2
1 2𝜋 4𝜋
2 cos 𝑥 + 1 = 0 ⇒ cos 𝑥 = − ⇒ 𝑥 ∈ { , }
2 3 3

2𝜋 4𝜋 3𝜋
𝑥∈{ , , }
3 3 2

3.57: Double Angle Identity for 𝒄𝒐𝒔


cos 2𝜃 = cos 2 𝜃 − sin2 𝜃
= 2 cos 2 𝜃 − 1
= 1 − 2 sin2 𝜃

Example 3.58
The number of elements in the set below is: (JEE Main, June 29, 2022-I)
𝑆 = {𝜃 ∈ [−4𝜋, 4𝜋]: 3 cos 2 2𝜃 + 6 cos 2𝜃 − 10 cos2 𝜃 + 5 = 0}

3 cos 2 2𝜃 + 6 cos 2𝜃 − 10 cos 2 𝜃 + 5 = 0


Two of the terms have 2𝜃 and one term has 𝜃. Hence, we convert that term to be in terms of 2𝜃.
cos 2𝜃+1
Substitute cos2 𝜃 = 2
:
cos 2𝜃 + 1
3 cos 2 2𝜃 + 6 cos 2𝜃 − 10 ( )+5=0
2
3 cos2 2𝜃 + 6 cos 2𝜃 − 5 cos 2𝜃 − 5 + 5 = 0
3 cos2 2𝜃 + cos 2𝜃 = 0

Use a change of variable. Let cos 2𝜃 = 𝑥:


1
3𝑥 2 + 𝑥 = 0 ⇒ 𝑥(3𝑥 + 1) = 0 ⇒ 𝑥 ∈ {− , 0 }
3

𝜃 ∈ [−4𝜋, 4𝜋] ⇒ 2𝜃 ∈ [−8𝜋, 8𝜋]


𝜋 3𝜋
𝑥 = cos 2𝜃 = 0 ⇒ 𝑃𝑟𝑖𝑛𝑐𝑖𝑝𝑎𝑙 𝑠𝑜𝑙𝑢𝑡𝑖𝑜𝑛𝑠 𝑎𝑟𝑒 2𝜃 ∈ { , } ⇒ 2 𝑆𝑜𝑙𝑢𝑡𝑖𝑜𝑛𝑠 𝑖𝑛 [0,2𝜋)
2 2
[−8𝜋, 8𝜋]
±(0 − 2𝜋), ±(2𝜋 − 4𝜋), ±(4𝜋 − 6𝜋), ±(6𝜋 − 8𝜋) ⇒ 8 𝐼𝑛𝑡𝑒𝑟𝑣𝑎𝑙𝑠 ⇒ 16 𝑆𝑜𝑙𝑢𝑡𝑖𝑜𝑛𝑠

1
𝑥 = cos 2𝜃 = − ⇒ 2 𝑆𝑜𝑙𝑢𝑡𝑖𝑜𝑛𝑠 𝑖𝑛 [0,2𝜋)
3
By the same logic as before:
⇒ 16 𝑆𝑜𝑙𝑢𝑡𝑖𝑜𝑛𝑠 𝑖𝑛 [−4𝜋, 4𝜋]

P a g e 103 | 213
Get all the files at: https://bit.ly/azizhandouts
Aziz Manva (azizmanva@gmail.com)

Total solutions
16 + 16 = 32

3.59: Sum Formula


𝜋
sin 𝜃 + cos 𝜃 = √2 sin (𝜃 + )
4

Factor √2 in the LHS:


1 1
𝐿𝐻𝑆 = √2 [(sin 𝜃) + cos 𝜃]
√2 √2

𝜋 𝜋 1
Substitute sin ( 4 ) = cos ( 4 ) = :
√2
𝜋 𝜋
= √2 [(sin 𝜃) cos ( ) + sin ( ) cos 𝜃]
4 4

Use the sum formula sin(𝛼 + 𝛽) = sin 𝛼 cos 𝛽 + sin 𝛽 cos 𝛼


𝜋
= √2 [sin (𝜃 + )] = 𝑅𝐻𝑆
4

Example 3.60
Let 𝑆 = {𝜃 ∈ [0,2𝜋): tan(𝜋 cos 𝜃) + tan(𝜋 sin 𝜃) = 0}. Then:
𝜋
∑ sin2 (𝜃 + ) =
4
𝜃∈𝑆
(JEE Main, Jan 24, 2023-II)

tan(𝜋 cos 𝜃) = − tan(𝜋 sin 𝜃)

Use − tan 𝜃 = tan(−𝜃):


tan(𝜋 cos 𝜃) = tan(−𝜋 sin 𝜃)

Since the period of tan 𝜃 is 𝜋:


𝜋 cos 𝜃 = 𝑛𝜋 − 𝜋 sin 𝜃 , 𝑛∈ℤ

Divide by 𝜋 both sides and rearrange:


sin 𝜃 + cos 𝜃 = 𝑛, 𝑛∈ℤ
√2 sin(𝜃 + 45°) = 𝑛, 𝑛∈ℤ
𝑛
sin(𝜃 + 45°) = , 𝑛∈ℤ
√2

−1 0 1
sin(𝜃 + 45°) = sin(𝜃 + 45°) = sin(𝜃 + 45°) =
√2 √2 √2
𝜃 + 45° ∈ {225°, 315°} 𝜃 + 45° ∈ {0°, 180°} 𝜃 + 45° ∈ {45°, 135°}
𝜃 ∈ {180°, 270°} 𝜃 ∈ {315°, 135°} 𝜃 ∈ {0°, 90°}
2 𝑆𝑜𝑙𝑢𝑡𝑖𝑜𝑛𝑠 2 𝑆𝑜𝑙𝑢𝑡𝑖𝑜𝑛𝑠 2 𝑆𝑜𝑙𝑢𝑡𝑖𝑜𝑛𝑠

1 sin2(𝜃 + 45°) = 0 1
sin2(𝜃 + 45°) = sin2 (𝜃 + 45°) =
2 2

P a g e 104 | 213
Get all the files at: https://bit.ly/azizhandouts
Aziz Manva (azizmanva@gmail.com)

𝜋 1 2×0=0 1
∑ sin2 (𝜃 + ) 2× =1 2× =1
4 2 2
𝜃∈𝑆

Total
=1+0+1=2

3.61: Sum and Difference Identities for 𝒕𝒂𝒏


tan 𝛼 + tan 𝛽
tan(𝛼 + 𝛽) =
1 − tan 𝛼 tan 𝛽
tan 𝛼 − tan 𝛽
tan(𝛼 − 𝛽) =
1 + tan 𝛼 tan 𝛽

Example 3.62
𝜋 𝜋 𝜋 3𝜋
𝑆 = [−𝜋, ) − {− , − , − }
2 2 4 2
The number of elements in the set 𝐴 = {𝜃 ∈ 𝑆: tan 𝜃 (1 + √5 tan(2𝜃)) = √5 − tan(2𝜃)} is: (JEE Main, 28 July,
2022-II)

tan 𝜃 + √5 tan 𝜃 tan(2𝜃) = √5 − tan(2𝜃)


tan 𝜃 + tan(2𝜃) = √5 − √5 tan 𝜃 tan(2𝜃)
tan 𝜃 + tan(2𝜃)
= √5
1 − tan 𝜃 tan(2𝜃)
tan 3𝜃 = √5
3𝜋
3𝜃 = tan−1 √5 + 𝑛𝜋, 𝑛 ∈ ℕ, 3𝑆 = [−3𝜋, )
2

Draw the graph (manually). There are


5 𝑠𝑜𝑙𝑢𝑡𝑖𝑜𝑛𝑠
B. Further Questions

Example 3.63: Pythagorean Identity


Find all solutions to:
2𝑥 2𝑥
16sin + 16cos = 10

P a g e 105 | 213
Get all the files at: https://bit.ly/azizhandouts
Aziz Manva (azizmanva@gmail.com)

Use sin2 𝑥 = 1 − cos 2 𝑥


2𝑥 2𝑥 2𝑥 16
16sin + 161−sin = 10 ⇒ 16sin + 2𝑥 = 10
16sin
2
Use a change of variable. Let 𝑦 = 16sin 𝑥 :
16
𝑦+ = 10 ⇒ 𝑦 2 + 16 = 10𝑦 ⇒ (𝑦 − 8)(𝑦 − 2) = 0
𝑦
𝟐
Case I: 𝒚 = 𝟏𝟔𝐬𝐢𝐧 𝒙 = 𝟖
2 3 √3
24sin 𝑥 = 23 ⇒ 4 sin2 𝑥 = 3 ⇒ sin2 𝑥 = ⇒ sin 𝑥 = ±
4 2
The solutions can be generalized to:
𝜋 2𝜋 4𝜋 5𝜋 𝜋 2𝜋
{ , , , } + 2𝑛𝜋, 𝑛 ∈ ℤ ⇒ { , } + 𝑛𝜋, 𝑛 ∈ ℤ
3 3 3 3 3 3
𝐬𝐢𝐧𝟐 𝒙
Case II: 𝒚 = 𝟏𝟔 =𝟐
2 1 1
24sin 𝑥 = 21 ⇒ 4 sin2 𝑥 = 1 ⇒ sin2 𝑥 = ⇒ sin 𝑥 = ±
4 2
Solutions are:
𝜋 5𝜋
{ , } + 𝑛𝜋, 𝑛 ∈ ℤ
6 6

Combine the two cases:


𝜋 𝜋 2𝜋 5𝜋
{ , , , } + 𝑛𝜋, 𝑛 ∈ ℤ
6 3 3 6

Example 3.64
2𝑥 2𝑥 𝑛𝜋
Find the number of solutions to 9sin + 9cos = 4√3 of the form , where 𝑛 is a two-digit prime.
3

2𝑥 2
9sin
+ 91−sin 𝑥 = 4√3
2 9
9sin 𝑥 + sin2 𝑥 = 4√3
9
9
𝑦 + = 4√3
𝑦
2
𝑦 − 4√3𝑦 + 9 = 0
𝑃𝑟𝑜𝑑𝑢𝑐𝑡 = 9, 𝑆𝑢𝑚 = −4√3 = −3√3 − √3
(𝑦 − 3√3)(𝑦 − √3) = 0
𝟐𝒙
Case I: 𝒚 = 𝟗𝐬𝐢𝐧 = 𝟑√𝟑
2𝑥 2𝑥 3 3 √3
9sin = 3√3 ⇒ 32 sin = 32 ⇒ 2 sin2 𝑥 = ⇒ sin 𝑥 = ±
2 2
𝟐𝒙
Case II: 𝒚 = 𝟗𝐬𝐢𝐧 = √𝟑
2𝑥 1 1 1 1
32 sin = 32 ⇒ 2 sin2 𝑥 = ⇒ sin2 𝑥 = ⇒ sin 𝑥 = ±
2 4 2

As in the previous example:


𝜋 𝜋 2𝜋 5𝜋
{ , , , } + 𝑛𝜋, 𝑛 ∈ ℤ
6 3 3 6

𝜋 2𝜋 3𝜋 4𝜋 5𝜋 6𝜋 7𝜋 3𝜋 6𝜋
{ , , , , , , ,…} − { , ,…}
3 3 3 3 3 3 3 3 3

P a g e 106 | 213
Get all the files at: https://bit.ly/azizhandouts
Aziz Manva (azizmanva@gmail.com)

All values of 𝑛 are available except the values where 𝑛 is a multiple of 3. The only multiple of 3 which is a prime
is 3 itself, and that is not valid since it is a single digit prime number.

Hence, we want the number of two-digit prime numbers which is:


25 − 4 = 21

Example 3.65: Sequences and Series


If 0 ≤ 𝑥 ≤ 100𝜋, find the sum of all solutions to sin 𝑥 + cos 𝑥 = −1.

Simplifying the Equation


Square both sides:
sin2 𝑥 + 2 sin 𝑥 cos 𝑥 + cos 2 𝑥 = 1
Use the Pythagorean Identity: sin 𝑥 + cos2 𝑥 = 1:
2

2 sin 𝑥 cos 𝑥 + 1 = 1
2 sin 𝑥 cos 𝑥 = 0
Use the double angle identity:
sin 2𝑥 = 0
Finding the Principal Solution
The relevant domain for principal solutions is:
0 ≤ 𝑥 < 2𝜋 ⇒ 0 ≤ 2𝑥 < 4𝜋
sin 𝑥 is zero at precisely integer multiples of 𝜋:
𝜋 3𝜋
2𝑥 ∈ {0, 𝜋, 2𝜋, 3𝜋} ⇒ 𝑥 ∈ {0, , 𝜋, }
2 2
Since we squared the equation, we need to check for extraneous solutions:
3𝜋
𝑥 ∈ {𝜋, }
2
Finding the Sum (𝒙 = 𝝅)
The sum of solutions in the range 0 ≤ 𝑥 ≤ 100𝜋 is:
𝜋 + 3𝜋 + 5𝜋 + ⋯ + 99𝜋
= 𝜋(1 + 3 + 5 + ⋯ + 99)
Use the formula for the sum of the first 𝑛 odd integers (99 is the 50𝑡ℎ odd integer):
= 𝜋(502 ) = 2500𝜋
𝟑𝝅
Finding the Sum (𝒙 = 𝟐
)
The sum of solutions in the range 0 ≤ 𝑥 ≤ 100𝜋 is:
3𝜋 3𝜋 3𝜋 3𝜋
+ ( + 2𝜋) + ( + 4𝜋) + ⋯ + ( + 98𝜋)
2 2 2 2
𝜋
Each term here is greater than the first solution by 2 . The final sum will be greater by:
𝜋
50 × = 25𝜋
2
The sum of solutions from this value is:
25𝜋 + 2500𝜋 = 2525𝜋
Final Answer
We add the two sums:
2500𝜋 + 2525𝜋 = 5025𝜋

C. Graph Based Equations

Example 3.66

P a g e 107 | 213
Aziz Manva (azizmanva@gmail.com)

The number of solutions of |cos 𝑥| = sin 𝑥, such that −4𝜋 ≤ 𝑥 ≤ 4𝜋 is: (JEE Main, July 25, 2022-I)

cos 𝑥 = ± sin 𝑥
sin 𝑥
= ±1
cos 𝑥
tan 𝑥 = ±1

𝑃𝑟𝑖𝑛𝑐𝑖𝑝𝑎𝑙 𝑆𝑜𝑙𝑢𝑡𝑖𝑜𝑛𝑠: 𝑥 ∈ {45°, 225°}


[0,2𝜋) → 2 𝑆𝑜𝑙𝑢𝑡𝑖𝑜𝑛𝑠
[0,4𝜋) → 4 𝑆𝑜𝑙𝑢𝑡𝑖𝑜𝑛𝑠
[−4𝜋, 4𝜋] → 8 𝑆𝑜𝑙𝑢𝑡𝑖𝑜𝑛𝑠

Example 3.67
Find the number of solutions to
𝑥
sin 𝑥 = ,𝑥 ≥ 0
8𝜋

𝑦 = sin 𝑥
𝑥
𝑦=
5

We will get intersections only when the line is in the range of 𝑦 = sin 𝑥
𝐹𝑜𝑟 𝑡ℎ𝑒 𝑠𝑡𝑟𝑎𝑖𝑔ℎ𝑡 𝑙𝑖𝑛𝑒: − 1 ≤ 𝑦 ≤ 1
𝑥
−1 ≤ ≤1
8𝜋
−8𝜋 ≤ 𝑥 ≤ 8𝜋

Since we want 𝑥 ≥ 0, we must have:


0 ≤ 𝑥 ≤ 8𝜋

The period for sin 𝑥 is 2𝜋. The number of cycles is:


8𝜋
=4
2𝜋

Each complete period results in two intersections.


𝑁𝑜. 𝑜𝑓 𝐼𝑛𝑡𝑒𝑟𝑠𝑒𝑐𝑡𝑖𝑜𝑛𝑠 = ⏟
4 × ⏟
2 =8
𝑁𝑜.𝑜𝑓 𝐼𝑛𝑡𝑒𝑟𝑠𝑒𝑐𝑡𝑖𝑜𝑛𝑠
𝐶𝑦𝑐𝑙𝑒𝑠 𝑝𝑒𝑟 𝑐𝑦𝑐𝑙𝑒

Challenge 3.68
Find the number of solutions to
𝑥
sin 𝑥 =
100𝜋

P a g e 108 | 213
Aziz Manva (azizmanva@gmail.com)

D. Logarithms

Challenge 3.69
If 𝑥 ∈ [0, 𝜋] find the sum of solutions to:
1 3
log 1 √ = + log 4 (√3 cos 𝜋𝑥)
2 2 cos(2𝜋𝑥) + 11 2

Use a change of variables. Substitute 𝑦 = 2 cos(2𝜋𝑥) + 11 and 𝑧 = √3 cos 𝜋𝑥


1 3
log 1 √ = + log 4 𝑧
2 𝑦 2

1
The LHS has base . The RHS has base 4. Work with the LHS to convert it to base 4.
2

1 1 1 1 1
𝐿𝐻𝑆 = log 1 √ = log 2−1 𝑦 −2 = (− ) × ( ) log 2 𝑦 = log 2 𝑦 = log 22 𝑦 = log 4 𝑦
2 𝑦 2 −1 2

Hence, the equation becomes:


3
log 4 𝑦 = + log 4 𝑧
2
Collate all terms on the LHS, and use the Quotient Rule:
𝑦 3
log 4 =
𝑧 2
Exponentiate both sides:
𝑦 3
= 42 = 8 ⇒ 𝑦 = 8𝑥
𝑧
Change back to the original variables:
2 cos(2𝜋𝑥) + 11 = 8√3 cos 𝜋𝑥
Substitute cos 2𝜋𝑥 = 2 cos2 𝜋𝑥 − 1:
2(2 cos2 𝜋𝑥 − 1) + 11 = 8√3 cos 𝜋𝑥
Expand and collate all terms on the LHS:
4 cos2 𝜋𝑥 − 8√3 cos 𝜋𝑥 + 9 = 0

Use change of variables one more time. Let cos 𝜋𝑥 = 𝑡:


4𝑡 2 − 8√3𝑡 + 9 = 0
𝑃𝑟𝑜𝑑𝑢𝑐𝑡 = 36, 𝑆𝑢𝑚 = −8√3

4𝑡 − 6√3𝑡 − 2√3𝑡 + 9 = 0
2𝑡(2𝑡 − 3√3) − √3(2𝑡 − 3√3) = 0
(2𝑡 − √3)(2𝑡 − 3√3) = 0
Use the zero-product property:
3√3
2𝑡 − 3√3 = 0 ⇒ 𝑡 = cos 𝜋𝑥 = > 1 ⇒ 𝑁𝑜 𝑣𝑎𝑙𝑖𝑑 𝑆𝑜𝑙𝑢𝑡𝑖𝑜𝑛𝑠
2

√3 √3
2𝑡 − √3 = 0 ⇒ 𝑡 = ⇒ cos 𝜋𝑥 =
2 2

P a g e 109 | 213
Aziz Manva (azizmanva@gmail.com)

Solutions in [0,2𝜋] are


𝜋 4𝜋
𝜋𝑥 ∈ { , }
3 3

Find the altered domain of 𝑥:


0 ≤ 𝑥 ≤ 𝜋 ⇒ 0 ≤ 𝜋𝑥 ≤ 𝜋 2
Since 𝜋 2 ≈ 9.85 > 9,we get solutions:
𝜋 4𝜋
𝜋𝑥 ∈ { + 2𝑛𝜋, + 2𝑛𝜋} , 𝑛 = 0,1,2,3,4
3 3
1 1
𝑥 ∈ { + 2𝑛, 1 + 2𝑛} , 𝑛 = 0,1,2,3,4
3 3
1 1 1 1 1 1 1 1 1 1
𝑆𝑢𝑚 = + 2 + 4 + 6 + 8 + 1 + 3 + 5 + 7 + 9

3 3 3 3 3 ⏟3 3 3 3 3
𝐹𝑖𝑟𝑠𝑡 𝑄𝑢𝑎𝑑𝑟𝑎𝑛𝑡 𝑆𝑒𝑐𝑜𝑛𝑑 𝑄𝑢𝑎𝑑𝑟𝑎𝑛𝑡

9 × 10
1 + 2 + ⋯+ 9 = = 45
2
1 10 1
× 10 = =3
3 3 3

1 1
45 + 3 = 48
3 3

E. Identities

Example 3.70: Subjective


sin 3𝜃 = 3 sin 𝜃 − 4 sin3 𝜃

The above is an identity. Equate the LHS and the RHS of the above identity to zero separately. Determine the
principal solutions for 𝜃. Interpret your answers.

Solve the Equation from the LHS


sin 3𝜃 = 0
−1 (0)
3𝜃 = sin + 180𝑘° = 0 + 180𝑘° = 180𝑘°
𝜃 = 60𝑘°

Principal solutions (𝜃 ∈ [0,2𝜋)) are


𝜃 ∈ {0,60°, 120°, 180°, 240°, 300°} + 360°

Solve the Equation from the RHS


3 sin 𝜃 − 4 sin3 𝜃 = 0

Let 𝛼 = sin 𝜃:
3𝛼 − 4𝛼 3 = 0
𝛼(3 − 4𝛼 2 ) = 0
𝛼 = 0 ⇒ sin 𝜃 = 0 ⇒ 𝜃 ∈ {0,180}
3 √3
3 − 4𝛼 2 = 0 ⇒ 𝛼 2 = ⇒ 𝛼 = ±
4 2
√3
sin 𝜃 = ⇒ 𝜃 ∈ {60°, 120°}
2
P a g e 110 | 213
Aziz Manva (azizmanva@gmail.com)

√3
sin 𝜃 = − ⇒ 𝜃 ∈ {240°, 300°}
2

Note that we obtained six solutions from the LHS, and six solutions from the RHS, and these match 1 to 1 (as
they should).

Example 3.71: Subjective


sin 5𝜃 = 5 sin 𝜃 − 20 sin3 𝜃 + 16 sin5 𝜃

The above is an identity. Equate the LHS and the RHS of the above identity to zero separately. Determine the
solutions for −90° ≤ 𝜃 ≤ 90°. Interpret your answers.

Solve the LHS Equation


sin 5𝜃 = 0
−1 (0)
5𝜃 = sin + 180𝑘° = 0 + 180𝑘° = 180𝑘°
𝜃 = 36𝑘°

Solutions in the range −90° ≤ 𝜃 ≤ 90°


𝜃 ∈ {−72°, −36°, 0,36°, 72°} + 360°

Solve the RHS Equation


5 sin 𝜃 − 20 sin3 𝜃 + 16 sin5 𝜃 = 0

Let 𝛼 = sin 𝜃:
5𝛼 − 20𝛼 3 + 16𝛼 5 = 0
𝛼(16𝛼 4 − 20𝛼 2 + 5) = 0

If
𝛼 = 0 ⇒ sin 𝜃 = 0 ⇒ 𝜃 = 0

If 𝛼 ≠ 0, let 𝛽 = 𝛼 2 :
16𝛽 2 − 20𝛽 + 5 = 0

Substitute 𝑎 = 16, 𝑏 = −20, 𝑐 = 5 in the quadratic formula:

20 ± √400 − (4)(16)(5) 5 ± √5 5 ± √5
𝛼2 = 𝛽 = = ⇒ 𝛼 = ±√
2(16) 8 8

Interpreting the answers

sin 0 sin 72 sin 36 sin(−36) sin(−72)


0
√5 + √5 √5 − √5 −√
5 − √5
−√
5 + √5
8 8 8 8

3.5 Inequalities
A. Inequalities

Example 3.72
P a g e 111 | 213
Aziz Manva (azizmanva@gmail.com)

Solve the following inequalities over the domain [0,2𝜋). Graph the related trigonometric function and shade
your final answer:
1
A. sin 𝜃 > 2
1
B. sin 𝜃 ≥ − 2
√3
C. cos 𝜃 < 2
D. csc 𝜃 < 2
E. |csc 𝜃| > 2

Part A
sin 𝜃 is positive in the first and second quadrant:

1 7𝜋 11𝜋
sin 𝜃 > − ⇒ 𝜃 ∈ [0, ] ∪ [ , 2𝜋)
2 6 6
1 𝜋 5𝜋
sin 𝜃 = ⇒𝜃={ , } Part C
2 6 6 𝜋 11𝜋
The region that we want is the shaded region 𝜃∈[ , ]
6 6
between the two points. Part D
Hence, the final answer is: 1 1
1 𝜋 5𝜋 csc 𝜃 < 2 ⇒ < 2 ⇒ sin 𝜃 >
sin 𝜃 > ⇒ < 𝜃 < sin 𝜃 2
2 6 6 And this is the same inequality as Part A. Hence, it
We can write the answer in interval notation as: has the same solution.
𝜋 5𝜋 Part E
𝜃∈( , )
6 6 Remove the absolute value sign:
Part B csc 𝜃 > 2 𝑂𝑅 ⏟
⏟ csc 𝜃 < −2
sin 𝜃 is negative in the third and fourth quadrant: 𝐶𝑎𝑠𝑒 𝐼 𝐶𝑎𝑠𝑒 𝐼𝐼
1 7𝜋 11𝜋
sin 𝜃 = − ⇒ 𝜃 = { , }
2 6 6 1 1
The region we want is above the line segment. That csc 𝜃 > 2 ⇒ > 2 ⇒ sin 𝜃 <
sin 𝜃 2
is, everything except the shaded region.

Example 3.73
Solve the following inequalities over the domain [0,2𝜋). Graph the related trigonometric function and shade
your final answer:
√3
A. sin2 𝜃 ≤ − 2
3
B. sin2 𝜃 ≥ 4

3 √3
Part A sin2 𝜃 ≥ ⇒ |sin 𝜃| ≥
4 2
The square of a positive real quantity cannot be Take square roots:
negative. √3 √3
𝜃∈𝜙 sin 𝜃 ≥ 𝑂𝑅 sin 𝜃 ≤ −
⏟ 2 ⏟ 2
Part B 𝐶𝑎𝑠𝑒 𝐼 𝐶𝑎𝑠𝑒 𝐼𝐼

P a g e 112 | 213
Aziz Manva (azizmanva@gmail.com)

𝑝𝑖 2𝜋 4𝜋 5𝜋
𝜃 ∈ [ , ]∪[ , ]
⏟3 3 ⏟3 3
𝐶𝑎𝑠𝑒 𝐼 𝐶𝑎𝑠𝑒 𝐼𝐼

Example 3.74
Solve the following inequalities over the domain [0,2𝜋). Graph the related trigonometric function and shade
your final answer:
A. tan 𝜃 < 1
B.

Part A

𝜋 𝜋 5𝜋 3𝜋
𝜃 ∈ [0, ) ∪ ( , ) ∪ ( , 2𝜋)
4 2 4 2

Example 3.75
Solve the inequalities below for 𝑥 ∈ ℝ:
√3
A. sin 𝑥 > 2
B. csc 𝑥 > 1
1
C. cos 𝑥 > 2
1
D. |sin 𝑥| ≥ 2
1
E. cos2 𝑥 ≤ 2

Part A
𝜋 2𝜋
( + 2𝑘𝜋, + 2𝑘𝜋) , 𝑘 ∈ ℤ
3 3
Part B
We consider two cases.
Case I:
1
sin 𝑥 < 0 ⇒ < 1 ⇒ 𝑁𝑜 𝑆𝑜𝑙𝑢𝑡𝑖𝑜𝑛𝑠
sin 𝑥
Case I: sin 𝑥 > 0
1
csc 𝑥 > 1 ⇒ > 1 ⇒ 1 > sin 𝑥 ⇒ sin 𝑥 < 1
sin 𝑥
For 𝑘 ∈ ℤ:
𝜋 5𝜋 7𝜋
(2𝑘𝜋, (2𝑘 + 1)𝜋) − { , , , … }
2 2 2

P a g e 113 | 213
Aziz Manva (azizmanva@gmail.com)

Part C
𝜋 𝜋
(2𝑘𝜋 − , 2𝑘𝜋 + ) , 𝑘 ∈ ℤ
3 3

Part D

3.6 Graphs: Modelling


A. Basics
We start by looking at the graph of 𝑦 = sin 𝑥. There are many applications and extensions of what we will learn.
Hence, this is an important set of concepts.

Example 3.76
The graph of sin 𝑥 is drawn alongside.
A. What is the value of sin 𝑥 at the
origin?
B. At what other places does it take
this value?
C. A periodic function is a function that repeats itself at regular intervals. What is the period of sin 𝑥?

Value of sin 𝑥 at the origin is 0. It is also zero at:


{… , −2𝜋, −𝜋, 0𝜋, 𝜋, 2𝜋, … } ⇒ {0 ± 𝑘𝜋, 𝑘 ∈𝕎}
𝑛𝜋, 𝑛∈ℤ

𝑃𝑒𝑟𝑖𝑜𝑑 = 2𝜋
B. Minimum and Maximum

3.77: Peaks and Troughs


➢ The top point/maximum of the graphs are its peaks.
➢ The bottom point/minimum of the graph are its troughs.

Example 3.78
Use the graph from the previous question.
A. What is the maximum value of sin 𝑥?
B. What is the smallest positive value of 𝑥 for which it achieves this maximum?
C. At what other places does it achieve the maximum?

P a g e 114 | 213
Aziz Manva (azizmanva@gmail.com)

D. What is the minimum value of sin 𝑥?


E. What is the smallest positive value of 𝑥 for which it achieves this maximum?
F. At what other places does it achieve the maximum?

Part A
Highest value that sin 𝑥 takes is 1
Part B

𝜋
sin
2
Part C
11𝜋 7𝜋 3𝜋 𝜋 5𝜋 9𝜋 13𝜋 𝜋
{… , − ,− ,− , , , , , … } ⇒ { + 2𝑘𝜋, 𝑘 ∈ ℤ}
2 2 2 2 2 2 2 2
➢ Lowest value that sin 𝑥 takes is −1
3𝜋
✓ { 2 ± 2𝑛𝜋, 𝑛 ∈ ℤ }

Part D
−1

3.79: Midline and Amplitude


The horizontal line that goes midway between the peaks and the troughs.
𝑀𝑎𝑥 + 𝑀𝑖𝑛
𝑀𝑖𝑑𝑙𝑖𝑛𝑒 =
2

The distance between a peak (or a trough) and the midline is called the amplitude.
𝑀𝑎𝑥 − 𝑀𝑖𝑛
𝐴𝑚𝑝𝑙𝑖𝑡𝑢𝑑𝑒 =
2

Example 3.80
A. What is the midline of sin 𝑥?
B. What is the amplitude of sin 𝑥?

𝑀𝑎𝑥 + 𝑀𝑖𝑛 1 − 1 0
𝑀𝑖𝑑𝑙𝑖𝑛𝑒 = = = =0
2 2 2
𝑀𝑎𝑥 − 𝑀𝑖𝑛 1 − (−1)
𝐴𝑚𝑝𝑙𝑖𝑡𝑢𝑑𝑒 = = =1
2 2
C. 𝒚 = 𝒔𝒊𝒏 𝒙 as a Function
We look at 𝑦 = sin 𝑥 as a function. If you have not studied functions before, look at the Note on functions before
proceeding further in this section. (The rest of the chapter should still be fine if you don’t know functions)

Example 3.81
Is the graph of 𝑦 = sin 𝑥 a function based on the vertical line test?

Any vertical line intersects the graph of 𝑦 = sin 𝑥 in a maximum of one place.
Hence, the graph of 𝑦 = sin 𝑥 is a function.

Example 3.82
Is 𝑦 = sin 𝑥 a function based on the algebraic test of a function?

P a g e 115 | 213
Aziz Manva (azizmanva@gmail.com)

Algebraically, we need a unique value of 𝑦 for each value of 𝑥.


This is true, since for any input 𝑥, we get a unique output 𝑦.

3.83: Domain
➢ The domain of a function 𝑦 = 𝑓(𝑥) is the set of acceptable 𝑥 values for the function.
➢ The range of a function 𝑦 = 𝑓(𝑥) is the set of 𝑦 values that the function takes.

Two standard types of expressions which restrict the domain of an expression are:
➢ Fractions: The denominator must be non-zero, since you cannot divide by zero.
➢ Square Roots: In the real number system, the expression inside a square root must be positive.

Example 3.84
A. What is the domain of 𝑓(𝑥) = sin 𝑥?
B. What is the range of 𝑓(𝑥) = sin 𝑥?

There are no restrictions on the value of 𝑥. In particular, 𝑦 = sin 𝑥 does not have square roots or fractions.
Hence, the
𝐷𝑜𝑚𝑎𝑖𝑛 𝑜𝑓 sin 𝑥 = 𝐷𝑓 = ℝ
The maximum value that sin 𝑥 can take is 1.

The minimum value that sin 𝑥 can take is −1.

sin 𝑥 is a continuous function. It takes all values between 1 and -1.

Hence, the
𝑅𝑎𝑛𝑔𝑒 𝑜𝑓 sin 𝑥 = 𝑅𝑓 = [−1,1]

Example 3.85: Intercepts


The 𝑥-intercepts of a function are the roots of the corresponding equation when 𝑓(𝑥) = 0. The 𝑦-intercepts of a
function are the point where 𝑥 = 0. What are the intercepts of 𝑓(𝑥) = sin 𝑥 ?

𝑥 intercept
𝑓(𝑥) = 0 ⇒ sin 𝑥 = 0 ⇒ 𝑥 ∈ {0 ± 𝑛𝜋, 𝑛 ∈ ℤ}

𝑦 intercept
𝑥 = 0 ⇒ sin 𝑥 = 0 ⇒ 𝑦 𝑖𝑛𝑡𝑒𝑟𝑐𝑒𝑝𝑡 = 0

Example 3.86: Symmetry


A circle centered at the origin and of radius 𝜋, is divided by the curve 𝑦 = sin 𝑥 into two parts. The area of one of
the parts is: (JMET 2011/80)

The graph of 𝑦 = sin 𝑥 is symmetrical about the y-axis.


Hence, the area of each part is equal.

Hence, required area:


𝐴𝑟𝑒𝑎 𝑜𝑓 𝐶𝑖𝑟𝑐𝑙𝑒 𝜋𝑟 2 𝜋 3
= = =
2 2 2

P a g e 116 | 213
Aziz Manva (azizmanva@gmail.com)

D. General Trigonometric Graph

3.87: General Trigonometric Graph


Any trigonometric graph of sin 𝑥 can be written in the form:
𝑦 = 𝑎 sin[𝑏(𝑥 + 𝑐)] + 𝑑
Where
2𝜋
𝑑 +𝑎,
⏟ ⏟
𝑑−𝑎, ⏟
𝑐 , ⏟
𝑎 , ⏟
𝑑 ,
𝑯𝒐𝒓𝒊𝒛𝒐𝒏𝒕𝒂𝒍 𝑨𝒎𝒑𝒍𝒊𝒕𝒖𝒅𝒆 𝑴𝒊𝒅𝒍𝒊𝒏𝒆
⏟𝑏
𝑴𝒂𝒙 𝑴𝒊𝒏
𝑺𝒉𝒊𝒇𝒕 𝑽𝒆𝒓𝒕𝒊𝒄𝒂𝒍 𝑺𝒉𝒊𝒇𝒕 𝑯𝒐𝒓𝒊𝒛𝒐𝒏𝒕𝒂𝒍 𝑺𝒄𝒂𝒍𝒊𝒏𝒈
𝑶𝑹 𝑷𝒆𝒓𝒊𝒐𝒅

𝑀𝑎𝑥 = 𝑀𝑖𝑑𝑙𝑖𝑛𝑒 + 𝐴𝑚𝑝𝑙𝑖𝑡𝑢𝑑𝑒 = 𝑑 + 𝑎


𝑀𝑖𝑛 = 𝑀𝑖𝑑𝑙𝑖𝑛𝑒 − 𝐴𝑚𝑝𝑙𝑖𝑡𝑢𝑑𝑒 = 𝑑 − 𝑎

𝑀𝑎𝑥 + 𝑀𝑖𝑛
𝑀𝑖𝑑𝑙𝑖𝑛𝑒 =
2
𝑀𝑎𝑥 − 𝑀𝑖𝑛
𝐴𝑚𝑝𝑙𝑖𝑡𝑢𝑑𝑒 =
2

2𝜋
𝜋=
𝑏
Divide by 𝜋 on both sides:
𝜋 2𝜋
=
𝜋 𝜋𝑏
2
1=
𝑏
𝑏=2

3.88: Graph Sketching Instructions


For all trigonometric graphs, mark
A. Maximum/Crest
B. Minimum/Trough
C. Midline
D. Amplitude
E. 𝑦 intercept
F. 𝑥 intercept

Example 3.89
P a g e 117 | 213
Aziz Manva (azizmanva@gmail.com)

Sketch 𝑦 = sin 𝑥 on plain paper.

E. Vertical Scaling/Amplitude: 𝒂

3.90: Vertical Scaling


The graph of 𝑘𝑓(𝑥) is the graph of 𝑓(𝑥) scaled vertically by a factor of 𝑘.

Example 3.91
Identify the scaling factor in the following functions compared to 𝑓(𝑥):
A. 2𝑓(𝑥)
𝑓(𝑥)
B. 𝜋

𝑉𝑒𝑟𝑡𝑖𝑐𝑎𝑙𝑙𝑦 𝑠𝑐𝑎𝑙𝑒𝑑 𝑏𝑦 𝑎 𝑓𝑎𝑐𝑡𝑜𝑟 𝑜𝑓 2


1
𝑉𝑒𝑟𝑡𝑖𝑐𝑎𝑙𝑙𝑦 𝑠𝑐𝑎𝑙𝑒𝑑 𝑏𝑦 𝑎 𝑓𝑎𝑐𝑡𝑜𝑟 𝑜𝑓 𝑂𝑅 𝑉𝑒𝑟𝑡𝑖𝑐𝑎𝑙𝑙𝑦 𝑠ℎ𝑟𝑢𝑛𝑘 𝑏𝑦 𝑎 𝑓𝑎𝑐𝑡𝑜𝑟 𝑜𝑓𝜋
𝜋

3.92: Amplitude
𝑦 = 𝑎 sin 𝑥
Has amplitude
𝑎

𝑦 = sin 𝑥 ⇒ 𝑀𝑎𝑥 = 1, 𝑀𝑖𝑛 = −1


𝑦 = 𝑎 sin 𝑥 scales 𝑦 = sin 𝑥 vertically by a factor of 𝑎:
𝑀𝑎𝑥 = 𝑎, 𝑀𝑖𝑛 = −𝑎
𝑀𝑎𝑥 − 𝑀𝑖𝑛 𝑎 − (−𝑎) 2𝑎
𝐴𝑚𝑝𝑙𝑖𝑡𝑢𝑑𝑒 = = = =𝑎
2 2 𝑎

Example 3.93
Identify the amplitude of the following, and then sketch the graph:
A. 𝑦 = 3 sin 𝑥
B. 𝑦 = 𝑒 sin 𝑥
C. 𝑦 = 𝜋 sin 𝑥
1
D. 𝑦 = sin 𝑥
𝑒

Parts A and B
3

Part C

P a g e 118 | 213
Aziz Manva (azizmanva@gmail.com)

Part D

F. Horizontal Scaling/Period: 𝒃

(Important) 3.94: Horizontal Scaling


1
The graph of 𝑓(𝑘𝑥) is the graph of 𝑓(𝑥) scaled horizontally by a factor of 𝑘.

1
Note that the function has 𝑘𝑥, and the scaling is by a factor of 𝑘.
In other words, the scaling is the reciprocal of the factor in the function.

3.95: Period
The period of sin 𝑏𝑥 is:
2𝜋
𝑃𝑒𝑟𝑖𝑜𝑑 =
⏟𝑏
𝑷𝒆𝒓𝒊𝒐𝒅

The period for 𝑦 = sin 𝑥 is 2𝜋.


𝑏 is a scaling factor. And since it is a horizontal scaling factor, if the function has 𝑏, then the period is scaled by
its reciprocal
1
𝑏

Example 3.96
Identify the period in the following, and then sketch the graph.
A. 𝑦 = sin 2𝑥
1
B. 𝑦 = sin 3 𝑥
C. 𝑦 = sin 𝜋𝑥
D. 𝑦 = sin 𝑒𝑥
𝑥
E. 𝑦 = sin 𝜋

Parts A and B
2𝜋 2𝜋
𝑃𝑒𝑟𝑖𝑜𝑑 = = =𝜋
𝑏 2
2𝜋 1
𝑃𝑒𝑟𝑖𝑜𝑑 = = 2𝜋 ÷ = 2𝜋 × 3 = 6𝜋
1 3
3
Parts C, D and E

2𝜋 2𝜋
𝑃𝑒𝑟𝑖𝑜𝑑 = = =2
𝑏 𝜋

P a g e 119 | 213
Aziz Manva (azizmanva@gmail.com)

2𝜋
𝑃𝑒𝑟𝑖𝑜𝑑 =
𝑒
2𝜋
𝑃𝑒𝑟𝑖𝑜𝑑 = = 2𝜋 2
1
𝜋
G. Horizontal Shift: 𝒄

3.97: Horizontal Shift


The graph of 𝑓(𝑥 + 𝑘) is shifted horizontally:
➢ left by 𝑘 units, 𝑘 > 0
➢ right by 𝑘 units, 𝑘 < 0

𝑓(𝑥) = sin(𝑥 − 1)
𝑓(1) = sin(1 − 1) = sin 0 = 0

3.98: Shifting Trig Graphs


The graph of 𝑦 = sin(𝑥 + 𝑐) is shifted
➢ Left by 𝑐 units for 𝑐 > 0
➢ Right by 𝑐 units for 𝑐 < 0

Examp
le 3.99
Identify the horizontal shift in the following, and then draw the graph:
A. 𝑦 = sin 𝑥
B. 𝑦 = sin(𝑥 + 𝜋)
𝜋
C. 𝑦 = sin (𝑥 − 2 )
D. 𝑦 = sin(𝑥 + 2𝜋)

𝑁𝑜 ℎ𝑜𝑟𝑖𝑧𝑜𝑛𝑡𝑎𝑙 𝑠ℎ𝑖𝑓𝑡
𝑆ℎ𝑖𝑓𝑡𝑒𝑑 𝑙𝑒𝑓𝑡 𝑏𝑦 𝜋 𝑈𝑛𝑖𝑡
𝜋
𝑆ℎ𝑖𝑓𝑡𝑒𝑑 𝑟𝑖𝑔ℎ𝑡 𝑏𝑦 𝑢𝑛𝑖𝑡𝑠
2
𝑆ℎ𝑖𝑓𝑡𝑒𝑑 𝐿𝑒𝑓𝑡 𝑏𝑦 2𝜋 𝑈𝑛𝑖𝑡𝑠 ⇒ 𝑁𝑜 𝑠ℎ𝑖𝑓𝑡 (𝑏𝑒𝑐𝑎𝑢𝑠𝑒 𝑝𝑒𝑟𝑖𝑜𝑑 𝑖𝑠 2𝜋)
H. Vertical Shift: 𝒅

3.100: Vertical Translations


For 𝑘 > 0:
𝑦 = 𝑓(𝑥) + 𝑘 → 𝑀𝑜𝑣𝑒𝑑 𝑢𝑝 𝑘 𝑢𝑛𝑖𝑡𝑠
𝑦 = 𝑓(𝑥) − 𝑘 → 𝑀𝑜𝑣𝑒𝑑 𝑑𝑜𝑤𝑛 𝑘 𝑢𝑛𝑖𝑡𝑠

Example 3.101
The following have been graphed in the diagram. Identify the graph and
state its color.
A. 𝑦 = sin 𝑥
B. 𝑦 = sin 𝑥 + 1
C. 𝑦 = sin 𝑥 − 1

P a g e 120 | 213
Aziz Manva (azizmanva@gmail.com)

𝐴: 𝑅𝑒𝑑 𝐺𝑟𝑎𝑝ℎ
𝐵: 𝐵𝑙𝑢𝑒 𝐺𝑟𝑎𝑝ℎ
𝐶: 𝐺𝑟𝑒𝑒𝑛 𝐺𝑟𝑎𝑝ℎ

Example 3.102
Identify the max and the min for the following graphs:
A. 𝑦 = sin 𝑥
B. 𝑦 = sin 𝑥 + 2
1
C. 𝑦 = sin 𝑥 − 2
D. 𝑦 = sin 𝑥 + 𝜋

𝑀𝑎𝑥 = 1, 𝑀𝑖𝑛 = −1
𝑀𝑎𝑥 = 1 + 2 = 3, 𝑀𝑖𝑛 = −1 + 2 = 1
1 1 1 3
𝑀𝑎𝑥 = 1 − = , 𝑀𝑖𝑛 = −1 − = −
2 2 2 2
𝑀𝑎𝑥 = 1 + 𝜋, 𝑀𝑖𝑛 = −1 + 𝜋

I. Max and Min

3.103: Max and Min


The max and min for 𝑦 = 𝑎 sin(𝑏𝑥 + 𝑐) + 𝑑 is:
𝑀𝑎𝑥 = 𝑎 + 𝑑
𝑀𝑖𝑛 = −𝑎 + 𝑑

3.104: Midline
𝑀𝑎𝑥 + 𝑀𝑖𝑛 (𝑎 + 𝑑) + (−𝑎 + 𝑑) 2𝑑
𝑀𝑖𝑑𝑙𝑖𝑛𝑒 = = = =𝑑
2 2 𝑑

Example 3.105
Find the max, min and midline for the following functions, and then graph them:
A. 𝑦 = 3 sin(𝑥) + 1
1
B. 𝑦 = 3 sin(𝑥) + 0.4
C. 𝑦 = 𝜋 sin(𝑥) + 𝑒

Part A
𝑎 = 3, 𝑑 = 1
𝑀𝑎𝑥 = 𝑎 + 𝑑 = 4
𝑀𝑖𝑛 = −𝑎 + 𝑑 = −3 + 1 = −2
𝑀𝑖𝑑𝑙𝑖𝑛𝑒 = 𝑑 = 1
Part B
1 4 2
𝑎 = , 𝑑 = 0.4 = =
3 10 5
1 2 5 6 11
𝑀𝑎𝑥 = 𝑎 + 𝑑 = + = + =
3 5 15 15 15
1 2 5 6 1
𝑀𝑖𝑛 = −𝑎 + 𝑑 = − + = − + =
3 5 15 15 15
𝑀𝑖𝑑𝑙𝑖𝑛𝑒 = 𝑑 = 0.4
Part C

P a g e 121 | 213
Aziz Manva (azizmanva@gmail.com)

𝑎 = 𝜋, 𝑑 = 𝑒
𝑀𝑎𝑥 = 𝑎 + 𝑑 = 𝜋 + 𝑒
𝑀𝑖𝑛 = −𝑎 + 𝑑 = −𝜋 + 𝑒
𝑀𝑖𝑑𝑙𝑖𝑛𝑒 = 𝑑 = 𝑒

J. Comparing Transformations

Example 3.106
Vayuna drew the same graph for the two functions below
𝑓(𝑥) = sin(𝑥) + 𝜋, 𝑔(𝑥) = sin(𝑥 + 𝜋)
A. Was she correct? If she wasn’t, explain the difference between the two graphs.
B. Draw the graphs of the two functions.

𝑓(𝑥) is a vertical translation. It moves 𝑦 = sin 𝑥 up by 𝜋 units


𝑔(𝑥) is a horizontal translation. It moves 𝑦 = sin 𝑥 left by 𝜋 units
K. Combining Transformations

Example 3.107
Graph:
A. 𝑦 = sin(3𝑥) + 2
1
B. 𝑦 = 2 sin ( 𝑥)
2

Part A
𝑀𝑎𝑥 = 𝑎 + 𝑑 = 1 + 2 = 3
𝑀𝑖𝑛 = −𝑎 + 𝑑 = −1 + 2 = 1
2𝜋 2𝜋 1
𝑃𝑒𝑟𝑖𝑜𝑑 = = , 𝐻𝑜𝑟𝑖𝑧𝑜𝑛𝑡𝑎𝑙 𝑆𝑐𝑎𝑙𝑖𝑛𝑔 𝐹𝑎𝑐𝑡𝑜𝑟 =
𝑏 3 3
𝐻𝑜𝑟𝑖𝑧𝑜𝑛𝑡𝑎𝑙 𝑆ℎ𝑖𝑓𝑡 = 0
𝑀𝑖𝑑𝑙𝑖𝑛𝑒 = 𝑑 = 2
Part B
𝑉𝑒𝑟𝑡𝑖𝑐𝑎𝑙 𝐷𝑖𝑙𝑎𝑡𝑖𝑜𝑛 𝐹𝑎𝑐𝑡𝑜𝑟 = 2
𝑀𝑎𝑥 = 𝑎 + 𝑑 = 2 + 0 = 2
𝑀𝑖𝑛 = −𝑎 + 𝑑 = −2 + 0 = −2
2𝜋
𝑃𝑒𝑟𝑖𝑜𝑑 = = 4𝜋, 𝐻𝑜𝑟𝑖𝑧𝑜𝑛𝑡𝑎𝑙 𝑆𝑐𝑎𝑙𝑖𝑛𝑔 𝐹𝑎𝑐𝑡𝑜𝑟 = 2
1
2
𝐻𝑜𝑟𝑖𝑧𝑜𝑛𝑡𝑎𝑙 𝑆ℎ𝑖𝑓𝑡 = 0
𝑀𝑖𝑑𝑙𝑖𝑛𝑒 = 𝑑 = 0

Example 3.108
1 1
𝑦 = sin ( 𝑥 + 𝜋) = sin ( (𝑥 + 2𝜋))
⏟ 2 ⏟ 2
𝑷𝒂𝒓𝒕 𝑨 𝑷𝒂𝒓𝒕 𝑩

Part A 𝑀𝑖𝑛 = −𝑎 + 𝑑 = −1 + 0 = −1
𝑉𝑒𝑟𝑡𝑖𝑐𝑎𝑙 𝐷𝑖𝑙𝑎𝑡𝑖𝑜𝑛 𝐹𝑎𝑐𝑡𝑜𝑟 = 1 2𝜋
𝑃𝑒𝑟𝑖𝑜𝑑 = = 4𝜋, 𝐻𝑜𝑟𝑖𝑧𝑜𝑛𝑡𝑎𝑙 𝑆𝑐𝑎𝑙𝑖𝑛𝑔 = 2
𝑀𝑎𝑥 = 𝑎 + 𝑑 = 1 + 0 = 1 1
2
P a g e 122 | 213
Aziz Manva (azizmanva@gmail.com)

𝐻𝑜𝑟𝑖𝑧𝑜𝑛𝑡𝑎𝑙 𝑆ℎ𝑖𝑓𝑡 = 𝜋 𝑡𝑜 𝑡ℎ𝑒 𝐿𝑒𝑓𝑡 𝐻𝑜𝑟𝑖𝑧𝑜𝑛𝑡𝑎𝑙 𝑆ℎ𝑖𝑓𝑡 = 2𝜋


𝑀𝑖𝑑𝑙𝑖𝑛𝑒 = 𝑑 = 0 𝑀𝑖𝑑𝑙𝑖𝑛𝑒 = 𝑑 = 0
You need to apply the transformations in the
reverse order of operations: You still need to apply the transformations in the
Shift 𝑦 = sin 𝑥 to the left by 𝜋 units: reverse order of operations.

Stretch it horizontally by a factor of 2:

Stretch it horizontally by a factor of 2:

Shift the above graph to the left by 2𝜋 units:

Part B
𝑀𝑎𝑥 = 𝑎 + 𝑑 = 1 + 0 = 1
𝑀𝑖𝑛 = −𝑎 + 𝑑 = −1 + 0 = −1
2𝜋
𝑃𝑒𝑟𝑖𝑜𝑑 = = 4𝜋
1
2

Example 3.109
𝜋 𝜋
𝑦 = 2 sin (2𝑥 + ) + 1 = 2 sin (2 (𝑥 + )) + 1
⏟ 2 ⏟ 4
𝑷𝒂𝒓𝒕 𝑨 𝑷𝒂𝒓𝒕 𝑩

Part A
𝑉𝑒𝑟𝑡𝑖𝑐𝑎𝑙 𝐷𝑖𝑙𝑎𝑡𝑖𝑜𝑛 𝐹𝑎𝑐𝑡𝑜𝑟 = 2
𝑉𝑒𝑟𝑡𝑖𝑐𝑎𝑙 𝑆ℎ𝑖𝑓𝑡 = 1
𝑀𝑎𝑥 = 𝑎 + 𝑑 = 2 + 1 = 3
𝑀𝑖𝑛 = −𝑎 + 𝑑 = −2 + 1 = −1
2𝜋 1
𝑃𝑒𝑟𝑖𝑜𝑑 = = 𝜋, 𝐻𝑜𝑟𝑖𝑧𝑜𝑛𝑡𝑎𝑙 𝑆𝑐𝑎𝑙𝑖𝑛𝑔 𝐹𝑎𝑐𝑡𝑜𝑟 =
2 2
𝜋
𝐻𝑜𝑟𝑖𝑧𝑜𝑛𝑡𝑎𝑙 𝑆ℎ𝑖𝑓𝑡 = 𝑡𝑜 𝑡ℎ𝑒 𝑙𝑒𝑓𝑡
2
𝑀𝑖𝑑𝑙𝑖𝑛𝑒 = 𝑑 = 1
The key is to be careful about the order in which you apply the horizontal shift and the horizontal scale.
You need to apply the transformations in the reverse order of operations.

P a g e 123 | 213
Aziz Manva (azizmanva@gmail.com)

➢ Graph I: 𝑦 = 2 sin(𝑥) + 1
𝜋 𝜋
➢ Graph II: Shift Graph I horizontally to the left by 2 units to get = 2 sin (𝑥 + 2 ) + 1
1
➢ Graph III: Scale Graph II horizontally by a factor of 2 to make its period 𝜋 from 2𝜋.

Example 3.110
Find the range of
A. 2 − 7 sin(45𝑥), where 𝑥 is in degrees

𝑀𝑖𝑑𝑙𝑖𝑛𝑒 = 2
𝐴𝑚𝑝𝑙𝑖𝑡𝑢𝑑𝑒 = 7
𝑀𝑎𝑥 = 2 + 7 = 9
𝑀𝑖𝑛 = 2 − 7 = −5
𝑅𝑎𝑛𝑔𝑒 ∈ [−5,9]

Example 3.111
Give an equation with Max is (4,2), Min is (8,-4)

𝑦 = 𝑎 sin(𝑏𝑥 + 𝑐) + 𝑑
2 − (−4)
𝐴𝑚𝑝𝑙𝑖𝑡𝑢𝑑𝑒 = =3=𝑎
2
−4 + 2
𝑀𝑖𝑑𝑙𝑖𝑛𝑒 = 𝑉𝑒𝑟𝑡𝑖𝑐𝑎𝑙 𝑆ℎ𝑖𝑓𝑡 = = −1 = 𝑑
2
𝑃𝑒𝑟𝑖𝑜𝑑 = 16

𝑃ℎ𝑎𝑠𝑒 𝑆ℎ𝑖𝑓𝑡 = 2
𝜋
𝑦 = 3 sin ( (𝑥 − 2)) − 1
4

Example 3.112
Give an equation with Max is (7,6), Min is (12,-2)
Input in degree

𝑦 = 𝑎 sin(𝑏𝑥 + 𝑐) + 𝑑
6 − (−2)
𝐴𝑚𝑝𝑙𝑖𝑡𝑢𝑑𝑒 = =4=𝑎
2
6−2
𝑀𝑖𝑑𝑙𝑖𝑛𝑒 = 𝑉𝑒𝑟𝑡𝑖𝑐𝑎𝑙 𝑆ℎ𝑖𝑓𝑡 = =2=𝑑
2
P a g e 124 | 213
Aziz Manva (azizmanva@gmail.com)

𝑀𝑎𝑥 𝑡𝑜 𝑀𝑖𝑛 = 12 − 7 = 5
𝑃𝑒𝑟𝑖𝑜𝑑 = 2(5) = 10

2𝜋 2𝜋 𝜋
= 10 ⇒ 𝑏 = =
𝑏 10 5

𝑃ℎ𝑎𝑠𝑒 𝑆ℎ𝑖𝑓𝑡 = 2
𝜋
𝑦 = 4 sin ( (𝑥 − 4.5)) + 2
5
180
Conversion factor from radians to degrees is 𝜋
:
180 𝜋
𝑦 = 4 sin ( × (𝑥 − 4.5)) + 2
𝜋 5
𝑦 = 4 sin(36(𝑥 − 4.5)) + 2

L. Modelling
Trigonometric equations exhibit periodicity. Many natural phenomena also exhibit periodicity. Hence,
trigonometric functions are useful while modelling
➢ Biology
✓ Population of certain animals over a period of time
➢ Weather:
✓ Temperature
✓ Tides
✓ Rainfall
➢ Circular Motion: When an object moves along the circumference of a circle, or it rotates, it is called
circular motion. An important case of circular motion is uniform circular motion, where the angular rate
of rotation is constant. Examples of circular motion include:
✓ The movement of a Ferris Wheel
✓ Rotation of the earth about its axis
➢ Simple Harmonic Motion: The motion of a frictionless spring which has a weight attached to it, and then
moved from its equilibrium position follows Hooke’s Law. If you plot the vertical position on the y axis,
and time on the 𝑥 axis, you get a sine curve.
➢ Waves can be modelled using trigonometric functions. Examples of waves include:
✓ Waves in the Ocean
✓ Sound Waves
✓ Electromagnetic Waves

M. Ferris Wheels

Example 3.113
A Ferris wheel has a diameter of 11 𝑚𝑒𝑡𝑒𝑟𝑠. Its center is 7.5 𝑚𝑒𝑡𝑒𝑟𝑠 above the ground. Vyas sits at the bottom of
the Ferris wheel at time 𝑡 = 0. The Ferris Wheel completes one revolution in 5 minutes. Model this using a
suitable trigonometric function of the form
𝑦 = 𝑎 sin(𝑏𝑥 + 𝑐) + 𝑑
A. What is the amplitude 𝑎?
B. What is the midline?
C. What is the vertical shift 𝑑?

P a g e 125 | 213
Aziz Manva (azizmanva@gmail.com)

D. What is the period? Hence, what is the value of 𝑏?


E. What is the horizontal shift 𝑐?
F. What is the function that models Vyas’ height at time 𝑡?

𝑀𝑎𝑥 − 𝑀𝑖𝑛 13 − 2 11
𝐴𝑚𝑝𝑙𝑖𝑡𝑢𝑑𝑒 = 𝑎 = = = = 5.5
2 2 2
Note that the amplitude is simply the radius of the Ferris wheel.

𝑀𝑎𝑥 + 𝑀𝑖𝑛 13 + 2 15
𝑀𝑖𝑑𝑙𝑖𝑛𝑒 = = = = 7.5
2 2 2
Note that the midline is simply the height of the center of the Ferris wheel.

The vertical shift matches the midline:


𝑉𝑒𝑟𝑡𝑖𝑐𝑎𝑙 𝑆ℎ𝑖𝑓𝑡 = 𝑑 = 7.5

𝑃𝑒𝑟𝑖𝑜𝑑 = 5 𝑚𝑖𝑛
Substitute the time take for the Ferris wheel to go around once (5 min) in the equation for Period, which is
2𝜋
𝑃𝑒𝑟𝑖𝑜𝑑 =
𝑏
2𝜋 2𝜋
5= ⇒𝑏=
𝑏 5

At time 𝑡 = 0, the height is ℎ = 2, which means the graph is at a minimum.


𝜋
𝐻𝑜𝑟𝑖𝑧𝑜𝑛𝑡𝑎𝑙 𝑆ℎ𝑖𝑓𝑡 = 𝑢𝑛𝑖𝑡𝑠 𝑡𝑜 𝑡ℎ𝑒 𝑙𝑒𝑓𝑡
2
2𝜋
Substitute 𝑎 = 5.5, 𝑑 = 7.5, 𝑏 = 5
in 𝑦 = 𝑎 sin(𝑏𝑥 + 𝑐) + 𝑑
2𝜋 𝜋
𝑦 = 5.5 sin ( 𝑡 + ) + 7.5
5 2

Example 3.114
You are on a Ferris wheel of diameter 30 meters. It makes one complete revolution every 90 seconds.
A. If you get on at the bottom, write a model for your vertical position assuming the wheel stands on a 2
meters high platform.
B. Calculate the height of your seat after 15 seconds.
C. When are you at a height of 28 meters?

𝑀𝑎𝑥 − 𝑀𝑖𝑛 30
𝐴𝑚𝑝𝑙𝑖𝑡𝑢𝑑𝑒 = = = 15
2 2
𝑀𝑖𝑑𝑙𝑖𝑛𝑒 = 𝐴𝑚𝑝𝑙𝑖𝑡𝑢𝑑𝑒 + 𝑀𝑖𝑛 = 2 + 15 = 17

N. Tides

Example 3.115
At a certain location, high tide occurred at 9 am, with a water height of 8.2 feet. The next low tide occurred at 3
pm, with a water height of 0.6 feet.
A. Write an equation that represents this situation starting from 𝑡𝑖𝑚𝑒 𝑡 = 0.

P a g e 126 | 213
Aziz Manva (azizmanva@gmail.com)

Let 𝑡 be the number of hours after midnight. We want an equation of the form:
𝑦 = 𝑎 𝑠𝑖𝑛 𝑏(𝑡 − 𝑐) + 𝑑
𝑀𝑎𝑥 − 𝑀𝑖𝑛 8.2 − 0.6 7.6
𝐴𝑚𝑝𝑙𝑖𝑡𝑢𝑑𝑒 = 𝑎 = = = = 3.8
2 2 2
𝑀𝑎𝑥 + 𝑀𝑖𝑛 8.2 + 0.6 8.8
𝑀𝑖𝑑𝑙𝑖𝑛𝑒 = 𝑑 = = = = 4.4
2 2 2

𝐻𝑖𝑔ℎ 𝑇𝑖𝑑𝑒 𝑡𝑜 𝐿𝑜𝑤 𝑇𝑖𝑑𝑒 = 6 𝐻𝑜𝑢𝑟𝑠


𝐻𝑖𝑔ℎ 𝑇𝑖𝑑𝑒 𝑡𝑜 𝐻𝑖𝑔ℎ 𝑇𝑖𝑑𝑒 = 12 𝐻𝑜𝑢𝑟𝑠
2𝜋 𝜋
𝑃𝑒𝑟𝑖𝑜𝑑 = = 12 ⇒ 𝑏 =
𝑏 6
So far, we have:
𝜋
𝑦 = 3.8 sin [ (𝑡 − 6)] + 4.4
6

Example 3.116
At 2: 00 𝑝𝑚, at high tide, you find that the depth of the water is 1.5 𝑚. At 7: 30 𝑝𝑚, at low tide, the depth of the
water is 1.1 𝑚. Assume that the depth varies sinusoidally with time.
a. Find a particular equation expressing depth as a function of time that has elapsed since 12: 00 𝑎𝑚, August 2.
b. Use your mathematical model to predict the depth of the water at 5: 00 𝑝𝑚 on August 3.
c. At what time does the first low tide occur on August 3?
d. What is the earliest time on August 3 that the water depth will be at 1.27 𝑚? Give your answer correct to
seconds.

Part A equation at 2.75 hours. Hence, we shift it right by


𝑀𝑎𝑥 = 1.5 0.25 hours to get:
𝑀𝑖𝑛 = 1.1 2𝜋
𝑀𝑎𝑥 + 𝑀𝑖𝑛 1.5 + 1.1 𝑓(𝑡) = sin ( (𝑡 − 0.25))
𝑑 = 𝑀𝑖𝑑𝑙𝑖𝑛𝑒 = = = 1.3 11
2 2 And add the amplitude and midline to get the final
𝑃𝑒𝑟𝑖𝑜𝑑 = 2(7.30 𝑝𝑚 − 2.00 𝑝𝑚) = 2(5.5) equation:
= 11 ℎ𝑜𝑢𝑟𝑠 2𝜋
2𝜋 2𝜋 𝑓(𝑡) = 0.2 sin ( (𝑡 − 0.25)) + 1.3
= 11 ⇒ 𝑏 = 11
𝑏 11
𝑀𝑎𝑥 − 𝑀𝑖𝑛 1.5 − 1.1 0.4 Part B
𝑎 = 𝐴𝑚𝑝𝑙𝑖𝑡𝑢𝑑𝑒 = = = 5: 00 𝑝𝑚 on August 3 is
2 2 2
= 0.2 41 ℎ𝑜𝑢𝑟𝑠 𝑎𝑓𝑡𝑒𝑟 12: 00 𝑎𝑚 𝑜𝑛 𝐴𝑢𝑔 2 ⇒ 𝑡 = 41
High Tide is at: We need to find:
2.00 𝑝𝑚 − 11 ℎ𝑜𝑢𝑟𝑠 = 3 𝑎𝑚 𝑓(41) = 1.11
Consider Part C
2𝜋 7.30 𝑝𝑚, 𝐴𝑢𝑔 2 + 11 𝐻𝑜𝑢𝑟𝑠 = 6.30 𝑎𝑚, 𝐴𝑢𝑔 3
𝑓(𝑡) = sin ( 𝑡)
11
Max, which is high tide, occurs in the above

O. Ocean Waves

Example 3.117
You observe a buoy in the ocean bobbing up and down a total distance of 8 feet. It starts at the bottom of a wave
and completes four cycles in one minute. Write a model that describes its vertical position in terms of 𝑡 minutes.

P a g e 127 | 213
Aziz Manva (azizmanva@gmail.com)

𝑀𝑎𝑥 − 𝑀𝑖𝑛 8
𝐴𝑚𝑝𝑙𝑖𝑡𝑢𝑑𝑒 = = =4
2 2
𝑀𝑖𝑑𝑙𝑖𝑛𝑒 = 𝑑 = 0
2𝜋 1
𝑃𝑒𝑟𝑖𝑜𝑑 = = ⇒ 𝑏 = 8𝜋
𝑏 4
The graph of sin x needs to be shifted one fourth of a cycle to the right, which is:
1 1 1
− × =−
4 4 16
Hence, the final answer is:
1
𝑦 = 4 sin (8𝜋 (𝑡 − ))
16

P. Biology

Example 3.118
Naturalists find that populations of some kinds of predatory animals vary periodically with time. Assume that
the population of foxes in a certain forest varies sinusoidally with time. Records started being kept at time 𝑡 = 0
year. A minimum number of 200 foxes appeared at 𝑡 = 2.9 years. The next maximum, 800 foxes, occurred at 𝑡 =
5.1 years.
A. Sketch the graph of this sinusoid.
B. Find a particular equation expressing the number of foxes as a function of time.
C. Predict the fox population when 𝑡 = 7, 8, 9, and 10 years.
D. Suppose foxes are declared a vulnerable species when their population drops below 300. Between what
two non-negative values of 𝑡 did the foxes become vulnerable?
Q. Sound

Example 3.119
The hum you hear on some radios when they are not tuned to a station is a sound wave of 60 cycles per second.
A. Is 60 cycles per second the period or is it the frequency? If it is the period, find the frequency. If it is the
frequency, find the period.
B. The wavelength of a sound wave is defined as the distance the wave travels in a time interval equal to
one period. If sound travels at 1100 𝑓𝑡/𝑠, find the wavelength of the 60-cycle-per-second hum.
C. The lowest musical note the human ear can hear is about 16 cycles per second. In order to play such a
note, a pipe on an organ must be exactly half as long as the wavelength. What length organ pipe would
be needed to generate a 16-cycle-per-second note?

Part A
𝑐𝑦𝑐𝑙𝑒𝑠
𝐹𝑟𝑒𝑞𝑢𝑒𝑛𝑐𝑦 = 𝑓 = 60
𝑠𝑒𝑐𝑜𝑛𝑑
1 1
𝑃𝑒𝑟𝑖𝑜𝑑 = = 𝑆𝑒𝑐𝑜𝑛𝑑𝑠
𝑓 60
Part B
𝑓𝑡
𝑆𝑝𝑒𝑒𝑑 𝑜𝑓 𝑆𝑜𝑢𝑛𝑑 = 1100
𝑠
The wave length is the distance travelled in one period:
𝑓𝑡 1 1100 110 55 1
𝑊𝑎𝑣𝑒𝑙𝑒𝑛𝑔𝑡ℎ = 𝐷𝑖𝑠𝑡𝑎𝑛𝑐𝑒 = 1100 × 𝑠𝑒𝑐 = = = = 18 𝑓𝑡
⏟ 𝑠𝑒𝑐 ⏟
60 60 6 3 3
𝑆𝑝𝑒𝑒𝑑 𝑇𝑖𝑚𝑒
Part C

P a g e 128 | 213
Aziz Manva (azizmanva@gmail.com)

1 1100 1 3
𝑃𝑖𝑝𝑒 𝐿𝑒𝑛𝑔𝑡ℎ = 𝑊𝑎𝑣𝑒𝑙𝑒𝑛𝑔𝑡ℎ × = × = 34
2 16 2 8

3.7 Graphs: Transformations


A. Basics

Example 3.120
A. The graph of cos 𝑥 is the graph of sin 𝑥
shifted by 𝑎 units. Write the graph in this
form.
B. What are the roots of 𝑦 = cos 𝑥?
C. What is the period of 𝑦 = cos 𝑥 ?
D. What is the amplitude?
E. What is the midline?
F. What is the domain of 𝑦 = cos 𝑥?
G. What is the range of 𝑦 = cos 𝑥?

Part A
𝜋
𝑦 = sin (𝑥 + )
2
Part B
3𝜋 𝜋 𝜋 3𝜋 5𝜋
𝑥 = {… , − , − , , , ,…}
2 2 2 2 2
𝑛𝜋
𝑥= , 𝑛 𝑖𝑠 𝑜𝑑𝑑 𝑖𝑛𝑡𝑒𝑔𝑒𝑟
2
Part C
Shifting the graph of 𝑦 = sin 𝑥 to the left (as we did in Part A) does not change its period. Hence,
𝑃𝑒𝑟𝑖𝑜𝑑 = 2𝜋
Part D
𝐴𝑚𝑝𝑙𝑖𝑡𝑢𝑑𝑒 = 1
Part E
𝑦=0
Part F
𝐷𝑜𝑚𝑎𝑖𝑛 = (−∞, ∞) = ℝ
Part G
𝑅𝑎𝑛𝑔𝑒 = [−1,1]
B. Graphing

Example 3.121
Identify the transformation applied to 𝑦 = cos 𝑥 to obtain the following functions.
A. 𝑦 = cos 𝑥 + 2
B. 𝑦 = cos(𝑥 + 2)
C. 𝑦 = 2 cos 𝑥
𝑥
D. 𝑦 = cos (2)

Example 3.122
Identify the midline, amplitude, period and range for the function below:

P a g e 129 | 213
Aziz Manva (azizmanva@gmail.com)

𝑥
𝑦 = 3 cos ( + 1) − 2
𝜋

𝑀𝑖𝑑𝑙𝑖𝑛𝑒: 𝑦 = −2
𝐴𝑚𝑝𝑙𝑖𝑡𝑢𝑑𝑒 = 3
2𝜋
𝑃𝑒𝑟𝑖𝑜𝑑 = = 2𝜋 2
1
𝜋
𝑅𝑎𝑛𝑔𝑒 = [1, −5]
C. 𝐭𝐚𝐧 𝒙

3.123: Domain
The domain of a function 𝑦 = 𝑓(𝑥) is the set of acceptable 𝑥 values for the function.

Two standard types of expressions which restrict the domain of an expression are:
➢ Fractions: The denominator must be non-zero, since you cannot
divide by zero.
➢ Square Roots: In the real number system, the expression inside a
square root must be positive.

Example 3.124
What is the domain of 𝑓(𝑥) = tan 𝑥?

sin 𝑥
tan 𝑥 =
cos 𝑥
The above is not defined when cos 𝑥 = 0, which is precisely when:
𝜋
𝑥 = + 𝑘𝜋, 𝑘∈ℤ
2

3.125: Domain of 𝒕𝒂𝒏 𝒙


𝜋
𝐷𝑜𝑚𝑎𝑖𝑛 = ℝ − { + 𝑘𝜋, 𝑘 ∈ℤ}
2

The domain of tan 𝑥 is all real numbers except the values when cos 𝑥 = 0, or the roots of 𝑦 = cos 𝑥.

Example 3.126
tan 𝑥
What is the domain of 𝑓(𝑥) = sec 𝑥?

sin 𝑥
tan 𝑥 cos 𝑥
=
sec 𝑥 1
cos 𝑥

Since we have cos 𝑥 in the denominator in two places, we cannot have


𝜋
cos 𝑥 = 0 ⇒ 𝑥 = { ± 𝑘𝜋, 𝑘 ∈ ℕ}
2

Example 3.127
tan 𝑥
What is the domain of 𝑓(𝑥) = sin 𝑥
?

P a g e 130 | 213
Aziz Manva (azizmanva@gmail.com)

sin 𝑥
tan 𝑥 cos 𝑥
=
sin 𝑥 sin 𝑥
𝜋
cos 𝑥 = 0 ⇒ 𝑥 = { ± 𝑘𝜋, 𝑘 ∈ ℕ} = 𝐴
2
sin 𝑥 = 0 ⇒ 𝑥 = {𝑘𝜋, 𝑘 ∈ ℤ} = 𝐵
𝐷𝑜𝑚𝑎𝑖𝑛 = ℝ − (𝐴 ∪ 𝐵)

Example 3.128
What is the range of 𝑦 = tan 𝑥

𝑅𝑎𝑛𝑔𝑒 = ℝ

Example 3.129
What are the roots of 𝑦 = tan 𝑥?

𝑥 = 𝑛𝜋, 𝑛∈ℤ

Example 3.130
𝑥
The period of 𝑦 = tan ( ) is the same as the period of 𝑦 = sin(𝑥𝑏𝜋). The value of 𝑎𝑏 is equal to a constant.
𝑎𝜋
Determine the constant.

𝜋 2𝜋 2 2
= ⇒ 𝜋 × 𝑎𝜋 = ⇒ 𝑎𝑏 = 2
1 𝑏𝜋 𝑏 𝜋
𝑎𝜋
D. Sums of Functions

Example 3.131
Period
Maximum and Minimum
Range

3.8 Angle Equations


A. Finding 𝒏 when 𝒏𝜽 = 𝟎

3.132: Point on the Unit Circle


A point P on the unit circle has coordinates
(𝑥, 𝑦) = (cos 𝜃 , sin 𝜃)
Where 𝜃 is the angle in standard position

Example 3.133
𝜋
Solve 𝑛𝜃 = 0 if 𝜃 = 4 and 𝑛 ∈ ℕ.

P a g e 131 | 213
Aziz Manva (azizmanva@gmail.com)

𝑛 = 0 is a solution but 0 is not a natural number.


Method I
𝑛𝜃 = 0, 𝑛 ∈ ℕ
𝜋
Consider the first few natural number multiples of 4 :
𝜋 2𝜋 3𝜋 4𝜋 5𝜋 6𝜋 7𝜋 8𝜋
{ , , , , , , , }
4 4 4 4 4 4 4 4

8𝜋
= 2𝜋 = 0
4
𝑛 = 8 𝑖𝑠 𝑎 𝑠𝑜𝑙𝑢𝑡𝑖𝑜𝑛

In fact,
𝑛 = 8𝑘, 𝑘 ∈ ℕ 𝑖𝑠 𝑎 𝑠𝑜𝑙𝑢𝑡𝑖𝑜𝑛

3.134: Adding 𝟐𝝅 to an angle


Adding any integer multiple of 2𝜋 to an angle does not change its value.
𝜃 + 2𝑘𝜋 = 𝜃, 𝑘 ∈ ℤ

Example 3.135
𝜋
Solve 𝑛𝜃 = 0 if 𝜃 = 4 and 𝑛 ∈ ℕ.

𝜋
𝑛( ) = 0
4
Since we can add 2𝜋 to any angle without changing its value, this equation should be written
𝜋
𝑛 ( ) = 0 + 2𝑘𝜋, 𝑘 ∈ ℕ
4

For 𝑘 ∈ ℕ:
𝜋
𝑛 ( ) = 2𝑘𝜋
4
4
Multiply both sides by 𝜋:
4
𝑛 = 2𝑘𝜋 × = 8𝑘
𝜋

Example 3.136
Solve 𝑛𝜃 = 0 if 𝑛 ∈ ℕ
𝜋
A. 𝜃 =
3
𝜋
B. 𝜃 = 8
𝜋
C. 𝜃 = 2

Part A
𝜋
𝑛 ( ) = 2𝑘𝜋 ⇒ 𝑛 = 6𝑘, 𝑘 ∈ ℕ
3
Part B
𝜋
𝑛 ( ) = 2𝑘𝜋 ⇒ 𝑛 = 16𝑘, 𝑘 ∈ ℕ
8
Part C

P a g e 132 | 213
Aziz Manva (azizmanva@gmail.com)

𝜋
𝑛 ( ) = 2𝑘𝜋 ⇒ 𝑛 = 4𝑘, 𝑘 ∈ ℕ
2

3.137: Fractions
At a minimum 𝑘 must be a natural number, but if your final answer has a denominator, then you may need to
apply a stronger condition.

Example 3.138
Solve 𝑛𝜃 = 0 if 𝑛 ∈ ℕ
3𝜋
A. 𝜃 = 4

3𝜋 8𝑘
𝑛(
) = 2𝑘𝜋 ⇒ 𝑛 =
4 3
But we have to be careful here. The LHS is a natural number. Hence, the RHS must also be a natural number.

8
𝑘=1⇒𝑛= ⇒ 𝑁𝑜𝑡 𝑎 𝑛𝑎𝑡𝑢𝑟𝑎𝑙 𝑛𝑢𝑚𝑏𝑒𝑟 ⇒ 𝑁𝑜𝑡 𝑉𝑎𝑙𝑖𝑑
3
8𝑘
For to be a natural number 𝑘 must be divisible by 3, and hence the final answer will be:
3
8𝑘
𝑛= , 𝑘 = 3𝑥, 𝑥 ∈ ℕ
3

Example 3.139
Solve 𝑛𝜃 = 0 if 𝑛 ∈ ℕ
5𝜋
A. 𝜃 = 3

5𝜋 6𝑘
𝑛( ) = 2𝑘𝜋 ⇒ 𝑛 =
3 5
Where
𝑘 𝑖𝑠 𝑎 𝑝𝑜𝑠𝑖𝑡𝑖𝑣𝑒 𝑚𝑢𝑙𝑡𝑖𝑝𝑙𝑒 𝑜𝑓 5
B. Finding 𝜽 when 𝒏𝜽 = 𝟎

Example 3.140
Solve 𝑛𝜃 = 0 if 𝑛 = 2 and 𝑛 ∈ ℕ.
Illustrate your solutions on the unit circle.

Example 3.141
Solve 𝑛𝜃 = 0 if 𝑛 = 4.
Illustrate your solutions on the unit circle.

𝑛𝜃 = 0
4𝜃 = 2𝑘𝜋, 𝑘 ∈ ℤ
𝑘
𝜃 = 𝜋, 𝑘 ∈ ℤ
2
𝜋 3𝜋
𝜃 = { , 𝜋, , 2𝜋} + 2𝐾𝜋, 𝐾 ∈ ℤ
2 2
Example 3.142

P a g e 133 | 213
Aziz Manva (azizmanva@gmail.com)

Solve 𝑛𝜃 = 0 if 𝑛 = 8.
Illustrate your solutions on the unit circle.

𝑘
8𝜃 = 2𝑘𝜋 ⇒ 𝜃 = 𝜋, 𝑘 ∈ ℤ
4
𝜋 2𝜋 8𝜋
𝜃={ , , … , } + 2𝐾𝜋, 𝐾 ∈ ℤ
4 4 4
𝜋 2𝜋 3𝜋 4𝜋
𝜃 = { , , , } + 𝐾𝜋, 𝐾 ∈ ℤ
4 4 4 4

3.143: General Solution for 𝜽


2𝑘𝜋
𝑛𝜃 = 0 ⇒ 𝜃 = ,𝑘 ∈ ℤ
𝑛

𝑛𝜃 = 0
𝑛𝜃 = 2𝑘𝜋
2𝑘𝜋
𝜃= ,𝑘 ∈ ℤ
𝑛
C. Solving general angles

Example 3.144
𝜋 𝜋
Solve 𝑛𝜃 = if 𝜃 = and 𝑛 ∈ ℕ.
2 4

𝜋
𝑛𝜃 =
2
𝜋
𝑛𝜃 = + 2𝑘𝜋
2
𝜋 𝜋
𝑛 ( ) = + 2𝑘𝜋
4 2
4
Multiply both sides by 𝜋:
𝑛 = 2 + 8𝑘, 𝑘 ∈ 𝕎

3.145: Questions with no solutions

Challenge 3.146
3𝜋 5𝜋
Solve 𝑛𝜃 = if 𝜃= and 𝑛 ∈ ℕ.
4 3

3𝜋
𝑛𝜃 =
4
5𝜋 3𝜋
𝑛( ) = + 2𝑘𝜋
3 4
12
Multiply both sides by 𝜋
:
20𝑛 = 9 + 24𝑘
9 + 24𝑘
𝑛=
20

P a g e 134 | 213
Aziz Manva (azizmanva@gmail.com)

20 + 24𝑘 − 11
𝑛=
20
24𝑘 − 11
𝑛 =1+
20

24𝑘
⏟ − 11
⏟ = 𝑆𝑜𝑚𝑒 𝑂𝑑𝑑 𝑁𝑢𝑚𝑏𝑒𝑟 ⇒ 𝑁𝑜𝑡 𝑑𝑖𝑣𝑖𝑠𝑖𝑏𝑙𝑒 𝑏𝑦 20
𝐸𝑣𝑒𝑛 𝑂𝑑𝑑
𝑛∈𝜙
3.9 Composite Trigonometric Functions
A. Basics

3.147: Domain of a Composite function


𝑓(𝑔(𝑥))

To determine the domain of a composite function


Step I: Determine the domain of the inner function. (Because to be a valid input, it must be in the domain of the
inner function:
𝐷𝑔 = 𝐷𝑜𝑚𝑎𝑖𝑛 𝑜𝑓𝑔

Step II: Determine the domain of the outer function


𝐷𝑓 = 𝐷𝑜𝑚𝑎𝑖𝑛 𝑜𝑓𝑓

Step III: Remove from 𝐷𝑔 all elements whose output is not in the domain of 𝑓.

Example 3.148
𝑓(𝑥) = tan 𝑥 , 𝑔(𝑥) = √𝑥
A. Determine the domain of 𝑔(𝑓(𝑥)).
B. Determine the domain of 𝑓(𝑔(𝑥)).

Part A
The composite function is given by:
𝑔(𝑓(𝑥)) = √tan 𝑥

Step I: Domain of Inner Function


𝑘𝜋
𝑓(𝑥) = tan 𝑥 ⇒ 𝐷tan 𝑥 = 𝐴𝑙𝑙 𝑟𝑒𝑎𝑙 𝑛𝑢𝑚𝑏𝑒𝑟𝑠 𝑒𝑥𝑐𝑒𝑝𝑡 , 𝑘 𝑖𝑠 𝑜𝑑𝑑 𝑖𝑛𝑡𝑒𝑔𝑒𝑟
2

Step II: Domain of Outer Function


𝑔(𝑥) = √𝑥 ⇒ 𝐷𝑔 𝑖𝑠 𝑥 ≥ 0
Hence, any valid input for the outer function must be non-negative.

Step III: Ensure range of inner function is a subset of domain of Outer Function
We must that the output (range) of the inner function meets the requirements of the outer function.
𝜋
tan 𝑥 ≥ 0 ⇒ 𝑥 ∈ [𝑛𝜋, 𝑛𝜋 + ) , 𝑛 ∈ ℤ
2
𝜋
𝐷𝑓(𝑔(𝑥)) 𝑖𝑠 𝑡ℎ𝑒 𝑠𝑒𝑡 [𝑛𝜋𝑥, 𝑛𝜋𝑥 + ) , 𝑛 ∈ ℤ
2

P a g e 135 | 213
Aziz Manva (azizmanva@gmail.com)

Part B
Inner Function:
𝐷√𝑥 = 𝑥 ≥ 0
Outer Function:
𝑘𝜋
𝐷tan 𝑥 = 𝐴𝑙𝑙 𝑟𝑒𝑎𝑙 𝑛𝑢𝑚𝑏𝑒𝑟𝑠 𝑒𝑥𝑐𝑒𝑝𝑡 , 𝑘 𝑖𝑠 𝑜𝑑𝑑 𝑖𝑛𝑡𝑒𝑔𝑒𝑟
2
Remove the values from the range of √𝑥 which do not lie in the domain of tan 𝑥:
𝑘𝜋 𝑘 2𝜋 2
√𝑥 = ⇒𝑥= , 𝑘 𝑖𝑠 𝑜𝑑𝑑 𝑖𝑛𝑡𝑒𝑔𝑒𝑟
2 4
𝑘 2𝜋 2
𝐷𝑓(𝜃) = {𝑥| 𝑥 ≥ 0 𝑎𝑛𝑑 𝑥 ≠ , 𝑘 𝑖𝑠 𝑜𝑑𝑑 𝑖𝑛𝑡𝑒𝑔𝑒𝑟}
4

Example 3.149
𝜋
𝑓(𝜃) = tan ( cos 𝜃)
2
Determine:
A. Domain
B. Range
C. Period

Part A: Domain
𝜋
Step I: Domain of inner function 2 cos 𝜃
𝜋
cos 𝜃 ℎ𝑎𝑠 𝑑𝑜𝑚𝑎𝑖𝑛 (−∞, ∞)
2
Step II: Domain of outer function:
𝑘𝜋
𝐷𝑜𝑚𝑎𝑖𝑛(tan 𝑥) ∈ ℝ / , 𝑘 𝑖𝑠 𝑜𝑑𝑑 𝑖𝑛𝑡𝑒𝑔𝑒𝑟
2
Step III: Ensure that the range of the inner function is a subset of the domain of the outer function
𝑘𝜋
The input for the 𝑡𝑎𝑛 function cannot be 2
. Hence, equate the inner function to the values which are not
allowed:
𝜋 𝑘𝜋
cos 𝜃 = , 𝑘 𝑖𝑠 𝑜𝑑𝑑 𝑖𝑛𝑡𝑒𝑔𝑒𝑟
2 2
cos 𝜃 = 𝑘, 𝑘 𝑖𝑠 𝑜𝑑𝑑 𝑖𝑛𝑡𝑒𝑔𝑒𝑟

The only valid value of 𝑘 is 𝑘 = ±1:


cos 𝜃 = ±1 ⇒ 𝜃 = 𝑛𝜋, 𝑛 ∈ ℤ
𝐷𝑓(𝜃) = 𝐴𝑙𝑙 𝑟𝑒𝑎𝑙 𝑛𝑢𝑚𝑏𝑒𝑟𝑠 𝑒𝑥𝑐𝑒𝑝𝑡 𝑛𝜋, 𝑛 ∈ ℤ

Part B: Range
𝜋
𝑔(𝜃) =cos 𝜃
2
𝜋 𝜋 𝜋
𝑅𝑎𝑛𝑔𝑒 𝑜𝑓 𝑔(𝜃) = cos 𝜃 ∈ (− , )
2 2 2
𝜋 𝜋
If tan 𝑥 has input (− 2 , 2 ), then it has range
(−∞, ∞)

Part C: Range
Period is
2𝜋

P a g e 136 | 213
Aziz Manva (azizmanva@gmail.com)

Example 3.150
𝑓(𝜃) = cos(sin 𝜃)
Determine:
A. Domain
B. Range
C. Period

Part A: Domain
𝐷𝑜𝑚𝑎𝑖𝑛 𝑜𝑓 sin 𝜃 : ℝ
𝑅𝑎𝑛𝑔𝑒 𝑜𝑓 sin 𝜃 = [−1,1]

𝐷𝑜𝑚𝑎𝑖𝑛 𝑜𝑓 cos 𝜃 : ℝ
Range of inner function (sin 𝜃) is a subset of domain of outer function (cos 𝜃). Hence, we do not need to remove
any values from the domain of sin 𝜃.
𝑅𝑎𝑛𝑔𝑒

𝑘𝜋
𝐷𝑜𝑚𝑎𝑖𝑛(tan 𝑥) ∈ ℝ / , 𝑘 𝑖𝑠 𝑜𝑑𝑑 𝑖𝑛𝑡𝑒𝑔𝑒𝑟
2

P a g e 137 | 213
Aziz Manva (azizmanva@gmail.com)

4. IDENTITIES AND MORE


4.1 The Pythagorean Identity
A. Basics

4.1: Square of a Trigonometric Function


(sin 𝜃)2 = sin2 𝜃

4.2: Pythagorean Identity


sin2 𝜃 + cos 2 𝜃 = 1

𝑜𝑝𝑝 2 𝑎𝑑𝑗 2 𝑜𝑝𝑝2 + 𝑎𝑑𝑗 2


𝐻𝑆 = sin2 𝜃 + cos 2 𝜃 = ( ) +( ) =
ℎ𝑦𝑝 ℎ𝑦𝑝 ℎ𝑦𝑝2

By the Pythagorean Theorem, 𝑜𝑝𝑝2 + 𝑎𝑑𝑗 2 = ℎ𝑦𝑝2 :


ℎ𝑦𝑝2
= = 1 = 𝑅𝐻𝑆
ℎ𝑦𝑝2

Alternate Method
Consider the unit circle.
Since it is a unit circle, the length of radius is 1.

Consider any point on the unit circle, and draw a right triangle, as shown.
𝑥 = cos 𝜃 , 𝑦 = sin 𝜃

By the Pythagorean Theorem,


sin2 𝜃 + cos2 𝜃 = 𝑦 2 + 𝑥 2 = 𝐻𝑦𝑝2 = 12 = 1

Example 4.3
3
If cos 𝜃 = − 5, then find sin 𝜃.

sin2 𝜃 + cos 2 𝜃 = 1
3 2
sin2 𝜃 + (− ) = 1
5
9 16
sin2 𝜃 = 1 − =
25 25
16 4
sin 𝜃 = ±√ = ±
25 5

cos 𝜃 is negative in 2𝑛𝑑 Quadrant and 3𝑟𝑑 Quadrant.


sin 𝜃 is positive in 2𝑛𝑑 Quadrant, and negative in 3rd Quadrant

B. Expressions

Example 4.4: Simplification

P a g e 138 | 213
Aziz Manva (azizmanva@gmail.com)

Simplify
A. sin2 𝑥 + 2 cos 2 𝑥
cot 𝑥
B. tan 𝑥
C. sin2 𝑥 cos2 𝑥 + sin4 𝑥

Part A
sin2 𝑥 + 2 cos2 𝑥 = sin2 𝑥 + cos 2 𝑥 + cos 2 𝑥 = cos2 𝑥 + 1
Part B
cot 𝑥 1
= cot 𝑥 × = cot 𝑥 × cot 𝑥 = cot 2 𝑥
tan 𝑥 tan 𝑥
Part C
Factor out sin2 𝑥 from each term:
sin2 𝑥 ⏟
(sin2 𝑥 + cos2 𝑥) = sin2 𝑥
=1

Example 4.5
(𝑎 + 𝑏)2 = 𝑎2 + 2𝑎𝑏 + 𝑏 2
(𝑎 − 𝑏)2 = 𝑎2 − 2𝑎𝑏 + 𝑏 2

Do as directed in the questions below. You might find the identities above useful.

Multiply
A. (sin 𝜙 + cos 𝜙)2
B. (3 sin 𝜙 − 2 cos 𝜙)2
Simplify
C. cos2 𝛼 + 6 cos 𝛼 + 9
D. 3 tan2 𝛾 − 6 tan 𝛾 + 3
sin2 𝑥+2 sin 𝑥 cos 𝑥+cos2 𝑥
E.
sin 𝑥+cos 𝑥
sin2 𝑥−2 sin 𝑥 cos 𝑥+cos2 𝑥
F. sin 𝑥−cos 𝑥
1+2 sin 𝛼 cos 𝛼
G.
sin 𝛼+cos 𝛼
1−2 sin 𝛼 cos 𝛼
H. sin 𝛼−cos 𝛼

Part A
(sin 𝜙 + cos 𝜙)2 = sin2 𝜙 + cos2 𝜙 + 2 sin 𝜙 cos 𝜙 = 1 + 2 sin 𝜙 cos 𝜙
Part B
9 sin2 𝜙 + 4 cos 2 𝜙 − 12 sin 𝜙 cos 𝜙
= 5 sin2 𝜙 + 4 sin2 𝜙 + 4 cos2 𝜙 − 12 sin 𝜙 cos 𝜙
= 5 sin2 𝜙 + 4 − 12 sin 𝜙 cos 𝜙
= 5 sin2 𝜙 − 12 sin 𝜙 cos 𝜙 + 4
Part C
Use a change of variable. Let 𝑎 = cos 𝛼:
𝑎2 + 6𝑎 + 9 = (𝑎 + 3)2
Change back to the original variable:
(cos 𝛼 + 3)2
Part D
3(tan2 𝛾 − 2 tan 𝛾 + 1) = 3(tan 𝛾 − 1)2
Part E

P a g e 139 | 213
Aziz Manva (azizmanva@gmail.com)

sin2 𝑥 + 2 sin 𝑥 cos 𝑥 + cos 2 𝑥 (sin 𝑥 + cos 𝑥)2


= = sin 𝑥 + cos 𝑥
sin 𝑥 + cos 𝑥 sin 𝑥 + cos 𝑥
Part F
sin2 𝑥 − 2 sin 𝑥 cos 𝑥 + cos 2 𝑥 (sin 𝑥 − cos 𝑥)2
= = sin 𝑥 − cos 𝑥
sin 𝑥 − cos 𝑥 sin 𝑥 − cos 𝑥
Part G
Substitute 1 = sin2 𝛼 + cos2 𝛼:
sin2 𝛼 + 2 sin 𝛼 cos 𝛼 + cos2 𝛼 (sin 𝛼 + cos 𝛼)2
= = sin 𝛼 + cos 𝛼
sin 𝛼 + cos 𝛼 sin 𝛼 + cos 𝛼
Part H
Substitute 1 = sin2 𝛼 + cos2 𝛼:
sin2 𝛼 − 2 sin 𝛼 cos 𝛼 + cos2 𝛼 (sin 𝛼 − cos 𝛼)2
= = sin 𝛼 − cos 𝛼
sin 𝛼 − cos 𝛼 sin 𝛼 − cos 𝛼

Example 4.6
𝑎2 − 𝑏 2 = (𝑎 + 𝑏)(𝑎 − 𝑏)

Do as directed in the questions below. You might find the identities above useful.

Multiply and simplify:


A. (sin 𝜃 + cos 𝜃)(sin 𝜃 − cos 𝜃)
B. (1 + sin 𝑥)(1 − sin 𝑥) + (1 + cos 𝑥)(1 − cos 𝑥)
Factor and Simplify
C. sin4 𝜙 − cos 4 𝜙
D. sin8 𝜙 − cos 8 𝜙
sin2 𝑥−cos2 𝑥
E. sin 𝑥−cos 𝑥
sin2 𝜙−cos2 𝜙
F. sin 𝜙+cos 𝜙
sin4 𝛽−cos4 𝛽
G. sin 𝛽+cos 𝛽

Part A
= sin2 𝜃 − cos2 𝜃
2
+ cos 2 𝜃 − cos2 𝜃 − cos 2 𝜃
= sin 𝜃 ⏟
𝑬𝒒𝒖𝒂𝒍 𝒕𝒐 𝒁𝒆𝒓𝒐
2
= 1 − 2 cos 𝜃
Part B
Multiply:
1 − sin2 𝑥 + 1 − cos2 𝑥 = 2 − (sin2 𝑥 + cos 2 𝑥)
Substitute sin2 𝑥 + cos 2 𝑥 = 1:
=2−1=1
Part C
Factor using difference of squares:
(sin2 𝜙 + cos 2 𝜙)(sin2 𝜙 − cos2 𝜙)
2 2
Substitute sin 𝜙 + cos 𝜙 = 1:
sin2 𝜙 − cos2 𝜙
Factor again using difference of squares:
(sin 𝜙 + cos 𝜙)(sin 𝜙 − cos 𝜙)
Part D
(sin4 𝜙 + cos 4 𝜙)(sin4 𝜙 − cos4 𝜙)

P a g e 140 | 213
Aziz Manva (azizmanva@gmail.com)

= (sin4 𝜙 + cos 4 𝜙)(sin 𝜙 + cos 𝜙)(sin 𝜙 − cos 𝜙)


Part E
sin2 𝑥 − cos 2 𝑥 (sin 𝑥 + cos 𝑥)(sin 𝑥 − cos 𝑥)
= = sin 𝑥 + cos 𝑥
sin 𝑥 − cos 𝑥 sin 𝑥 − cos 𝑥
Part F
(sin 𝜙 + cos 𝜙)(sin 𝜙 − cos 𝜙)
= sin 𝜙 − cos 𝜙
sin 𝜙 + cos 𝜙
Part G
(sin2 𝛽 − cos2 𝛽)(sin2 𝛽 + cos 2 𝛽)
sin 𝛽 + cos 𝛽
Substitute 1 = sin2 𝛽 + cos 2 𝛽:
sin2 𝛽 − cos 2 𝛽 (sin 𝛽 + cos 𝛽)(sin 𝛽 − cos 𝛽)
= = sin 𝛽 − cos 𝛽
sin 𝛽 + cos 𝛽 sin 𝛽 + cos 𝛽

Example 4.7
𝑎3 + 𝑏 3 = (𝑎 + 𝑏)(𝑎2 − 𝑎𝑏 + 𝑏 2 )
𝑎3 − 𝑏 3 = (𝑎 − 𝑏)(𝑎2 + 𝑎𝑏 + 𝑏 2 )

Do as directed in the questions below. You might find the identities above useful.
Simplify:
sin3 𝜃+cos3 𝜃
A.
sin 𝜃+cos 𝜃
sin3 𝛾−cos3 𝛾
B. 1−sin2 𝛾 cos2 𝛾

Part A
Apply the formula for the sum of cubes:
(sin 𝜃 + cos 𝜃)(sin2 𝜃 − sin 𝜃 cos 𝜃 + cos 2 𝜃)
sin 𝜃 + cos 𝜃
= sin2 𝜃 − sin 𝜃 cos 𝜃 + cos2 𝜃
1 − sin 𝜃 cos 𝜃
Part B
Apply the formula for the difference of cubes:
(sin 𝛾 − cos 𝛾)(sin2 𝛾 + sin 𝛾 cos 𝛾 + cos 2 𝛾)
(1 − sin 𝛾 cos 𝛾)(1 + sin 𝛾 cos 𝛾)
(sin 𝛾 − cos 𝛾)(1 + sin 𝛾 cos 𝛾)
=
(1 − sin 𝛾 cos 𝛾)(1 + sin 𝛾 cos 𝛾)
sin 𝛾 − cos 𝛾
=
1 − sin 𝛾 cos 𝛾

Example 4.8
A. sin2 𝜃 + 5 sin 𝜃 + 6
B. 2 cos2 𝛼 + cos 𝛼 − 3
C.

Part A
Let sin 𝜃 = 𝑎
𝑎2 + 5𝑎 + 6
(𝑎 + 2)(𝑎 + 3)
Change back to the original variable:
P a g e 141 | 213
Aziz Manva (azizmanva@gmail.com)

(sin 𝜃 + 2)(sin 𝜃 + 3)
Part B
Use a change of variable. Let cos 𝛼 = 𝑎:
2𝑎2 + 𝑎 − 3
𝑃𝑟𝑜𝑑𝑢𝑐𝑡 = −6 = (3)(−2), 𝑆𝑢𝑚 = 1

2𝑎2 + 3𝑎 − 2𝑎 − 3
𝑎(2𝑎 + 3) − 1(2𝑎 + 3)
(2𝑎 + 3)(𝑎 − 1)
Change back to the original variable:
(2 cos 𝛼 + 3)(cos 𝛼 − 1)

C. Rationalization

4.9: Non-negative Square Roots


We will make the assumption that all quantities inside a square root sign are positive. If we remove this
assumption, we will have to redo all the questions that are given below.

Example 4.10
1 − cos 𝛼

1 + cos 𝛼

Method I
√1 − cos 𝛼 √1 + cos 𝛼 √1 − cos 2 𝛼 √sin2 𝛼 sin 𝛼
= × = = =
√1 + cos 𝛼 √1 + cos 𝛼 1 + cos 𝛼 1 + cos 𝛼 1 + cos 𝛼

Method II
√1 − cos 𝛼 √1 − cos 𝛼 1 − cos 𝛼 1 − cos 𝛼 1 − cos 𝛼
= × = = =
√1 + cos 𝛼 √1 − cos 𝛼 √1 − cos 2 𝛼 √sin2 𝛼 sin 𝛼

Show that the two answers are the same.


sin 𝛼 1 − cos 𝛼 (sin 𝛼)(1 − cos 𝛼) (sin 𝛼)(1 − cos 𝛼) 1 − cos 𝛼
× = = =
1 + cos 𝛼 1 − cos 𝛼 1 − cos 2 𝛼 sin2 𝛼 sin 𝛼

Example 4.11
√sin2 𝛼

√sin2 𝛼 = |sin 𝛼|

√𝑥 2 = |𝑥|
𝑥 > 0: √𝑥 2 = 𝑥
𝑥 < 0: √𝑥 2 = −𝑥

For Quadrant IV and I:

P a g e 142 | 213
Aziz Manva (azizmanva@gmail.com)

𝑐𝑜𝑠𝑥 > 0: √(1 + sin 𝑥)(1 − sin 𝑥) = √1 − sin2 𝑥 = √cos 2 𝑥 = cos 𝑥


For Quadrant II and III:
cos 𝑥 < 0: √(1 + sin 𝑥)(1 − sin 𝑥) = √1 − sin2 𝑥 = √cos2 𝑥 = − cos 𝑥

For all Quadrants:


√(1 + sin 𝑥)(1 − sin 𝑥) = √1 − sin2 𝑥 = √cos 2 𝑥 = |cos 𝑥|

Example 4.12
When 𝛼 = 180°, determine the value of
1 − cos 𝛼

1 + cos 𝛼

Substitute cos 180° = −1


1 − (−1) 2
√ = √ ⇒ 𝑁𝑜𝑡 𝐷𝑒𝑓𝑖𝑛𝑒𝑑
1 + (−1) 0

D. Identities Derived from Pythagorean

4.13: Identities derived from the Pythagorean Identity


tan2 𝜃 + 1 = sec 2 𝜃
1 + cot 2 𝜃 = csc 2 𝜃

Divide both sides of the Pythagorean Identity by cos2 𝜃:


sin2 𝜃 cos 2 𝜃 1
sin2 𝜃 + cos 2 𝜃 = 1 ⇒ + = ⇒ tan2 𝜃 + 1 = sec 2 𝜃
cos 𝜃 cos 𝜃 cos 2 𝜃
2 2

Divide both sides of the Pythagorean Identity by cos2 𝜃:


sin2 𝜃 cos 2 𝜃 1
sin2 𝜃 + cos 2 𝜃 = 1 ⇒ + = ⇒ 1 + cot 2 𝜃 = csc 2 𝜃
sin 𝜃 sin 𝜃 sin2 𝜃
2 2

Example 4.14
(sec 𝑥 + tan 𝑥)(sec 𝑥 − tan 𝑥)

= sec 2 𝑥 − tan2 𝑥 = tan2 𝑥 + 1 − tan2 𝑥 = 1


E. Proving Identities
There are three main ways you can work with and prove an identity
➢ Work with the LHS, and show that it is equal to the RHS
➢ Work with the RHS and show that it is equal to the LHS
➢ Work with both sides and show that they are equal

Example 4.15
Prove that:
csc 𝑥 − 1 1
cos2 𝑥 = ( ) (1 + )
csc 𝑥 csc 𝑥

P a g e 143 | 213
Aziz Manva (azizmanva@gmail.com)

1
Work with the RHS. Substitute csc 𝑥 = sin 𝑥:
1
−1
𝑅𝐻𝑆 = ( sin 𝑥 ) (1 + sin 𝑥)
1
sin 𝑥
1 − sin 𝑥 sin 𝑥
=( )( ) (1 + sin 𝑥)
sin 𝑥 1
= 1 − sin2 𝑥
= cos2 𝑥 = 𝐿𝐻𝑆

Example 4.16: Multiply by One


Show that:
1
sec 𝜃 + tan 𝜃 =
sec 𝜃 − tan 𝜃

1 1
𝑅𝐻𝑆 = =
1 sin 𝜃 1 − sin 𝜃
cos 𝜃 − cos 𝜃 cos 𝜃
Move the fraction in the denominator to the numerator by taking the reciprocal:
cos 𝜃
1 − sin 𝜃
We do not want a denominator, so multiply the numerator and the denominator by 1 + sin 𝜃:
cos 𝜃 (1 + sin 𝜃) cos 𝜃 + cos 𝜃 sin 𝜃
=
(1 − sin 𝜃)(1 + sin 𝜃) ⏟ 1 − sin2 𝜃
∵𝐜𝐨𝐬 𝟐 𝜽=𝟏−𝐬𝐢𝐧𝟐 𝜽
2 2
Substitute cos 𝜃 = 1 − sin 𝜃
cos 𝜃(1 + sin 𝜃) 1 + sin 𝜃
= = sec 𝜃 + tan 𝜃 = 𝐿𝐻𝑆
cos2 𝜃 cos 𝜃

Example 4.17
1
Consider the following set of steps used to show that sec 𝜃 + tan 𝜃 = sec 𝜃−tan 𝜃

Start with the identity to be proved:


1
sec 𝜃 + tan 𝜃 =
sec 𝜃 − tan 𝜃
Cross-multiply and expand:
(sec 𝜃 − tan 𝜃)(sec 𝜃 + tan 𝜃) = 1
sec 2 𝜃 − tan2 𝜃 = 1
Expand using the definition:
1 sin2 𝜃
− =1
cos2 𝜃 cos2 𝜃
Multiply both sides by cos2 𝜃:
1 − sin2 𝜃 = cos2 𝜃
Re-arrange:
cos2 𝜃 + sin2 𝜃 = 1
The above is a known identity.
Hence, the proof is complete.

Are the steps correct? If the steps are not correct, what can you do to fix it.

P a g e 144 | 213
Aziz Manva (azizmanva@gmail.com)

Each of the steps used above is reversible, and hence, the identity is correct.

Example 4.18
1
Show that csc 𝑎 − cot 𝑎 =
csc 𝑎 + cot 𝑎

1 − cos 𝑎 1 1 1 1
𝐿𝐻𝑆 = = = = =
sin 𝑎 sin 𝑎 (1 + cos 𝑎) sin 𝑎 + sin 𝑎 cos 𝑎 1 cos 𝑎 csc 𝑎 + cot 𝑎
(1 − cos 𝑎)(1 + cos 𝑎) sin2 𝑎 sin 𝑎 + sin 𝑎

Example 4.19: Solving Simultaneous Equations

Example 4.20: Nested Fractions


sec 𝑥 + csc 𝑥
𝑆ℎ𝑜𝑤 𝑡ℎ𝑎𝑡 = sin 𝑥 + cos 𝑥
tan 𝑥 + cot 𝑥

We start with the left-hand side since it is more complicated:


1 1 sin 𝑥 + cos 𝑥
sec 𝑥 + csc 𝑥 cos 𝑥 + sin 𝑥
𝐿𝐻𝑆 = = = sin 𝑥 cos 𝑥 = sin 𝑥 + cos 𝑥 = 𝑅𝐻𝑆
tan 𝑥 + cot 𝑥 sin 𝑥 + cos 𝑥 sin2 𝑥 + cos2 𝑥
cos 𝑥 sin 𝑥 sin 𝑥 cos 𝑥

Example 4.21

(sin 𝑥 tan 𝑥)2 = (tan 𝑥 + sin 𝑥)(tan 𝑥 − sin 𝑥)

sin2 𝑥 1
𝑅𝐻𝑆 = tan2 𝑥 − sin2 𝑥 = 2
− sin2 𝑥 = sin2 𝑥 ( 2 − 1) = sin2 𝑥 (sec 2 𝑥 − 1) = (sin 𝑥 tan 𝑥)2
cos 𝑥 cos 𝑥

F. Application: Trig Substitutions


The Pythagorean identities are useful in eliminating square roots from algebraic expressions using a change of
variable. This technique is exploited in Calculus to replace an expression that has a square root with one that
doesn’t.

4.22: Substitutions
Show that the following substitutions work:
A. 𝑥 = 𝑎 sin 𝜃 ⇒ √𝑎2 − 𝑥 2 = 𝑎 cos 𝜃
B. 𝑥 = 𝑎 tan 𝜃 ⇒ √𝑎2 + 𝑥 2 = 𝑎 sec 𝜃
C. 𝑥 = 𝑎 sec 𝜃 ⇒ √𝑥 2 − 𝑎2 = 𝑎 tan 𝜃

Part A
√𝑎 2 − 𝑥 2
Substitute 𝑥 = 𝑎 sin 𝜃:
√𝑎2 − 𝑎2 sin2 𝜃 = √𝑎2 (1 − sin2 𝜃) = 𝑎√1 − sin2 𝜃
2 2
Substitute 1 − sin 𝜃 = cos 𝜃:

P a g e 145 | 213
Aziz Manva (azizmanva@gmail.com)

𝑎√cos 2 𝜃 = 𝑎 cos 𝜃 8
Part B
√𝑎 2 + 𝑥 2
Substitute 𝑥 = 𝑎 tan 𝜃:
√𝑎2 + 𝑎2 tan2 𝜃 = √𝑎2 (1 + tan2 𝜃) = 𝑎√(1 + tan2 𝜃)
Here we make use of tan 𝜃 + 1 = sec 2 𝜃:
2

= 𝑎√sec 2 𝜃 = 𝑎 sec 𝜃
Part C
√𝑥 2 − 𝑎 2
Substitute 𝑥 = 𝑎 sec 𝜃:
√𝑎2 sec 2 𝜃 − 𝑎2 = √𝑎2 (sec 2 𝜃 − 1) = 𝑎√sec 2 𝜃 − 1
Here we make use of tan2 𝜃 + 1 = sec 2 𝜃:
= 𝑎√tan2 𝜃 = 𝑎 tan 𝜃

Example 4.23
Use a trigonometric substitution in terms of 𝜃 to remove the square root, and find the values of sin 𝜃 , cos 𝜃, and
tan 𝜃.
A. √1 − 𝑥 2
𝑥2
B.
√25−𝑥 2
C. √4 + 𝑥 2
1
D. 2 2
𝑥 √16+𝑥
E. √𝑥 2 − 100
F. √3 − 𝑥 2
G. √16 − 9𝑥 2

Part A
Substitute 𝑥 = sin 𝜃:
√1 − 𝑥 2 = √1 − sin2 𝜃 = √cos 2 𝜃 = cos 𝜃

𝑥
sin 𝜃 =
1
𝑎𝑑𝑗
cos 𝜃 = = √1 − 𝑥 2
ℎ𝑦𝑝
𝑜𝑝𝑝 𝑥
tan 𝜃 = =
𝑎𝑑𝑗 √1 − 𝑥 2

Part B
𝑥2
√25 − 𝑥 2
Substitute 𝑥 = 5 sin 𝜃:
(5 sin 𝜃)2 25 sin2 𝜃 25 sin2 𝜃 5 sin2 𝜃
= = =
√25 − (5 sin 𝜃)2 √25 − 25 sin2 𝜃 5√cos2 𝜃 cos 𝜃

8 Specifically √𝑥 2 = |𝑥|. For the time being, let us assume that cos 𝜃 is positive.

P a g e 146 | 213
Aziz Manva (azizmanva@gmail.com)

𝑥
𝑥 = 5 sin 𝜃 ⇒ sin 𝜃 =
5
2 2
𝑥 + 𝑎𝑑𝑗 = 25 ⇒ 𝑎𝑑𝑗 = √25 − 𝑥 2
𝑎𝑑𝑗 √25 − 𝑥 2
cos 𝜃 = =
ℎ𝑦𝑝 5
𝑜𝑝𝑝 𝑥
tan 𝜃 = =
𝑎𝑑𝑗 √25 − 𝑥 2
Part C
Use a change of variable. Let 𝑥 = 2 𝑡𝑎𝑛 𝜃:
√(4 + 4 tan2 𝜃) = √(4 + 4 tan2 𝜃) = √4(1 + tan2 𝜃) = 2√sec 2 𝜃 = 2 sec 𝜃

𝑥
𝑥 = 2 tan 𝜃 ⇒ tan 𝜃 =
2
Draw a reference triangle
𝐻𝑦𝑝𝑜𝑡𝑒𝑛𝑢𝑠𝑒 = √4 + 𝑥 2
𝑜𝑝𝑝 𝑥
sin 𝜃 = =
ℎ𝑦𝑝 √4 + 𝑥 2
𝑎𝑑𝑗 2
cos 𝜃 = =
ℎ𝑦𝑝 √4 + 𝑥 2

Part D

Part E

Part F

Part G

16
√16 − 9𝑥 2 = √9 ( − 𝑥2)
9
3
Substitute 𝑥 = 2 sin 𝜃:

Example 4.24: Completing the square


A. Complete the square below.
B. Use a change of variable to bring it in line with the formula above.
C. Use a trigonometric substitution in terms of 𝜃 to remove the square root, and find the values of
sin 𝜃 , cos 𝜃, and
tan 𝜃.
√−𝑥 2 + 2𝑥 + 5

Part A
√−𝑥 2 + 2𝑥 + 5 = √4 − (𝑥 − 1)2

P a g e 147 | 213
Aziz Manva (azizmanva@gmail.com)

Part B
Let 𝑢 = 𝑥 − 1:
√4 − 𝑢2

Part C
Let 𝑢 = sin 𝜃

Example 4.25
√4𝑥 2 + 4𝑥 − 48
A. Complete the square.
B. Use a change of variable to bring it in line with the formula above.
C. Factor the expression inside the square root to make the coefficient of 𝑥 as 1.
D. Use a trigonometric substitution in terms of 𝜃 to remove the square root, and find the values of
sin 𝜃 , cos 𝜃, and tan 𝜃.

𝒏
G. Expressions of power 𝟐

Example 4.26

H. Trigonometric Expressions
If the expression inside the square root is itself trigonometric, you can still use a change of variable to eliminate
the square root.

Example 4.27
Use a trigonometric substitution in terms of 𝜃 to remove the square root, and find the values of sin 𝜃 , cos 𝜃, and
tan 𝜃.
√1 + sin2 𝑥

4.2 Double Angle and Half Angle Identities


A. Double Angle Identities for 𝒔𝒊𝒏

4.28: Double Angle Identity


sin(2𝜃) = 2 sin 𝜃 cos 𝜃

Use sin(𝛼 + 𝛽) = sin 𝛼 cos 𝛽 + cos 𝛼 sin 𝛽:


sin(2𝜃) = sin(𝜃 + 𝜃) = sin 𝜃 cos 𝜃 + cos 𝜃 sin 𝜃 = 2 sin 𝜃 cos 𝜃

Example 4.29: Physics

P a g e 148 | 213
Aziz Manva (azizmanva@gmail.com)

1
The range of a projectile is given by 𝑅 = 𝑢𝑥 𝑡 + 𝑎𝑥 𝑡 2, where the
2
2𝑢 sin 𝜃
displacement = 𝑢𝑥 = 𝑢 cos 𝜃, the flight is given by 𝑡 = 𝑔
, the
acceleration
𝐴𝑐𝑐𝑒𝑙𝑒𝑟𝑎𝑡𝑖𝑜𝑛 = 𝑎𝑥 = 0
𝜃 = 𝐿𝑎𝑢𝑛𝑐ℎ 𝐴𝑛𝑔𝑙𝑒
Show that
𝑢2 sin 2𝜃
𝑔

Substitute the given values in the given equation:


1 2𝑢 sin 𝜃 1
𝑅 = 𝑢𝑥 𝑡 + 𝑎𝑥 𝑡 2 = (𝑢 cos 𝜃) ( ) + (0)𝑡 2
2 𝑔 2

The second term will become zero, and carry out the multiplication in the first term:
𝑢2 (2 sin 𝜃 cos 𝜃)
=
𝑔

Substitute 2 sin 𝜃 cos 𝜃 = sin 2𝜃 using the double angle formula:


𝑢2 sin 2𝜃
=
𝑔

Example 4.30: Geometry


In a right triangle the square of the hypotenuse is equal to twice the product of the legs. One of the acute angles
of the triangle is: (AHSME 1959/15)

Method I: Trigonometry
ℎ2 = 2𝑙1 𝑙2

𝑙1 𝑙2
Substitute sin 𝜃 = ℎ
⇒ 𝑙1 = ℎ sin 𝜃 , cos 𝜃 = ℎ
⇒ 𝑙2 = ℎ cos 𝜃:
2
ℎ = 2(ℎ sin 𝜃)(ℎ cos 𝜃)
1 = 2 sin 𝜃 cos 𝜃

Using the double angle identity:


sin 2𝜃 = 1

Solving for 𝜃
2𝜃 = 90° ⇒ 𝜃 = 45°

Method II: Pythagoras Theorem


Let the legs be 𝑥 and 𝑦. Then:
𝑥 2 + 𝑦 2 = 2𝑥𝑦 ⇒ (𝑥 − 𝑦)2 = 0 ⇒ 𝑥 − 𝑦 = 0 ⇒ 𝑥 = 𝑦

Hence, the triangle is an isosceles triangle. And we know that it is right-angled. Combine the two to get:
𝑅𝑖𝑔ℎ𝑡 − 𝑎𝑛𝑔𝑙𝑒𝑑 𝑖𝑠𝑜𝑐𝑒𝑙𝑒𝑠 𝑡𝑟𝑖𝑎𝑛𝑔𝑙𝑒 ⇒ 45 − 45 − 90

Example 4.31
Prove that

P a g e 149 | 213
Aziz Manva (azizmanva@gmail.com)

sin 2𝜃 = (sin 𝜃 + cos 𝜃)2 + tan2 𝜃 − sec 2 𝜃

Start with the RHS. Substitute tan2 𝜃 − sec 2 𝜃 = −1:


(sin 𝜃 + cos 𝜃)2 − 1
Open the brackets:
sin2 𝜃 + 2 sin 𝜃 cos 𝜃 + cos2 𝜃 − 1
2 2
Substitute sin 𝜃 + cos 𝜃 = 1
2 sin 𝜃 cos 𝜃 + 1 − 1
= 2 sin 𝜃 cos 𝜃
By the double angle identity for sine:
= sin 2𝜃 = 𝐿𝐻𝑆

Example 4.32: Telescoping


The value of the expression below as a single real number: (JEE Main, Jan 10, 2019-II)
𝜋 𝜋 𝜋 𝜋
cos 2 ∙ cos 3 ∙ … ∙ cos 10 ∙ sin 10
2 2 2 2
𝜋 𝜋 𝜋 𝜋 𝜋
cos 2
∙ cos 3 ∙ … ∙ cos 9 ∙ cos 10 ∙ sin 10
2 2 2 2 2
1
Use the double angle identity 2 ∙ sin(2𝜃) = sin 𝜃 cos 𝜃 on the last two terms:
𝜋 𝜋 𝜋 1 𝜋
cos 2 ∙ cos 3 ∙ … ∙ cos 9 ∙ ( ∙ sin 9 )
2 2 2 2 2

sin(2𝜃)
Use the double angle identity = sin 𝜃 cos 𝜃 again on the last two terms:
2
𝜋 𝜋 𝜋 1 𝜋
cos 2 ∙ cos 3 ∙ … ∙ cos 8 ∙ ( 2 ∙ sin 8 )
2 2 2 2 2

Keep repeating. There are a total of ten terms, which combine nine to get:
1 𝜋 1 1
9
sin = 9 ∙ 1 =
2 2 2 512

4.33: Telescoping Identity


sin 2𝑛 𝐴
cos 𝐴 ∙ cos 2𝐴 ∙ cos 22 𝐴 ∙ … ∙ cos 2𝑛−1 𝐴 =
2𝑛 sin 𝐴

2 sin 𝐴
𝐿𝐻𝑆 = × cos 𝐴 ∙ cos 2𝐴 ∙ cos 22 𝐴 ∙ … ∙ cos 2𝑛−1 𝐴
2 sin 𝐴

2 sin 2𝐴
= ∙ cos 2𝐴 ∙ cos 22 𝐴 ∙ … ∙ cos 2𝑛−1 𝐴
22 sin 𝐴
2 sin 22 𝐴
= 3 ∙ cos 22 𝐴 ∙ … ∙ cos 2𝑛−1 𝐴
2 sin 𝐴

Keep repeating the process to telescope:


sin 2𝑛 𝐴
= = 𝑅𝐻𝑆
2𝑛 sin 𝐴

P a g e 150 | 213
Aziz Manva (azizmanva@gmail.com)

B. Double Angle Identities for 𝒄𝒐𝒔

4.34: Double Angle Identity


The double angle identity for cos 2𝜃 has three versions, all of which are important.
cos 2𝜃 = cos2 𝜃 − sin2 𝜃 = 1 − 2 sin2 𝜃 = 2 cos2 𝜃 − 1

Use cos(𝑎 + 𝑏) = cos 𝑎 cos 𝑏 − sin 𝑎 sin 𝑏:


cos 2𝜃 = cos(𝜃 + 𝜃) = cos 𝜃 cos 𝜃 − sin 𝜃 sin 𝜃 = cos2 𝜃 − sin2 𝜃
cos 2𝜃 = (1 − sin2 𝜃) − sin2 𝜃 = cos 2 𝜃 − (1 − cos2 𝜃) = 2 cos 2 𝜃 − 1

cos 2𝜃 = 1 − 2 sin2 𝜃
2 sin2 𝜃 = 1 − cos 2𝜃
1 − cos 2𝜃
sin2 𝜃 =
2

cos 2𝜃 = 2 cos2 𝜃 − 1
2 cos 2 𝜃 = 1 + cos 2𝜃
1 + cos 2𝜃
cos 2 𝜃 =
2

Example 4.35
cos 2𝑥 sin 2𝑥
− = − sec 𝑥
cos 𝑥 sin 𝑥

Use the double angle identities in the LHS:


cos 2 𝑥 − sin2 𝑥 2 𝐬𝐢𝐧 𝒙 cos 𝑥
= −
cos 𝑥 𝐬𝐢𝐧 𝒙
cos2 𝑥 − sin2 𝑥
= − 2 cos 𝑥
cos 𝑥

Add the fractions:


cos 2 𝑥 − sin2 𝑥 − 2 cos2 𝑥
=
cos 𝑥
Simplify:
−(sin2 𝑥 + cos 2 𝑥)
=
cos 𝑥
Substitute sin2 𝑥 + cos 2 𝑥 = 1
= − sec 𝑥 = 𝑅𝐻𝑆

4.36: Reduction of Higher Powers


The double angle identities can be used to “reduce” the powers of trigonometric functions by writing them in
lower powers.

Challenge 4.37
Reduce the LHS to show that it is equal to the RHS:
3 − 4 cos 2𝑥 + cos 4𝑥
sin4 𝑥 =
8

P a g e 151 | 213
Aziz Manva (azizmanva@gmail.com)

𝐿𝐻𝑆 = sin4 𝑥
Substitute sin4 𝑥 = (sin2 𝑥)2 :
= (sin2 𝑥)2

1−cos 2𝜃
Substitute cos 2𝜃 = 1 − 2 sin2 𝜃 ⇒ 2 sin2 𝜃 = 1 − cos 2𝜃 ⇒ sin2 𝜃 =
2
1 − cos 2𝑥 2
( )
2

Expand using (𝑎 − 𝑏)2 = 𝑎2 − 2𝑎𝑏 + 𝑏 2


1 − 2 cos 2𝑥 + cos2 2𝑥
=
4
1+cos 2𝑥
Substitute cos 2𝑥 = 2 cos 2 𝑥 − 1 ⇒ cos 2 𝑥 = 2
:
1 + cos 4𝑥
1 − 2 cos 2𝑥 + ( 2 )
4

Simplify:
3 − 4 cos 2𝑥 + cos 4𝑥
=
8

4.38: Finding values of trigonometric functions


The double angle identities can be used to find the values of certain important trigonometric functions.

Example 4.39
Find cos 15°

1 + cos 2𝑥
cos 𝑥 = √
2
Substitute 𝑥 = 15°
1 1 √3 √3 + 2 1
cos 15° = √ (cos 30 + 1) = √ ( + 1) = √( ) = √√3 + 2
2 2 2 4 2

4.40: Identity
1 + cos 𝜃 𝜃
= cos2
2 2

cos 2α = 2 cos2 α − 1
1 + cos 2α = 2 cos 2 α
1 + cos 2α = 2 cos 2 α

𝜃
Let 2𝛼 = 𝜃 ⇒ 𝛼 = 2
𝜃
1 + cos 𝜃 = 2 cos2
2

Example 4.41
Simplify the expression below and write in the form 𝑎 cos 𝛼:

P a g e 152 | 213
Aziz Manva (azizmanva@gmail.com)

√2 + √2 + √2 + 2 cos 𝜃

Factor 2 from the innermost root:

√2 + √2 + √2(1 + cos 𝜃)

𝜃
Substitute 1 + cos 𝜃 = 2 cos 2 :
2

𝜃 𝜃
= √2 + √2 + √2 (2 cos2 ) = √2 + √2 + 2 cos
2 2

Repeat the process:

𝜃 𝜃 𝜃
= √2 + √2 (1 + 1 cos ) = √2 + √2 (2 cos2 ) = √2 + 2 cos
2 4 4

Repeat the process one more time:


𝜃 𝜃 𝜃
= √2 (1 + cos ) = √2 (2 cos2 ) = 2 cos
4 8 8

4.42: Double Angle Identity for 𝒕𝒂𝒏


2 tan 𝜃
tan 2𝜃 =
1 − tan2 𝜃

Example 4.43
cos(2𝑥 + 2𝑥) = cos2 2𝑥 − sin2 2𝑥 = (cos2 𝑥 − sin2 𝑥)2 − (2 sin 𝑥 cos 𝑥)2
= cos 4 𝑥 − 2 cos2 𝑥 sin2 𝑥 + sin4 𝑥 − 4 sin2 𝑥 cos 2 𝑥
=⏟cos 4 𝑥 − 6 cos 2 𝑥 sin2 𝑥 + sin4 𝑥
𝑽𝒆𝒓𝒔𝒊𝒐𝒏 𝟏
= cos 4 𝑥 + 2 cos2 𝑥 sin2 𝑥 + sin4 𝑥 − 8 cos2 𝑥 sin2 𝑥
= cos 4 𝑥 + 2 cos2 𝑥 sin2 𝑥 + sin4 𝑥 − 8 cos2 𝑥 sin2 𝑥
= (sin2 𝑥 + cos2 𝑥)2 − 8 cos2 𝑥 sin2 𝑥
=⏟1 − 8 cos 2 𝑥 sin2 𝑥
𝑽𝒆𝒓𝒔𝒊𝒐𝒏 𝟐

cos 4𝑥 = cos(2𝑥 + 2𝑥) = 2 cos 2 2𝑥 − 1 = 2(2 cos2 𝑥 − 1)2 − 1


= 2(4 cos 4 𝑥 − 4 cos 2 𝑥 + 1) − 1 = ⏟
8 cos 4 𝑥 − 8 cos2 𝑥 + 1
𝑽𝒆𝒓𝒔𝒊𝒐𝒏 𝟑

C. Half Angle Identities


The half angle identities are used to find calculate trigonometric functions for “half” of an angle for which you
know the value.

P a g e 153 | 213
Aziz Manva (azizmanva@gmail.com)

𝜽
4.44: 𝒔𝒊𝒏
𝟐

𝜃 1 − cos 𝜃
sin = ±√
2 2

cos 2𝜃 = 1 − 2 sin2 𝜃
2 sin2 𝜃 = 1 − cos 2𝜃
1 − cos 2𝜃
sin2 𝜃 =
2
𝜃 1 − cos 𝜃
sin2 =
2 2
𝜃 1 − cos 𝜃
sin = ±√
2 2

𝜽
4.45: 𝒄𝒐𝒔
𝟐

𝜃 1 + cos 𝜃
cos = ±√
2 2

Notes:
➢ When making use of this identity, the sign is determined by the quadrant in which the angle lies.
➢ cos 𝜃 > 0 ⇒ 𝜃 𝑖𝑛 𝑄𝐼 𝑜𝑟 𝑄𝐼𝑉

cos 2𝜃 = 2 cos2 𝜃 − 1
2 cos 2 𝜃 = 1 + cos 2𝜃
1 + cos 2𝜃
cos 2 𝜃 =
2
𝜃 1 + cos 𝜃
cos2 =
2 2
𝜃 1 + cos 𝜃
cos = ±√
2 2

Example 4.46
𝜋
Find the value of cos 8 = cos 22.5°.

Note that
45
➢ 22.5 = 2
, and hence we are in a position to make use of the half angle identity for 𝑐𝑜𝑠.
𝜋
➢ cos is in the first quadrant, and hence positive.
8

𝜋 𝜋 √2
𝜋 4 1 + cos 4 √1 + 2 2 + √2 1 2 + √2 1
cos = cos ( ) = √ = = √( )× =√ = √2 + √2
8 2 2 2 2 2 4 2

𝜽
4.47: 𝒕𝒂𝒏
𝟐

P a g e 154 | 213
Aziz Manva (azizmanva@gmail.com)

𝜃 sin 𝜃
tan =
2 1 + cos 𝜃

𝜃 1 − cos 𝜃
𝜃 sin 2 ±√ 2 1 − cos 𝜃
tan = = = ±√
2 cos 𝜃 1 + cos 𝜃 1 + cos 𝜃
2 ±√ 2

1 − cos 𝜃 1 + cos 𝜃 1 − cos 2 𝜃 𝑠𝑖𝑛2 𝜃 sin 𝜃


=±√ × = ±√ = ± √ = ± | |
1 + cos 𝜃 1 + cos 𝜃 (1 + cos 𝜃)2 (1 + cos 𝜃)2 1 + cos 𝜃

√𝑥 2 = |𝑥|
𝜃 sin 𝜃
tan =
2 1 + cos 𝜃

Example 4.48
A ladder kept in support of a wall makes an angle of 22.5 with the ground. The distance between the bottom of
the wall and the foot of the ladder is 2. What is the length of the ladder? (JMET 2011/85)
Write your answer in the form √𝑎 + 𝑏√𝑐, where 𝑎, 𝑏, 𝑐 ∈ ℕ and c has no perfect square factor.

2 2
𝑐𝑜𝑠 22.5 = ⇒𝑥=
𝑥 𝑐𝑜𝑠 22.5

2 4 16
𝑥= = =√ = √8(2 − √2) = √16 − 8√2
1√ √2 + √2 2 + √2
2 + √2
2

4.49: 𝒕𝒂𝒏
𝜃 1 − cos 𝜃
tan = ±√
2 1 + cos 𝜃

4.50: Connecting cosines with Circumradius


If 𝑨, 𝑩 and 𝑪 are three angles in a triangle and 𝑹 = 𝒄𝒊𝒓𝒄𝒖𝒎𝒓𝒂𝒅𝒊𝒖𝒔 then:
𝑨 𝑩 𝑪 𝒔
𝐜𝐨𝐬 𝐜𝐨𝐬 𝐜𝐨𝐬 =
𝟐 𝟐 𝟐 𝟒𝑹

Finding an expression for each term


Use the half-angle identity:
𝐴 cos 𝐴 + 1
cos =√
2 2
𝑏2 +𝑐 2 −𝑎2
Substitute cos 𝐴 = 2𝑏𝑐
and then simplify:
𝑏 2 + 𝑐 2 − 𝑎2
√ +1 𝑏 2 + 𝑐 2 + 2𝑏𝑐 − 𝑎2
2𝑏𝑐 =√
2 4𝑏𝑐

P a g e 155 | 213
Aziz Manva (azizmanva@gmail.com)

Factor the numerator, and then use difference of squares:


(𝑏 + 𝑐)2 − 𝑎2 (𝑎 + 𝑏 + 𝑐)(𝑎 + 𝑏 + 𝑐 − 2𝑎)
√ =√
4𝑏𝑐 4𝑏𝑐
𝑎+𝑏+𝑐
Substitute 𝑠 = 2
, and then simplify:

(2𝑠)(2𝑠 − 2𝑎) 𝑠(𝑠 − 𝑎)


=√ =√
4𝑏𝑐 𝑏𝑐

Similarly, we can show that


𝐵 𝑠(𝑠 − 𝑏) 𝐶 𝑠(𝑠 − 𝑐)
cos =√ , cos =√
2 𝑎𝑐 2 𝑎𝑏

Finding the product


𝐴 𝐵 𝐶 𝑠 3 (𝑠 − 𝑎)(𝑠 − 𝑏)(𝑠 − 𝑐) 𝑠
cos cos cos = √ 2 2 2
= √𝑠(𝑠 − 𝑎)(𝑠 − 𝑏)(𝑠 − 𝑐)
2 2 2 𝑎 𝑏 𝑐 𝑎𝑏𝑐
Substitute Δ = √𝑠(𝑠 − 𝑎)(𝑠 − 𝑏)(𝑠 − 𝑐) to get:
𝐴 𝐵 𝐶 𝑠Δ Δ
cos cos cos = =𝑠×
2 2 2 𝑎𝑏𝑐 𝑎𝑏𝑐
𝑎𝑏𝑐 𝑎𝑏𝑐 Δ 1
And we know that 𝑅 = 4Δ
⇒ 4𝑅 = Δ
⇒ 𝑎𝑏𝑐 = 4𝑅:
𝐴 𝐵 𝐶 𝑠Δ 1 𝑠
cos cos cos = =𝑠× =
2 2 2 𝑎𝑏𝑐 4𝑅 4𝑅

4.51: Ratio of inradius and circumradius in terms of sines


𝑨 𝑩 𝑪 𝒓
𝐬𝐢𝐧 𝐬𝐢𝐧 𝐬𝐢𝐧 =
𝟐 𝟐 𝟐 𝟒𝑹

In Δ𝐼𝐵𝐷:
𝐵 𝐼𝐷 𝑟 𝐵
tan = = ⇒ 𝐵𝐷 = 𝑟 cot
2 𝐵𝐷 𝐵𝐷 2
𝐶 𝐼𝐷 𝑟 𝐶
tan = = ⇒ 𝐵𝐷 = 𝑟 cot
2 𝐵𝐷 𝐵𝐷 2

In Δ𝐴𝐵𝐶:
𝐵 𝐶
𝐵 𝐶 cos 2 cos 2
𝑎 = 𝐵𝐶 = 𝐵𝐷 + 𝐵𝐶 = 𝑟 cot + 𝑟 cot = 𝑟 ( + )
2 2 𝐵 𝐶
sin 2 sin 2
Add the fractions, and then use the double angle identity:
𝐶 𝐵 𝐵 𝐶 𝐵 𝐶
sin 2 cos 2 + sin 2 cos 2 sin ( 2 + 2 )
= 𝑟( )=𝑟
𝐵 𝐶 𝐵 𝐶
sin 2 sin 2 sin 2 sin 2

Since 𝐴, 𝐵 and 𝐶 are the angles of a triangle, we must have:


𝐴 𝐵 𝐶 𝜋 𝐵 𝐶 𝜋 𝐴
𝐴+𝐵+𝐶 =𝜋 ⇒ + + = ⇒ + = −
2 2 2 2 2 2 2 2

P a g e 156 | 213
Aziz Manva (azizmanva@gmail.com)

𝐵 𝐶 𝜋 𝐴 𝐴
Hence, we can substitute sin ( 2 + 2 ) = sin ( 2 − 2 ) = cos 2 to get:
𝐴
cos 2
𝑎=𝑟
𝐵 𝐶
sin sin
2 2
𝑎 𝐴 𝐴
We can also substitute for 𝑎 in terms of half-angles since = 2𝑅 ⇒ 𝑎 = 2𝑅 sin 𝐴 = 4𝑅 sin cos :
sin 𝐴 2 2
𝑨
𝐴 𝑨 𝐜𝐨𝐬 𝟐 𝐴 𝐵 𝐶 𝑟
4𝑅 sin 𝐜𝐨𝐬 = 𝑟 ⇒ sin sin sin =
2 𝟐 𝐵 𝐶 2 2 2 4𝑅
sin sin
2 2

4.52: Relation between inradius and circumradius


𝑹 ≥ 𝟐𝒓

Using Jensen’s Inequality, it can be shown that:


𝐴 𝐵 𝐶 1
sin sin sin ≤
2 2 2 8
𝐴 𝐵 𝐶 𝑟
And substitute sin 2 sin 2 sin 2 = 4𝑅
𝑟 1
≤ ⇒ 𝑅 ≥ 2𝑟
4𝑅 8

4.3 Sum and Difference Identities


A. Calculating Values

4.53: 𝒔𝒊𝒏 and 𝒄𝒐𝒔


sin(𝛼 ± 𝛽) = sin 𝛼 cos 𝛽 ± cos 𝛼 sin 𝛽
cos(𝛼 ± 𝛽) = cos 𝛼 cos 𝛽 ∓ sin 𝛼 sin 𝛽

Note that:
➢ Sine of a sum, RHS has a plus sign.
➢ Sine of a difference, RHS has a negative sign
➢ Cosine of a sum, RHS has a plus sign.
➢ Cosine of a difference, RHS has a negative sign

Example 4.54
Find the following:
A. sin 75°
B. sin 15°
C. cos 75°
D. cos 15°

sin(𝛼 + 𝛽) = sin 𝛼 cos 𝛽 + cos 𝛼 sin 𝛽

√2 √3 √2 1 √6 √2 √6 + √2
sin 75° = sin(45 + 30) = ( ) ( ) + ( ) ( ) = + =
⏟ 2 ⏟ 2 ⏟ 2 ⏟2 4 4 4
sin 45 cos 30 cos 45 sin 30

P a g e 157 | 213
Aziz Manva (azizmanva@gmail.com)

√2 √3 √2 1 √6 √2 √6 − √2
sin(45 − 30) = ( ) ( ) − ( ) ( ) = − =
⏟ 2 ⏟ 2 ⏟ 2 ⏟2 4 4 4
sin 45 cos 30 cos 45 sin 30
√2 √3 √2 1 √6 − √2
cos(45 + 30) = ( )( ) − ( )( ) =
2 2 2 2 4
√2 √3 √2 1 √6 + √2
cos(45 − 30) = ( ) ( ) + ( ) ( ) =
2 2 2 2 4

4.55: 𝒕𝒂𝒏
tan 𝛼 + tan 𝛽
tan(𝛼 + 𝛽) =
1 − tan 𝛼 tan 𝛽
tan 𝛼 − tan 𝛽
tan(𝛼 − 𝛽) =
1 + tan 𝛼 tan 𝛽

Use the definition and then the formula:


sin(𝛼 + 𝛽) sin 𝛼 cos 𝛽 + cos 𝛼 sin 𝛽
tan(𝛼 + 𝛽) = =
cos(𝛼 + 𝛽) cos 𝛼 cos 𝛽 − sin 𝛼 sin 𝛽

Divide numerator and denominator by cos 𝛼 cos 𝛽, and simplify:


sin 𝛼 cos 𝛽 cos 𝛼 sin 𝛽
+ tan 𝛼 + tan 𝛽
cos 𝛼 cos 𝛽 cos 𝛼 cos 𝛽
= =
cos 𝛼 cos 𝛽 sin 𝛼 sin 𝛽 1 − tan 𝛼 tan 𝛽

cos 𝛼 cos 𝛽 cos 𝛼 cos 𝛽

Example 4.56
A. tan 75°
B. tan 15°

√3 3 + √3 2
1+ 3 3 3 + √3 (3 + √3) 9 + 6√3 + 3
tan(45 + 30) = = = = = = 2 + √3
√3 3 − √3 3 − √3 6 6
1 − (1) ( 3 ) 3
1 √3 + 1
1+
√3 √3 + 1
tan(45 + 30) = = √3 =
1
1 − (1) ( ) √3 − 1 √3 − 1
√3 √3

1 √3 − 1
1−
√3 √3 − 1
tan(45 − 30) = = √3 =
1
1 + (1) ( ) √3 + 1 √3 + 1
√3 √3

√3 3 − √3 2
1− 3 3 3 − √3 (3 − √3) 9 − 6√3 + 3
tan(45 − 30) = = = = = = 2 − √3
√3 3 + √3 3 + √3 6 6
1 + (1) ( 3 ) 3

Example 4.57: Review

P a g e 158 | 213
Aziz Manva (azizmanva@gmail.com)

In Δ𝐴𝐵𝐶 is right-angled with ∠𝐴 = 𝛼, ∠𝐵 = 90°, ∠𝐶 = 𝛽, 𝐴𝐵 = 5, 𝐵𝐶 = 12. Find:


A. sin(𝛼 + 𝛽)
B. sin(𝛼 − 𝛽)
C. cos(𝛼 + 𝛽)
D. cos(𝛼 − 𝛽)
E. tan(𝛼 + 𝛽)
F. tan(𝛼 − 𝛽)

Example 4.58: Simple Harmonic Oscillator


In physics, a simple harmonic oscillator is used to model alternating current, the movement of an ideal spring
and waves. The equation of a simple harmonic oscillator:
2𝜋
𝑥 = 𝑎 cos ( 𝑡 + 𝜙)
𝑇
Where
𝑇 = 𝑇𝑖𝑚𝑒 𝑝𝑒𝑟𝑖𝑜𝑑 𝑜𝑓 𝑜𝑠𝑐𝑖𝑙𝑙𝑎𝑡𝑖𝑜𝑛
𝜙 = 𝑝ℎ𝑎𝑠𝑒 𝑠ℎ𝑖𝑓𝑡

2𝜋
A. Use a change of variable. Substitute 𝑎𝑛𝑔𝑢𝑙𝑎𝑟 𝑓𝑟𝑒𝑞𝑢𝑒𝑛𝑐𝑦 = 𝜔 = 𝑇
to simplify the equation.
B. Show that (𝑎 cos 𝜙)(cos 𝜔𝑡) − (𝑎 sin 𝜙)(sin 𝜔𝑡) is also the equation of a simple harmonic oscillator.

𝑥 = 𝑎 cos(𝜔𝑡 + 𝜙)
𝑎[(cos 𝜙)(cos 𝜔𝑡) − (sin 𝜙)(sin 𝜔𝑡)] = 𝑎 cos(𝜔𝑡 + 𝜙)
B. Identities

Example 4.59
sin(90 − 𝜃) = cos 𝜃
𝜋
cos ( − 𝜃) = sin 𝜃
2

Using sin(𝛼 − 𝛽) = sin 𝛼 cos 𝛽 − cos 𝛼 sin 𝛽:


𝐿𝐻𝑆 = sin(90 − 𝜃) = sin 90° cos 𝜃 − cos 90° sin 𝜃 = (1) cos 𝜃 − (0) sin 𝜃 = cos 𝜃 = 𝑅𝐻𝑆

Using cos(𝛼 − 𝛽) = cos 𝛼 cos 𝛽 + sin 𝛼 sin 𝛽:


𝜋 𝜋
𝐿𝐻𝑆 = cos cos(𝜃) + sin ( ) sin 𝜃 = 0 × cos(𝜃) + 1 × sin 𝜃 = sin 𝜃
2 2

Example 4.60
Prove the following identities:
sin(𝛼 + 𝛽) tan 𝛼 + tan 𝛽
𝐴: =
sin(𝛼 − 𝛽) tan 𝛼 − tan 𝛽
cos(𝛼 + 𝛽) 1 − tan 𝛼 tan 𝛽
𝐵: =
cos(𝛼 − 𝛽) 1 + tan 𝛼 tan 𝛽

Part A
Use the formula to expand the LHS and then divide by cos 𝛼 cos 𝛽 in the numerator and denominator:

P a g e 159 | 213
Aziz Manva (azizmanva@gmail.com)

1
(sin 𝛼 cos 𝛽 + cos 𝛼 sin 𝛽) ×
cos 𝛼 cos 𝛽 tan 𝛼 + tan 𝛽
𝐿𝐻𝑆 = = = 𝑅𝐻𝑆
1 tan 𝛼 − tan 𝛽
(sin 𝛼 cos 𝛽 − cos 𝛼 sin 𝛽) ×
cos 𝛼 cos 𝛽
Part B
Use the formula to expand the LHS, and since we want RHS to begin with a 1, divide by cos 𝛼 cos 𝛽 in the
numerator and denominator:
1
(cos 𝛼 cos 𝛽 − sin 𝛼 sin 𝛽) ×
cos 𝛼 cos 𝛽 1 − tan 𝛼 tan 𝛽
𝐿𝐻𝑆 = = = 𝑅𝐻𝑆
1 1 + tan 𝛼 tan 𝛽
(cos 𝛼 cos 𝛽 + sin 𝛼 sin 𝛽) ×
cos 𝛼 cos 𝛽

Example 4.61
Identities like the ones below are useful in Calculus9. Prove them:
sin(𝑥 + ℎ) − sin 𝑥 cos ℎ − 1 sin ℎ
𝐴: = sin 𝑥 ( ) + cos 𝑥 ( )
ℎ ℎ ℎ
cos(𝑥 + ℎ) − cos 𝑥 cos ℎ − 1 sin ℎ
𝐵: = cos 𝑥 ( ) − sin 𝑥 ( )
ℎ ℎ ℎ

Use the formula to expand, rearrange and factor, and then split the fraction:
sin 𝑥 cos ℎ + sin ℎ cos 𝑥 − sin 𝑥 cos ℎ − 1 sin ℎ
𝐴: 𝐿𝐻𝑆 = = sin 𝑥 ( ) + cos 𝑥 ( ) = 𝑅𝐻𝑆
ℎ ℎ ℎ
cos 𝑥 cos ℎ − sin 𝑥 sin ℎ − cos 𝑥 cos ℎ − 1 sin ℎ
𝐵: 𝐿𝐻𝑆 = = cos 𝑥 ( ) − sin 𝑥 ( ) = 𝑅𝐻𝑆
ℎ ℎ ℎ

Example 4.62
Prove:
sin(𝑎 + 𝑏) sin(𝑎 − 𝑏) = sin2 𝑎 − sin2 𝑏

Use the sum and difference formula:


𝐿𝐻𝑆 = [sin 𝑎 cos 𝑏 + sin 𝑏 cos 𝑎][sin 𝑎 cos 𝑏 − sin 𝑏 cos 𝑎]

Use (𝑥 + 𝑦)(𝑥 − 𝑦) = 𝑥 2 − 𝑦 2 :
sin2 𝑎 cos2 𝑏 − sin2 𝑏 cos2 𝑎
= sin2 𝑎 (1 − sin2 𝑏) − sin2 𝑏 (1 − sin2 𝑎)
= sin2 𝑎 − sin2 𝑎 sin2 𝑏 − sin2 𝑏 + sin2 𝑎 sin2 𝑏
= sin2 𝑎 − sin2 𝑏 = 𝑅𝐻𝑆

Example 4.63
If cos(𝑎 + 𝑏) = cos 𝑎 cos 𝑏 show that:
sin2 (𝑎 + 𝑏) = (sin 𝑎 + sin 𝑏)2

Expand the LHS using the sum formula for 𝑠𝑖𝑛:


(sin 𝑎 cos 𝑏 + sin 𝑏 cos 𝑎)2

Square:
= sin2 𝑎 cos2 𝑏 + 2 sin 𝑎 sin 𝑏 cos 𝑎 cos 𝑏 + sin2 𝑏 cos 2 𝑎

9 For calculating derivatives of trigonometric functions using the limit definition.

P a g e 160 | 213
Aziz Manva (azizmanva@gmail.com)

Substitute cos 𝑎 cos 𝑏 − sin 𝑎 sin 𝑏 = cos 𝑎 cos 𝑏 ⇒ sin 𝑎 sin 𝑏 = 0:


= sin2 𝑎 cos2 𝑏 + sin2 𝑏 cos2 𝑎

Substitute cos2 𝑏 = 1 − sin2 𝑏 , cos 2 𝑎 = 1 − sin2 𝑎:


= sin2 𝑎 (1 − sin2 𝑏) + sin2 𝑏 (1 − sin2 𝑎)

Expand and simplify:


= sin2 𝑎 − sin2 𝑎 sin2 𝑏 + sin2 𝑏 − sin2 𝑎 sin2 𝑏
= sin2 𝑎 + sin2 𝑏

Substitute sin 𝑎 sin 𝑏 = 0:


= sin2 𝑎 + 2 sin 𝑎 sin 𝑏 + sin2 𝑏
= (sin 𝑎 + sin 𝑏)2 = 𝑅𝐻𝑆

Example 4.64
Prove:
sin(𝑎 − 𝑏) sin(𝑏 − 𝑐) sin(𝑐 − 𝑎)
+ + =0
sin 𝑎 sin 𝑏 sin 𝑏 sin 𝑐 sin 𝑐 sin 𝑎

Rearrange:
sin(𝑎 − 𝑏) sin(𝑏 − 𝑐) sin(𝑐 − 𝑎)
+ =−
sin 𝑎 sin 𝑏 sin 𝑏 sin 𝑐 sin 𝑐 sin 𝑎

sin(𝑎 − 𝑏) sin 𝑐 + sin(𝑏 − 𝑐) sin 𝑎


𝐿𝐻𝑆 =
sin 𝑎 sin 𝑏 sin 𝑐

[sin 𝑎 cos 𝑏 − sin 𝑏 cos 𝑎] sin 𝑐 + [sin 𝑏 cos 𝑐 − sin 𝑐 cos 𝑏] sin 𝑎
=
sin 𝑎 sin 𝑏 sin 𝑐

[sin 𝑎 cos 𝑏 sin 𝑐 − sin 𝑏 sin 𝑐 cos 𝑎] + [sin 𝑎 sin 𝑏 cos 𝑐 − sin 𝑎 cos 𝑏 sin 𝑐]
=
sin 𝑎 sin 𝑏 sin 𝑐
sin 𝑎 sin 𝑏 cos 𝑐 − sin 𝑏 sin 𝑐 cos 𝑎
=
sin 𝑎 sin 𝑏 sin 𝑐
sin(𝑎 − 𝑐) − sin(𝑐 − 𝑎)
= = = 𝑅𝐻𝑆
sin 𝑎 sin 𝑐 sin 𝑎 sin 𝑐
C. Expressions

Example 4.65
Determine the value of cos(𝐴 + 𝐵) if
𝜋 𝜋 𝜋 𝜋
sin ( − 𝐴) sin ( − 𝐵) = cos ( − 𝐴) cos ( − 𝐵)
2 2 2 2

𝜋 𝜋
Using the cofunction identities sin ( − 𝜃) = cos 𝜃 , cos ( − 𝜃) = sin 𝜃:
2 2
cos 𝐴 cos 𝐵 = sin 𝐴 sin 𝐵
cos 𝐴 cos 𝐵 − sin 𝐴 sin 𝐵 = 0

Using the sum formula for 𝑐𝑜𝑠:


cos(𝐴 + 𝐵) = 0

P a g e 161 | 213
Aziz Manva (azizmanva@gmail.com)

Example 4.66
Simplify
A. cos(𝐴 + 𝐵) cos 𝐵 + sin(𝐴 + 𝐵) sin 𝐵
B. sin(𝐴 − 𝐵) cos 𝐵 + cos(𝐴 − 𝐵) sin 𝐵

Part A
= [cos 𝐴 cos 𝐵 − sin 𝐴 sin 𝐵] cos 𝐵 + [sin 𝐴 cos 𝐵 + sin 𝐵 cos 𝐴] sin 𝐵
= [cos 𝐴 cos2 𝐵 − sin 𝐴 sin 𝐵 cos 𝐵] + [sin 𝐴 sin 𝐵 cos 𝐵 + sin2 𝐵 cos 𝐴]

Simplify, factor, and use the Pythagorean Identity:


= cos 𝐴 cos 2 𝐵 + sin2 𝐵 cos 𝐴
= cos 𝐴 (cos 2 𝐵 + sin2 𝐵) = cos 𝐴
Part B
= [sin 𝐴 cos 𝐵 − sin 𝐵 cos 𝐴] cos 𝐵 + [cos 𝐴 cos 𝐵 + sin 𝐴 sin 𝐵] sin 𝐵
= [sin 𝐴 cos 2 𝐵 − sin 𝐵 cos 𝐴 cos 𝐵] + [sin 𝐵 cos 𝐴 cos 𝐵 + sin 𝐴 sin2 𝐵]
= sin 𝐴 cos 2 𝐵 + sin 𝐴 sin2 𝐵
= sin 𝐴

Example 4.67
Simplify
sin(𝐴 + 𝐵) − sin(𝐴 − 𝐵)
cos(𝐴 + 𝐵) + cos(𝐴 − 𝐵)

Expand using the sum and difference formula:


(sin 𝐴 cos 𝐵 + sin 𝐵 cos 𝐴) − (sin 𝐴 cos 𝐵 − sin 𝐵 cos 𝐴)
=
(cos 𝐴 cos 𝐵 − sin 𝐴 sin 𝐵) + (cos 𝐴 cos 𝐵 + sin 𝐴 sin 𝐵)

Simplify:
2 sin 𝐵 cos 𝐴 sin 𝐵
= = = tan 𝐵
2 cos 𝐴 cos 𝐵 cos 𝐵

D. Identities in Numerical Problems

4.68: Identities in Numerical Problems


Identities can be applied to numerical problems.
Recognizing that an identity is to be applied can be the most difficult part.

Example 4.69
√40
Given that tan 𝜃 = 3
evaluate:
𝜋 𝜋 𝜋 𝜋
cos (𝜃 + ) cos (𝜃 − ) − sin (𝜃 + ) sin (𝜃 − )
12 12 12 12
𝜋 𝜋
Use a change of variable. Let 𝐴 = 𝜃 + 12 , 𝐵 = 𝜃 − 12:
= cos 𝐴 cos 𝐵 − sin 𝐴 sin 𝐵

Use the difference formula for 𝑐𝑜𝑠, and changing back to the original variable:
𝜋 𝜋
= cos(𝐴 + 𝐵) = cos (𝜃 + + 𝜃 − ) = cos 2𝜃 = 2 cos 2 𝜃 − 1
12 12

P a g e 162 | 213
Aziz Manva (azizmanva@gmail.com)

√40 3 9
Substitute from a reference triangle tan 𝜃 = 3
⇒ cos 𝜃 = 7 ⇒ cos2 𝜃 = 49
9 18 27
= 2( ) − 1 = −1 = −
49 49 49

Example 4.70
1
𝑍= tan[cos −1(3𝑋 − 2𝑌)]
2√3

Find the solution set in terms of 𝜃 for the equation above given that:
𝜋 𝜋 𝜋 𝜋
𝑍 = sin ( − 𝜃) cos ( + 𝜃) + cos ( − 𝜃) sin ( + 𝜃) , 𝜃∈ℝ
12 12 12 12
𝑋 = 𝑀𝑎𝑥𝑖𝑚𝑢𝑚 𝑣𝑎𝑙𝑢𝑒 𝑜𝑓 𝑍
𝑌 = 𝑀𝑖𝑛𝑖𝑚𝑢𝑚 𝑣𝑎𝑙𝑢𝑒 𝑜𝑓 𝑍

𝜋 𝜋
Use a change of variable. Let 𝐴 = 12 − 𝜃, 𝐵 = 12 + 𝜃
𝜋 𝜋 𝜋 1
𝑍 = sin 𝐴 cos 𝐵 + cos 𝐴 sin 𝐵 = sin(𝐴 + 𝐵) = sin ( − 𝜃 + + 𝜃) = sin = = 𝐶𝑜𝑛𝑠𝑡𝑎𝑛𝑡
12 12 6 2

Since 𝑍 is a constant:
1 1
𝑋=𝑌= ⇒ 3𝑋 − 2𝑌 = 3𝑋 − 2𝑋 = 𝑋 =
2 2

We can then show the given equation is an identity:


1 1 1 𝜋 1 1
𝑅𝐻𝑆 = tan [cos −1 ( )] = tan [ ] = ∙ √3 = = 𝑍 = 𝐿𝐻𝑆
2√3 2 2√3 3 2√3 2

Hence:
𝜃∈ℝ

Example 4.71
2 sin(𝛼 − 𝜃) cos 𝛼 = sin(𝑛𝛼 − 𝜃) − sin 𝜃
Given that the above is an identity determine the value of 𝑛, where 𝑛 is a real number.

Use the difference identity on the LHS for sin(𝛼 − 𝜃):


𝐿𝐻𝑆 = [2 sin 𝛼 cos 𝜃 − 2 cos 𝛼 sin 𝜃] cos 𝛼

Distribute the cos 𝛼 to each term:


= 2 sin 𝛼 cos 𝛼 cos 𝜃 − 2 cos 2 𝛼 sin 𝜃

Use the double angle identity to substitute 2 sin 𝛼 cos 𝛼 = 2 sin 2𝛼 , 2 cos2 𝛼 = cos 2𝛼 + 1:
= sin 2𝛼 cos 𝜃 − (cos 2𝛼 + 1) sin 𝜃
= sin 2𝛼 cos 𝜃 − cos 2𝛼 sin 𝜃 − sin 𝜃

Use the difference identity on the LHS:


= sin(2𝛼 − 𝜃) − sin(𝜃)

𝑛=2

P a g e 163 | 213
Aziz Manva (azizmanva@gmail.com)

4.4 More Identities


A. Sum of Sinusoidal Functions

4.72: Sum of Sinusoidal Functions


For all 𝑥 and constants 𝑎, 𝑏, 𝐶 it is true that:
𝑎 sin 𝑥 + 𝑏 cos 𝑥 = 𝐴 sin(𝑥 + 𝐶)
Where:
𝐴 = √𝑎2 + 𝑏 2
𝑎 𝑏 𝑏
𝐶 = cos−1 ( ) = sin−1 ( ) = tan−1 ( )
√𝑎2 + 𝑏 2 √𝑎2 + 𝑏 2 𝑎

Suppose that for some constants 𝐴, 𝐵, 𝐶 to be determined, it is true that:


𝑎 sin 𝑥 + 𝑏 cos 𝑥 = 𝐴 sin(𝑥 + 𝐶)

Use the sum formula for 𝑠𝑖𝑛 on the RHS:


𝑎 sin 𝑥 + 𝑏 cos 𝑥 = 𝐴 sin 𝑥 cos 𝐶 + 𝐴 sin 𝐶 cos 𝑥

Use the method of undetermined coefficients. Comparing like terms:


𝐴 cos 𝐶 = 𝑎, 𝐴 sin 𝐶 = 𝑏

𝑎2 + 𝑏 2 = (𝐴 cos 𝑐)2 + (𝐴 sin 𝑐)2 = 𝐴2 cos 2 𝑐 + 𝐴2 sin2 𝐶 = 𝐴2 ⇒ 𝐴 = √𝑎2 + 𝑏 2

Note that we did not take the negative square root because we did not need to.

𝑎 𝑎 𝑎
cos 𝐶 = = ⇒ 𝐶 = cos −1 ( )
𝐴 √𝑎2 + 𝑏 2 √𝑎2 + 𝑏 2
𝑏 𝑏 𝑏
sin 𝐶 = = ⇒ 𝐶 = sin−1 ( )
𝐴 √𝑎2 + 𝑏 2 √𝑎2 + 𝑏 2
𝑏
sin 𝐶 𝐴 𝑏 𝑏
tan 𝐶 = = = ⇒ 𝐶 = tan−1 ( )
cos 𝐶 𝑎 𝑎 𝑎
𝐴

Example 4.73
Convert using the formula into a single 𝑠𝑖𝑛 function:
A. sin 𝑥 + cos 𝑥
B. sin 𝑥 − √3 cos 𝑥

𝑏
𝑎 sin 𝑥 + 𝑏 cos 𝑥 = √𝑎2 + 𝑏 2 sin (𝑥 + tan−1 ( ))
𝑎
Part A: Substitute 𝑎 = 𝑏 = 1:
1 𝜋
sin 𝑥 + cos 𝑥 = √12 + 12 sin (𝑥 + tan−1 ( )) = √2 sin (𝑥 + )
1 4

Part B: Substitute 𝑎 = 1, 𝑏 = −√3:


2 √3 𝜋
sin 𝑥 − √3 cos 𝑥 = √12 + (−√3) sin (𝑥 + tan−1 (− )) = 4 sin (𝑥 − )
1 3

P a g e 164 | 213
Aziz Manva (azizmanva@gmail.com)

Example 4.74
3 sin 𝑥 + 4 cos 𝑥
Determine the
A. Minimum value and maximum value
B. Range
C. Period

4 4
3 sin 𝑥 + 4 cos 𝑥 = √32 + 42 sin (𝑥 + tan−1 ( )) = 5 sin (𝑥 + tan−1 ( ))
3 3

𝑀𝑖𝑛 𝑉𝑎𝑙𝑢𝑒 = −5
𝑀𝑎𝑥 𝑉𝑎𝑙𝑢𝑒 = 5

Range as two interpretations:


𝑅𝑎𝑛𝑔𝑒 𝑜𝑓 𝑡ℎ𝑒 𝑓𝑢𝑛𝑐𝑡𝑖𝑜𝑛: − 5 ≤ 𝑥 ≤ 5
𝑅𝑎𝑛𝑔𝑒 𝑓𝑟𝑜𝑚 𝑠𝑡𝑎𝑡𝑖𝑠𝑡𝑖𝑐𝑠 = 𝑀𝑎𝑥 − 𝑚𝑖𝑛 = 5 − (−5) = 10

Period
2𝜋
= = 2𝜋
1

Example 4.75
Determine sin 𝐶
𝑝
2 sin 𝑥 + 3 cos 𝑥 = 𝐴 sin(𝐵𝑥 + 𝐶) , 𝐶 = cos−1 ( )
√𝑞
where the prime factorization of 𝑞 has no perfect squares.

𝑎
𝑎 sin 𝑥 + 𝑏 cos 𝑥 = √𝑎2 + 𝑏 2 sin (𝑥 + cos−1 ( ))
√𝑎2 + 𝑏2

Substitute √𝑎2 + 𝑏 2 = √22 + 32 = √4 + 9 = √13:


2
2 sin 𝑥 + 3 cos 𝑥 = √13 sin (𝑥 + cos−1 ( ))
√13

2 2 3
𝐶 = cos−1 ( ) ⇒ cos 𝐶 = ⇒ sin 𝐶 =
√13 √13 √13

4.76: Sum of Sinusoidal Functions


For all 𝑥 and constants 𝑎, 𝑏, 𝐶 it is true that:
𝑎 sin 𝑥 + 𝑏 cos 𝑥 = 𝐴 cos(𝑥 − 𝐶)
Where:
𝐴 = √𝑎2 + 𝑏 2
𝑎 𝑏 𝑏
𝐶 = cos−1 ( ) = sin−1 ( ) = tan−1 ( )
√𝑎2 + 𝑏 2 √𝑎2 + 𝑏 2 𝑎

Suppose that for some constants 𝐴, 𝐵, 𝐶 to be determined, it is true that:


𝑎 sin 𝑥 + 𝑏 cos 𝑥 = 𝐴 cos(𝑥 − 𝐶)

Use the sum formula for 𝑐𝑜𝑠 on the RHS:

P a g e 165 | 213
Aziz Manva (azizmanva@gmail.com)

𝑎 sin 𝑥 + 𝑏 cos 𝑥 = 𝐴 cos 𝑥 cos 𝐶 + 𝐴 sin 𝐶 sin 𝑥

Use the method of undetermined coefficients. Comparing like terms:


𝐴 cos 𝐶 = 𝑏, 𝐴 sin 𝐶 = 𝑎

𝑎2 + 𝑏 2 = (𝐴 sin 𝑐)2 + (𝐴 cos 𝑐)2 = 𝐴2 sin2 𝑐 + 𝐴2 𝑐𝑜𝑠 2 𝐶 = 𝐴2 ⇒ 𝐴 = √𝑎2 + 𝑏 2

Note that we did not take the negative square root because we did not need to.

𝑏 𝑎 𝑏
cos 𝐶 = = ⇒ 𝐶 = cos −1 ( )
𝐴 √𝑎2 + 𝑏 2 √𝑎2 + 𝑏 2
𝑎 𝑎 𝑎
sin 𝐶 = = ⇒ 𝐶 = sin−1 ( )
𝐴 √𝑎 + 𝑏
2 2 √𝑎 + 𝑏 2
2
𝑎
sin 𝐶 𝐴 𝑎 𝑎
tan 𝐶 = = = ⇒ 𝐶 = tan−1 ( )
cos 𝐶 𝑏 𝑏 𝑏
𝐴
B. Conditional Identities

4.77: Angles of a Triangle


𝐴+𝐵+𝐶 =𝜋

C. Reduction Identities

4.78: Triple Angle Identity


sin 3𝑥 = 3 sin 𝑥 − 4 sin3 𝑥

Split 3𝑥 as 2𝑥 + 𝑥 in the LHS:


sin 3𝑥 = sin(2𝑥 + 𝑥)

Use the sum identity:


= sin 2𝑥 cos 𝑥 + sin 𝑥 cos 2𝑥

Use the double angle identities:


= (2 sin 𝑥 cos 𝑥) cos 𝑥 + sin 𝑥 (1 − 2 sin2 𝑥)
= 2 sin 𝑥 cos2 𝑥 + sin 𝑥 − 2 sin3 𝑥

Use the Pythagorean Identity:


= 2 sin 𝑥 (1 − sin2 𝑥) + sin 𝑥 − 2 sin3 𝑥
= 2 sin 𝑥 − 2 sin3 𝑥 + sin 𝑥 − 2 sin3 𝑥
= 3 sin 𝑥 − 4 sin3 𝑥 = 𝑅𝐻𝑆

D. Product to Sum Identities

4.79: Product to Sum Formula


2 sin 𝛼 cos 𝛽 = sin(𝛼 + 𝛽) + sin(𝛼 − 𝛽)
2 cos 𝛼 sin 𝛽 = sin(𝛼 + 𝛽) − sin(𝛼 − 𝛽) =

P a g e 166 | 213
Aziz Manva (azizmanva@gmail.com)

We can set up some cancellation here by expanding using the formula for the sum and differences of sines:
sin 𝛼 cos 𝛽 + 𝐜𝐨𝐬 𝜶 𝐬𝐢𝐧 𝜷 + ⏟
⏟ sin 𝛼 cos 𝛽 − 𝐜𝐨𝐬 𝜶 𝐬𝐢𝐧 𝜷 = 2 sin 𝛼 cos 𝛽
sin(𝛼+𝛽) sin(𝛼−𝛽)

𝐬𝐢𝐧 𝜶 𝐜𝐨𝐬 𝜷 + cos 𝛼 sin 𝛽 − [𝐬𝐢𝐧


⏟ ⏟ 𝜶 𝐜𝐨𝐬 𝜷 − cos 𝛼 sin 𝛽 ] = 2 cos 𝛼 sin 𝛽
sin(𝛼+𝛽) sin(𝛼−𝛽)

Example 4.80
𝑃𝑟𝑜𝑑𝑢𝑐𝑡 𝑡𝑜 𝑆𝑢𝑚 𝐹𝑜𝑟𝑚𝑢𝑙𝑎: 2 sin 𝛼 cos 𝛽 = sin(𝛼 + 𝛽) + sin(𝛼 − 𝛽)
Prove the product to sum formula using the sum to product formula.

Apply the sum to product formula on the RHS:


(𝛼 + 𝛽) + (𝛼 − 𝛽) (𝛼 + 𝛽) − (𝛼 − 𝛽)
𝑅𝐻𝑆 = 2 [sin ( ) cos ( )]
2 2
2𝛼 2𝛽
2 [sin ( ) cos ( )]
2 2
2[sin 𝛼 cos 𝛽]
E. Sum to Product Identities

4.81: 𝒄𝒐𝒔
𝛼+𝛽 𝛼−𝛽
cos 𝛼 + cos 𝛽 = 2 cos ( ) cos ( )
2 2

F. A product identity

4.82: Product
𝜋 2𝜋 3𝜋 (𝑛 − 1)𝜋 𝑛
sin ( ) sin ( ) sin ( ) . . . sin ( ) = 𝑛−1 , 𝑛∈ℕ
𝑛 𝑛 𝑛 𝑛 2

Example 4.83
𝜋 𝜋 3𝜋 𝜋 7𝜋
sin ( ) sin ( ) sin ( ) sin ( ) … sin ( )
8 4 8 2 8

𝜋 2𝜋 3𝜋 4𝜋 7𝜋
sin ( ) sin ( ) sin ( ) sin ( ) … sin ( )
8 8 8 8 8
𝑛
Substitute 𝑛 = 8 = 23 in 2𝑛−1 :
23 23 1 1
8−1
= 7
= 4=
2 2 2 16

Example 4.84
𝜋 2𝜋 3𝜋 15𝜋
𝑎 = sin ( ) sin ( ) sin ( ) … sin ( )
16 16 16 16
17𝜋 18𝜋 31𝜋
𝑏 = sin ( ) sin ( ) . . . sin ( )
16 16 16
Find 𝑎𝑏

P a g e 167 | 213
Aziz Manva (azizmanva@gmail.com)

𝑛
Substitute 𝑛 = 16 = 24 in 2𝑛−1 :
𝜋 2𝜋 3𝜋 15𝜋 24 24 1
𝑎 = sin ( ) sin ( ) sin ( ) … sin ( ) = 16−1 = 15 = 11
16 16 16 16 2 2 2

𝑛
Substitute 𝑛 = 16 = 24 in 2𝑛−1 :
𝜋 2𝜋 15𝜋
𝑏 = sin (𝜋 + ) sin (𝜋 + ) … sin (𝜋 + )
16 16 16
𝜋 2𝜋 15𝜋
= [− sin ( )] [− sin ( )] … [− sin ( )]
16 16 16
𝜋 2𝜋 3𝜋 15𝜋
= − sin ( ) sin ( ) sin ( ) … sin ( )
16 16 16 16
= −𝑎

1 2 1
𝑎𝑏 = −𝑎2 = − ( 11
) = − 22
2 2

4.85: Product Notation


𝑘=𝑛−1
𝑘𝜋 𝑛
∏ sin ( ) = 𝑛−1 , 𝑛∈ℕ
𝑛 2
𝑘=1

Example 4.86
𝜋 2𝜋 6𝜋
sin ( ) ∙ sin ( ) ∙ … ∙ sin ( )
7 7 7

The property above is applicable with 𝑛 = 7:


𝑘=6
𝑘𝜋 7 7
∏ sin ( )= 6=
7 2 64
𝑘=1

Example 4.87
Determine the value of the expression given that that it has 𝑛 terms:
𝜋 2𝜋 3𝜋
sin ( ) ∙ sin ( ) ∙ sin ( ) ∙ … ∙
𝑛 𝑛 𝑛

𝜋 2𝜋 3𝜋 𝑛𝜋
sin ( ) ∙ sin ( ) ∙ sin ( ) ∙ … ∙ sin ( )
𝑛 𝑛 𝑛 𝑛
𝑛𝜋
sin ( ) = sin 𝜋 = 0
𝑛
𝐸𝑥𝑝𝑟𝑒𝑠𝑠𝑖𝑜𝑛 = 0

Example 4.8810
Determine the value of the expression below as a real number:

10 This lengthier explanation makes use only of standard trigonometric properties.

P a g e 168 | 213
Aziz Manva (azizmanva@gmail.com)

𝜋 2𝜋 3𝜋
sin ( ) ∙ sin ( ) ∙ sin ( )
7 7 7

Let
𝜋 2𝜋 3𝜋
𝑥 = sin ( ) ∙ sin ( ) ∙ sin ( )
7 7 7

Square both sides:


𝜋 2𝜋 3𝜋 𝜋 2𝜋 3𝜋
𝑥 2 = sin ( ) ∙ sin ( ) ∙ sin ( ) ∙ sin ( ) ∙ sin ( ) ∙ sin ( )
7 7 7 7 7 7

Use the property sin 𝜃 = sin(𝜋 − 𝜃) on the last three terms:


𝜋 𝜋 6𝜋
sin ( ) = sin (𝜋 − ) = sin ( )
7 7 7
2𝜋 2𝜋 5𝜋
sin ( ) = sin (𝜋 − ) = sin ( )
7 7 7
3𝜋 3𝜋 4𝜋
sin ( ) = sin (𝜋 − ) = sin ( )
7 7 7

Make the above substitutions:


𝜋 2𝜋 3𝜋 4𝜋 5𝜋 6𝜋
𝑥 2 = sin ( ) ∙ sin ( ) ∙ sin ( ) ∙ sin ( ) ∙ sin ( ) ∙ sin ( )
7 7 7 7 7 7
2
7
𝑥 =
64
√7
𝑥=
8

P a g e 169 | 213
Aziz Manva (azizmanva@gmail.com)

5. INVERSE TRIG FUNCTIONS


5.1 Inverse Trigonometric Functions (ITF)
A. Inverse Sine Functions
After you are done with this section you should not only understand the inverse trigonometric functions, but
also have their 𝑔𝑟𝑎𝑝ℎ𝑠, 𝑑𝑜𝑚𝑎𝑖𝑛, 𝑟𝑎𝑛𝑔𝑒 and 𝑝𝑟𝑜𝑝𝑒𝑟𝑡𝑖𝑒𝑠 memorized.

5.1: Inverse Sine Function


A. Does the function 𝑦 = sin 𝑥 meet the horizontal line test
for an invertible function? Hence, does it have an
inverse?
B. Identify a restricted domain for the sin function such that
the entire range of sin x is achieved, 0 lies in the domain,
and the function meets the horizontal line test. What
quadrants does this represent on the unit circle?
C. Graph 𝑦 = sin−1 𝑥 by reflecting 𝑦 = sin 𝑥 across the line 𝑦 = 𝑥. State its domain and range.
D. When is sin−1 𝑥 > 0? < 0? neither positive nor negative?

Part A
The sine function does not satisfy the horizontal line test. Hence, there is no inverse for the function:
𝑦 = sin 𝑥
However, we can make the function invertible by restricting the domain.
Part B
𝑅𝑎𝑛𝑔𝑒 𝑜𝑓 sin 𝑥 𝑖𝑠 [−1,1]
The restricted domain is:
𝜋 𝜋
𝑦 = −1 ⇒ 𝑥 = − , 𝑦=1⇒𝑥=
⏟ 2 ⏟ 2
𝑺𝒕𝒂𝒓𝒕 𝑷𝒐𝒊𝒏𝒕 𝑬𝒏𝒅 𝑷𝒐𝒊𝒏𝒕
It represents
𝜋
0 < 𝑥 < ⇔ 0° < 𝑥 < 90° ⇔ 𝑰𝒔𝒕 𝑄𝑢𝑎𝑑𝑟𝑎𝑛𝑡
2
𝜋
− < 𝑥 < 0 ⇔ −90° < 𝑥 < 0° ⇔ 𝑰𝑽𝒕𝒉 𝑄𝑢𝑎𝑑𝑟𝑎𝑛𝑡
2
Part C
𝜋 𝜋
𝑦 = sin−1 𝑥 , −1 ≤ 𝑥 < 1 ,
⏟ − ≤𝑦≤
⏟2 2
𝐷𝑜𝑚𝑎𝑖𝑛
𝑅𝑎𝑛𝑔𝑒
Part D
sin−1 𝑥 is positive when 𝑥 is in the first quadrant in the unit circle. We can write
this in interval notation:
𝜋
(0,90°] ⇔ (0, ]
2
sin−1 𝑥 is negative when 𝑥 is in the fourth quadrant in the unit circle. We can
write this in interval notation:
𝜋
[−90°, 0) ⇔ [− , 0)
2
In other words, for 𝑥 ∈ 𝐷sin−1 𝑥 :
𝑥 > 0 ⇒ sin−1 𝑥 > 0
𝑥 < 0 ⇒ sin−1 𝑥 < 0

5.2: Inverse Sine Function


P a g e 170 | 213
Aziz Manva (azizmanva@gmail.com)

The sine inverse function satisfies the cancellation equations:


sin(sin−1 𝑥) = 𝑥, −1 ≤ 𝑥 ≤ 1
−1 (sin
𝜋 𝜋
sin 𝑥) = 𝑥, − ≤𝑥≤
2 2

Example 5.3

Evaluating 𝒔𝒊𝒏−𝟏 G. sin (sin−1 )


1
2
Answer in both degrees and radians. If your answer √3
is negative, write your final answer as an equivalent H. sin (sin−1 (− 2
))
positive angle. I. sin(sin−1 0.24)
1
A. sin−1 ( ) J. sin(sin−1 1.2)
2
K. sin−1(sin 45°)
−1 √3
B. sin ( 2 ) 𝜋
L. sin−1 (sin 3 )
1
C. sin−1 ( ) 11
√2 M. sin−1 (sin 6
𝜋)
1
D. sin−1 (− 2) Cancellations using 𝐬𝐢𝐧(𝟏𝟖𝟎 − 𝜽) = 𝒔𝒊𝒏 𝜽

√3 Evaluate. Write your answer in the same notation
E. sin−1 (− 2 )
(degrees/radians) as in the question.
1
F. sin−1 (− 2) N. sin−1(sin 120°)
Basic Cancellations O. sin−1(sin 119°)
3𝜋
Evaluate. Write your answer in the same notation P. sin−1 (sin 5
)
(degrees/radians) as in the question.

Evaluating 𝒔𝒊𝒏−𝟏 √3 √3
1 𝜋 sin (sin−1 (− )) = −
sin−1 ( ) = 30° = 2 2
2 6 −1
sin(sin 0.24) = 0.24
√3 𝜋 sin(sin−1 1.2)
sin−1 ( ) = 60° =
2 3 → 𝑁𝑜𝑡 𝑑𝑒𝑓𝑖𝑛𝑒𝑑 𝑠𝑖𝑛𝑐𝑒 1.2 𝑖𝑠 𝑛𝑜𝑡 𝑖𝑛 𝑡ℎ𝑒 𝑑𝑜𝑚𝑎𝑖𝑛 𝑜𝑓 sin−1 𝑥
1 𝜋 sin−1 (sin 45°) = 45°
sin−1 ( ) = 45° = 𝜋 𝜋
√2 4
sin−1 (sin ) =
1 𝜋 3𝜋 3 3
sin−1 (− ) = −45° = − = 11 11
√2 4 4 sin−1 (sin 𝜋) = 𝜋
√3 𝜋 2𝜋 6 6
sin−1 (− ) = −60° = − = Using 𝐬𝐢𝐧(𝟏𝟖𝟎 − 𝜽)
2 3 3
sin−1(sin(180 − 60)) = sin−1(sin 60) = 60°
1 𝜋 5𝜋
sin−1 (− ) = −30° = − = sin−1(sin(180 − 61)) = sin−1(sin 61) = 61°
2 6 6 3𝜋 2𝜋 2𝜋
First and Fourth Quadrants sin−1 (sin (𝜋 − )) = sin−1 (sin ( )) =
1 1 5 5 5
sin (sin−1 ) =
2 2

Example 5.4
Evaluate ℎ(𝑥), 0 ≤ 𝑥 ≤ 2𝜋 if:
𝑓(𝑥) = sin 𝑥 , 𝑔(𝑥) = sin−1 𝑥 , ℎ(𝑥) = 𝑔(𝑓(𝑥))

ℎ(𝑥) = 𝑔(𝑓(𝑥)) = sin−1 (sin 𝑥)

P a g e 171 | 213
Aziz Manva (azizmanva@gmail.com)

If 𝑥 is in the first or the fourth quadrant, then the function and its inverse will cancel:
𝜋 3𝜋
0≤𝑥≤ , ≤ 𝑥 ≤ 2𝜋 ⇒ ℎ(𝑥) = 𝑥
2 4

If 𝑥 is in the second or the third quadrant, then 𝑥 is not in the domain of sin−1 𝑥, so we can’t cancel directly. We
need to find a corresponding value:
sin 𝑥 = sin(180 − 𝑥)
ℎ(𝑥) = sin−1(sin 𝑥) = sin−1(sin 180 − 𝑥) = 180 − 𝑥

𝜋 3𝜋
𝑥, 0 ≤ 𝑥 ≤ , ≤ 𝑥 ≤ 2𝜋
ℎ(𝑥) = { 2 4
𝜋 3𝜋
180 − 𝑥, ≤ 𝑥 <
2 4

5.5: Reciprocal Property


1
sin−1 𝑥 = csc −1 , 𝑥 ∈ [−1,1]
𝑥
1
csc −1 𝑥 = sin−1 , 𝑥 ∈ 𝑅 − (−1,1)
𝑥
𝜋 𝜋
𝜃 = csc −1 𝑥 , − <𝜃<
2 2

1 1
csc 𝜃 = 𝑥 ⇒ sin 𝜃 = ⇒ 𝜃 = sin−1
𝑥 𝑥

5.6: Odd Function


An odd function is a function that satisfies 𝑓(−𝑥) = −𝑓(𝑥). The inverse sine
function is an odd function. That is, it satisfies:
sin−1(−𝑦) = − sin−1 𝑦

𝜃 = sin−1(−𝑦)
Apply the sin function both sides:
sin 𝜃 = −𝑦 ⇒ 𝑦 = − sin 𝜃 ⇒ 𝑦 = sin(−𝜃)
Apply the sin-1 function to both sides:
sin−1 𝑦 = −𝜃 ⇒ 𝜃 = − sin−1 𝑦 ⇒ sin−1 (−𝑦) = − sin−1 𝑦

B. Inverse Cosine Function

5.7: Inverse Cosine Function

Example 5.8

Evaluating 𝒄𝒐𝒔−𝟏 A.
1
cos−1 ( )
2
Answer in both degrees and radians. If your answer
−1 √3
is negative, write your final answer as an equivalent B. cos ( 2 )
positive angle. C.
1
cos−1 ( 2)

P a g e 172 | 213
Aziz Manva (azizmanva@gmail.com)

D. cos−1 (−
1
) J. cos(cos−1 1.2)
√2
K. cos−1(cos 45°)
√3
E. cos−1 (− 2 ) 𝜋
L. cos−1 (cos 3 )
1
F. cos−1 (− 2) M. cos−1 (cos
11
𝜋)
6
Basic Cancellations Cancellations using 𝐜𝐨𝐬(𝟏𝟖𝟎 − 𝜽) = −𝒄𝒐𝒔 𝜽
Evaluate. Write your answer in the same notation
Evaluate. Write your answer in the same notation
(degrees/radians) as in the question. (degrees/radians) as in the question.
1
G. cos (cos−1 ) N. cos−1(cos 120°)
2
√3 O. cos−1(cos 119°)
H. cos (cos−1 (− 2
)) 3𝜋
−1 P. cos−1 (cos )
I. cos(cos 0.24) 5

1
1 cos−1 (− ) = 135°
cos −1 ( ) = 60° √2
2 √3
√3 cos−1 (− ) = 150°
cos−1 ( ) = 30° 2
2
1
cos −1 ( ) = 45°
√2

C. Inverse Tan Function

5.9: Inverse Tan Function

Example 5.10

Evaluating 𝒄𝒐𝒔−𝟏 G. sin (sin−1 2)


1

Answer in both degrees and radians. If your answer


√3
is negative, write your final answer as an equivalent H. sin (sin−1 (− 2
))
positive angle. I. sin(sin−1 0.24)
A. tan−1 (√3) J. sin(sin−1 1.2)
√3 K. sin−1(sin 45°)
B. tan−1 ( 3 ) 𝜋
L. sin−1 (sin 3 )
C. tan−1 (1)
11
D. tan−1 (−√3) M. sin−1 (sin 𝜋)
6
√3 Cancellations using 𝐬𝐢𝐧(𝟏𝟖𝟎 − 𝜽) = 𝒔𝒊𝒏 𝜽
E. tan−1 (− 3
)
Evaluate. Write your answer in the same notation
F. tan−1 (−1)
(degrees/radians) as in the question.
Basic Cancellations
N. sin−1(sin 120°)
Evaluate. Write your answer in the same notation
O. sin−1(sin 119°)
(degrees/radians) as in the question. 3𝜋
P. sin−1 (sin 5
)

5.11: Inverse Function-Inverse Cofunction Identities


𝜋
cos −1 𝑥 = − sin−1 𝑥
2
𝜋
cot −1 𝑥 = − tan−1 𝑥
2
P a g e 173 | 213
Aziz Manva (azizmanva@gmail.com)

𝜋
csc −1 𝑥 = − sec −1 𝑥
2

𝜃 = sin−1 (𝑥) ⇒ sin 𝜃 = 𝑥


𝜋 𝜋 𝜋
cos ( − 𝜃) = 𝑥 ⇒ − 𝜃 = cos −1 𝑥 ⇒ = 𝜃 + cos −1 𝑥 = sin−1 (𝑥) + cos −1 𝑥
2 2 2

D. Trig Functions of Inverse Trig Functions

Example 5.12
sin(cos−1 𝑥)
Use a change of variable. Let:
𝜃 = cos−1 𝑥
Now draw a right-angled triangle with 𝜃 as one of the angles
(𝑠𝑒𝑒 𝑑𝑖𝑎𝑔𝑟𝑎𝑚):
𝑥 𝐴𝑑𝑗
cos 𝜃 = 𝑥 = =
1 𝐻𝑦𝑝
Use the Pythagorean Theorem to find the opposite side.
𝑂𝑝𝑝 = √𝐻𝑦𝑝2 − 𝐴𝑑𝑗 2 = √1 − 𝑥 2
Finally:
√1 − 𝑥 2
sin(cos −1 𝑥) = sin 𝜃 = = √1 − 𝑥 2
1
1 − 𝑥 2 ≥ 0 ⇒ 1 ≥ 𝑥 2 ⇒ 𝑥 2 ≤ 1 ⇒ 𝑥 ∈ [−1,1]

cos(sin−1 𝑥)

Let:
𝜃 = sin−1 𝑥
Take sin of both sides:
𝑥
sin 𝜃 = 𝑥 =
1
Draw a reference triangle. Then:
cos(sin−1 𝑥) = cos 𝜃 = √1 − 𝑥 2

tan(sin−1 𝑥)

sin(sin−1 𝑥) 𝑥
tan(sin−1 𝑥) = −1
=
cos(sin 𝑥) √1 − 𝑥 2
𝑥
𝜃 = sin−1 𝑥 ⇒ sin 𝜃 = 𝑥 =
1
−1
𝑥
tan(sin 𝑥) = tan 𝜃 =
√1 − 𝑥 2

sec(cos−1 𝑥)

Let
1 1
𝜃 = cos−1 𝑥 ⇒ cos 𝜃 = 𝑥 ⇒ sec 𝜃 = =
cos 𝑥 𝑥

P a g e 174 | 213
Aziz Manva (azizmanva@gmail.com)

1
sec(cos−1 𝑥) = sec 𝜃 =
𝑥

Example 5.13: Application of Reference Triangles


𝑥
tan (sin−1 )
𝑥+1
𝑥 𝑥
𝜃 = sin−1 ⇒ sin 𝜃 =
𝑥+1 𝑥+1
𝑥 𝑥
tan (sin−1 ) = tan 𝜃 =
𝑥+1 √2𝑥 + 1
𝑥
tan (sec −1 )
2

𝑥 𝑥 2
𝜃 = sec −1 ⇒ sec 𝜃 = ⇒ cos 𝜃 =
2 2 𝑥

𝑜𝑝𝑝 𝑠𝑖𝑑𝑒 = √𝑥 2 − 4
√𝑥 2 − 4
tan 𝜃 =
2

Example 5.14
Show that if sin(sin−1 𝑥 + cos −1 𝑦) = 2𝑥𝑦, then 𝑥 and 𝑦 both lie on the unit circle.

𝐿𝐻𝑆 = 𝑥𝑦 + √(1 − 𝑥 2 )(1 − 𝑦 2 )

𝑥𝑦 + √(1 − 𝑥 2 )(1 − 𝑦 2 ) = 2𝑥𝑦


√(1 − 𝑥 2 )(1 − 𝑦 2 ) = 𝑥𝑦
(1 − 𝑥 2 )(1 − 𝑦 2 ) = 𝑥 2 𝑦 2
1 − 𝑥2 − 𝑦2 + 𝑥2𝑦2 = 𝑥2𝑦2
1 − 𝑥2 − 𝑦2 = 0
1 = 𝑥2 + 𝑦2

Example 5.15: Identities


We can also simplify trigonometric functions of inverse trigonometric functions using trigonometric identities.
A. sin(cos−1 𝑥)
B. cos(sin−1 𝑥)

Part A
Substitute 𝜃 = cos−1 𝑥 in the Pythagorean Identity sin2 𝜃 + cos2 𝜃 = 1:
sin2(cos−1 𝑥) + cos 2 (cos−1 𝑥) = 1
sin2 (cos−1 𝑥) + 𝑥 2 = 1
sin2 (cos−1 𝑥) = 1 − 𝑥 2
sin(cos−1 𝑥) = √1 − 𝑥 2
Part B
Let:
cos2 𝜃 = 1 − sin2 𝜃 = 1 − 𝑥 2 ⇒ cos 𝜃 = √1 − 𝑥 2

P a g e 175 | 213
Aziz Manva (azizmanva@gmail.com)

𝜃 = sin−1 𝑥
cos(sin−1 𝑥) = cos 𝜃 = √1 − 𝑥 2

Example 5.16: Addition Formula


4 5
cos (tan−1 − cos−1 )
3 13

Use the formula cos(𝛼 − 𝛽) = cos 𝛼 cos 𝛽 + sin 𝛼 sin 𝛽:


4 5 4 5
= cos (tan−1 ) cos (cos−1 ) + sin (tan−1 ) sin (cos−1 )
3 13 3 13

4 4 3 4
tan 𝜃 = ⇒ 𝜃 = tan−1 = cos−1 = sin−1
3 3 5 5
5 5 12
cos 𝜃 = ⇒ 𝜃 = cos −1 = sin−1
13 13 13

4 3 3
cos (tan−1 ) = cos (cos−1 ) =
3 5 5
−1
5 5
cos (cos )=
13 13
4 4 4
sin (tan−1 ) = sin (sin−1 ) =
3 5 5
5 12 12
sin (cos −1 ) = sin (sin−1 ) =
13 13 13

3 5 4 12 3 48 15 48 63
= × + × = + = + =
5 13 5 13 13 65 65 65 65

Example 5.17: Double Angle Formula


sin(2 sin−1 𝑥)

Apply the formula sin 2𝜃 = 2 sin 𝜃 cos 𝜃:


2 sin(sin−1 𝑥) cos(sin−1 𝑥)

5.18: Tan Addition Identity for Inverses


𝑥+𝑦
tan−1 𝑥 + tan−1 𝑦 = tan−1 ( ), 𝑥𝑦 < 1
1 − 𝑥𝑦
𝑥+𝑦
tan−1 𝑥 − tan−1 𝑦 = tan−1 ( ), 𝑥𝑦 > −1
1 + 𝑥𝑦

tan 𝛼+tan 𝛽
Take the tan inverse of both sides of the identity tan(𝛼 + 𝛽) = 1−tan 𝛼 tan 𝛽:
tan 𝛼 + tan 𝛽
𝛼 + 𝛽 = tan−1 ( )
1 − tan 𝛼 tan 𝛽
Let tan 𝛼 = 𝑥 ⇒ 𝛼 = tan−1 𝑥 , tan 𝛽 = 𝑦 ⇒ 𝛽 = tan−1 𝑦:
𝑥+𝑦
tan−1 𝑥 + tan−1 𝑦 = tan−1 ( )
1 − 𝑥𝑦
The proof of the second identity is similar, but it makes use of

P a g e 176 | 213
Aziz Manva (azizmanva@gmail.com)

tan 𝛼 − tan 𝛽
tan(𝛼 − 𝛽) =
1 + tan 𝛼 tan 𝛽

Example 5.19
4 2
A. The value of tan [cos−1 + tan−1 ] is: (JEE 1983)
5 3
4 2
B. The value of sin [cos−1 5 + tan−1 3] is: (CBSE Grade 12 2019)
1 1 1 1 𝜋
C. Prove that tan−1 5 + tan−1 7 + tan−1 3 + 𝑡𝑎𝑛−1 8 = 4
(CBSE Grade 12 2016)
1 1 1
D. If tan−1 (1+1∙2) + tan−1 (1+2∙3) + ⋯ + tan−1 (1+𝑛(𝑛+1) ) = tan−1 𝜃, then find the value of 𝜃. (CBSE Grade
12 2015)

Part D
𝑥+𝑦
Use the identity tan−1 𝑥 − tan−1 𝑦 = tan−1 (1+𝑥𝑦) to expand each term:
2−1
tan−1 ( ) = 𝐭𝐚𝐧−𝟏 𝟐 − tan−1 1
1+1∙2
3−2
tan−1 ( ) = 𝐭𝐚𝐧−𝟏 𝟑 − 𝐭𝐚𝐧−𝟏 𝟐
1+2∙3
.
.
.
(𝑛 + 1) − 𝑛
tan−1 ( ) = tan−1 (𝑛 + 1) − 𝐭𝐚𝐧−𝟏 𝒏
1 + 𝑛(𝑛 + 1)

Note that we have a telescoping series. Add the terms:


(𝑛 + 1) − 1 𝑛
tan−1 (𝑛 + 1) − tan−1 1 = tan−1 ( ) = tan−1 ( )
1 + 1(𝑛 + 1) 𝑛+2

E. Applications

5.20:
𝑏
𝑎 sin 𝜃 + 𝑏 cos 𝜃 = √𝑎2 + 𝑏 2 sin (𝜃 + tan−1 ( ))
𝑎

Let
𝑏 𝑏
𝐶 = tan−1 ( ) ⇒ tan 𝐶 =
𝑎 𝑎

Use the sum formula on the RHS:


𝑅𝐻𝑆 = √𝑎2 + 𝑏 2 [sin 𝜃 cos 𝐶 + cos 𝜃 sin 𝐶]

𝑎 𝑏
Substitute cos 𝐶 = , sin 𝐶 =
√𝑎 2 +𝑏2 √𝑎 2 +𝑏2
𝑎 𝑏
= √𝑎2 + 𝑏 2 [sin 𝜃 + cos 𝜃 ]
√𝑎2+ 𝑏2 √𝑎2 + 𝑏2
= 𝑎 sin 𝜃 + 𝑏 cos 𝜃 = 𝐿𝐻𝑆

P a g e 177 | 213
Aziz Manva (azizmanva@gmail.com)

Example 5.21
𝜋
sin 𝜃 + cos 𝜃 = √2 sin (𝜃 + )
4
A. Using the formula
B. Without the formula

Part A
Using 𝑎 = 𝑏 = 1
√𝑎2 + 𝑏 2 = √12 + 12 = √2
𝑏 1 𝜋
tan−1 ( ) = tan−1 ( ) =
𝑎 1 4

𝑏 𝜋
√𝑎2 + 𝑏 2 sin (𝜃 + tan−1 ( )) = √2 sin (𝜃 + )
𝑎 4

Part B
Factor √2 in the LHS:
1 1
𝐿𝐻𝑆 = √2 [(sin 𝜃) + cos 𝜃]
√2 √2
𝜋 𝜋 1
Substitute sin ( ) = cos ( ) = :
4 4 √2
𝜋 𝜋
= √2 [(sin 𝜃) cos ( ) + sin ( ) cos 𝜃]
4 4
Use the sum formula sin(𝛼 + 𝛽) = sin 𝛼 cos 𝛽 + sin 𝛽 cos 𝛼
𝜋
= √2 [sin (𝜃 + )] = 𝑅𝐻𝑆
4

Example 5.22
√12 sin 3𝛼 + 2 cos 3𝛼

Use a change of variable. Let 3𝛼 = 𝜃:


√12 sin 𝜃 + 2 cos 𝜃

Using 𝑎 = √12, 𝑏 = 2
2
√𝑎2 + 𝑏 2 = √(√12) + 22 = √12 + 4 = √16 = 4
2 2 1 𝜋
tan−1 ( ) = tan−1 ( ) = tan−1 ( )=
√12 2√3 √3 6

𝑏 𝜋 𝜋
√𝑎2 + 𝑏 2 sin (𝜃 + tan−1 ( )) = 4 sin (𝜃 + ) = 4 sin (3𝛼 + )
𝑎 6 6

5.23:
𝑎
𝑎 sin 𝜃 + 𝑏 cos 𝜃 = √𝑎2 + 𝑏 2 cos (𝜃 − tan−1 ( ))
𝑏

P a g e 178 | 213
Aziz Manva (azizmanva@gmail.com)

Let
𝑎 𝑎
𝐶 = tan−1 ( ) ⇒ tan 𝐶 =
𝑏 𝑏

Use the sum formula on the RHS:


𝑅𝐻𝑆 = √𝑎2 + 𝑏 2 [cos 𝜃 cos 𝐶 + sin 𝜃 sin 𝐶]

𝑏 𝑎
Substitute cos 𝐶 = , sin 𝐶 =
√𝑎 2 +𝑏2 √𝑎 2 +𝑏2
𝑏 𝑎
= √𝑎2 + 𝑏 2 [cos 𝜃 + sin 𝜃 ]
√𝑎2 + 𝑏 2 √𝑎2 + 𝑏 2

Simplify:
= 𝑎 sin 𝜃 + 𝑏 cos 𝜃 = 𝐿𝐻𝑆

Example 5.24
𝜋
sin 𝜃 + cos 𝜃 = √2 sin (𝜃 + )
4
𝑎
√𝑎2 + 𝑏 2 cos (𝜃 − tan−1 ( ))
𝑏
Substitute 𝑎 = 𝑏 = 1
𝑎 𝜋
= √12 + 12 cos (𝜃 − tan−1 ( )) = √2 cos (𝜃 − )
𝑏 4

Example 5.25
√12 sin 3𝛼 + 2 cos 3𝛼

Using 𝑎 = √12, 𝑏 = 2:
2
√𝑎2 + 𝑏 2 = √(√12) + 22 = √12 + 4 = √16 = 4
√12 2√3 𝜋
tan−1 ( ) = tan−1 ( ) = tan−1 (√3) =
2 2 3

𝑎 𝜋
√𝑎2 + 𝑏 2 cos (𝜃 − tan−1 ( )) = 4 cos (3𝛼 − )
𝑏 3
5.2 Equations
A. Basics

Example 5.26
The number of solutions of sin−1 𝑥 = 2 tan−1 𝑥 is equal to: (JEE Main, April 13, 2023-II)

Let
𝜃
𝜃 = 2 tan−1 𝑥 ⇒ = tan−1 𝑥
2

Take the 𝑡𝑎𝑛 of the both sides and draw a reference triangle:
𝜃 𝜃 𝑥 𝜃 1
tan = 𝑥 ⇒ sin = , cos =
2 2 √1 + 𝑥 2 2 √1 + 𝑥 2

P a g e 179 | 213
Aziz Manva (azizmanva@gmail.com)

𝜃 𝜃 𝑥 1 2𝑥
sin 𝜃 = 2 sin cos = 2 ∙ ∙ =
2 2 2
√1 + 𝑥 √1 + 𝑥 2 1 + 𝑥2
2𝑥
Substitute 𝜃 = sin−1 (1+𝑥2 ) in the given equation:
2𝑥
sin−1 𝑥 = sin−1 ( )
1 + 𝑥2

2𝑥
𝑥=
1 + 𝑥2
𝑥 + 𝑥 3 = 2𝑥
𝑥3 − 𝑥 = 0
𝑥(𝑥 2 − 1) = 0
𝑥 ∈ {−1,0,1}
3 𝑆𝑜𝑙𝑢𝑡𝑖𝑜𝑛𝑠

5.3 Domain and Range


A. Multiple Domains

5.27: Intersection of Multiple Domains


If there is more than one part to the function, each part must be checked separately.
The final domain is the 𝑖𝑛𝑡𝑒𝑟𝑠𝑒𝑐𝑡𝑖𝑜𝑛 of all the parts.

Example 5.28

P a g e 180 | 213
Aziz Manva (azizmanva@gmail.com)

The domain of the function below is: (JEE Main 2002, 2004)
sin−1 (𝑥 − 3)
𝑓(𝑥) =
√9 − 𝑥 2

Expression inside the square root must be positive. (Zero not allowed since it is in the denominator).
9 − 𝑥2 > 0
𝑥2 < 9
−3 < 𝑥 < 3

The input for sin−1 𝑥 must be between −1 and 1


−1 ≤ 𝑥 − 3 ≤ 1
2≤𝑥≤4

Take the intersection of the above two:


[2,3)

Example 5.29
The interval for which the function below is defined is: (JEE Main 2007, Adapted)
2 𝑥
𝑓(𝑥) = 4−𝑥 + cos−1 ( − 1) + log(cos 𝑥)
3
2
4−𝑥 has no restrictions.
cos−1 𝑥 must be between −1 and 1.
𝑥
−1 ≤ − 1 ≤ 1 ⇒ 0 ≤ 𝑥 ≤ 6
3

The input for the log function must be positive:


𝜋 𝜋 3𝜋 5𝜋
cos 𝑥 > 0 ⇒ 𝑥 ∈ 𝑄𝐼, 𝑄𝐼𝑉 ⇒ (− , ) ∪ ( , )
2 2 2 2

The final answer is:


𝜋 3𝜋
[0, ) ∪ ( , 6]
2 2

B. Compound Inequalities

5.30: Compound Inequality


The compound inequality 𝑎 < 𝑏 < 𝑐 can be split into the equivalent equalities
𝑎 < 𝑏, 𝑏<𝑐

Example 5.31
The domain of the function
−1
𝑥 2 − 3𝑥 + 2
𝑓(𝑥) = sin ( 2 ) (𝑱𝑬𝑬 𝑴𝒂𝒊𝒏, 𝑱𝒖𝒍𝒚 𝟐𝟗, 𝟐𝟎𝟐𝟐 − 𝑰𝑰)
𝑥 + 2𝑥 + 7

𝑥 2 − 3𝑥 + 2
−1 ≤ ≤1
𝑥 2 + 2𝑥 + 7

Split into two inequalities:

P a g e 181 | 213
Aziz Manva (azizmanva@gmail.com)

𝑥 2 − 3𝑥 + 2
≤1
𝑥 2 + 2𝑥 + 7
𝑥 2 − 3𝑥 + 2 ≤ 𝑥 2 + 2𝑥 + 7
−5𝑥 ≤ 5
𝑥 ≥ −1

𝑥 2 − 3𝑥 + 2
≥ −1
𝑥 2 + 2𝑥 + 7
𝑥 2 − 3𝑥 + 2 ≥ −𝑥 2 − 2𝑥 − 7
2𝑥 2 − 𝑥 + 9 ≥ 0
2 2
𝑏 − 4𝑎𝑐 = (−1) − 4(2)(9) < 0 ⇒ 𝐴𝑙𝑙 𝑟𝑒𝑎𝑙 𝑛𝑢𝑚𝑏𝑒𝑟𝑠 𝑎𝑟𝑒 𝑣𝑎𝑙𝑖𝑑

Hence, the final answer is the intersection of the above two:


𝑥 ∈ [−1, ∞)

Example 5.32
The domain of
𝑥 2 − 4𝑥 + 2
𝑓(𝑥) = cos −1 ( ) (𝑱𝑬𝑬 𝑴𝒂𝒊𝒏, 𝑱𝒖𝒍𝒚 𝟐𝟖, 𝟐𝟎𝟐𝟐 − 𝑰)
𝑥2 + 3

The condition for the domain of 𝑎𝑟𝑐𝑐𝑜𝑠 is


𝑥 2 − 4𝑥 + 2
−1 ≤ ≤1
𝑥2 + 3

Since 𝑥 2 + 3 > 0, multiply throughout by 𝑥 2 + 3:


−𝑥 2 − 3 ≤ 𝑥 2 − 4𝑥 + 2 ≤ 𝑥 2 + 3
−𝑥 2 − 5 ≤ 𝑥 2 − 4𝑥 ≤ 𝑥 2 + 1

Split the inequality. First part is:


𝑥 2 − 4𝑥 ≤ 𝑥 2 + 1
−4𝑥 ≤ 1
1
𝑥≥−
4

Second part is:


−𝑥 2 − 5 ≤ 𝑥 2 − 4𝑥
2𝑥 2 − 4𝑥 + 5 ≥ 0
𝐷𝑖𝑠𝑐𝑟𝑖𝑚𝑖𝑛𝑎𝑛𝑡 = (−4)2 − 4(2)(5) = 16 − 40 < 0
𝐴𝑙𝑙 𝑅𝑒𝑎𝑙 𝑁𝑢𝑚𝑏𝑒𝑟𝑠

The final answer is:


1
𝑥≥−
4
1
[− , ∞)
4

C. Multiple Domains with Compound Inequalities

Example 5.33

P a g e 182 | 213
Aziz Manva (azizmanva@gmail.com)

15: The domain of the function below is:


𝑥 2 − 5𝑥 + 6
cos −1 ( )
𝑥2 − 9
𝑓(𝑥) =
ln(𝑥 2 − 3𝑥 + 2 )

Example 5.34
If the domain of the function
10𝑥 + 6
𝑓(𝑥) = ln(4𝑥 2 + 11𝑥 + 6) + sin−1(4𝑥 + 3) + cos−1 ( )
3
Is (𝛼, 𝛽], then 36|𝛼 + 𝛽| = (JEE Main, April 15, 2023-I)

The input for a logarithm must be positive.


4𝑥 2 + 11𝑥 + 6 > 0
3
(4𝑥 + 3)(𝑥 + 2) = 0 ⇒ 𝑥 ∈ {−2, − }
4

3
(−∞, −2) ∪ (− , ∞)
4

The input for sin−1 𝑥 must be from −1 to 1:


−1 ≤ 4𝑥 + 3 ≤ 1
−4 ≤ 4𝑥 ≤ −2
1
−1 ≤ 𝑥 ≤ −
2
1
𝑥 ∈ [−1, − ]
2

The input for cos−1 𝑥 must be from −1 to 1:


10𝑥 + 6
∈ [−1,1]
3
10𝑥 + 6 ∈ [−3,3]
10𝑥 ∈ [−9, −3]
9 3
𝑥 ∈ [− , − ]
10 10

The intersection of the three individual domains is:

P a g e 183 | 213
Aziz Manva (azizmanva@gmail.com)

3 1
(− , − ]
4 2
3 1 5
36|𝛼 + 𝛽| = 36 |− − | = 36 ∙ = 45
4 2 4

Example 5.35
The domain of the function below is: (JEE Main Aug 31, 2021-II)
3𝑥 2 + 𝑥 − 1 𝑥−1
𝑓(𝑥) = sin−1 ( ) + cos −1
( )
(𝑥 − 1)2 𝑥+1

𝐜𝐨𝐬 −𝟏 𝒙 must be between −𝟏 and 𝟏


𝑥−1
Split the inequality −1 ≤ 𝑥+1 ≤ 1 into two parts.
First Part
𝑥−1 −2 2
≤1⇒ ≤0⇒ ≥0
𝑥+1 𝑥+1 𝑥+1
𝐷𝑒𝑔𝑟𝑒𝑒 = 0 − 1 → 𝑂𝑑𝑑 ⇒ −𝑣𝑒 𝑎𝑡 − ∞
𝐶𝑟𝑖𝑡𝑖𝑐𝑎𝑙 𝑃𝑜𝑖𝑛𝑡𝑠 ∈ {−1}

Since the expression is not defined at 𝑥 = −1, remove this from the solution set:
(−1, ∞)

Second Part
𝑥−1 2𝑥
≥ −1 ⇒ ≥0
𝑥+1 𝑥+1
𝐶𝑟𝑖𝑡𝑖𝑐𝑎𝑙 𝑃𝑜𝑖𝑛𝑡𝑠 ∈ {0, −1}
𝐷𝑒𝑔𝑟𝑒𝑒 = 1 − 1 = 0 ⇒ 𝐸𝑣𝑒𝑛 ⇒ +𝑣𝑒 𝑎𝑡 − ∞

Since the expression is not defined at 𝑥 = −1, remove this from the solution set:
(−∞, −1) ∪ [0, ∞)

Take the intersection of the solutions from the first and


second part:
[0, ∞)

𝐬𝐢𝐧−𝟏 𝒙 must be between −𝟏 and 𝟏


3𝑥 2 + 𝑥 − 1
−1 ≤ ≤1
(𝑥 − 1)2
𝑥 = 1 is a critical point since it makes the denominator zero.
Since (𝑥 − 1)2 > 0, multiply throughout by (𝑥 − 1)2 = (𝑥 2 − 2𝑥 + 1):
−(𝑥 2 − 2𝑥 + 1) ≤ 3𝑥 2 + 𝑥 − 1 ≤ (𝑥 2 − 2𝑥 + 1)

Split the inequality into two parts.


First Part
3𝑥 2 + 𝑥 − 1 ≤ (𝑥 2 − 2𝑥 + 1)
2𝑥 2 + 3𝑥 − 2 ≤ 0
(2𝑥 − 1)(𝑥 + 2) ≤ 0
1
𝐶𝑟𝑖𝑡𝑖𝑐𝑎𝑙 𝑝𝑜𝑖𝑛𝑡𝑠 ∈ −2
⏟ , , ⏟1
𝑂𝑑𝑑

2 𝐸𝑣𝑒𝑛
𝑀𝑢𝑙𝑡𝑖𝑝𝑙𝑖𝑐𝑖𝑡𝑦 𝑂𝑑𝑑 𝑀𝑢𝑙𝑡𝑖𝑝𝑙𝑖𝑐𝑖𝑡𝑦
𝑀𝑢𝑙𝑡𝑖𝑝𝑙𝑖𝑐𝑖𝑡𝑦

P a g e 184 | 213
Aziz Manva (azizmanva@gmail.com)

𝐷𝑒𝑔𝑟𝑒𝑒 = 2 − 2 = 0 ⇒ 𝐸𝑣𝑒𝑛
1
[−2, ]
2

Second Part
3𝑥 2 + 𝑥 − 1 ≥ −(𝑥 2 − 2𝑥 + 1)
𝑥(4𝑥 − 1) ≥ 0
1
𝐶𝑟𝑖𝑡𝑖𝑐𝑎𝑙 𝑝𝑜𝑖𝑛𝑡𝑠 ∈ ⏟
0 , , ⏟
1
𝑂𝑑𝑑

4 𝐸𝑣𝑒𝑛
𝑀𝑢𝑙𝑡𝑖𝑝𝑙𝑖𝑐𝑖𝑡𝑦 𝑂𝑑𝑑 𝑀𝑢𝑙𝑡𝑖𝑝𝑙𝑖𝑐𝑖𝑡𝑦
𝑀𝑢𝑙𝑡𝑖𝑝𝑙𝑖𝑐𝑖𝑡𝑦

𝐷𝑒𝑔𝑟𝑒𝑒 = 2 − 2 = 0 ⇒ 𝐸𝑣𝑒𝑛 ⇒ +𝑣𝑒 𝑎𝑡 − ∞


1
(−∞ ,0] ∪ [ , 1) ∪ (1, ∞)
4
Take the intersection of the first and second part:
1 1
[−2,0] ∪ [ , ]
4 2

Take the intersection of the above with the solution set


from the cos−1 𝑥 part [0, ∞)
1 1
{0} + [ , ]
4 2

D. Nested Functions

Example 5.36
The domain of the function:
1
2 sin−1 ( 2 )
cos−1 ( 4𝑥 − 1 ) (𝑱𝑬𝑬 𝑴𝒂𝒊𝒏, 𝑱𝒖𝒏𝒆 𝟐𝟗, 𝟐𝟎𝟐𝟐 − 𝑰)
𝜋

1
Expression in the fraction 4𝑥2 −1 cannot have a zero denominator:
1 1 1
4𝑥 2 − 1 = 0 ⇒ 𝑥 2 = ⇒ 𝑥 ∈ {− , }
4 2 2

Outside Function:
Check the domain of the outside function:
1
2 sin−1 ( 2 )
−1 ≤ 4𝑥 −1 ≤1
𝜋
𝜋
Multiply both sides by :
2
𝜋 1 𝜋
− ≤ sin−1 ( 2 )≤
2 4𝑥 − 1 2
Inside Function:
Take the 𝑠𝑖𝑛 of all parts. Note this is precisely the domain requirement of the inside function:

P a g e 185 | 213
Aziz Manva (azizmanva@gmail.com)

1
−1 ≤ ≤1
4𝑥 2 −1

Split the inequality.


1
Case I: Subtract 1 from both sides of 4𝑥2 −1 ≤ 1
1
−1≤0
4𝑥 2 −1
1 −4𝑥 2 + 1
+ ≤0
4𝑥 2 − 1 4𝑥 2 − 1
−4𝑥 2 + 2
≤0
4𝑥 2 − 1

Divide by 4 both sides:


1
−𝑥 2 + 2 √2 1 1 √2
≤ 0 ⇒ 𝑅𝑜𝑜𝑡𝑠 ∈ {− ,− , , }
1 1 2 2 2 2
(𝑥 + ) (𝑥 − )
2 2

Check with −2:


1 1
−𝑥 2 + 2 −(−2)2 + 2 −𝑣𝑒
= = = −𝑣𝑒
1 1 1 1 (−𝑣𝑒)(−𝑣𝑒)
(𝑥 + 2) (𝑥 − 2) (−2 + 2) (−2 − 2)

The solution set for the interval is:


√2 1 1 √2
𝐶𝑎𝑠𝑒 𝐼: (−∞, − ] ∪ [− , ] ∪ [ , ∞)
2 2 2 2

1
Case II Add 1 to both sides of 4𝑥2 −1 ≥ −1:
1 4𝑥 2 − 1
+ ≥0
4𝑥 2 − 1 4𝑥 2 − 1
4𝑥 2
≥0
4𝑥 2 − 1

Divide both sides by 4:


𝑥2
≥0
1 1
(𝑥 + 2) (𝑥 − 2)
1 1
𝑅𝑜𝑜𝑡𝑠 ∈ {− , 0, }
2 2

P a g e 186 | 213
Aziz Manva (azizmanva@gmail.com)

Check with −1:


𝑥2 (−1)2 +𝑣𝑒
= = = +𝑣𝑒
1 1 1 1 (−𝑣𝑒)(−𝑣𝑒)
(𝑥 + 2) (𝑥 − 2) (−1 + 2) (−1 − 2)

The solution set for the interval is


1 1
𝐶𝑎𝑠𝑒 𝐼𝐼: (−∞, − ] ∪ {0} ∪ [ , ∞)
2 2
√2 1 1 √2
𝐶𝑎𝑠𝑒 𝐼: (−∞, − ] ∪ [− , ] ∪ [ , ∞)
2 2 2 2
1 1
𝑍𝑒𝑟𝑜 𝐷𝑒𝑛𝑜𝑚𝑖𝑛𝑎𝑡𝑜𝑟 𝑛𝑜𝑡 𝑎𝑙𝑙𝑜𝑤𝑒𝑑: {− , }
2 2

Take the intersection of Case I and II:


√2 √2
(−∞, − ) ∪ {0} ∪ ( , ∞)
2 2

E. Greatest Integer Functions

Example 5.37
The domain of the function 𝑓(𝑥) = sin−1 [2𝑥 2 − 3] + log 2 (log 1 (𝑥 2 − 5𝑥 + 5)) where [𝑡] is the greatest integer
2
function is: (JEE Main, July 2022-II)

Domain of 𝐬𝐢𝐧−𝟏 𝒙
The input for sin−1 𝑥 must be between −1 and 1:
−1 ≤ [2𝑥 2 − 3] ≤ 1

Remove the greatest integer function:


−1 ≤ 2𝑥 2 − 3 < 2

P a g e 187 | 213
Aziz Manva (azizmanva@gmail.com)

2 ≤ 2𝑥 2 < 5
5
1 ≤ 𝑥2 <
2

5 5 5
𝑥2 < ⇒ −√ < 𝑥 < √
2 2 2

𝑥 2 ≥ 1 ⇒ 𝑥 ≥ 1 𝑂𝑅 𝑥 ≤ −1
√10 √10
(− , −1] ∪ [1, )
2 2
Domain of the log function
For the inner log function:
𝑥 2 − 5𝑥 + 5 > 0
5 ± √25 − 4(1)(5) 5 ± √5
𝑥= =
2(1) 2
5 − √5 5 + √5
(−∞, )∪( , ∞)
2 2
5 − √5 5 − 2.23 2.77
≈ = = 1.39
2 2 2
5 + √5 5 + 2.23 2.77
≈ = = 3.615
2 2 2

For the outer log function:


log 1(𝑥 2 − 5𝑥 + 5) > 0
2

Using log properties, take the reciprocal of the base:


− log 2 (𝑥 2 − 5𝑥 + 5) > 0
𝑥 2 − 5𝑥 + 4 < 0
(𝑥 − 4)(𝑥 − 1) < 0
𝑥 ∈ (1,4)

The final answer is:

P a g e 188 | 213
Aziz Manva (azizmanva@gmail.com)

5 − √5
𝑥 ∈ (1, )
2

F. 𝐬𝐞𝐜 −𝟏 𝒙

5.38: Domain of 𝐬𝐞𝐜 −𝟏 𝒙


The domain of sec −1 𝑥 is the range of sec 𝑥
𝐷sec−1 𝑥 = (−∞, −1] ∪ [1, ∞)

𝑅𝑎𝑛𝑔𝑒cos 𝑥 ∈ [−1,1]
𝑅𝑎𝑛𝑔𝑒sec 𝑥 ∈ (−∞, −1] ∪ [1, ∞)

The domain of an inverse function is the range of the original function:


𝐷sec−1 𝑥 = 𝑅sec 𝑥 = (−∞, −1] ∪ [1, ∞)

Example 5.39
2𝑥
Determine the domain of the function 𝑓(𝑥) = sec −1 (5𝑥+3) (JEE Main, April 10, 2023-II, Adapted)

2𝑥
ℎ𝑎𝑠 𝑑𝑒𝑔𝑟𝑒𝑒 = 1 − 1 = 0 ⇒ 𝐸𝑣𝑒𝑛
5𝑥 + 3

2𝑥 −3𝑥 − 3 𝑥+1
≥1⇒ ≥0⇒ ≤0
5𝑥 + 3 5𝑥 + 3 5𝑥 + 3

3
𝑅𝑜𝑜𝑡𝑠 ∈ {−1, − }
5
3
[−1, − )
5
3
where − 5 was excluded since it is a denominator root, and makes the expression undefined.

2𝑥 7𝑥 + 3
≤ −1 ⇒ ≤0
5𝑥 + 3 5𝑥 + 3

3 3
𝑅𝑜𝑜𝑡𝑠 ∈ {− , − },
5 7
3 3
(− , − ]
5 7
3
where − 5 was excluded since it is a denominator root, and makes the
expression undefined.

3 3 3
[−1, − ) ∪ (− , − ]
5 5 7

P a g e 189 | 213
Aziz Manva (azizmanva@gmail.com)

6. FURTHER TOPICS
6.1 Parametric Equations
A. Basics

6.1: Function
𝑦 = 𝑓(𝑥) is a function if there is rule such that for every valid 𝑥 in the domain, there is an output 𝑦.

Some curves (such as circles) are not functions.


Some curves are difficult to represent using functions.

6.2: Parametric Representation


In a parametric representation, we describe a graph in the (𝑥, 𝑦) coordinate using a third variable (usually
called 𝑡). 𝑥 and 𝑦 are functions of t. That is
𝑦 = 𝑓(𝑡), 𝑥 = 𝑓(𝑡)

B. Circles

6.3: Equation of a Circle


The cartesian equation of a circle center at origin and radius 𝑟 is:
𝑥2 + 𝑦2 = 𝑟2

Draw a circle with center at the origin, and radius 𝑟.


Let 𝑃 be a point on the circle with coordinates
(𝑥, 𝑦)

Using the distance formula, the distance between 𝑃 and the origin is:
√(𝑥 − 0)2 + (𝑦 − 0)2 = 𝑟

Simplify:
√𝑥 2 + 𝑦 2 = 𝑟

And finally, the equation of the circle is:


𝑥2 + 𝑦2 = 𝑟2

6.4: Pythagorean Identity


sin2 𝜃 + cos 2 𝜃 = 1

The Pythagorean identity is useful in parameterization of circles and ellipses.

Example 6.5
Eliminate the parameter 𝑡 from 𝑥 = cos 𝑡 , 𝑦 = sin 𝑡 and determine the shape that it represents.

𝑥 2 + 𝑦 2 = cos 2 𝑡 + sin2 𝑡 = 1
𝑥2 + 𝑦2 = 1

This is the equation of the unit circle centered at origin.

P a g e 190 | 213
Aziz Manva (azizmanva@gmail.com)

Example 6.6
Eliminate the parameter 𝑡 from the system of equations below and show that it represents a circle.
A. 𝑥 = 𝑟 cos 𝑡 , 𝑦 = 𝑟 sin 𝑡

𝑥 2 + 𝑦 2 = 𝑟 2 cos 2 𝑡 + 𝑟 2 sin2 𝑡 = 𝑟 2
𝑥2 + 𝑦2 = 𝑟2

This is the equation of a circle of radius 𝑟 centred at origin.

6.7: Equation of a Circle


The equation of a circle with center (ℎ, 𝑘) and radius 𝑟 is:
(𝑥 − ℎ)2 + (𝑦 − 𝑘)2 = 𝑟 2

Example 6.8
Eliminate the parameter 𝑡 from the system of equations below and show that it represents a circle.
A. 𝑥 = 2 + 3 cos 𝑡 , 𝑦 = 3 sin 𝑡 − 1

cos2 𝑡 + sin2 𝑡 = 1
𝑥−2 𝑦+1
Substitute cos 𝑡 = , sin 𝑡 = :
3 3
(𝑥 − 2)2 (𝑦 + 1)2
+ =1
32 32
(𝑥 − 2)2 + (𝑦 + 1)2 = 32

Circle of radius 3 centred at (2, −1).

Example 6.9
Eliminate the parameter 𝑡 from the system of equations below and show that it represents a circle.
A. 𝑥 = ℎ + 𝑟 cos 𝑡 , 𝑦 = 𝑘 + 𝑟 sin 𝑡

(𝑥 − ℎ)2 + (𝑦 − 𝑘)2 = 𝑟 2

𝑥 − ℎ = 𝑟 cos 𝑡
𝑦 − 𝑘 = 𝑟 sin 𝑡

𝐿𝐻𝑆 = (𝑥 − ℎ)2 + (𝑦 − 𝑘)2 = (𝑟 cos 𝑡)2 + (𝑟 sin 𝑡)2 = 𝑟 2 = 𝑅𝐻𝑆

𝑥−ℎ 𝑦−𝑘
Substitute cos 𝑡 = 𝑟
, sin 𝑡 = 𝑟
in the identity cos2 𝑡 + sin2 𝑡 = 1:
(𝑥 − ℎ)2 (𝑦 − 𝑘)2
+ = 1 ⇒ (𝑥 − ℎ)2 + (𝑦 − 𝑘)2 = 𝑟 2
𝑟2 𝑟2
Circle of radius 𝑟 centred at (ℎ, 𝑘).

P a g e 191 | 213
Aziz Manva (azizmanva@gmail.com)

Example 6.10
Ferris Wheel

C. Ellipses

6.11: Equation of an Ellipse


The cartesian equation of an ellipse centered at the origin is:
𝑥2 𝑦2
+ =1
𝑎2 𝑏 2

A circle is a special case of an ellipse with 𝑎 = 𝑏.

Example 6.12
Eliminate the parameter 𝑡 from the system of equations below and identify the nature of the function it
represents.
A. 𝑥 = √5 cos 𝑡 , 𝑦 = 3 sin 𝑡

𝑎2 cos2 𝑡 𝑏 2 sin2 𝑡
cos2 𝑡 + sin2 𝑡 = 1 ⇒ + =1
⏟ 𝑎2 𝑏2
𝐸𝑞𝑢𝑎𝑡𝑖𝑜𝑛 𝐼

𝑥2
𝑥 = √5 cos 𝑡 ⇒ 𝑥 2 = 5 cos 2 𝑡 ⇒ cos2 𝑡 =
5
2
𝑦
𝑦 = 3 sin 𝑡 ⇒ 𝑦 2 = 9 sin2 𝑡 ⇒ sin2 𝑡 =
9

𝑥2 𝑦2
Substitute cos2 𝑡 = , sin2 𝑡 = in cos2 𝑡 + sin2 𝑡 = 1:
5 9
𝑥2 𝑦2
+ = 1 ⇒ 9𝑥 2 + 5𝑦 2 = 45
5 9

6.13: Equation of an Ellipse


𝑥2 𝑦2
𝑥 = 𝑎 cos 𝑡 , 𝑦 = 𝑏 sin 𝑡 ⇔ + =1
𝑎2 𝑏 2

cos2 𝑡 + sin2 𝑡 = 1
𝑎2 cos2 𝑡 𝑏 2 sin2 𝑡
+ =1
𝑎2 𝑏2

Substitute 𝑥 = 𝑎 cos 𝑡 , 𝑦 = 𝑏 sin 𝑡 in Equation I to get an ellipse centered at the origin:


𝑥2 𝑦2
+ =1
𝑎2 𝑏 2

6.14: Equation of an Ellipse


The equation of an ellipse centered at (ℎ, 𝑘) with axes 𝑎 and 𝑏 is
(𝑥 − ℎ)2 (𝑦 − 𝑘)2
+ =1
𝑎2 𝑏2

Example 6.15
P a g e 192 | 213
Aziz Manva (azizmanva@gmail.com)

Eliminate the parameter 𝑡 from the system of equations below and identify the nature of the function it
represents.
1. 𝑥 = ℎ + 𝑎 cos 𝑡 , 𝑦 = 𝑘 + 𝑏 sin 𝑡

Substitute:
𝑥 − ℎ = 𝑎 cos 𝑡 ⇒ (𝑥 − ℎ)2 = 𝑎2 cos 2 𝑡
𝑦 − 𝑘 = 𝑏 sin 𝑡 ⇒ (𝑦 − 𝑘)2 = 𝑏 2 sin2 𝑡
In Equation I to get an ellipse centered at (ℎ, 𝑘)
(𝑥 − ℎ)2 (𝑦 − 𝑘)2
+ =1
𝑎2 𝑏2
D. Parabola

6.16: Parabola
𝑦 = 𝑎𝑥 2 + 𝑏𝑥 + 𝑐

𝑦 = 𝑥2
In a parabola, 𝑜𝑛𝑒
⏟ 𝑞𝑢𝑎𝑛𝑡𝑖𝑡𝑦 is related to the square of 𝑎𝑛𝑜𝑡ℎ𝑒𝑟
⏟ 𝑞𝑢𝑎𝑛𝑡𝑖𝑡𝑦.
𝒚 𝒙
The parameter in one variable is the square of what it is in the other variable.

6.17: Algebraic Parametrization

6.18: Restrictions

Example 6.19
Eliminate the parameter 𝑡 from the system of equations below and show it represents a parabola, or part of a
parabola.
𝑥 = √𝑡, 𝑦=𝑡

𝑥 = √𝑡 ⇒ 𝑥 2 = 𝑡
Substitute:
𝑦 = 𝑡 ⇒ 𝑦 = 𝑥2

This is a parabola that only holds for


1. 𝑥 > 0 since the square root function has nonnegative output:
𝑅𝑎𝑛𝑔𝑒 𝑜𝑓 √𝑡 = [0, ∞)
2. 𝑡 > 0 since the square root function has nonnegative domain:
𝐷𝑜𝑚𝑎𝑖𝑛 𝑜𝑓 √𝑡 = [0, ∞)

6.20: Parabola
𝑥 = 𝑎𝑦 2 + 𝑏𝑦 + 𝑐

Example 6.21
Eliminate the parameter 𝑡 from the system of equations below and show it represents a parabola, or part of a

P a g e 193 | 213
Aziz Manva (azizmanva@gmail.com)

parabola.
𝑥 = 𝑡2, 𝑦 =𝑡+2

𝑦 =𝑡+2⇒𝑡 =𝑦−2

𝑥 = 𝑡 2 = (𝑦 − 2)2 = 𝑦 2 − 4𝑦 + 4
Parabola with no restrictions

6.22: Exponential Parametrization

Example 6.23
Eliminate the parameter 𝑡 from the system of equations below and show it represents a parabola, or part of a
parabola.
𝑥 = 𝑒 𝑡 − 1, 𝑦 = 𝑒 2𝑡

Method I
𝑥 = 𝑒𝑡 − 1
𝑒𝑡 = 𝑥 + 1
𝑒 2𝑡 = (𝑥 + 1)2
𝑦 = (𝑥 + 1)2
To establish the interval for 𝑥, note that:
𝑥 = 𝑓(𝑡) = 𝑒 𝑡 − 1
𝑒 𝑡 > 0 ⇒ 𝑒 𝑡 − 1 > −1 ⇒ 𝑥 > −1
𝑅𝑎𝑛𝑔𝑒 𝑖𝑠 (−1, ∞)

Method II
𝑒𝑡 = 𝑥 + 1
𝑡 = ln(𝑥 + 1)
2
𝑦 = 𝑒 2𝑡 =𝑒 2𝑙𝑛(𝑥+1)
= 𝑒 𝑙𝑛(𝑥+1) = (𝑥 + 1)2

That is the right side of the parabola, not including its vertex.

6.24: Trigonometric Parametrization

Example 6.25
Eliminate the parameter 𝑡 from the system of equations below and show it represents a parabola, or part of a
parabola.
𝜋 𝜋
A. 𝑥 = sin 𝑡 , 𝑦 = cos 2𝑡 , − 2 ≤ 𝑡 ≤ 2

𝑦 = cos 2𝑡 = 1 − 2 sin2 𝑡 = 1 − 2𝑥 2

𝜋 𝜋
Over the domain − 2 ≤ 𝑡 ≤ 2 , 𝑡 has the range [−1,1], starting at −1 and going to 1.
Hence, the parabola begins at (−1, −1) and ends at (1, −1)

Example 6.26

P a g e 194 | 213
Aziz Manva (azizmanva@gmail.com)

Eliminate the parameter 𝑡 from the system of equations below and identify the nature of the function it
represents.

𝑦 2 = 4𝑎𝑥
𝑦 = 2𝑎𝑡 ⇒ 𝑦 = 4𝑎2 𝑡 2 = (4𝑎)(𝑎𝑡 2 )
2

𝑥 = 𝑎𝑡 2
E. Hyperbola

6.27: Equation of a Rectangular Hyperbola


The equation of a rectangular hyperbola is:
𝑥𝑦 = 𝑘
Where
𝑘 𝑖𝑠 𝑐𝑜𝑛𝑠𝑡𝑎𝑛𝑡 𝑜𝑓 𝑝𝑟𝑜𝑝𝑜𝑟𝑡𝑖𝑜𝑛𝑎𝑙𝑖𝑡𝑦

𝑃𝑉 = 𝐶𝑜𝑛𝑠𝑡𝑎𝑛𝑡
𝑁𝑜. 𝑜𝑓 𝐴𝑝𝑝𝑙𝑒𝑠 × 𝑃𝑟𝑖𝑐𝑒 𝑝𝑒𝑟 𝐴𝑝𝑝𝑙𝑒 = 𝑇𝑜𝑡𝑎𝑙 𝑀𝑜𝑛𝑒𝑦

Example 6.28
Eliminate the parameter 𝑡 from the system of equations below and identify the nature of the function it
represents.
1
A. 𝑥 = sin 𝑡 , 𝑦 = csc 𝑡 = sin 𝑡
B. 𝑥 = √𝑘 sin 𝑡 , 𝑦 = √𝑘 csc 𝑡
C. 𝑥 = 𝑘 sin 𝑡 , 𝑦 = csc 𝑡
D. 𝑥 = sin 𝑡 , 𝑦 = 𝑘 csc 𝑡

Part A
1
𝑥𝑦 = (sin 𝑡) ( )=1
sin 𝑡
Rectangular Hyperbola with constant of proportionality 1

Parts B, C and D
𝑥𝑦 = 𝑘
Rectangular Hyperbola with constant of proportionality k

6.29: Equation of a Hyperbola


The equation of a hyperbola
𝑥2 𝑦2
− = 𝑘, 𝑇𝑟𝑎𝑛𝑠𝑣𝑒𝑟𝑠𝑒 𝐴𝑥𝑖𝑠 𝐻𝑜𝑟𝑖𝑧𝑜𝑛𝑡𝑎𝑙
𝑎2 𝑏 2
2 2
𝑦 𝑥
− = 𝑘, 𝑇𝑟𝑎𝑛𝑠𝑣𝑒𝑟𝑠𝑒 𝐴𝑥𝑖𝑠 𝐻𝑜𝑟𝑖𝑧𝑜𝑛𝑡𝑎𝑙
𝑎2 𝑏 2

Compare with the equation of an ellipse:


𝑥2 𝑦2
+ =1
𝑎2 𝑏 2

Example 6.30
Eliminate the parameter 𝑡 from the system of equations below and identify the nature of the function it
represents.

P a g e 195 | 213
Aziz Manva (azizmanva@gmail.com)

1 1
A. 𝑥 = 𝑡 + , 𝑦 = 𝑡 −
𝑡 𝑡
B. 𝑥 = √3 + 𝑡 2 , 𝑦 = 𝑡

Part A
From the given equations:
2
𝑥
⏟+ 𝑦 = 2𝑡 , ⏟𝑥 − 𝑦 =
𝑡
𝑬𝒒𝒖𝒂𝒕𝒊𝒐𝒏 𝑰
𝑬𝒒𝒖𝒂𝒕𝒊𝒐𝒏 𝑰𝑰

Multiply Equation I and II:


2
(𝑥 + 𝑦)(𝑥 − 𝑦) = (2𝑡) ( )
𝑡
𝑥2 − 𝑦2 = 4

Example 6.31
𝑇𝑟𝑎𝑛𝑠𝑣𝑒𝑟𝑠𝑒 𝐴𝑥𝑖𝑠 𝐻𝑜𝑟𝑖𝑧𝑜𝑛𝑡𝑎𝑙: 𝑥 = 𝑎 sec 𝑡 , 𝑦 = 𝑏 tan 𝑡
𝑇𝑟𝑎𝑛𝑠𝑣𝑒𝑟𝑠𝑒 𝐴𝑥𝑖𝑠 𝑉𝑒𝑟𝑡𝑖𝑐𝑎𝑙: 𝑥 = 𝑏 tan 𝑡 , 𝑦 = 𝑎 sec 𝑡

𝑥 2 𝑦 2 (𝑎 sec 𝑡)2 (𝑏 tan 𝑡)2 𝑎2 sec 2 𝑡 𝑏 2 tan2 𝑡


− = − = − = sec 2 𝑡 − tan2 𝑡 = 1
𝑎2 𝑏 2 𝑎2 𝑏2 𝑎2 𝑏2

𝑦 2 𝑥 2 (𝑎 sec 𝑡)2 (𝑏 tan 𝑡)2 𝑎2 sec 2 𝑡 𝑏 2 tan2 𝑡


− = − = − = sec 2 𝑡 − tan2 𝑡 = 1
𝑎2 𝑏 2 𝑎2 𝑏2 𝑎2 𝑏2
F. Lines

6.32: Line

Example 6.33
Eliminate the parameter 𝑡 from each system of equations below and identify the nature of the function it
represents.
A. 𝑥 = 3 + 𝑡, 𝑦 = 2 + 5𝑡
B. 𝑥 = 𝑎 + 𝑡, 𝑦 = 𝑏 + 𝑚𝑡

Part A
Substitute 𝑥 = 3 + 𝑡 ⇒ 𝑡 = 𝑥 − 3 in the second equation:
𝑦 = 2 + 5𝑡 = 2 + 5(𝑥 − 3) = 5𝑥 − 13
This is a line with slope 5, and 𝑦-intercept -13.

Part A
Substitute 𝑥 = 𝑎 + 𝑡 ⇒ 𝑡 = 𝑥 − 𝑎 in the second equation:
𝑦 = 𝑏 + 𝑚𝑡 = 𝑏 + 𝑚(𝑥 − 𝑎) = 𝑚𝑥 + 𝑏 − 𝑚𝑎
This is a line with slope 𝑚, and 𝑦-intercept 𝑏 − 𝑎.

Example 6.34
Convert the equation 𝑦 = 4𝑥 + 7 into parametric form.

𝑡 = 𝑥 ⇒ 𝑦 = 4𝑡 + 7
𝑡 = 𝑥 + 2 ⇒ 𝑥 = 𝑡 − 2 ⇒ 𝑦 = 4𝑡 + 7

P a g e 196 | 213
Aziz Manva (azizmanva@gmail.com)

G. Further Questions

Example 6.35
Show that 𝑥 2 𝑦 + 𝑎2 𝑦 = 𝑎𝑏𝑥, 𝑎𝑏 > 0 (Newton’s Serpentine) can be represented parametrically as:
𝑥 = 𝑎 cot 𝑡 , 𝑦 = 𝑏 sin 𝑡 cos 𝑡

𝑥 2 𝑦 + 𝑎2 𝑦 − 𝑎𝑏𝑥 = 0

Substitute 𝑥 = 𝑎 cot 𝑡 , 𝑦 = 𝑏 sin 𝑡 cos 𝑡 in the LHS:


(𝑎2 cot 2 𝑡)(𝑏 sin 𝑡 cos 𝑡) + 𝑎2 (𝑏 sin 𝑡 cos 𝑡) − 𝑎𝑏(𝑎 cot 𝑡)

Factor out 𝑎2 𝑏 cot 𝑡:


(𝑎2 𝑏 cot 𝑡)[(cot 𝑡)(sin 𝑡 cos 𝑡) + (sin 𝑡 cos 𝑡 tan 𝑡) − 1]

Simplify:
(𝑎2 𝑏 cot 𝑡)[cos 2 𝑡 + sin2 𝑡 − 1]
(𝑎2 𝑏 cot 𝑡)[1 − 1] = (𝑎2 𝑏 cot 𝑡)[0] = 0 = 𝑅𝐻𝑆

Example 6.36
Eliminate the parameter 𝜃 from the system of equations below and solve for 𝑦:
𝑥 = 2 + cos 𝜃 𝑓𝑜𝑟 1 < 𝑥 < 3, 𝑦 = 2 csc 𝜃 𝑓𝑜𝑟 𝑦 > 2

2 2 4
= 𝑦 ⇒ sin 𝜃 = ⇒ sin2 𝜃 = 2
sin 𝜃 𝑦 ⏟ 𝑦
𝑬𝒒𝒖𝒂𝒕𝒊𝒐𝒏 𝑰
cos 2 𝜃 = 𝑥 2 − 4𝑥 + 4
cos 𝜃 = 𝑥 − 2 ⇒ ⏟
𝑬𝒒𝒖𝒂𝒕𝒊𝒐𝒏 𝑰𝑰

Add Equations I and II and substitute sin2 𝜃 + cos 2 𝜃 = 1:


4
1 = 2 + 𝑥 2 − 4𝑥 + 4
𝑦
4
= −(𝑥 2 − 4𝑥 + 3)
𝑦2

Take the reciprocal of both sides. Then, multiply both sides by 4, take the square root both sides, and reject the
negative square root:
𝑦2 1 2
=− 2 ⇒𝑦=
4 𝑥 − 4𝑥 + 3 √−(𝑥 2 − 4𝑥 + 3)

Check the domain:


−(𝑥 2 − 4𝑥 + 3) > 0 ⇒ 𝑥 2 − 4𝑥 + 3 < 0 ⇒ (𝑥 − 3)(𝑥 − 1) < 0 ⇒ 𝑥 ∈ (1,3)

Check the range:


For 𝑥 ∈ (1,3), −(𝑥 2 − 4𝑥 + 3) is a downward parabola with vertex (2,2). It takes values in the range (0,2) and
2
hence 𝑦 = takes values > 2.
√−(𝑥 2 −4𝑥+3)

P a g e 197 | 213
Aziz Manva (azizmanva@gmail.com)

6.2 Polar Coordinates; Circles


A. Directed Distance
The rectangular coordinate system uses a grid layout to identify points. An alternate system specifies the
location of a point as a combination of:
➢ Distance from the origin
➢ Angle in which the distance is to be travelled

Polar coordinates get used extensively in:


➢ Vectors
➢ Complex Numbers
➢ Calculus

6.37: Cartesian Coordinates


➢ The Cartesian coordinate plane consists of an 𝑥 axis and a
𝑦 axis perpendicular to the 𝑥 axis.
➢ The axes divide the plane into quadrants.
➢ Any point on the plane is uniquely identified by its
coordinates
➢ Any coordinates uniquely identify a point on the plane
➢ Cartesian coordinates are also called rectangular
coordinates

6.38: Distance on the Real Number Line


Points a distance 𝑑 on the real number line from the origin
are given by:
|𝑥| = 𝑑

|𝑥| = 3 ⇒ 𝑥 = ±3

Example 6.39
Identify the point on the real number line:
A. 3 units to the right of zero
B. 3 units to the left of zero
C. −3 units to the right of zero
D. −3 units to the left of zero

3 𝑢𝑛𝑖𝑡𝑠 𝑡𝑜 𝑡ℎ𝑒 𝑟𝑖𝑔ℎ𝑡 𝑜𝑓 𝑧𝑒𝑟𝑜 = 3


3 𝑢𝑛𝑖𝑡𝑠 𝑡𝑜 𝑡ℎ𝑒 𝑙𝑒𝑓𝑡 𝑜𝑓 𝑧𝑒𝑟𝑜 = −3
−3 𝑢𝑛𝑖𝑡𝑠 𝑡𝑜 𝑡ℎ𝑒 𝑟𝑖𝑔ℎ𝑡 𝑜𝑓 𝑧𝑒𝑟𝑜 = −3
−3 𝑢𝑛𝑖𝑡𝑠 𝑡𝑜 𝑡ℎ𝑒 𝑙𝑒𝑓𝑡 𝑜𝑓 𝑧𝑒𝑟𝑜 = 3

6.40: 𝑫𝒊𝒓𝒆𝒄𝒕𝒆𝒅 𝑫𝒊𝒔𝒕𝒂𝒏𝒄𝒆 = 𝒓


The directed distance 𝑟 is defined as the distance travelled from the origin in a straight line in the direction
indicated.
➢ 𝑟 > 0 means you travel along the direction indicated
➢ 𝑟 < 0 means you travel |𝑟| unit diametrically opposite (180° 𝑡𝑢𝑟𝑛) along the direction indicated

P a g e 198 | 213
Aziz Manva (azizmanva@gmail.com)

Example 6.41
Identify the point that lies on the line 𝑦 = 𝑥:
A. That is reached by travelling 3 units at an angle of 45° with the positive 𝑥 axis.
B. That is reached by travelling 3 units at an angle of 225° with the positive 𝑥 axis.
C. That is reached by travelling −3 units at an angle of 225° with the positive 𝑥 axis.

3 33 3
𝐴: ( ,
) = ( √2, √2)
√2 √2 2 2
3 3 3 3
𝐵: (− , − ) = (− √2, − √2)
√2 √2 2 2
3 3 3 3
𝐶: ( , ) = ( √2, √2)
√2 √2 2 2
B. Polar Coordinates

6.42: Polar Coordinates


Polar coordinates define a point in the form:
(𝑟, 𝜃)
Where
𝑟 = 𝑑𝑖𝑟𝑒𝑐𝑡𝑒𝑑 𝑑𝑖𝑠𝑡𝑎𝑛𝑐𝑒
𝜃 = 𝑎𝑛𝑔𝑙𝑒 𝑚𝑎𝑑𝑒 𝑏𝑦 𝑡ℎ𝑒 𝑝𝑜𝑖𝑛𝑡 𝑤𝑖𝑡ℎ 𝑝𝑜𝑙𝑎𝑟 𝑎𝑥𝑖𝑠
𝑃𝑜𝑙𝑎𝑟 𝑎𝑥𝑖𝑠 = 𝑝𝑜𝑠𝑖𝑡𝑖𝑣𝑒 𝑑𝑖𝑟𝑒𝑐𝑡𝑖𝑜𝑛 𝑜𝑓 𝑥 𝑎𝑥𝑖𝑠

Example 6.43
Convert from polar coordinates to Cartesian Coordinates:
𝜋
A. (3, 3 )
𝜋
B. (2, 4 )

𝜋 3 3
(3, ) = ( , √3)
⏟ 3 ⏟2 2
𝑷𝒐𝒍𝒂𝒓 𝑭𝒐𝒓𝒎 𝑹𝒆𝒄𝒕𝒂𝒏𝒈𝒖𝒍𝒂𝒓
𝑭𝒐𝒓𝒎

𝜋
(2, ) = ⏟
(√2, √2)
⏟ 4
𝑹𝒆𝒄𝒕𝒂𝒏𝒈𝒖𝒍𝒂𝒓
𝑷𝒐𝒍𝒂𝒓 𝑭𝒐𝒓𝒎
𝑭𝒐𝒓𝒎

6.44: Different Values of 𝜽


(𝑟, 𝜃) = (𝑟, 𝜃 + 𝑛360°), 𝑛 ∈ ℤ

6.45: Converting from positive to negative 𝒓


(𝑟, 𝜃) = (−𝑟, 𝜃 + 𝜋) = (−𝑟, 𝜃 − 𝜋)

Example 6.46
𝜋
A. (−3, 4 )
𝜋
B. (2, − 4 )
𝜋
C. (−2, − 4 )

P a g e 199 | 213
Aziz Manva (azizmanva@gmail.com)

Part A
𝜋
(−3, ) = (−3,45°)
4

Use the property (𝑟, 𝜃) = (−𝑟, 𝜃 + 𝜋):


3 3
(3,45° + 180°) = (3,225°) = (− √2, − √2)
2 2
Part B
𝜋
(2, − ) = (2, −45°) = (−√2,
⏟ −√2)
4
𝑹𝒆𝒄𝒕𝒂𝒏𝒈𝒖𝒍𝒂𝒓
𝑭𝒐𝒓𝒎
Part C
𝜋
(−2, − ) = (2, −45° + 180°) = (2,135°) = ⏟
(−√2, √2)
4
𝑹𝒆𝒄𝒕𝒂𝒏𝒈𝒖𝒍𝒂𝒓
𝑭𝒐𝒓𝒎

Example 6.47
Given that |𝜃| < 2𝜋, Write (1, √3) in polar form (𝑟, 𝜃) with:
A. 𝑟 > 0, 𝜃 > 0
B. 𝑟 < 0, 𝜃 > 0
C. 𝑟 > 0, 𝜃 < 0
D. 𝑟 < 0, 𝜃 < 0

Combine the answers from all parts and compare.

The absolute can be removed to get the equivalent restriction for the range of 𝜃:
−2𝜋 < 𝜃 < 2𝜋

The distance from the origin


2
= 𝑟 = √12 + (√3) = √4 = 2
Part A
Since we get a 30 − 60 − 90 triangle the angle is
𝜃 = 60°
𝜋
(𝑟, 𝜃) = (2,60°) = (2, )
3
Part B
Using the property (𝑟, 𝜃) = (−𝑟, 𝜃 + 𝜋):
𝜋 𝜋 4𝜋
(2, ) = (−2, + 𝜋) = (−2, )
3 3 3
Part C
𝜋
We replace 𝜃 = 3 with an equivalent negative angle
𝜋 𝜋 5𝜋
(2, ) = (2, − 2𝜋) = (2, − )
3 3 3
Part D
Modify the answer from Part A:
𝜋 𝜋 2𝜋
(2, ) = (−2, − 𝜋) = (−2, − )
3 3 3

P a g e 200 | 213
Aziz Manva (azizmanva@gmail.com)

Or we can modify the answer from Part B:


4𝜋 4𝜋 2𝜋
(−2, ) = (−2, − 2𝜋) = (−2, − )
3 3 3

Writing all four answers together:


𝜋 4𝜋 5𝜋 2𝜋
(1, √3)
⏟ = (2, ) = (−2, ) = (2, − ) = (−2, − )
3 3 3 3
𝑹𝒆𝒄𝒕𝒂𝒏𝒈𝒖𝒍𝒂𝒓
𝑭𝒐𝒓𝒎

C. Equation of a Circle
In rectangular form, you would have encountered the equation of a line in multiple forms. They represent the
same shape, but the form of the equation is different.
𝑦 = 𝑚𝑥 + 𝑐 ⇔ ⏟
⏟ 𝑦 − 𝑦1 = 𝑚(𝑥 − 𝑥1 )
𝑺𝒍𝒐𝒑𝒆 𝑰𝒏𝒕𝒆𝒓𝒄𝒆𝒑𝒕 𝑷𝒐𝒊𝒏𝒕 𝑺𝒍𝒐𝒑𝒆 𝑭𝒐𝒓𝒎
𝑭𝒐𝒓𝒎

𝑦 − 𝑦1 = 𝑚𝑥 − 𝑚𝑥1
𝑦 = 𝑚𝑥 − 𝑚𝑥1 + 𝑦1
Substitute 𝑐 = −𝑚𝑥1 + 𝑦1 :
𝑦 = 𝑚𝑥 + 𝑐

Similarly, equations in polar coordinates can have more than one form.

6.48: Circle at Origin


A circle with center at the origin and radius 𝑅 is the set of points that are exactly
𝑅 units aways from the origin.

6.49: Circle
The equation of a circle in polar coordinates is given by:
𝑟=𝑅
Where
𝑟 𝑖𝑠 𝑎 𝑣𝑎𝑟𝑖𝑎𝑏𝑙𝑒 (𝑑𝑖𝑟𝑒𝑐𝑡𝑒𝑑 𝑑𝑖𝑠𝑡𝑎𝑛𝑐𝑒)
𝑅 𝑖𝑠 𝑡ℎ𝑒 𝑟𝑎𝑑𝑖𝑢𝑠 𝑜𝑓 𝑡ℎ𝑒 𝑐𝑖𝑟𝑐𝑙𝑒

Example 6.50
A. Determine the shape formed by 𝑟 = 3.
𝜋 3𝜋
B. Graph this shape. Make sure to identify the points where 𝜃 ∈ {0, 2 , 𝜋, 2
}

Part A
Circle with radius 3 and center at the origin

Part B

6.51: Circle
The equation of a circle in polar coordinates is given by:
𝑟 = −𝑅
Where
𝑅 𝑖𝑠 𝑡ℎ𝑒 𝑟𝑎𝑑𝑖𝑢𝑠 𝑜𝑓 𝑡ℎ𝑒 𝑐𝑖𝑟𝑐𝑙𝑒

P a g e 201 | 213
Aziz Manva (azizmanva@gmail.com)

(𝑟, 𝜃) = (−𝑟, 𝜃 + 180°)


Since there is no restriction on the angle, even if we take 𝑟 to be negative, the entire circle is covered.

Example 6.52
A. Determine the shape formed by 𝑟 = −3.
𝜋 3𝜋
B. Graph this shape. Make sure to identify the points where 𝜃 ∈ {0, 2 , 𝜋, 2
}

Example 6.53
𝑘𝑚 𝑘𝑚
X travels at 10 ℎ𝑟 . 𝑌 travels at 5 ℎ𝑟 . Y is 5 𝑘𝑚 away from 𝑋. Y travels away 𝑋. At the same time, X goes to catch Y.
When will X catch 𝑌, and how much time will it take?

𝑇𝑖𝑚𝑒: 1 𝐻𝑜𝑢𝑟
𝐷𝑖𝑠𝑡𝑎𝑛𝑐𝑒 𝑓𝑜𝑟 𝑌: 5 𝑘𝑚
𝐷𝑖𝑠𝑡𝑎𝑛𝑐𝑒 𝑓𝑜𝑟 𝑋: 10 𝑘𝑚

𝐷𝑖𝑠𝑡𝑎𝑛𝑐𝑒
𝑇𝑖𝑚𝑒 =
𝑆𝑝𝑒𝑒𝑑
𝐺𝑎𝑝 5 5
𝑇𝑖𝑚𝑒 = = = =1
𝑆𝑝𝑒𝑒𝑑 10 − 5 5

Example 6.54
𝜃
Spaceship 𝑋 travels along the curve 𝑟 = 3, 𝑡 = 𝜃. Spaceship 𝑌 travels along the curve 𝑟 = −3, 𝑡 = . 𝑡 represents
2
time in minutes and 0 ≤ 𝑡 ≤ 2𝜋.
A. Write 𝜃 as a function of 𝑡 for each of the spaceships.
B. Determine if and when the paths of the two spaceships intersect.

Part A

P a g e 202 | 213
Aziz Manva (azizmanva@gmail.com)

For Spaceship X, angular Speed


𝑟𝑎𝑑
= 𝐴𝑛𝑔𝑙𝑒 𝑡𝑟𝑎𝑣𝑒𝑙𝑙𝑒𝑑 𝑝𝑒𝑟 𝑢𝑛𝑖𝑡 𝑡𝑖𝑚𝑒 = 1
𝑚𝑖𝑛
For Spaceship 𝑌, angular Speed
𝑟𝑎𝑑
= 𝐴𝑛𝑔𝑙𝑒 𝑡𝑟𝑎𝑣𝑒𝑙𝑙𝑒𝑑 𝑝𝑒𝑟 𝑢𝑛𝑖𝑡 𝑡𝑖𝑚𝑒 = 2
𝑚𝑖𝑛
Part B
Starting point is:
𝑋 = (3,0), 𝑌 = (−3,0)
The relative speed is:
2−1 = 1
The time taken by 𝑋 to catch up with 𝑌 is
𝐴𝑛𝑔𝑙𝑒 𝜋
𝑇𝑖𝑚𝑒 = = =𝜋
𝐴𝑛𝑔𝑢𝑙𝑎𝑟 𝑆𝑝𝑒𝑒𝑑 2 − 1

Example 6.55
Explain the difference in the circle 𝑟 = 2 and the circle 𝑟 = −2 by considering the restriction
0 ≤ 𝜃 ≤ 90°

𝑟 = 2 begins at (2,0) and proceeds counterclockwise.


𝑟 = −2 begins at (−2, −1) and proceeds counterclockwise.

In general, for any value of 𝜃, 𝑟 = 2 and 𝑟 = −2 are at diametrically


opposite points on the circle.

6.56: Polar to Cartesian Coordinates


𝑥 = 𝑟 cos 𝜃 , 𝑦 = 𝑟 sin 𝜃

𝑥 2 + 𝑦 2 = 𝑟 2 cos 2 𝜃 + 𝑟 2 sin2 𝜃 = 𝑟 2
𝑥2 + 𝑦2 = 𝑟2

6.57: Cartesian to Polar Coordinates


The equation of a circle with center at the origin is given in Cartesian Coordinates by
𝑥 2 + 𝑦 2 = 𝑅 2. We can convert this into polar coordinates to show that
𝑟 = ±𝑅

𝑥 2 + 𝑦 2 = 𝑅2
(𝑟 cos 𝜃)2 + (𝑟 sin 𝜃)2 = 𝑅 2
𝑟 2 cos2 𝜃 + 𝑟 2 sin2 𝜃 = 4
𝑟 2 (cos2 𝜃 + sin2 𝜃) = 4
𝑟2 = 𝑅2
𝑟 = ±𝑅

Example 6.58

P a g e 203 | 213
Aziz Manva (azizmanva@gmail.com)

The equation of a circle touching the origin is given in Cartesian coordinates by:
(𝑥 − 1)2 + (𝑦 − 1)2 = 4

Convert the equation into polar coordinates.

Expand the brackets:


𝑥 2 − 2𝑥 + 1 + 𝑦 2 − 2𝑦 + 1 = 4

Rearrange and subtract 2 from both sides:


𝑥 2 + 𝑦 2 − 2(𝑥 + 𝑦) = 2

Substitute 𝑥 2 + 𝑦 2 = 𝑟 2 , 𝑥 = 𝑟 cos 𝜃 , 𝑦 = 𝑟 sin 𝜃:


𝑟 2 − 2𝑟(cos 𝜃 + sin 𝜃) = 2

𝑟[1 − 2(cos 𝜃 + sin 𝜃)] = 2


2
𝑟=
1 − 2(cos 𝜃 + sin 𝜃)

Example 6.59
Graph the polar curve:
𝑟 2 − 5𝑟 + 6 = 0

This is a quadratic. Factor and solve:


(𝑟 − 2)(𝑟 − 3) = 0 ⇒ 𝑟 ∈ {2,3}

This corresponds to 2 distinct circles with radii 2 and 3.

Example 6.60
Graph the region that satisfies:
𝑟 2 − 5𝑟 + 6 < 0, 𝑟∈ℝ

(𝑟 − 2)(𝑟 − 3) < 0
2<𝑟<3

And the region is the shaded area between the circles with radii 2 and 3 (graphed
alongside).

6.61: Disc

P a g e 204 | 213
Aziz Manva (azizmanva@gmail.com)

A circular disc in the coordinate plane with


center at origin and radius 𝑅 is given by
𝐵𝑜𝑢𝑛𝑑𝑎𝑟𝑦 𝐼𝑛𝑐𝑙𝑢𝑑𝑒𝑑: 𝑟 ≤ 𝑅
𝐵𝑜𝑢𝑛𝑑𝑎𝑟𝑦 𝐸𝑥𝑐𝑙𝑢𝑑𝑒𝑑: 𝑟 < 𝑅

Example 6.62
The diagram alongside shows two circles. Write
the equation of the circles in polar form.

𝐿𝑒𝑓𝑡 𝐶𝑖𝑟𝑐𝑙𝑒: 𝑟 < 2


𝑅𝑖𝑔ℎ𝑡 𝐶𝑖𝑟𝑐𝑙𝑒: 𝑟 ≤ 1

6.63: Annulus
Annulus is the region between two concentric circles.

White shaded region in the adjoining diagram.

6.64: Inequality for an Annulus


𝑅1 ≤ 𝑟 ≤ 𝑅2
Where
𝑅1 = 𝑅𝑎𝑑𝑖𝑢𝑠 𝑜𝑓 𝑖𝑛𝑛𝑒𝑟 𝑐𝑖𝑟𝑐𝑙𝑒
𝑅2 = 𝑅𝑎𝑑𝑖𝑢𝑠 𝑜𝑓 𝑜𝑢𝑡𝑒𝑟 𝑐𝑖𝑟𝑐𝑙𝑒

Endpoints included versus not included:


(𝑟 − 𝑅1 )(𝑟 − 𝑅2 ) ≤ 0

D. Equation of a Line

6.65: Equation of a Line


The equation of a line in polar coordinates going through the origin is
given by
𝜃 = 𝜃1
Where
𝜃1 = 𝑎𝑛𝑔𝑙𝑒 𝑚𝑎𝑑𝑒 𝑏𝑦 𝑙𝑖𝑛𝑒 𝑤𝑖𝑡ℎ 𝑥 𝑎𝑥𝑖𝑠

Since there is no restriction on 𝑟


𝑟 ∈ (−∞, ∞)

Example 6.66
Determine the equation of a line that passes the origin and:
A. (0,3)
B. (−√2, −√2)

Part A
(0,3) lies on the 𝑦 axis. Angle made by (0,3) with the 𝑥 axis is 90°.

P a g e 205 | 213
Aziz Manva (azizmanva@gmail.com)

𝜋
𝜃 = 90° 𝑂𝑟 𝜃 =
2
Part B
(−√2, −√2) = (𝑟, 225°)
5𝜋
𝜃 = 225° 𝑂𝑅 𝜃 =
4

6.67: Equation of a Line-II


If 𝜃 = 𝜃1 is the equation of a line, then the equation is also represented by:
𝜃 = 𝜃1 + 180° = 𝜃1 + 𝜋

Turning a line around by 180° results in the same line.


Hence, both equations are equally valid.

6.68: Converting from Rectangular Equations to Polar Equations


To convert from rectangular equations to polar coordinates use the following substitutions:
𝑥 = 𝑟 cos 𝜃
𝑦 = 𝑟 sin 𝜃

6.69: Horizontal and Vertical Lines


We can convert lines from Cartesian coordinates into lines in polar coordinates:
𝑐 𝑐
𝑥=𝑐⇒𝑟= , 𝑦=𝑐⇒𝑟=
cos 𝜃 sin 𝜃
Where
𝑐 𝑖𝑠 𝑎𝑛𝑦 𝑐𝑜𝑛𝑠𝑡𝑎𝑛𝑡, 𝑐 ∈ ℝ

Vertical Line
𝑐
𝑥 = 𝑐 ⇒ 𝑟 cos 𝜃 = 𝑐 ⇒ 𝑟 =
cos 𝜃
Horizontal Line
𝑐
𝑦 = 𝑐 ⇒ 𝑟 sin 𝜃 = 𝑐 ⇒ 𝑟 =
sin 𝜃

Example 6.70
Write the following Cartesian equations in polar form:
A. 𝑥 = 3
B. 𝑦 = 2

3
𝑟 cos 𝜃 = 3 ⇒ 𝑟 =
cos 𝜃
2
𝑟 sin 𝜃 = 2 ⇒ 𝑟 =
sin 𝜃

6.71: General Equation of a Line


The equation of a line that does not pass through the origin:
𝑐
𝑟=
sin 𝜃 − 𝑚 cos 𝜃

𝑦 = 𝑚𝑥 + 𝑐 ⇒ 𝑦 − 𝑚𝑥 = 𝑐

P a g e 206 | 213
Aziz Manva (azizmanva@gmail.com)

Substitute 𝑦 = 𝑟 sin 𝜃 , 𝑥 = 𝑟 cos 𝜃:


𝑟 sin 𝜃 − 𝑚 𝑟 cos 𝜃 = 𝑐

Factor 𝑟:
𝑟(sin 𝜃 − 𝑚 cos 𝜃) = 𝑐

Solve for 𝑟:
𝑐
𝑟=
sin 𝜃 − 𝑚 cos 𝜃

E. Inequalities
Just as equations lead to shapes in the coordinate, inequalities represent intervals on the number line, and lead
to regions in the coordinate plane.

6.72: Region between two lines

Example 6.73
𝑥
Describe the region between the lines 𝑦 = 𝑥 and 𝑦 = using polar coordinates. (Assume the region includes
√3
its boundaries).

𝑦 = 𝑥 ⇒ 𝜃 = 45°
𝑥
𝑦= ⇒ 𝜃 = 30°
√3
The region that we want is:
30° ≤ 𝜃 ≤ 45°

This region has infinite area.

Converting from Polar to Cartesian and vice versa


x=r cos theta
y=r sin theta
r^2 = x^2 + y^2
tan theta = y/x

Example 6.74
Convert from Polar to Cartesian
A. 𝑟 sin 𝜃 = 5
B. 𝑟 cos 𝜃 = 7
C. 𝑟 2 sin 2𝜃 = 1
D. 𝑟 2 sin2 𝜃 = 1

Parts A and B
y=5, Equation of horizontal line
x=7, Equation of vertical line

Part C
𝑟 2 sin 2𝜃 = 1

P a g e 207 | 213
Aziz Manva (azizmanva@gmail.com)

Substitute 2 sin 𝜃 cos 𝜃 = sin 2𝜃


2𝑟 2 sin 𝜃 cos 𝜃 = 1
1
(𝑟 sin 𝜃)(𝑟 cos 𝜃) =
2
1
𝑥𝑦 =
2
𝑅𝑒𝑐𝑡𝑎𝑛𝑔𝑢𝑙𝑎𝑟 𝐻𝑦𝑝𝑒𝑟𝑏𝑜𝑙𝑎

Part D
𝑟 2 sin2 𝜃 = 1
(𝑟 sin 𝜃)2 = 1
𝑦2 = 1
𝑦 = ±1
Part E
𝑟 2 𝑐𝑜𝑠 2 𝜃 − 𝑟 2 𝑠𝑖𝑛2 𝜃 = 1
𝑥2 − 𝑦2 = 1
𝐻𝑦𝑝𝑒𝑟𝑏𝑜𝑙𝑎

Example 6.75
Convert from Cartesian to Polar. Identify the equation.
A. 𝑥 2 + 𝑦 2 = 𝑐
B.
3
C. 𝑟 = 4 𝑐𝑜𝑠 𝜃 − 2 𝑠𝑖𝑛 𝜃

Part A
Convert from Cartesian to Polar. Identify it is as standard equation
𝑥2 + 𝑦2 = 𝑐
𝑟 = √𝑐
Part B
Convert from Polar to Cartesian: Circle
r = a cos theta
r^2 = a r cos theta
x^2 + y^2 = ax
x^2 - ax + y^2 = 0
x^2 - ax +a^2/4 + y^2 = a^2/4
(x-a/4)^2 + y^2 = a^2/4

Part C
Eliminate fractions:
4𝑟 cos 𝜃 − 2𝑟 sin 𝜃 = 3
4𝑥 − 2𝑦 = 3
3
𝑦 = 2𝑥 −
2

6.3 Graphing in Polar Coordinates


A. Summary

P a g e 208 | 213
Aziz Manva (azizmanva@gmail.com)

B. Archimedes Spirals

6.76: Archimedes Spiral


The polar curve where the distance from the origin is given by the angle itself is the
equation:
𝑟=𝜃

C. Circles

6.77: Circles
A circle of radius 𝑅 centered at the origin has the equation:
𝑟=𝑅
𝑟 = −𝑅

6.78: Circles: Trigonometric Version


A circle with diameter 𝑎 and center at the origin has the equation
𝑟 = 𝑎 cos 𝜃

➢ Oriented "right": if a>0


➢ Oriented "left": if a<0

𝑥 = 𝑟 cos 𝜃 = (𝑎 cos 𝜃) cos 𝜃 = 𝑎 cos2 𝜃


𝑦 = 𝑟 sin 𝜃 = (𝑎 cos 𝜃) sin 𝜃

𝜋
When 𝜃 =
4
𝜋 1 2 2
𝑥 = 2 cos2 ( ) = 2 ( ) = = 1
4 √2 2
𝜋 𝜋
𝑦 = (2) cos ( ) sin ( ) = 1
4 4

6.79: Circles: Trigonometric Version


A circle with diameter 𝑎 and center at the origin has the equation
𝑟 = 𝑎 sin 𝜃

➢ Oriented "top": if a>0


➢ Oriented "bottom": if a<0

D. Limacons

6.80: Limacons
Limacons are an important class of curves. They have the form:
𝑟 = 𝑎 ± 𝑏 cos 𝜃
𝑟 = 𝑎 ± 𝑏 sin 𝜃

𝑎 > 0, 𝑏 > 0

P a g e 209 | 213
Aziz Manva (azizmanva@gmail.com)

6.81: Cardoid
𝑎
A limacon is a cardoid if 𝑏 = 1. That is
𝑎=𝑏

6.82: Inner Loop Limacons


A limacon has an inner loop if
𝑎
<1
𝑏

6.83: One Loop Limacons


A limacon has one loop if
𝑎
1< <2
𝑏

This is also called a 𝑑𝑖𝑚𝑝𝑙𝑒𝑑 limacon.


𝑎 = 4, 𝑏 = 3

6.84: Simple Closed Limacons


A simple closed limacon has
𝑎
>2
𝑏

E. Lemniscates

6.85: Lemniscates
This is a polar curve that resembles the infinity symbol. It is centered at the origin.
𝑟 2 = 𝑎2 cos2 𝜃
𝑟 2 = 𝑎2 sin2 𝜃

𝑎≠0

F. Roses

6.86: Graphing Polar Equations


To graph polar equations, we make a table of values.
When the values start to repeat, we stop the graph.

6.87: Graphing Roses


𝑟 = sin 𝑛𝜃 , 𝑛 𝑖𝑠 𝑒𝑣𝑒𝑛 ⇒ 2𝑛 𝑝𝑒𝑡𝑎𝑙𝑠
0 ≤ 𝜃 ≤ 2𝜋

𝑟 = sin 𝑛𝜃 , 𝑛 𝑖𝑠 𝑜𝑑𝑑 ⇒ 𝑛 𝑝𝑒𝑡𝑎𝑙𝑠

P a g e 210 | 213
Aziz Manva (azizmanva@gmail.com)

0≤𝜃≤𝜋

Example 6.88

The table shows a summary of some important types of polar curves.

Polar Curves

Circles Limacons Other Curves

Cardoids
𝑟=𝑛 Leminscates
𝑎=𝑏

Inner Loop: Archimedes


𝑟 = 𝑎 cos 𝜃 𝑎
𝑏
<1 Spiral

One Loop:
𝑟 = 𝑎 𝑠𝑖𝑛 𝜃 𝑎 Roses
1<𝑏<2

Simple
𝑎
𝑏
<2

P a g e 211 | 213
Aziz Manva (azizmanva@gmail.com)

G. Symmetry

6.4 Trigonometric Series


A. Trigonometric Series

6.89: Arithmetic Series


𝑆 = 𝛼 + (𝛼 + 𝛽) + (𝛼 + 2𝛽) + [𝛼 + (𝑛 − 1)𝛽]

Where
𝐹𝑖𝑟𝑠𝑡 𝑇𝑒𝑟𝑚 = 𝛼
𝐶𝑜𝑚𝑚𝑜𝑛 𝑑𝑖𝑓𝑓𝑒𝑟𝑒𝑛𝑐𝑒 = 𝛽

If we take an arithmetic sequence, and apply a trigonometric function to each term of the series, we will get a
trigonometric series.

6.90: Sum of Sine Series


The sum of a trigonometric series with 𝑛 terms:
𝑛𝛽
sin ( 2 ) (𝑛 − 1)𝛽
𝑆 = sin 𝛼 + sin(𝛼 + 𝛽) + ⋯ + sin(𝛼 + (𝑛 − 1)𝛽) = sin (𝛼 + )
𝛽 2
sin ( 2 )

Step I: Expand
Begin with the given expression. Let:
𝑆 = sin 𝛼 + sin(𝛼 + 𝛽) + ⋯ + sin(𝛼 + (𝑛 − 1)𝛽)

𝛽
Multiply both sides by 2 ∙ sin ( 2 ). The reason for this manipulation will become clear in the next step:
𝛽 𝛽 𝛽 𝛽
𝑆 × 2 ∙ sin ( ) = 2 sin 𝛼 sin ( ) + 2 sin(𝛼 + 𝛽) sin ( ) + ⋯ + 2 sin(𝛼 + (𝑛 − 1)𝛽) sin ( )
2 ⏟ 2 ⏟ 2 ⏟ 2
1𝑠𝑡 𝑇𝑒𝑟𝑚 2𝑛𝑑 𝑇𝑒𝑟𝑚 𝑁 𝑡ℎ 𝑇𝑒𝑟𝑚

Step II: Telescope


Use the product to sum formula (2 sin 𝑎 sin 𝑏 = cos(𝑎 − 𝑏) − cos(𝑎 + 𝑏)) to expand each term:

P a g e 212 | 213
Aziz Manva (azizmanva@gmail.com)

𝛽 𝛽 𝛽
1𝑠𝑡 𝑇𝑒𝑟𝑚: 2 sin 𝛼 sin ( ) = cos (𝛼 − ) − cos (𝛼 + )
2 2 2
𝛽 𝛽 3𝛽
2𝑛𝑑 𝑇𝑒𝑟𝑚: 2 sin(𝛼 + 𝛽) sin ( ) = cos (𝛼 + ) − cos (𝛼 + )
2 2 2
.
.
.
𝛽 (2𝑛 − 3)𝛽 (2𝑛 − 1)𝛽
𝑁 𝑡ℎ 𝑇𝑒𝑟𝑚: 2 sin(𝛼 + (𝑛 − 1)𝛽) sin ( ) = cos (𝛼 + ) − cos (𝛼 + )
2 2 2

The second term of each expansion cancels with the first term of the next expansion:
𝛽 𝛽 (2𝑛 − 1)𝛽
𝑆 × 2 ∙ sin ( ) = [cos (𝛼 − )] − cos (𝛼 + )
2 2 2
Step III: Convert back to sin form
2 sin 𝑎 sin 𝑏 = cos(𝑎 − 𝑏) − cos(𝑎 + 𝑏)

Apply the reverse of the product to sum formula to the above expression. Find 𝑎 and 𝑏 such that:
𝛽 (2𝑛 − 1)𝛽
𝑎−𝑏 =𝛼− , 𝑎+𝑏 =𝛼+
⏟ 2 ⏟ 2
𝑬𝒒𝒖𝒂𝒕𝒊𝒐𝒏 𝑰 𝑬𝒒𝒖𝒂𝒕𝒊𝒐𝒏 𝑰𝑰

This is a system of equations in two variables, which we can solve using standard techniques., Add the two
equations to eliminate 𝑏 from the system:
𝛽 (2𝑛 − 1)𝛽
2𝑎 = 𝛼 − + 𝛼 + = 2𝛼 + (𝑛 − 1)𝛽
2 2

(𝑛−1)𝛽
Solve for 𝑏 by substituting 𝑎 = 𝛼 + 2
in the first equation:
𝛽 (𝑛 − 1)𝛽 𝛽 (𝑛 − 1)𝛽 𝛽 𝑛𝛽
𝑏 = 𝑎 − (𝛼 − ) = 𝛼 + − (𝛼 − ) = + =
2 2 2 2 2 2
Step III: Final Answer
Substitute the values of 𝑎 and 𝑏 in the equality above:
𝛽 𝑛𝛽 (𝑛 − 1)𝛽
𝑆 × 2 ∙ sin ( ) = 2 sin ( ) sin (𝛼 + )
2 2 2

Isolate and solve for 𝑆:


𝑛𝛽
sin ( 2 ) (𝑛 − 1)𝛽
𝑆= sin (𝛼 + )
𝛽 2
sin ( )
2
6.5 Further Topics
A. Hyperbolic Functions
These are covered in the Note on Logarithms

91 Examples

P a g e 213 | 213

You might also like